Final Exam: Review

Réussis tes devoirs et examens dès maintenant avec Quizwiz!

A patient with a diagnosis of advanced Alzheimer disease who is unable to follow directions requires an inhaled bronchodilator. Which of the following medication delivery systems is most appropriate for this patient? a) metered-dose inhaler with spacer b) nebulizer c) metered-dose inhaler without spacer d) dry powder inhaler

B

A patient with a head injury has bloody drainage from the ear. To determine whether CSF is present in the drainage, the nurse a. examines the tympanic membrane for a tear b. tests the fluid for a halo sign on a white dressing c. tests the fluid with a glucose identifying strip or stick d. collects 5 mL of fluid in a test tube and sends it to the laboratory for analysis

B

a nurse is caring for a client who has increased ICP and new prescription for mannitol. for which of the following adverse effects should the nurse monitor? A. hyperglycemia B. hyponatremia C. hypervolemia D. oliguria

B

The nurse is performing an assessment on a client who has returned from the dialysis unit following hemodialysis. The client is complaining of headache and nausea and is extremely restless. Which of the following is the most appropriate nursing action? a. Monitor the client. b. Notify the physician. c. Elevate the head of the bed. d. Medicate the client for nausea.

B Disequilibrium syndrome may be caused by the rapid decreases in the blood urea nitrogen level during hemodialysis. These changes can cause cerebral edema that leads to increased intracranial pressure. The client is exhibiting early signs of disequilibrium syndrome and appropriate treatments with anticonvulsive medications and barbiturates may be necessary to prevent a life-threatening situation. The physician must be notified.

The client's rhythm strip shows a heart rate of 76 beats/min, one P wave occurring before each QRS complex, a PR interval measuring 0.24 second, and a QRS complex measuring 0.08 second. How does the nurse interpret this rhythm strip? a. Normal sinus rhythm b. Sinus bradycardia c. Sinus rhythm with first-degree atrioventricular (AV) block d. Sinus rhythm with premature ventricular contractions

C

You are providing care to a patient experiencing chest pain when coughing or breathing in. The patient has pericarditis. The physician has ordered the patient to take Ibuprofen for treatment. How will you administer this medication? A. strictly without food B. with a full glass of juice C. with a full glass of water D. with or without food

C

A newly graduated RN has just finished a 6-week orientation to the oncology unit. Which of these clients would be most appropriate to assign to the new graduate? A. A 30-year-old with acute lymphocytic leukemia who will receive combination chemotherapy today B. A 40-year-old with chemotherapy-induced nausea and vomiting who has had no urine output for 16 hours C. A 45-year-old with pancytopenia who will require IV administration of erythropoietin (Procrit) D. A 72-year-old with tumor lysis syndrome who is receiving normal saline IV at a rate of 250 mL/hr

C A new nurse after a 6-week oncology orientation possesses the skills to care for clients with pancytopenia and with administration of medications to stimulate the bone marrow. the other options are too complex

The patient with CKD is brought to the emergency department with Kussmaul respirations. What does the nurse know about CKD that could cause this patient's Kussmaul respirations? a. Uremic pleuritis is occurring. b. There is decreased pulmonary macrophage activity. c. They are caused by respiratory compensation for metabolic acidosis. d. Pulmonary edema from heart failure and fluid overload is occurring.

C Kussmaul respirations occur with severe metabolic acidosis when the respiratory system is attempting to compensate by removing carbon dioxide with exhalations.Uremic pleuritis would cause a pleural friction rub.Decreased pulmonary macrophage activity increases the risk of pulmonary infection. Dyspnea would occur with pulmonary edema.

A patient who has acute glomerulonephritis is hospitalized with hyperkalemia. Which information will the nurse monitor to evaluate the effectiveness of the prescribed calcium gluconate IV? a. Urine volume b. Calcium level c. Cardiac rhythm d. Neurologic status

C The calcium gluconate helps prevent dysrhythmias that might be caused by the hyperkalemia. The nurse will monitor the other data as well, but these will not be helpful in determining the effectiveness of the calcium gluconate

A 67-year-old male patient with acute pancreatitis has a nasogastric (NG) tube to suction and is NPO. Which information obtained by the nurse indicates that these therapies have been effective? a. Bowel sounds are present. b. Grey Turner sign resolves. c. Electrolyte levels are normal. d. Abdominal pain is decreased.

D

A 68-yr-old male patient diagnosed with sepsis is orally intubated on mechanical ventilation. Which nursing action is most important? a. Use the open-suctioning technique. b. Administer morphine for discomfort. c. Limit noise and cluster care activities. d. Elevate the head of the bed 30 degrees.

D

After extubation of a patient, which finding would the nurse report to the health care provider immediately? A.) Respiratory rate of 25 breaths/min B.) Patient has difficulty speaking C.) Oxygen saturation of 93% D.) Crowing noise during inspiration

D

After the respiratory therapist performs suctioning on a patient who is intubated, the unlicensed assistive personnel (UAP) measures vital signs for the patient. Which vital sign value should the UAP be instructed to report to the RN immediately? a. Heart rate of 98 beats/min b. Respiratory rate of 24 breaths/min c. Blood pressure of 168/90 mm Hg d. Tympanic temperature of 101.4°F (38.6°C)

D

As a knowledgeable nurse, you know that the primary goals of antiretroviral therapy (ART) include all, EXCEPT: A. Reduce HIV-associated morbidity and prolong the duration and quality of survival B. Restore and preserve immunologic function C. Maximally and durably suppress plasma HIV viral load D. Elimination of HIV entirely from the body

D

Assisted Control Ventilation is best for which patient? A. The patient with sleep apnea B. The patient trying to wean from mechanical ventilation C. The patient who is receiving neuromuscular blocking agents D. The patient who has respiratory drive but cannot sustain normal tidal volume

D

At 11 p.m., a male client is admitted to the emergency department. He has a respiratory rate of 44 breaths/minute. He's anxious, and wheezes are audible. The client is immediately given oxygen by face mask and methylprednisolone (Depo-medrol) I.V. At 11:30 p.m., the client's arterial blood oxygen saturation is 86% and he's still wheezing. The nurse should plan to administer: a. alprazolam (Xanax). b. propranolol (Inderal) c. morphine. d. albuterol (Proventil).

D

Client is 12 hours postoperative craniotomy. Nurse observes and increase in urine output up to 220cc/hour for the past 2 hours. Increased UO may be indicative of ... A.Improved renal function B.Hypovolemic shock C.Normal response post-craniotomy D.Diabetes Insipidus.

D

Client sustained closed head injury. Nurse assess for which early sign of impending neurological deterioration? A.Loss of corneal reflex B.Increased visual acuity C.Bilateral pupil equality and reactivity D.Ipsilateral pupil dilation.

D

Nadolol (Corgard) is prescribed for a patient with chronic stable angina and left ventricular dysfunction. To determine whether the drug is effective, the nurse will monitor for a. decreased blood pressure and heart rate. b. fewer complaints of having cold hands and feet. c. improvement in the strength of the distal pulses. d. the ability to do daily activities without chest pain.

D

On physical assessment of a patient with pericarditis, you may hear what type of heart sound? A. S3 or S4 B. mitral murmur C. pleural friction rub D. pericardial friction rub

D

What is the primary reason for administering morphine to a client with an MI? A) To sedate the client B) To decrease the client's pain C) To decrease the client's anxiety D) To decrease oxygen demand on the client's heart

D

Which is the primary consideration when preparing to administer thrombolytic therapy to a patient who is experiencing an acute myocardial infarction (MI)? a.) History of heart disease. b.) Sensitivity to aspirin. c.) Size and location of the MI. d.) Time since onset of symptoms.

D

Which of the following conditions is the predominant cause of angina? A) Increased preload B) Decreased afterload C) Coronary artery spasm D) Inadequate oxygen supply to the myocardium

D

Which serum laboratory value indicates to the nurse that the patient's CKD is getting worse? a. decreased BUN b. decreased sodium c. decreased creatinine d. decreased calculated GFR

D

a client with a spinal cord injury is prone to experiencing autonomic dysreflexia. the nurse should avoid which measure to minimize the risk of occurrence? A. strict adherence to bowel retraining program B. keeping the linen wrinkle-free under the client C. preventing unnecessary pressure on the lower limbs D. limiting bladder cauterization to once every 12 hours

D

A nurse is monitoring a client with an oral endotracheal tube inserted that is attached to mechanical ventilation. The nurse assesses the client and notes that the client has unequal breath sounds. On the basis of this assessment finding, the nurse would first: A. Contact the physician B. Suction the endotracheal tube C. Apply humidified oxygen to the client D. Check the depth marking at the client's lips

D "If it is determined that breath sounds in the client with an endotracheal tube attached to mechanical ventilation are unequal, the nurse would first check the depth marking at the client's lips to evaluate the endotracheal tube for proper depth. If the tube is deeper or shallower than it should be, repositioning of the tube will be necessary. The nurse would then notify the physician, who may prescribe a chest x-ray to verify placement and then reposition the tube as needed. If the tube is displaced, suctioning the client would not remedy the problem. Humidified oxygen should already be in place for a client receiving mechanical ventilation."

A client who is undergoing chemotherapy for breast cancer reports problems with concentration and memory. Which intervention is indicated at this time? A. Explain that this occurs in some clients and is usually permanent. B. Encourage the client that a small glass of wine may help her relax. C. Protect the client from infection. D. Allow the client an opportunity to express her feelings.

D Although no specific intervention for the side effect is known, therapeutic communication and listening may be helpful to the client.

A client had a transurethral resection of the prostate (TURP) yesterday. The staff nurse notes that the hemoglobin is 8.2 g/dL. What is the nurse's best action? A. Notify the charge nurse as soon as possible. B. Irrigate the catheter with 30 mL normal saline. C. Document the assessment in the medical record. D. Prepare for a blood transfusion.

D Blood transfusions are commonly given after a TURP surgery; a blood transfusion is warranted for a hemoglobin reading of 8.2 g/dL. The nurse is capable of managing this situation with the physician, especially since blood transfusions after a TURP are common. Irrigating the catheter is necessary only if the color of the drainage indicates bleeding or there is a presence of clots. Documentation should be done, but it is not the first priority.

After a colonoscopy, a client reports that he is experiencing abdominal fullness and cramping. What is the nurse's best action? A. Insert a rectal tube to assist in passing of flatus. B. Notify the physician immediately because a bowel perforation is suspected. C. Have the client consume only liquids for 4 to 6 hours after the test. D. Explain to the client that this feeling can be expected for several hours after the test.

D Feelings of abdominal fullness, cramping, and the passing of flatus are normally expected after a colonoscopy.

What is the most appropriate nursing intervention to help an HIV-infected patient adhere to a treatment regimen? a. "Set up" a drug pillbox for the patient every week. b. Give the patient a video and a brochure to view and read at home. c. Tell the patient that the side effects of the drugs are bad but that they go away after a while. d. Assess the patient's routines and find adherence cues that fit into the patient's life circumstances.

D The best approach to improve adherence to a treatment regimen is to learn about the patient's life and assist with problem solving within the confines of that life.

A 27-year-old female patient is admitted to the hospital for evaluation of right lower quadrant abdominal pain with nausea and vomiting. Which action should the nurse take? a. Encourage the patient to sip clear liquids. b. Assess the abdomen for rebound tenderness. c. Assist the patient to cough and deep breathe. d. Apply an ice pack to the right lower quadrant.

D The patient's clinical manifestations are consistent with appendicitis, and application of an ice pack will decrease inflammation at the area. Checking for rebound tenderness frequently is unnecessary and uncomfortable for the patient. The patient should be NPO in case immediate surgery is needed. The patient will need to know how to cough and deep breathe postoperatively, but coughing will increase pain at this time.

C

End - stage renal disease puts you at risk for? A. Hypokalemia B. Hyponatremia C. Hyperkalemia D. Hypernatremia

C

The 65-year-old patient has a potassium laboratory value of 5.0 mEq/L. How does the nurse interpret this value? a. High for the patient's age b. Low for the patient's age c. Normal for the patient's age d. Dependent upon the medical diagnosis

BCD

The nurse caring for the patient with hypercalcemia anticipates orders for which medications? (Select all that apply) a. Magnesium sulfate b. Calcitonin (Calcimar) c. Furosemide (Lasix) d. Plicamycin (Mithracin) e. Calcium gluconate f. Aluminum hydroxide

A patient has been diagnosed with chronic mitral regurgitation. When interviewing the patient, which of these should the healthcare provider expect the patient to report?Please choose from one of the following options. 1. Frequent dyspnea 2. Occasional syncope 3. Swelling of the feet 3. Insomnia 3 nights/week

1

The client with chronic alcoholism has chronic pancreatitis and hypomagnesemia.What should the nurse assess when administering magnesium sulfate to the client? 1. Deep tendon reflexes. 2. Arterial blood gases. 3. Skin turgor. 4. Capillary refill time.

1

The nurse determines that a client with a tracheostomy tube needs suctioning if which finding is noted? 1.Rhonchi are auscultated. 2.Pleural friction rub is heard. 3.Fine crackles are auscultated. 4.Pulse oximetry reading is 96%.

1

The nurse is admitting a client diagnosed with protein-calorie malnutrition secondary to AIDS. Which intervention should be the nurse's first intervention? 1. Assess the client's body weight and ask what the client has been able to eat. 2. Place in contact isolation and don a mask and gown before entering the room. 3. Check the HCP's orders and determine what laboratory tests will be done. 4. Teach the client about total parenteral nutrition and monitor the subclavianIV site.

1

The nurse is caring for a client in acute addisonian crisis. Which laboratory data would the nurse expect to find? 1. Hyperkalemia 2. Reduced blood urea nitrogen (BUN) 3. Hypernatremia 4. Hyperglycemia

1 In adrenal insufficiency, the client has hyperkalemia due to reduced aldosterone secretion. BUN increases as the glomerular filtration rate is reduced. Hyponatremia is caused by reduced aldosterone secretion. Reduced cortisol secretion leads to impaired glyconeogenesis and a reduction of glycogen in the liver and muscle, causing hypoglycemia.

A client is diagnosed with syndrome of inappropriate antidiuretic hormone (SIADH). The nurse informs the client that the physician will prescribe diuretic therapy and restrict fluid and sodium intake to treat the disorder. If the client does not comply with the recommended treatment, which complication may arise? 1. Cerebral edema 2. Hypovolemic shock 3. Severe hyperkalemia 4. Tetany

1 Noncompliance with treatment for SIADH may lead to water intoxication from fluid retention caused by excessive antidiuretic hormone. This, in turn, limits water excretion and increases the risk for cerebral edema. Hypovolemic shock results from, severe deficient fluid volume; in contrast, SIADH causes excess fluid volume. The major electrolyte disturbance in SIADH is dilutional hyponatremia, not hyperkalemia. Because SIADH doesn't alter renal function, potassium excretion remains normal; therefore, severe hyperkalemia doesn't occur. Tetany results from hypocalcemia, an electrolyte disturbance not associated with SIADH.

The nurse is evaluating the adequacy of a burn-injured patient's nutritional intake. Which of the following laboratory values is the best indicator of a need to adjust the nutritional program? 1. glycosuria 2. creatine phosphokinase (CPK) 3. BUN levels 4. hemoglobin 5. serum sodium levels

1 Rationale: Glucose in the urine is seen after a major burn injury. It signals the need to reevaluate the patient's nutritional plan. Creatine phosphokinase is used to identify the presence of muscle injuries. BUN levels are used to evaluate kidney function. Hemoglobin levels will fluctuate with the stages of the burn injury dependent upon the fluid status. Serum sodium levels are not indicative of nutritional status.

A patient with third-degree burns to his right arm is scheduled for passive range of motion to the extremity every two hours. Which of the following should the nurse do prior to this exercise session? 1. Medicate for pain. 2. Empty the patient's in-dwelling catheter collection bag. 3. Change the patient's bed linens. 4. Change the dressing on the burn.

1 Rationale: The nurse should anticipate this patient's needs for analgesia and administer pain medication to promote the patient's comfort during the exercise session. Arm exercise is not related to the amount of urine in the catheter bag. Linen changes do not impact range of motion activities. The burn's dressing is changed according to the physician's orders or as needed.

A client is wearing a continuous cardiac monitor, which begins to sound its alarm. A nurse sees no electrocardiogram complexes on the screen. The first action of the nurse is to: 1.Check the client status and lead placement 2.Press the recorder button on the electrocardiogram console. 3.Call the physician 4.Call a code blue

1 Sudden loss of electrocardiogram complexes indicates ventricular asystole or possible electrode displacement. Accurate assessment of the client and equipment is necessary to determine the cause and identify the appropriate intervention.

The client diagnosed with Addison's disease is admitted to the emergency departmentafter a day at the lake. The client is lethargic, forgetful, and weak. Which interventionshould be the emergency department nurse's first action? 1. Start an IV with an 18-gauge needle and infuse NS rapidly. 2. Have the client wait in the waiting room until a bed is available. 3. Perform a complete head-to-toe assessment. 4. Collect urinalysis and blood samples for a CBC and calcium level.

1 This client has been exposed to wind and sun at the lake during the hours prior to being admitted to the emergency department. This predisposes the client to dehydration and an Addisonian crisis. Rapid IV fluid replacement is necessary.

A patient is coming into the emergency department with third-degree burns over 25% of his body. The nurse should prepare which of the following solutions for intravenous infusion for this patient? 1. warmed lactated Ringer's 2. 5% dextrose in water 3. 5% dextrose in 0.45 normal saline 4. 5% dextrose in normal saline

1 Rationale: Warmed Ringer's lactate solution is the intravenous fluid most widely used during the first 24 hours after a burn injury because it most closely approximates the body's extracellular fluid composition.

When teaching a patient regarding desmopressin (DDAVP), the nurse will inform the patient to monitor for which potential side effects? 1. Headache 2. Weight gain 3. Nasal irritation 4. Hyperglycemia 5. Hypotension

123

The nurse is preparing a client diagnosed with rule-out meningitis for a lumbarpuncture. Which interventions should the nurse implement? Select all that apply. 1. Obtain an informed consent from the client or significant other. 2. Have the client empty the bladder prior to the procedure. 3. Place the client in a side-lying position with the back arched. 4. Instruct the client to breathe rapidly and deeply during the procedure. 5. Explain to the client what to expect during the procedure.

1235

The nurse is performing an admission assessment on a client diagnosed with diabetes insipidus. Which findings should the nurse expect to note during the assessment? (SATA) 1. Extreme polyuria 2. Excessive thirst 3. Elevated systolic blood pressure 4. Low urine specific gravity 5. Bradycardia 6. Elevated serum potassium level

124 Signs and symptoms of diabetes insipidus include an abrupt onset of extreme polyuria, excessive thirst, dry skin and mucous membranes, tachycardia, and hypotension. Diagnostic studies reveal low urine specific gravity and osmolarity and elevated serum sodium. Serum potassium levels are likely to be decreased, not increased.

A nurse is caring for a child who had a laproscopic appendectomy. What interventions should the nurse document on the child's clinical record? Select all that apply. 1) Intake and Output 2) Measurement of Pain 3) Tolerance to low-residue diet 4) Frequency of dressing changes 5) Auscultation of bowel sounds

125

The nurse is completing an assessment on a client who is being admitted for a diagnostic workup for primary hyperparathyroidism. Which client complaint would be characteristic of this disorder? Select all that apply. 1. Polyuria 2. Headache 3. Bone pain 4. Nervousness 5. Weight gain

13 The role of parathyroid hormone (PTH) in the body is to maintain serum calcium homeostasis. In hyperparathyroidism, PTH levels are high, which causes bone resorption (calcium is pulled from the bones). Hypercalcemia occurs with hyperparathyroidism. Elevated serum calcium levels produce osmotic diuresis and thus polyuria. This diuresis leads to dehydration (weight loss rather than weight gain). Loss of calcium from the bones causes bone pain. Options 2, 4, and 5 are not associated with hyperparathyroidism. Some gastrointestinal symptoms include anorexia, nausea, vomiting, and constipation.

The nurse is assessing the client diagnosed with meningococcal meningitis. Which assessment data would warrant notifying the HCP? 1. Purpuric lesions on the face. 2. Complaints of light hurting the eyes. 3. Dull, aching, frontal headache. 4. Not remembering the day of the week.

1`

A client with a large goiter is scheduled for a subtotal thyroidectomy to treat thyrotoxicosis. Saturated solution of potassium iodide (SSKI) is prescribed preoperatively for the client. The expected outcome of using this drug is that it helps: 1.slow progression of exophthalmos 2. reduce the vascularity of the thyroid gland. 3. decrease the body's ability to store thyroxine. 4. increase the body's ability to excrete thyroxine.

2

A lumbar puncture is performed on a child suspected of having bacterial meningitis. CSF is obtained for analysis. A nurse reviews the results of the CSF analysis and determines that which of the following results would verify the diagnosis? 1. Cloudy CSF, decreased protein, and decreased glucose 2. Cloudy CSF, elevated protein, and decreased glucose 3. Clear CSF, elevated protein, and decreased glucose 4. Clear CSF, decreased pressure, and elevated protein

2

Which medication should be available to provide emergency treatment if a client develops tetany after a subtotal thyroidectomy? 1. Sodium phosphate 2. calcium gluconate 3. echothiophate iodide 4. sodium bicarbonate

2

C

The nurse is caring for several patients at risk for falls because of fluid and electrolyte imbalances. Which task related to patient safety and fall prevention does the nurse delegate to the UAP? a. Assess for orthostatic hypotension b. Orient the patient to the environment. c. Help the incontinent patient to toilet every 1 to 2 hours. d. Encourage family members or significant other to stay with the patient.

C

The nurse is caring for the patient with hypernatremia caused by fluid and sodium losses. What type of IV solution is best for treating this patient? a. Hypotonic 0.225% sodium chloride. b. Small-volume infusions of hypertonic (2% to 3%) saline. c. Isotonic sodium chloride (NaCl) d. Isotonic Ringer's lactate

A client is diagnosed with Cushing's syndrome. When reviewing the recent laboratory results, the nurse should expect an excess of which substance? 1. Calcium 2. Cortisol 3. Epinephrine 4. Norepinephrine

2 Cushing's syndrome is characterized by an excess of cortisol, a glucocorticoid. Glucocorticoids are produced by the adrenal cortex. Calcium would be decreased in this disorder. Epinephrine and norepinephrine are produced by the adrenal medulla.

After several diagnostic tests, a client is diagnosed with diabetes insipidus. The nurse understands that which symptom is indicative of this disorder? 1. Diarrhea 2. Polydipsia 3. Weight gain 4. Blurred vision

2 Polydipsia and polyuria are classic symptoms of diabetes insipidus. The urine is pale in color, and its specific gravity is low. Anorexia and weight loss occur. Diarrhea, weight loss, and blurred vision are not manifestations of the disorder.

The client diagnosed with hyperthyroidism is prescribed propylthiouracil (PTU). Which statement by the client warrants immediate intervention by the nurse? 1. "I seem to be drowsy and sleepy all the time." 2. "I have a sore throat and have had a fever." 3. "I have gained 2 pounds since I started taking PTU." 4. "Since taking PTU I am not as hot as I used to be."

2 Propylthiouracil (PTU) is a hyperthyroid treatment. The antithyroid medication may affect the body's ability to defend itself against bacteria and viruses; therefore, the nurse should intervene if the client has any type of sore throat, fever, chills, malaise, or weakness.

The client has just had emergency intubation for respiratory distress. Immediately after endotracheal tube insertion, which of the following actions by the nurse is most appropriate? 1. Tape the tube securely in place 2. Assess for bilateral breath sounds 3. Call for chest x-ray to determine placement 4. Assure the client that alternative communication means will be provided

2 The first action by the nurse is to assess for bilateral breath sounds as an initial indication of correct tube placement. The nurse would next secure the tube and then call for chest x-ray to confirm tube placement. Once the client's airway and breathing have been attended to, then the nurse can assure the client about alternative communication means

A nurse is caring for a client following an adrenalectomy and is monitoring for signs of adrenal insufficiency. Which of the following, if noted in the client, indicates signs and symptoms related to adrenal insufficiency? Select all that apply. 1. Double vision 2. Hypotension 3. Mental status changes 4. Weakness 5. Fever

2345 The nurse should be alert to signs and symptoms of adrenal insufficiency in a client following adrenalectomy. These signs and symptoms include weakness, hypotension, fever, and mental status changes. Double vision is generally not associated with this condition.

Sublingual nitroglycerin tablets begin to work within 1 to 2 minutes. How should the nurse instruct the client to use the drug when chest pain occurs? 1.Take one tablet every 2 to 5 minutes until the pain stops. 2.Take one tablet and rest for 10 minutes. Call the physician if pain persists after 10 minutes. 3.Take one tablet, then an additional tablet every 5 minutes for a total of 3 tablets. Call the physician if pain persists after three tablets. 4.Take one tablet. If pain persists after 5 minutes, take two tablets. If pain still persists 5 minutes later, call the physician.

3

The client diagnosed with ARF is admitted to the intensive care unit and placed on a therapeutic diet. Which diet is most appropriate for the client? 1.A high-potassium and low-calcium diet. 2.A low-fat and low-cholesterol diet. 3.A high-carbohydrate and restricted-protein diet. 4.A regular diet with six (6) small feedings a day.

3

The client diagnosed with hypothyroidism is prescribed the thyroid hormone levothyroxine (Synthroid). Which assessment data indicate the medication has been effective? 1. The client has a three (3)-pound weight gain. 2. The client has a decreased pulse rate. 3. The client's temperature is WNL. 4. The client denies any diaphoresis.

3

The client had a right-sided chest tube inserted two (2) hours ago for a pneumothorax. Which action should the nurse take if there is no fluctuation (tidaling) in the water-seal compartment? 1. Obtain an order for a stat chest x-ray. 2. Increase the amount of wall suction. 3. Check the tubing for kinks or clots. 4. Monitor the client's pulse oximeter reading.

3

The nurse is admitting a client diagnosed with primary adrenal cortex insufficiency (Addison's disease). Which clinical manifestations should the nurse expect to assess? 1. Moon face, buffalo hump, and hyperglycemia. 2. Hirsutism, fever, and irritability. 3. Bronze pigmentation, hypotension, and anorexia. 4. Tachycardia, bulging eyes, and goiter.

3

The nurse is assessing a client in an outpatient clinic. Which assessment data are a risk factor for developing pheochromocytoma? 1. A history of skin cancer. 2. A history of high blood pressure. 3. A family history of adrenal tumors. 4. A family history of migraine headaches.

3

The nurse is assessing the client diagnosed with bacterial meningitis. Which clinicalmanifestations would support the diagnosis of bacterial meningitis? 1. Positive Babinski's sign and peripheral paresthesia. 2. Negative Chvostek's sign and facial tingling. 3. Positive Kernig's sign and nuchal rigidity. 4. Negative Trousseau's sign and nystagmus.

3

The nurse is discussing the endocrine system with the client. Which endocrine gland secretes epinephrine and norepinephrine? 1. The pancreas. 2. The adrenal cortex. 3. The adrenal medulla. 4. The anterior pituitary gland

3

The nurse should teach the client with Graves' disease to prevent corneal irritation from mild exophthalmos by: 1. massaging the eyes at regular intervals. 2. instilling an ophthalmic anesthetic as prescribed. 3. wearing dark-colored glasses. 4. Covering both eyes with moistened gauze pads.

3

The patient, newly diagnosed with hypothyroidism, seems very anxious to begin her drug regimen. The nurse's instructions include: 1. "Be certain that no dose is skipped." 2. "If a dose is skipped one day, double the dose the next day." 3. "Know the signs and symptoms of hyperthyroidism." 4. "You will be able to notice the benefits of thyroid replacement therapy right away."

3

When conducting a health history with a female client with thyrotoxicosis, the nurse should ask about which changes in the menstrual cycle? 1. dysmenorrhea 2. metrorrhagia 3. oligomenorrhea 4. menorrhagia

3

A client has been diagnosed with hypoparathyroidism. The nurse teaches the client to include foods in the diet that are: 1. High in phosphorus and low in calcium 2. Low in phosphorus and low in calcium 3. Low in phosphorus and high in calcium 4. High in phosphorus and high in calcium

3 Hypoparathyroidism results in hypocalcemia. A therapeutic diet for this disorder is one that is high in calcium but low in phosphorus because these two electrolytes have inverse proportions in the body. All of the other options are unrelated to this disorder and are incorrect.

The emergency department nurse is caring for a client in an Addisonian crisis. Which intervention should the nurse implement first? 1. Draw serum electrolyte levels. 2. Administer methylprednisolone IV. 3. Start an 18-gauge catheter with normal saline. 4. Ask the client what medications he or she is taking

3 The nurse must treat an Addisonian crisis as all other shock situations. An IV and fluid replacement are imperative to prevent or treat shock. This is the first action

The nurse is admitting a client with acute appendicitis to the emergency department. The client has abdominal pain of 10 on a pain scale of 1 to 10. The client will be going to surgery as soon as possible. The nurse should: 1. Contact the surgeon to request an order for a narcotic for the pain. 2. Maintain the client in a recumbent position. 3. Place the client on nothing-by-mouth (NPO) status. 4. Apply heat to the abdomen in the area of the pain.

3 The nurse should place the client on NPO status in anticipation of surgery. The nurse can initiate pain relief strategies, such as relaxation techniques, but the surgeon will likely not order narcotic medication prior to surgery. The nurse can place the client in a position that is most comfortable for the client. Heat is contraindicated because it may lead to perforation of the appendix.

A nurse would expect to note which interventions in the plan of care for a client with hypothyroidism? Select all that apply. 1. Provide a cool environment for the client. 2. Instruct the client to consume a high-fat diet. 3. Instruct the client about thyroid replacement therapy. 4. Encourage the client to consume fluids and high-fiber foods in the diet. 5. Instruct the client to contact the health care provider if episodes of chest pain occur. 6. Inform the client that iodine preparations will be prescribed to treat the disorder.

345 The clinical manifestations of hypothyroidism are the result of decreased metabolism from low levels of thyroid hormone. Interventions are aimed at replacement of the hormones and providing measures to support the signs and symptoms related to a decreased metabolism. The nurse encourages the client to consume a well-balanced diet that is low in fat for weight reduction and high in fluids and high-fiber foods to prevent constipation. The client often has cold intolerance and requires a warm environment. The client would notify the health care provider if chest pain occurs since it could be an indication of overreplacement of thyroid hormone. Iodine preparations are used to treat hyperthyroidism. These medications decrease blood flow through the thyroid gland and reduce the production and release of thyroid hormone.

B

The older adult patient needs an oral potassium solution, but is refusing it because it has a strong and unpleasant taste. What is the best strategy the nurse uses to administer the drug? a. Tell the patient that failure to take the drug could result in serious heart problem b. Ask the patient's preference of juice and mix the drug with a small amount c. Mix the solution into food on the patient's meal tray and encourage the patient to eat everything d. Offer the drug to the patient several times and then document the patient's refusal.

D

The patient has a magnesium level of 0.8 mg/dL. Which treatment does the nurse expect to be ordered for this patient? a. Intramuscular magnesium sulfate b. Increased intake of fruits and vegetables c. Oral preparations of magnesium sulfate d. IV magnesium sulfate and discontinuation of diuretic therapy

B

The patient has a serum sodium level of 126 mEq/L. What assessment findings does the nurse expect to see in this patient? a. Constipation and paralytic ileus b. Watery diarrhea with abdominal cramping c. Muscle cramping and spasticity d. Tachypnea and diminished breath sounds

D

The patient is at risk for fluid volume excess and dependent edema. Which task does the nurse delegate to the UAP? a. Massage the legs and heels to stimulate circulation. b. Evaluate the effectiveness of a pressure-reducing-mattress. c. Assess the coccyx, elbows, and hips daily for signs of redness. d. Assist the patient to change position every 2 hours.

two days after her cholecystectomy, a client has been experiencing nausea and vomiting. the client has a t tube in place. for what electrolyte imbalance will the nurse monitor? 1. hypernatremia 2. hyperkalemia 3. hypervolemia 4. hypokalemia

4 gastric secretions are high in postassium

A

The patient on the medical-surgical unit suddenly has a severely elevated magnesium level. What does the nurse anticipate will happen next with the patient's care? a. Immediate transfer to the intensive care unit b. Discontinuation of magnesium sources so the patient can recover c. Administration of diuretics such as furosemide d. Administration of IV magnesium binder

A

The patient with a recent history of anterior neck injury reports muscle twitching and spasms with tingling in the lips, nose, and ears. The nurse suspects these symptoms may be caused by which condition? a. Hypocalcemia b. Hypokalemia c. Hyponatremia d. Hypomagnesemia

B

The patient with low potassium must have an IV potassium infusion. The pharmacy sends a 250 mL IV bag of dextrose in water with 40 mEq of potassium. The label is marked "to infuse over 1 hour." What does the nurse do? a. Obtain a pump and administer the solution b. Double-check the physician's order and call the pharmacy c. Hold the infusion because there is an error in labeling d. Recalculate the rate so that it is safe for the patient

A critical care nurse is caring for a client with an endotracheal tube who is on a ventilator. The nurse knows that meticulous airway management of this patient is necessary. What is the main rationale for this? A) Maintaining a patent airway B) Preventing the need for suctioning C) Maintaining the sterility of the patients airway D) Increasing the patients lung compliance

A

A kidney transplant recipient complains of having fever, chills, and dysuria over the past 2 weeks. What is the first action the nurse should take? a. assess temperature and initiate workup to rule out infection b. reassure the patient that this is common after transplantation c. provide warm cover for the patient and give 1 g acetaminophen orally d. notify the nephrologist that the patient has developed symptoms of acute rejection

A

A patient has ARDS resulting from sepsis. Which measure is most likely to be implemented to maintain cardiac output? A. Administer crystalloid fluids or colloid solutions. B. Position the patient in the Trendelenburg position. C. Perform chest physiotherapy and assist with staged coughing. D. Place the patient on fluid restriction, and administer diuretics.

A

A patient has been started on lamotrigine (Lamictal). How does the nurse instruct this patient to take the medication to decrease the incidence of gastric irritation? A. With milk or food B. Between meals with a glass of orange juice C. At bedtime D. One hour before meals or 2 hours after meals

A

A patient needing vascular access for hemodialysis asks the nurse what the differences are between an arteriovenous (AV) fistula and a graft. The nurse explains that one advantage of the fistula is that it a. is much less likely to clot. b. increases patient mobility. c. can accommodate larger needles. d. can be used sooner after surgery.

A

A physician prescribes levothyroxine sodium (Synthroid), 0.15 mg orally daily, for a client with hypothyroidism. The nurse will prepare to administer this medication: a) in the morning to prevent insomnia b) only when the client complains of fatigue and cold intolerance c) at various times during the day to prevent tolerance from occurring d) three times daily in equal doses of 0.5 mg each to ensure consistent serum drug levels

A

An ESRD patient receiving HD is considering asking a relative to donate a kidney for transplantation. In assisting the patient to make a decision about treatment, the nurse informs the patient that a. successful transplantation usually provides a better quality of life than that offered by dialysis b. if rejection of the transplanted kidney occurs, no further treatment for the renal failure is available c. HD replaces the normal function of the kidneys, and patients do not have to live with the continual fear of rejection d. the immunosuppressive therapy following transplantation makes the person ineligible to receive other forms of treatment if the kidney fails

A

During the immediate postoperative care of a recipient of a kidney transplant, what is a priority for the nurse to do? a. regulate fluid intake hourly based on urine output b. monitor urine-tinged drainage on abdominal dressing c. medicate the patient frequently for incisional flank pain d. remove the urinary catheter to evaluate the ureteral implant

A

The nurse is caring for a client who is diagnosed with impaired gas exchange. While performing a physical assessment of the client, which data is the nurse likely to find, keeping in mind the client's diagnosis? a) high respiratory rate b) low pulse rate c) high temperature d) low blood pressure

A

The nurse is caring for a male client with a chest tube. If the chest drainage system is accidentally disconnected, what should the nurse plan to do? A. Place the end of the chest tube in a container of sterile saline B. Apply an occlusive dressing and notify the physician. C. Clamp the chest tube immediately D. Secure the chest tube with tape

A

The nurse is reviewing a client's record and notes that the HCP has documented that the client has CKD. On review of the laboratory results, the nurse most likely would expect to note which finding? a. elevated creatinine level b. decreased hemoglobin level c. decreased RBC d. increased number of WBC in the urine

A

The purpose of adding PEEP to positive pressure ventilation is to a. increase functional residual capacity and improve oxygenation b. increase FIO2 in an attempt to wean the patient and avoid O2 toxicity c. determine if the patient is in synchrony with the ventilator or needs to be paralyzed d. determine is the patient is able to be weaned and avoid the risk of pneumomediastinum

A

To evaluate the effectiveness of antiretroviral therapy (ART), which laboratory test result will the nurse review? a. Viral load testing b. Enzyme immunoassay c. Rapid HIV antibody testing d. Immunofluorescence assay

A

Two hours after surgery the nurse assesses a patient who had a chest tube inserted during surgery. There is 200 mL of dark-red drainage in the chest tube at this time. What is the appropriate action for the nurse to perform? A: Record the amount and continue to monitor drainage B: Notify the health care provider C: Strip the chest tube starting at the chest D: Increase the suction by 10 mm Hg

A

What are the most common early clinical manifestations of ARDS? A. Dyspnea and tachypnea B. Cyanosis and apprehension C. Hypotension and tachycardia D. Respiratory distress and frothy sputum

A

D

What is the intervention of choice for the patient with mild hypernatremia caused by excessive fluid loss? a. IV infusion of 10 units of insulin in 50 mL of 10% dextrose. b. Replacement of table salt with salt substitute c. Furosemide (Lasix) 20 mg IV d. Increased oral water intake

Clients with mitral stenosis would likely manifest symptoms associated with congestion in the? a. Pulmonary circulation b. Descending aorta c. Bundle of His d. Superior vena cava

A Congestion occurs in the pulmonary circulation due to the inefficient emptying of the left ventricle and the lack of a competent valve to prevent back flow into the pulmonary vein.

The nurse is caring for a client with advanced heart failure who develops asystole. The nurse corrects the new graduate when the graduate offers to perform which intervention? a. Defibrillation b. Cardiopulmonary resuscitation (CPR) c. Administration of atropine d. Administration of oxygen

A Defibrillation interrupts the heart rhythm and allows normal pacemaker cells to take over: in asystole, there is no rhythm to interrupt; therefore this intervention is not used.

Which of the following nursing diagnoses would be the most important yet relevant nursing diagnosis for the patient diagnosed with having a pulmonary contusion? A) Fluid Volume Overload B) Imbalanced Nutrition: Less than body requirements C) Acute Pain D) Risk for Infection

A Fluid volume overload would be appropriate for this client because of the fluid build-up occurring in the lungs (AEB: Crackles, decreased breath sounds, etc.). This build-up is caused by the bruising and edema pulling fluid from the vascular spaces.

Which information would be most important to help the nurse determine if the patient needs human immunodeficiency virus (HIV) testing? a. Patient age b. Patient lifestyle c. Patient symptoms d. Patient sexual orientation

A The current Center for Disease Control (CDC) policy is to offer routine testing for HIV to all individuals age 13 to 64. Although lifestyle, symptoms, and sexual orientation may suggest increased risk for HIV infection, the goal is to test all individuals in this age range.

A client with COPD has a physician's prescription stating, "Adjust oxygen to SpO2 at 90% to 92%." Which nursing action can be delegated to a nursing assistant working under the supervision of an RN? A. Adjust the position of the oxygen tubing B. Assess for signs and symptoms of hypoventilation C. Change the O2 flow rate to keep SpO2 as prescribed D. Choose which O2 delivery device should be used for the client

A The scope of a nursing assistant's work includes positioning of oxygen tubing for client comfort.

A patient is receiving continuous enteral nutrition through a small-bore silicone feeding tube. What should the nurse plan for when this patient has a computed tomography (CT) scan ordered? a. Shut the feeding off 30 to 60 minutes before the scan. b. Ask the health care provider to reschedule the CT scan. c. Connect the feeding tube to continuous suction during the scan. d. Send the patient to CT scan with oral suction in case of aspiration.

A The tube feeding should be shut off 30 to 60 minutes before any procedure requiring the patient to lie flat. Because the CT scan is ordered for diagnosis of patient problems, rescheduling is not usually an option. Prevention, rather than treatment, of aspiration is needed. Small-bore feeding tubes are soft and collapse easily with aspiration or suction, making nasogastric suction of gastric contents unreliable.

For a female client with newly diagnosed cancer, the nurse formulates a nursing diagnosis of Anxiety related to the threat of death secondary to cancer diagnosis. Which expected outcome would be appropriate for this client? a. "Client verbalizes feelings of anxiety." b. "Client doesn't guess at prognosis." c. "Client uses any effective method to reduce tension." d. "Client stops seeking information."

A Verbalizing feelings is the client's first step in coping with the situational crisis. It also helps the health care team gain insight into the client's feelings, helping guide psychosocial care. Option B is inappropriate because suppressing speculation may prevent the client from coming to terms with the crisis and planning accordingly. Option C is undesirable because some methods of reducing tension, such as illicit drug or alcohol use, may prevent the client from coming to terms with the threat of death as well as cause physiologic harm. Option D isn't appropriate because seeking information can help a client with cancer gain a sense of control over the crisis.

B

Which clinical condition can result from hypocalcemia? a. Stimulated cardiac muscle contraction b. Increased intestinal and gastric motility c. Decreased peripheral nerve excitability d. Increased bone density

ACDE

Which conditions cause the patient to be at risk for hypernatremia? (Select all that apply) a. Renal failure b. Immobility c. Use of corticosteroids d. Watery diarrhea e. Cushing's syndrome

C

Which food provides both calcium and vitamin D for the patient in need of supplemental diet therapy for hypocalcemia? a. Eggs b. Broccoli c. Milk d. Tofu

A

Which medication order does the nurse clarify before administering the drug to the patient with hypocalcemia? a. Magnesium sulfate 1 g IM every 6 hours for four doses b. Aluminum hydroxide (AlternaGEL) 15 mL orally three times a day and at bedtime c. Calcium carbonate 1000 mg orally after meals and at bedtime d. Calcium gluconate 5 mEq IV PRN for tetany

A

Which of the following mechanisms is the most important regulator of fluid intake? a. Thirst b. Electrolytes c. Renin-angiotensin d. Kidneys

D

Which patient is most at risk for developing hypernatremia? a. Patient who dislikes drinking milk and lacks calcium in the diet. b. Patient who is receiving total parenteral nutrition related to gastrointestinal surgery c. Patient with excessive diarrhea and vomiting from food poisoning d. Older adult patient with decreased sensitivity to thirst

A

A nurse is assessing a client with a suspected diagnosis of hypocalcemia. Which of the following clinical manifestations would the nurse expect to note in the client? a. Twitching b. Hypoactive bowel sounds c. Negative Trousseau's sign d. Hypoactive deep tendon reflexes

B

A nurse is preparing to care for a client with a potassium deficit. The nurse reviews the client's record and determines that the client was at risk for developing the potassium deficit because the client: a. Sustained tissue damage b. Requires nasogastric suction c. Has a history of Addison's disease d. Is taking a potassium-sparing diuretic

A

A nurse is reading a physician's progress notes in the client's record and reads that the physician has documented "insensible fluid loss of approximately 800 mL daily." The nurse interprets that this type of fluid loss can occur through: a. The skin b. Urinary output c. Wound drainage d. The gastrointestinal tract

The nurse is teaching a client about possible seizure triggers. Which information should the nurse​ include? (Select all that​ apply.) A.Specific odors B.Flashing lights C.Menstruation D.Lactose consumption E.Fever

ABCE

a patient is diagnosed with mitral stenosis and new onset a.fib. which interventions could the nurse delegate to unlicensed assitive personnel? (select all that apply) a. obtain and record daily weight b. determine apical-radial pulse rate c. observe for overt signs of bleeding d. obtain and record vital signs, including pulse ox e. teach patient how to purchase medic alert bracelet

ACD

A nurse cares for a client who is recovering from a right-sided heart catheterization. For which complications of this procedure should the nurse assess? (Select all that apply.) a. Thrombophlebitis b. Stroke c. Pulmonary embolism d. Myocardial infarction e. Cardiac tamponade

ACE Complications from a right-sided heart catheterization include thrombophlebitis, pulmonary embolism, and vagal response. Cardiac tamponade is a risk of both right- and left-sided heart catheterizations. Stroke and myocardial infarction are complications of left-sided heart catheterizations.

Which of the following are the 5 characteristics of ARDS? A. Dyspnea B. Myasthenia Gravis C. Refractory hypoxemia D. Cyanosis E. Dense pulmonary infiltrates on CXR F. Decreased pulmonary compliance G. Non-cardiac pulmonary edema H. Chest pain

ACEFG

The nurse is monitoring a client with increased intracranial pressure (ICP). What indicators are the most critical for the nurse to monitor? select all that apply. A. systolic blood pressure B. urine output C. breath sounds D. cerebral perfusion pressure

AD

What characteristics describe positive pressure ventilators? Select all that apply a. require an artificial airway b. applied to outside of body c. most similar to physiologic ventilation d. most frequently used with critically ill patients e. frequently used in the home for neuromuscular or nervous system disorders

AD

A

Which is a preventive measure for patients at risk for developing hypocalcemia? a. Increase daily dietary calcium intake to 1000 mg b. Increase intake of phosphorus c. Apply sunblock and wear protective clothing whenever outdoors d. Administer calcium-containing IV fluids to patients receiving multiple blood transfusions

ABCD

Which patient conditions cause the patient to be at risk for hypocalcemia? (Select all that apply) a. Crohn's disease b. Acute pancreatitis c. Removal or destruction of parathyroid glands d. Immobility e. Use of digitalis

C

Which patient in the medical surgical unit is most likely to have increased aldosterone secretion? a. Patient who has excessive salt ingestion b. Patient who drinks a lot of water c. Patient who loses a lot of fluid and sodium d. Patient who loses potassium and water

C

Which patient is at greatest risk of developing hypocalcemia? a. 30-year-old Asian woman with breast cancer b. 45-year-old Caucasian man with hypertension and diuretic therapy c. 60-year-old Aftrican American woman with a recent ileostomy d. 70-year-old Caucasian man on long-term lithium therapy

BCD

Which potassium levels are within normal limits? (Select all that apply) a. 2.0 mmol/L b. 3.5 mmol/L c. 4.5 mmol/L d. 5.0 mmol/L e. 6.0 mmol/L

BDE

Which statements about the function of the lymphatic system are true? (Select all that apply) a. Lymph fluid contains more protein than plasma. b. Lymph flow is slower than blood flow c. Lymph flow is enhanced by a pump system d. Lymphatic vessels carry lymph fluid toward the heart e. Lymph fluid is filtered by lymph nodes. f. The lymphatic system takes lymph to the kidneys for excretion.

A child is admitted to the pediatric unit with a diagnosis of infective endocarditis. Which of the following interventions should the healthcare provider anticipate? Select all that apply. 1. Seizure precautions 2. Contact isolation 3.Echocardiogram 4. Intravenous antibiotics 5. Blood cultures

345

The amount of air inspired and expired with each breath is called: a. tidal volume. b. residual volume. c. vital capacity. d. dead-space volume.

A

The child who receives a bone marrow transplant will require a. meticulous personal hygiene b. multiple peripheral sites for IV therapy c. less chemotherapy before the transplant d. a room with laminar air flow

A

The dialysis solution is warmed before use in peritoneal dialysis primarily to a. encourage the removal of serum urea b. force potassium back into the cells c. add extra warmth to the body d. promote abdominal muscle relaxation

A

The registered nurse is teaching a nursing student about the importance of observing for bone marrow suppression during chemotherapy. Select the person who displays bone marrow suppression. A. Client with hemoglobin of 7.4 and hematocrit of 21.8 B. Client with diarrhea and potassium level of 2.9 mEq/L C. Client with 250,000 platelets D. Client with 5000 white blood cells/mm3

A

a nurse is caring for a client who just experienced a generalized seizure. which of the following actions should the nurse perform first? A.keep the client in a side lying position B.document the duration of the seizure C.reorient the client to the environment D.provide client hygiene

A

When preparing a patient for a capsule endoscopy study, what should the nurse do? A) Ensure the patient understands the required bowel preparation. B) Have the patient return to the procedure room for removal of the capsule. C) Teach the patient to maintain a clear liquid diet throughout the procedure. D) Explain to the patient that conscious sedation will be used during placement of the capsule.

A A capsule endoscopy study involves the patient performing a bowel prep to cleanse the bowel before swallowing the capsule. The patient will be on a clear liquid diet for 1 to 2 days before the procedure and will remain NPO for 4 to 6 hours after swallowing the capsule. The capsule is disposable and will pass naturally with the bowel movement, although the monitoring device will need to be removed.

A client has a new tracheostomy and is receiving 60% oxygen via tracheostomy collar. Which assessment finding requires immediate action by the nurse? a. Constant, nonproductive coughing b. Blood-tinged sputum c. Rhonchi in upper lobes d. Dry mucous membranes

A Causes and manifestations of lung injury from oxygen toxicity include nonproductive cough, substernal chest pain, GI upset, and dyspnea. Blood-tinged sputum is expected in clients with new tracheostomies. Rhonchi in upper lobes indicates sputum that can be expectorated and is not an emergent problem. Dry mucous membranes should be lubricated, and the client's hydration status can be checked.

The nurse is caring for a client who has developed cardiac tamponade. Which of the following assessments would the nurse anticipate finding? a. Distended neck veins b. Widening pulse pressure c. Bradycardia d. Pleural friction rub

A In cardiac tamponade, intrapericardial pressures rise to a point at which venous blood cannot flow into the heart. As a result, venous pressure rises and the neck veins become distended.

Which of the following patients are MOST at risk for developing endocarditis? Select-all-that-apply: A. A 25 year old male who reports using intravenous drugs on a daily basis. B. A 55 year old male who is post-opt from aortic valve replacement. C. A 63 year old female who is newly diagnosed with hyperparathyroidism and is taking Aspirin. D. A 66 year old female who recently had an invasive dental procedure performed 1 month ago and is having a fever.

ABD

The nurse includes which of the following in teaching regarding the warning signs of cancer? Select all that apply. A. Persistent constipation B. Scab present for 6 months C. Curdlike vaginal discharge D. Axillary swelling E. Headache

ABD Change in bowel habit, A sore that does not heal, A lump or thickening in the breast or elsewhere is a warning signal of cancer.

The nurse is administering an oral antiseizure medication to an adult client. Which intervention should the nurse implement when administering this​ medication? (Select all that​ apply.) A.Monitoring the client for seizure activity B.Monitoring oxygen levels C.Assessing the client for slurred speech D.Administering antiseizure medication 2 hours after antacids are administered E.Asking the client for a list of home medications

ACE

Which findings will the nurse expect when caring for a patient with chronic kidney disease (CKD)? Select all that apply a. Anemia b. Dehydration c. Hypertension d. Hypercalcemia e. Increased risk for fractures

ACE

A client is having a seizure. Which nursing intervention is of immediate​ importance? A.Administering medication B.Maintaining the airway C.Placing a padded tongue blade in the​ client's mouth D.Intubating the client

B

A client with chronic renal failure is receiving hemodialysis three times a week. In order to protect the fistula, the nurse should a. take the BP in the arm with the fistula b. report the loss of a thrill or bruit on the arm with the fistula c. maintain a pressure dressing on the shunt d. start a second IV in the arm with the fistula

B

A female client with interstitial lung disease is prescribed prednisone (Deltasone) to control inflammation. During client teaching, the nurse stresses the importance of taking prednisone exactly as prescribed and cautions against discontinuing the drug abruptly. A client who discontinues prednisone abruptly may experience: a. hyperglycemia and glycosuria. b. acute adrenocortical insufficiency. c. GI bleeding. d. restlessness and seizures.

B

A male adult client with cystic fibrosis is admitted to an acute care facility with an acute respiratory infection. Prescribed respiratory treatment includes chest physiotherapy. When should the nurse perform this procedure? a. Immediately before a meal b. At least 2 hours after a meal c. When bronchospasms occur d. When secretions have mobilized

B

A patient with end-stage renal disease (ESRD) secondary to diabetes mellitus has arrived at the outpatient dialysis unit for hemodialysis. Which assessments should the nurse perform as a priority before, during, and after the treatment? a. Level of consciousness b. Blood pressure and fluid balance c. Temperature, heart rate, and blood pressure d. Assessment for signs and symptoms of infection

B

Acute Respiratory Distress Syndrome (ARDS) can be defined as _____________________________. A. Sudden life-threatening deterioration of gas exchange in the lungs B. Non-cardiac pulmonary edema with increasing hypoxemia despite treatment with O2 C. Sudden life-threatening pulmonary edema that causes a deterioration of gas exchange despite treatment with O2

B

Synthetic thyroid hormone T₃: A. Levothyroxine B. Liothyronine C. Liotrix D. Thyroid

B

The client, age 8, is prescribed valproic acid (Depokene) for treatment of a seizure disorder. The nurse should monitor the client closely for: A.Vitamin B deficiency. B. Restlessness and agitation. C. Hyperthermia. D. Respiratory distress

B

The nurse identifies that the client is unable to cough to produce a sputum specimen and must be suctioned. Which suctioning route is preferred? A. Nasopharyngeal B. Nasotracheal C. Oropharyngeal D. Orotracheal

B

The nurse obtains a 6-second rhythm strip and charts the following analysis: atrial rate 70, regular; ventricular rate 40, regular; QRS 0.04 sec; no relationship between P waves and QRS complexes; atria and ventricles beating independently of each other. Which of the following would be a correct interpretation of this rhythm strip? a. Sinus dysrhythmias b. Third-degree heart block c. Wenckebach phenomenon d. Premature ventricular contractions

B

The patient has been receiving scheduled doses of phenytoin (Dilantin) and begins to experience diplopia. You immediately assess the patient for A. an aura. B. nystagmus or confusion. C. abdominal pain or cramping. D. irregular pulse or palpitations

B

A nurse is completing preoperative teaching for a client who will undergo a laparoscopic cholecystectomy. Which of the following should be included in the teaching? A. "The scope will be passed through your rectum." B. "You may have shoulder pain after surgery." C. "The T-tube will remain in place for 1 to 2 weeks." D. "You should limit how often you walk for 1 to 2 weeks."

B Shoulder pain occurs due to free air that is introduced into the abdomen during laparoscopic surgery.

Client is having tonic-clonic seizure. Nurse should take which of the following actions? SELECT ALL THAT APPLY A.Restrain client B.Maintain airway. C.Turn client to side. D.Place tongue blade in mouth E.Protect client from injury.

BCE

A nurse is caring for a client who has a benign brain tumor. The client asks the nurse if he can expect this same type of tumor to occur in other areas of his body. Which of the following is an appropriate response by the nurse? A. It can spread to breasts and kidneys B. It can develop in your GI tract C. It is limited to brain tissue D. It probably started in another area of your body and spread to your brain

C

A nurse is providing discharge teaching to a client who has a new prescription for aluminum hydroxide(Amphojel). The nurse should advise the client to A. take the medication with food. B. monitor for diarrhea. C. wait 1 to 2 hr before taking other oral medications. D. maintain a low-fiber diet.

C

A patient with a head injury has an arterial blood pressure is 92/50 mm Hg and an intracranial pressure of 18 mm Hg. Which action by the nurse is appropriate? a. Document and continue to monitor the parameters. b. Elevate the head of the patient's bed. c. Notify the health care provider about the assessments. d. Check the patient's pupillary response to light.

C

The patient with CKD is brought to the ED with Kussmaul respirations. What does the nurse know about CKD that could cause this patient's Kussmaul respirations? a. uremic pleuritis is occurring b. there is decreased pulmonary macrophage activity c. they are caused by respiratory compensation for metabolic acidosis d. pulmonary edema from HF and fluid overload is occurring

C

The patient with bacterial meningitis has irritation of cranial nerve (CN) II. What symptom would you expect the patient to have? A. Inability to hear whispered words B. Abnormal pronation and supination test result C. Papilledema D. Dysphagia

C

The patient with breast cancer is having teletherapy radiation treatments after her surgery. What should the nurse teach the patient about the care of her skin? A. Use Dial soap to feel clean and fresh. B. Scented lotion can be used on the area. C. Avoid heat and cold to the treatment area. D. Wear the new bra to comfort and support the area.

C Avoiding heat and cold in the treatment area will protect it. Only mild soap and unscented, nonmedicated lotions may be used to prevent skin damage. The patient will want to avoid wearing tight-fitting clothing such as a bra over the treatment field and will want to expose the area to air as often as possible.

Which information obtained by the nurse who is admitting the patient for magnetic resonance imaging (MRI) will be most important to report to the health care provider before the MRI? a.The patient has an allergy to shellfish. b.The patient has a history of atherosclerosis. c.The patient has a permanent ventricular pacemaker. d.The patient took all the prescribed cardiac medications today.

C MRI is contraindicated for patients with implanted metallic devices such as pacemakers. The other information also will be reported to the health care provider but does not impact on whether or not the patient can have an MRI.

Which intervention will be most helpful for the client with mucositis? A. Administering a biological response modifier B. Encouraging oral care with commercial mouthwash C. Providing oral care with a disposable mouth swab D. Maintaining NPO until the lesions have resolved

C Mouth swabs are soft and disposable and therefore clean. Commercial mouthwashes should be avoided because they may contain alcohol or other drying agents that may further irritate the mucosa.

A 26-year-old woman has been admitted to the emergency department with nausea and vomiting. Which action could the RN delegate to unlicensed assistive personnel (UAP)? a. Auscultate the bowel sounds. b. Assess for signs of dehydration. c. Assist the patient with oral care. d. Ask the patient about the nausea.

C Oral care is included in UAP education and scope of practice. The other actions are all assessments that require more education and a higher scope of nursing practice.

Which value indicates clinical hypoxemia and the need to increase oxygen delivery? A. Hemoglobin of 22 g/dL B. PaCO2 of 30 mm Hg C. PaO2 of 65 mm Hg D. Oxygen saturation of 88%

C PaO2 of 65 mm Hg indicates low levels of oxygen in the arterial blood; this is termed hypoxemia.

A patient is taking prednisolone and fludrocortisone (Florinef). When teaching this patient about dietary intake, the nurse will instruct the patient to consume a diet a. high in carbohydrates. b. high in fat. c. high in protein. d. low in potassium

C Patients receiving fludrocortisone are at risk for negative nitrogen balance and should consume a high-protein diet.

The nurse is providing care to a client with acute respiratory distress syndrome​ (ARDS). Which independent intervention should the nurse prepare to perform for this​ client? (Select all that​ apply.) A. Order a Foley catheter to monitor urine output. B. Prescribe analgesia for pain. C. Maintain the head of the bed at 30 degrees. D. Recommend a prone position to facilitate oxygenation. E. Auscultate heart and lung sounds.

CDE

An 83-year-old female patient was found lying on the bathroom floor. She said she fell 2 days ago and has not been able to take her heart medicine or eat or drink anything since then. What conditions could be causing prerenal AKI in this patient (select all that apply)? a. Anaphylaxis b. Renal calculi c. Hypovolemia d. Nephrotoxic drugs e. Decreased cardiac output

CE Because the patient has had nothing to eat or drink for 2 days, she is probably dehydrated and hypovolemic.Decreased cardiac output (CO) is most likely because she is older and takes heart medicine, which is probably for heart failure or hypertension. Both hypovolemia and decreased CO cause prerenal AKI. Anaphylaxis is also a cause of prerenal AKI but is not likely in this situation. Nephrotoxic drugs would contribute to intrarenal causes of AKI and renal calculi would be a postrenal cause of AKI.

A male client undergoes a laryngectomy to treat laryngeal cancer. When teaching the client how to care for the neck stoma, the nurse should include which instruction? a. "Keep the stoma uncovered." b. "Keep the stoma dry." c. "Have a family member perform stoma care initially until you get used to the procedure." d. "Keep the stoma moist."

D

A nurse is caring for a client with neutropenia who has a suspected infection. Which intervention does the nurse implement first? A. Hydrates the client with 1000 mL of IV normal saline B. Initiates the administration of prescribed antibiotics C. Obtains requested cultures D. Places the client on Bleeding Precautions

D

A nurse is preparing to hang the initial bag of the parenteral nutrition (PN) solution via the central line of a malnourished client. The nurse ensure the availability of which medical equipment before hanging the solution? A. Glucometer. B. Dressing tray. C. Nebulizer. D. Infusion Pump

D

A patient has been newly diagnosed with emphysema. In discussing his condition with the nurse, which of his statements would indicate a need for further education? A: "I'll make sure that I rest between activities so I don't get so short of breath." B: "I'll rest for 30 minutes before I eat my meal." C: "If I have trouble breathing at night, I'll use two to three pillows to prop up." D: "If I get short of breath, I'll turn up my oxygen level to 6 L/min."

D

A patient taking phenytoin (Dilantin) has started attending college and reports frequently drinking alcohol with friends. What does the nurse monitor for in this patient? A. Clinical manifestations of phenytoin toxicity B. Hyperglycemia C. Hypertension D. Increased seizure activity

D

A wife indicates she has been providing care to her husband, who was diagnosed with bacterial meningococcal meningitis. What is your most important action related to the wife? A. Teach airborne isolation precautions. B. Assess her for respiratory infection. C. Explain the signs of meningitis. D. Provide prophylactic antibiotics.

D

Abnormal extension (Decerebrate) posturing is characterized by which of the following? A.Extension of extremities and pronation of the arms B.Flexion of extremities and pronation of arms C.Upper extremity flexion with lower extremity extension D.Upper extremity extension with lower extremity flexion.

D

The gallbladder is found on the __________ side of the body and is located under the ____________. It stores __________.* A. right; pancreas; bilirubin B. left; liver; bile C. right; thymus' bilirubin D. right; liver; bile

D

While the nursing student changes a patients trach dressing, the nurse observes the student using a pair of scissors to cut a 4X4 gauze pad to make a split dressing that will fit around the trach tube. What is the nurses best action? A. give the student positive reinforcement for use of materials and technique B. report the student to the instructor for remediation of the skill C. change the dressing immediately after the student has left the room D. direct the student in the correct use of materials and explain the rationale

D

a client has clear fluid leaking from the nose following a basilar skull fracture. Which finding would alert the nurses the cerebrospinal fluid is present? A. fluid is clear and tests negative for glucose B. fluid is grossly bloody in appearance and has a pH of 6 C. fluid clumps together on the dressing and has a pH of 7 D. fluid separates into concentric rings and tests positive for glucose

D

a client recovering from a head injury is participating in care. the nurse determines that the client understands measures to prevent elevations in intracranial pressure if the nurse observes the client doing which activity? A. blowing the nose B. isometric exercises C. coughing vigorously D. exhaling during repositioning

D

A nurse is planning care for a client who is scheduled for a tracheostomy procedure. What equipment should the nurse plan to have at the bedside when the client returns from surgery? 1.Obturator 2.Oral airway 3.Epinephrine (Adrenalin) 4.Tracheostomy set with the next larger size

1

The nurse is providing care to a patient with a third-degree burn on his left thigh and left forearm. During wound care, the nurse applies Elase to the burned areas. Which of the following types ofwound debridement is this nurse using? 1. enzymatic 2. mechanical 3. surgical 4. topical

1 Rationale: Enzymatic debridement involves the use of a topical agent to dissolve and remove necrotic tissue. An enzyme such as Elase is applied in a thin layer directly to the wound and covered with one layer of fine mesh gauze. A topical antimicrobial agent is then applied and covered with a bulky wet dressing. Mechanical debridement may be performed by applying and removing gauze dressings, hydrotherapy, irrigation, or using scissors and tweezers. Surgical debridement is the process of excising the wound to the fascia or removing thin slices of the burn to the level of viable tissue. Topical treatments are key in the care of a burn but do not involve debridement.

Plans for nursing interventions for a client in the acute stage of bacterial endocarditis should include which of the following interventions? a. Aggressive physical therapy b. Strict fluid restriction c. Daily ECGs d. Administration of analgesics as needed

D Antibiotics to combat the bacterial infection and analgesics for aches that may occur are appropriate interventions for acute-stage bacterial endocarditis. Adequate fluid intake is important also. However, aggressive physical therapy is contraindicated in the acute stage because the client needs to reserve some physical resources for recovery. Daily ECGs are not necessary for acute bacterial endocarditis.

Which serum laboratory value indicates to the nurse that the patient's CKD is getting worse? a. Decreased BUN b. Decreased sodium c. Decreased creatinine d. Decreased calculated glomerular filtration rate (GFR)

D As GFR decreases, BUN and serum creatinine levels increase. Although elevated BUN and creatinine indicate that waste products are accumulating, the calculated GFR is considered a more accurate indicator of kidney function than BUN or serum creatinine.

A patient who takes high-dose aspirin to treat arthritis will need to take prednisone to treat an acute flare of symptoms. What action will the nurse perform? a. Observe the patient for hypoglycemia. b. Monitor closely for increased urine output. c. Observe the patient for hypotension. d. Request an order for enteric-coated aspirin.

D Glucocorticoids can increase gastric distress, so an enteric-coated aspirin product is indicated. Glucocorticoids increase the risk of hypoglycemia, fluid retention, and hypertension.

The nurse is caring for a client on a telemetry unit with a regular heart rhythm and rate of 60; a P wave precedes each QRS complex, and the PR interval is 0.24 second. Additional vital signs are as follows: blood pressure 118/68, respiratory rate 16, and temperature 98.8° F. The following medications are available on the medication record. What action should the nurse take? a. Administer atropine. b. Administer digoxin. c. Administer clonidine. d. Continue to monitor.

D The client is displaying sinus rhythm with first-degree atrioventicular heart block; this is usually asymptomatic and does not require treatment. Atropine is used in emergency treatment of symptomatic bradycardia. This client has normal vital signs. Digoxin is used in the treatment of atrial fibrillation, which is, by definition, an irregular rhythm. Clonidine is used in the treatment of hypertension; a side effect is bradycardia.

A nurse prepares to palpate a client's thyroid gland. Which action should the nurse take when performing this assessment? a. Stand in front of the client instead of behind the client. b. Ask the client to swallow after palpating the thyroid. c. Palpate the right lobe with the nurses left hand. d. Place the client in a sitting position with the chin tucked down

D The client should be in a sitting position with the chin tucked down as the examiner stands behind the client. The nurse feels for the thyroid isthmus while the client swallows and turns the head to the right, and the nurse palpates the right lobe with the right hand. The technique is repeated in the opposite fashion for the left lobe.

A nurse is conducting a follow-up home visit to a client who has been discharged with a parenteral nutrition(PN). Which of the following should the nurse most closely monitor in this kind of therapy? A. Blood pressure and temperature. B. Blood pressure and pulse rate. C. Height and weight. D. Temperature and weight.

D The client's temperature is monitored to identify signs of infection which is one of the complications of this therapy. While the weight is monitored to detect hypervolemia and to determine the effectiveness of this nutritional therapy.

After teaching a group of students about how to perform peritoneal dialysis, which statement would indicate to the instructor that the students need additional teaching? a) "The effluent should be allowed to drain by gravity." b) "It is important to use strict aseptic technique." c) "The infusion clamp should be open during infusion." d) "It is appropriate to warm the dialysate in a microwave."

D The dialysate should be warmed in a commercial warmer and never in a microwave oven. Strict aseptic technique is essential. The infusion clamp is opened during the infusion and clamped after the infusion. When the dwell time is done, the drain clamp is opened and the fluid is allowed to drain by gravity into the drainage bag.

Which of the following may be included in the diet of the client with chronic renal failure? a) orange slices b) watermelon slices c) cantaloupe slices d) apple slices

D the client with renal failure should be given low potassium diet because of hyperkalemia. Apple contains very little potassium. So, it can be given to the client.

A patient has a vegetative growth on the mitral valve which has caused the valve to become stenotic. This increases the risk of which of the following problems? Select all that apply. 1. Atrial fibrillation 2. Decreased peripheral pulses 3. Orthopnea 4. Stroke 5. Pulmonary embolism

1234

The nurse is planning the care of a client diagnosed with syndrome of inappropriate antidiuretic hormone (SIADH). Which interventions should be implemented? Select all that apply. 1. Restrict fluids per health-care provider order. 2. Assess level of consciousness every two (2) hours. 3. Provide an atmosphere of stimulation. 4. Monitor urine and serum osmolality. 5. Weigh the client every three (3) days.

124

The nurse is monitoring the chest tube drainage system in a client with a chest tube. The nurse notes intermittent bubbling in the water seal chamber. Which is the most appropriate nursing action? 1.Check for an air leak. 2.Document the findings. 3.Notify the health care provider. 4.Change the chest tube drainage system.

2

Which statement best describes the scientific rationale for alternating a nonnarcoticantipyretic and a nonsteroidal anti-inflammatory drug (NSAID) every two (2) hoursto a female client diagnosed with bacterial meningitis? 1. This regimen helps to decrease the purulent exudate surrounding the meninges. 2. These medications will decrease intracranial pressure and brain metabolism. 3. These medications will increase the client's memory and orientation. 4. This will help prevent a yeast infection secondary to antibiotic therapy.

2

D

During the shift report, the nurse discovers that the patient has low sodium. What gastrointestinal change does the nurse expect to find during the physical assessment? a. Minimal bowel sounds with frequent episodes of vomitting. b. Absent bowel sounds with pronounced abdominal distention. c. Hypoactive bowel sounds and complaints of constipation. d. Hyperactive bowel sounds and abdominal cramps.

The nurse is instructing a client with diabetes mellitus about peritoneal dialysis. The nurse tells the client that it is important to maintain the prescribed dwell time for the dialysis because of the risk of: 1. Infection. 2. Hyperglycemia. 3. Hypophosphatemia. 4. Disequilibrium syndrome.

2 An extended dwell time increases the risk of hyperglycemia in the client with diabetes mellitus as a result of absorption of glucose from the dialysate and electrolyte changes. Diabetic clients may require extra insulin when receiving peritoneal dialysis.

The nurse is admitting a client to the neurological intensive care unit who is postoperative transsphenoidal hypophysectomy . Which data would warrant immediate intervention? 1. The client is alert to name but is unable to tell the nurse the location. 2. The client has an output of 2500 mL since surgery and an intake of 1000 mL. 3. The client's vital signs are T 97.6, P 88, R 20, and BP 130/80. 4. The client has a 3-cm amount of dark-red drainage on the turban dressing.

2 The output is more than double the intake in a short time. This client could be developing diabetes insipidus, a complication of trauma to the head.

which client problem has priority for the client dx with acute pancreatitis? 1. risk for fluid volume deficit 2. alteration in comfort 3. imbalanced nutrition: less than body requirements 4. knowledge deficit

2 autodigestion of the pancreas results in severe epigastric pain, accompanied by nausea, vomiting, abdominal tenderness and muscle guarding

B

On admission, the patient with pulmonary edema weighed 151 lbs; now the patient's weight is 149 lbs. Assuming the patient was weighed both times with the same clothing, same scale, and same time of day, how many milliliters of fluid does the nurse estimate the patient has lost? a. 500 b. 1000 c. 2000 d. 2500

B

The nurse is caring for the child at risk for dehydration secondary to diarrhea, vomiting, and fever. The child is alert, quiet, and clinging to the parent. What is the best nursing intervention to rehydrate this patient? a. Give an oral rehydration solution such as oralyte or rehydralyte. b. Have the parent give small sips of preferred diluted fluids every 5 to 10 minutes. c. Obtain an order for IV access and an isotonic solution such as normal saline. d. Encourage the child to take as much water as possible and offer popsicles.

C

The nurse is caring for the patient with hypovolemia secondary to severe diarrhea and vomiting. In evaluating the respiratory system for this patient, what does the nurse expect to assess? a. No changes, because the respiratory system is not involved. b. Hypoventilation, because the respiratory system is trying to compensate for low pH c. Increased respiratory rate, because the body perceives hypovolemia as hypoxia d. Normal respiratory rate, but a decreased oxygen saturation

C

The nurse is reviewing orders for several patients who have risk for fluid volume excess. For which patient condition does the nurse question an order for diuretics? a. Pulmonary edema b. Congestive heart failure c. End-stage renal disease d. Ascites

B

The nurse is teaching the patient about hypokalemia. Which statement by the patient indicates a correct understanding of the treatment of hypokalemia? a. "My wife does all the cooking. She shops for food high in calcium." b. "When I take the liquid potassium in the evening, I'll eat a snack beforehand." c. "I will avoid bananas, orange juice, and salt substitutes." d. "I hate being stuck with needles all the time to monitor how much sugar I can eat."

ABC

The patient with congestive heart failure is receiving a loop diuretic. The nurse monitors for which electrolyte imbalances? (Select all that apply) a. Hypocalcemia b. Hypokalemia c. Hyponatremia d. Hypercalcemia e. Hyperkalemia f. Hypernatremia

A

The patient's serum sodium value is 149 mEq/L. Which of the following nursing interventions is most appropriate for this patient? (Select all that apply) a. Encourage the patient to eat a low-salt diet b. Administer a 0.45% NaCl IV solution c. Hold all doses of glucocorticoids d. Notify the health care provider e. Have patient drink as much water as possible

The client admitted to rule out pancreatic islet tumors complains of feeling weak,shaky, and sweaty. Which should be the first intervention implemented by the nurse? 1. Start an IV with D5W. 2. Notify the health-care provider. 3. Perform a bedside glucose check. 4. Give the client some orange juice.

3

Which laboratory value should be monitored by the nurse for the client diagnosed with diabetes insipidus? 1. Serum sodium. 2. Serum calcium 3. Urine glucose. 4. Urine white blood cells.

1

The client diagnosed with ARDS is in respiratory distress and the ventilator is malfunctioning. Which intervention should the nurse implement first? 1. Notify the respiratory therapist immediately 2. Ventilate with a manual resuscitation bag 3. Request STAT ABGs 4. Auscultate the client's lung sounds

2

Which signs or symptoms should the nurse report immediately because they indicate thrombocytopenia secondary to cancer chemotherapy? Select all that apply. A. Bruises B. Fever C. Petechiae D. Epistaxis E. Pallor

ACD Fever is a sign of infection secondary to neutropenia.Pallor is a sign of anemia.

The client is diagnosed with hypothyroidism. Which signs/symptoms would the nurse expect the client to exhibit? 1. Complaints of extreme fatigue and hair loss. 2. Exophthalmos and complaints of nervousness. 3. Complaints of profuse sweating and flushed skin. 4. Tetany and complaints of stiffness of the hands.

1

Which is a priority nursing diagnosis for a patient receiving desmopressin (DDAVP)? 1. Risk for injury 2. Acute pain 3. Excess fluid volume 4. Deficient knowledge regarding medication

3

n planning care for the newly admitted pt with acute pancreatitis, which pt outcome should receive highest priority? a. pt expresses satisfaction with pain control b. pt verbalizes understanding of medications for home c. pt increases activity intolerance d. pt maintains normal bowl function

A

The patient with CKD is receiving dialysis, and the nurse observes excoriations on the patient's skin. What pathophysiologic changes in CKD most likely occur that can contribute to this finding? Select all that apply a. dry skin b. sensory neuropathy c. vascular calcifications d. calcium-phosphate skin deposits e. uremic crystallization from high BUN

ABD

a nures in the critical care unit is completing an admission assessment of a client who has a gunshot wound to the head. which of the following assessment findings are indicative of increased ICP (select all that apply) A. headache B. dilated pupils C. tachycardia D. decorticate posturing E. hypotension

ABD

A patient with endocarditis has listed in their medical history "Roth Spots". You know that this is a complication of infective endocarditis and presents as? A. Non-tender spots found on the feet and hands B. Red and tender lesions found in the eyes C. Retinal hemorrhages with white centers D. Purplish spots found on the forearms and groin

C

After assisting with a needle biopsy of the liver at a patient's bedside, the nurse should a. put pressure on the biopsy site using a sandbag. b. elevate the head of the bed to facilitate breathing. c. place the patient on the right side with the bed flat. d. check the patient's postbiopsy coagulation studies.

C

Before seeing a newly assigned female client with respiratory alkalosis, the nurse quickly reviews the client's medical history. Which condition is a predisposing factor for respiratory alkalosis? a. Myasthenia gravis b. Type 1 diabetes mellitus c. Extreme anxiety d. Narcotic overdose

C

Children who have profound anemia during induction therapy sohuld a. strictly limit their activities b. regulate their own activity with adult supervision c. receive blood transfusions until the hemoglobin level reaches 10 d. receive chemotherapy until the hemoglobin level reaches 10

C

Client preparing for lumbar puncture. Nurse will assist client into which position for procedure? A.Prone, slight Trendelenburg position B.Prone, pillow under abdomen C.Side-lying with legs pulled up and head bent down onto chest. D.Side-lying with pillow under hip

C

During hemodialysis, a patient complains of nausea and dizziness. Which action should the nurse take first? a. Slow down the rate of dialysis. b. Obtain blood to check the blood urea nitrogen (BUN) level. c. Check the patients blood pressure. d. Give prescribed PRN antiemetic drugs.

C

During hemodialysis, the patient develops light-headedness and nausea. What should the nurse do first? a. Administer hypertonic saline. b. Administer a blood transfusion. c. Decrease the rate of fluid removal. d. Administer antiemetic medications.

C

For a client who is having respiratory symptoms of unknown etiology, the diagnostic test that is most invasive is: A. Pulse oximetry to determine oxygen saturation levels B. Throat cultures with sterile swabs C. Bronchoscopy of the bronchial trees D. Computed tomography of the lung fields

C

In the administration of a drug such as levothyroxine (Synthroid), the nurse should teach the client: A) That therapy typically lasts about 6 months. B) That weekly laboratory tests for T4 levels will be required. C) To report weight loss, anxiety, insomnia, and palpitations. D) That the drug may be taken every other day if diarrhea occurs.

C

In which patient is carbamazepine (Tegretol) contraindicated? A. Patient with new onset of seizures B. Patient with an ulcer C. Patient with chronic hepatitis B D. Patient with diabetes mellitus

C

Maintenance of fluid balance in the patient with ARDS involves A. hydration using colloids. B. administration of surfactant. C. mild fluid restriction and diuretics as necessary. D. keeping the hemoglobin level greater than 12 g/dL (120 g/L).

C

Measures indicated in the conservative therapy of CKD include a. decreased fluid intake, carbohydrate intake, and protein intake. b. increased fluid intake; decreased carbohydrate intake and protein intake. c. decreased fluid intake and protein intake; increased carbohydrate intake. d. decreased fluid intake and carbohydrate intake; increased protein intake.

C

Screening for HIV infection generally involves a. laboratory analysis of blood to detect HIV antigen. b. electrophoretic analysis for HIV antigen in plasma. c. laboratory analysis of blood to detect HIV antibodies. d. analysis of lymph tissues for the presence of HIV RNA.

C

Strategies to prevent ventilator associated pneumonia include: A) rotating the patient's position every 2 hours with HOB at 10 degrees. B) daily oral care with peroxide. C) peptic ulcer disease prophylaxis. D) biweekly assessment of readiness to extubate.

C

The client in end-stage of renal failure had undergone kidney transplant. Which of the following assessment findings indicate kidney transplant rejection? a) increased urinary output, BUN = 15 mg/dL b) HCT = 50%, Hgb = 17 g/dl c) decreased urinary output, sudden weight gain d) decreased urinary output, sudden weight loss

C

The nurse has identified a nursing diagnosis of pain related to inflammatory process for a patient who has acute pericarditis. Which nursing intervention planned by the nurse ismost appropriate for this problem? a. Force fluids to 3000 mL/day to decrease fever and inflammation b. Teach the patient to take shallow, rapid respirations to control the pain c. Position the patient in Fowler's position, leaning forward on a padded overbed table d. Consult with the physician to provide intravenous narcotic analgesics for pain control

C

The nurse is caring for a 22-year-old patient who came to the emergency department with acute respiratory distress. Which information about the patient requires the most rapid action by the nurse? a. Respiratory rate is 32 breaths/min. b. Pattern of breathing is shallow. c. The patient's PaO2 is 45 mm Hg. d. The patient's PaCO2 is 34 mm Hg.

C

The nurse is caring for a client after a bronchoscopy and biopsy. Which finding, if noted in the client, should be reported immediately to the health care provider? a. Dry cough b. Hematuria c. Bronchospasm d. Blood-streaked sputum

C

The nurse is caring for a client who had an episode of​ near-drowning 5 days ago. This​ morning, the nurse noted rhonchi in the lower lung lobes on auscultation. Which action by the nurse is best​? A. Monitoring vital signs and oxygen saturation every 2 hours B. Documenting the findings as normal C. Notifying the healthcare provider D. Preparing for intubation

C

The nurse notes that a patient has incisional pain, a poor cough effort, and scattered rhonchi after a thoracotomy. Which action should the nurse take first? a. Assist the patient to sit upright in a chair. b. Splint the patient's chest during coughing. c. Medicate the patient with prescribed morphine. d. Observe the patient use the incentive spirometer.

C

The nurse obtains the following data when assessing a patient who experienced an ST-segment-elevation myocardial infarction (STEMI) 2 days previously. Which information is most important to report to the health care provider? a. The troponin level is elevated. b. The patient denies ever having a heart attack. c. Bilateral crackles are auscultated in the mid-lower lobes. d. The patient has occasional premature atrial contractions (PACs).

C

The nurse on the clinical unit is assigned to four patients. Which patient should she assess first? a. patient with a skull fracture whose nose is bleeding b. elderly patient with a stroke who is confused and whose daughter is present c. patient with meningitis who is suddenly agitated and reporting a HA of 10 on a 0 to 10 scale d. patient who had a craniotomy for a brain tumor who is now 3 days postoperative and has had continued emesis

C

The nurse would question a prescription for steroids in a patient with which condition? A. Asthma B. Spinal cord injury C. Diabetes mellitus D. Rheumatoid arthritis

C

When a patient who has had progressive chronic kidney disease (CKD) for several years is started on hemodialysis, which information about diet will the nurse include in patient teaching? a. Increased calories are needed because glucose is lost during hemodialysis. b. Unlimited fluids are allowed since retained fluid is removed during dialysis. c. More protein will be allowed because of the removal of urea and creatinine by dialysis. d. Dietary sodium and potassium are unrestricted because these levels are normalized by dialysis.

C

When taking the blood pressure (BP) on the right arm of a patient with severe acute pancreatitis, the nurse notices carpal spasms of the patient's right hand. Which action should the nurse take next? a. Ask the patient about any arm pain. b. Retake the patient's blood pressure. c. Check the calcium level in the chart. d. Notify the health care provider immediately.

C

Which data obtained when assessing a patient who had a kidney transplant 8 years ago and who is receiving the immunosuppressants tacrolimus (Prograf), cyclosporine (Sandimmune), and prednisone (Deltasone) will be of most concern to the nurse? a. The blood glucose is 144 mg/dL. b. The patients blood pressure is 150/92. c. There is a nontender lump in the axilla. d. The patient has a round, moonlike face.

C

Which parameter will be most important for the nurse to consider when titrating the IV fluid infusion rate immediately after a patient has had kidney transplantation? a. Heart rate b. Blood urea nitrogen (BUN) level c. Urine output d. Creatinine clearance

C

A 64-year-old male patient who is receiving radiation to the head and neck as treatment for an invasive malignant tumor complains of mouth sores and pain. Which intervention should the nurse add to this patient's plan of care? A. Weigh the patient every month to monitor for weight loss. B. Cleanse the mouth every 2 to 4 hours with hydrogen peroxide. C. Provide high-protein and high-calorie, soft foods every 2 hours. D. Apply palifermin (Kepivance) liberally to the affected oral mucosa.

C A patient with stomatitis should have soft, nonirritating foods offered frequently. The diet should be high in protein and high in calories. Saline or water should be used to cleanse the mouth (not hydrogen peroxide). Palifermin is administered intravenously as a growth factor to stimulate cells on the surface layer of the mouth to grow. Patients should be weighed at least twice each week to monitor for weight loss.

A client has an appendectomy. This is an example of what kind of surgery? a. Diagnostic b. palliative c. ablative d. constructive

C Appendectomy is an example of ablative surgery. Diagnostic confirms or establishes a diagnosis, palliative relieves or reduces pain, and constructive restores function or appearance.

The nurse is caring for a client with burns. Which question does the nurse ask the client and family to assess their coping strategies? A. "Do you support each other?" B. "How do you plan to manage this situation?" C. "How have you handled similar situations before?" D. "Would you like to see a counselor?"

C Asking how the client and family have handled similar situations in the past assesses whether the client's and the family's coping strategies may be effective. "Yes-or-no" questions such as "Do you support each other?" are not very effective in extrapolating helpful information. The client and family in this situation probably are overwhelmed and may not know how they will manage; asking them how they plan to manage the situation does not assess coping strategies. Asking the client and the family if they would like to see a counselor also does not assess their coping strategies.

A nurse plans care for a client with Cushing's disease. Which action should the nurse include in this client's plan of care to prevent injury? a. Pad the side rails of the client's bed. b. Assist the client to change positions slowly. c. Use a lift sheet to change the client's position. d. Keep suctioning equipment at the clients bedside.

C Cushing's syndrome or disease greatly increases the serum levels of cortisol, which contributes to excessive bone demineralization and increases the risk for pathologic bone fracture. Padding the side rails and assisting the client to change position may be effective, but these measures will not protect her as much as using a lift sheet. The client should not require suctioning.

The patient arrives to the ED and you are told by the reporting nurse that the patient is suspected of having flail chest. Which of the following would the nurse assess for first? A) Palpate the thorax for a crackling, grating sound B) Ask pt. pain level and location C) Monitor respirations D) Assess blood pressure and heart rate

C In order to look for s/s of flail chest, the most important assessment sign to watch for is paradoxical chest movement, which could be found by monitoring respirations. Palpating the thorax could cause further damage to the ribs. It would be very important to assess pain and bp and hr (bleeding) but these will not help confirm the suspected diagnosis.

The nurse is taking care of the patient with a pneumothorax. Which of the following, if found in the patients history, would be most contributory to the development of this pneumothorax? A) MVA involvement approximately 2 weeks ago. B) Hx of diabetes, HTN, and asthma C) Insertion of subclavian line yesterday D) Daily use of albuterol and corticosteroid inhaler

C Insertion of a subclavian line is often associated with traumatic pneumothorax. Some other procedures that may also cause this condition include throracentesis, endotracheal intubation, or transbronchial lung biopsy

A client undergoing long term PD at home is currently experiencing a reduced outflow from the dialysis catheter. To determine if the catheter is obstructed, the nurse should inquire whether the client has a. diarrhea b. vomiting c. flatulence d. constipation

D

A male client has an abnormal result on a Papanicolaou test. After admitting, he read his chart while the nurse was out of the room, the client asks what dysplasia means. Which definition should the nurse provide? a. Presence of completely undifferentiated tumor cells that don't resemble cells of the tissues of their origin b. Increase in the number of normal cells in a normal arrangement in a tissue or an organ c. Replacement of one type of fully differentiated cell by another in tissues where the second type normally isn't found d. Alteration in the size, shape, and organization of differentiated cells

D

A male client is having a tonic-clonic seizures. What should the nurse do first? a. Elevate the head of the bed. b. Restrain the client's arms and legs. c. Place a tongue blade in the client's mouth. d. Take measures to prevent injury

D

A nurse is educating a patient in anticipation of a procedure that will require a water-sealed chest drainage system. What should the nurse tell the patient and the family that this drainage system is used for? A) Maintaining positive chest-wall pressure B) Monitoring pleural fluid osmolarity C) Providing positive intrathoracic pressure D) Removing excess air and fluid

D

A nurse is making a home health visit and finds the client experiencing right lower quadrant abdominal pain, which has decreased in intensity over the last day. The client also has a rigid abdomen and a temperature of 103.6 F. The nurse should intervene by: a) administer Tylenol (acetaminophen) for the elevated temperature b) advising the client to increase oral fluids c) asking the client when she last had a bowel movement d) notifying the physician

D

A nurse is teaching a client who has chronic kidney disease and is to begin hemodialysis. Which of the following information should the nurse include in the teaching? a. hemodialysis restores kidney function b. hemodialysis replaces hormonal function of the renal system c. hemodialysis allows an unrestricted diet d. hemodialysis returns a balance to blood electrolytes

D

A patient has been diagnosed with severe iron deficiency anemia. During physical assessment for which of the following symptoms would the nurse assess to determine the patient's oxygen status? A: Increased breathlessness but increased activity tolerance B: Decreased breathlessness and decreased activity tolerance C: Increased activity tolerance and decreased breathlessness D: Decreased activity tolerance and increased breathlessness

D

A patient with a 25-year history of type 1 diabetes mellitus is reporting fatigue, edema, and an irregular heartbeat. On assessment, the nurse notes newly developed hypertension and uncontrolled blood sugars. Which diagnostic study is most indicative of chronic kidney disease (CKD)? a. Serum creatinine b. Serum potassium c. Microalbuminuria d. Calculated glomerular filtration rate (GFR)

D

A patient with hyperthyroidism is taking propylthiouracil (PTU). The nurse will monitor the patient for: A) gingival hyperplasia and lycopenemia. B) dyspnea and a dry cough. C) blurred vision and nystagmus. D) fever and sore throat.

D

Which action by a 70-year-old patient who is using peritoneal dialysis (PD) indicates that the nurse should provide more teaching about PD? a. The patient leaves the catheter exit site without a dressing. b. The patient plans 30 to 60 minutes for a dialysate exchange. c. The patient cleans the catheter while taking a bath each day. d. The patient slows the inflow rate when experiencing abdominal pain.

C Patients are encouraged to take showers rather than baths to avoid infections at the catheter insertion side. The other patient actions indicate good understanding of peritoneal dialysis

The nurse caring for a patient with an endotracheal tube recognizes several disadvantages of an endotracheal tube. What would the nurse recognize as a disadvantage of endotracheal tubes? A) Cognition is decreased. B) Daily arterial blood gases (ABGs) are necessary. C) Slight tracheal bleeding is anticipated. D) The cough reflex is depressed.

D

The nurse is caring for a patient who is receiving desmopressin acetate (DDAVP). Which assessments are important while caring for this patient? a. Blood pressure and serum potassium b. Heart rate and serum calcium c. Lung sounds and serum magnesium d. Urine output and serum sodium

D

The nurse is caring for an adult who is diagnosed with acute respiratory distress syndrome​ (ARDS) after a​ near-drowning episode last week. Which type of medication should the nurse anticipate the provider ordering to help open​ alveoli? A. Corticosteroids B. Nonsteroidal​ anti-inflammatory drugs​ (NSAIDs) C. Inhaled nitric oxide D. Surfactant therapy

D

The nurse is reviewing the results of the patient's diagnostic testing. Of the following results, the finding that falls within expected or normal limits is: A. Palpable, elevated hardened area around a tuberculosis skin testing site. B. Sputum for culture and sensitivity identifies mycobacterium tuberculosis C. Presence of acid fast bacilli in sputum D. Arterial oxygen tension (PaO2) of 95 mmHg

D

The nurse presents a cancer prevention program to teens. Which of the following will have the greatest impact in cancer prevention? A. Avoid asbestos. B. Wear sunscreen. C. Get the human papilloma virus (HPV) vaccine. D. Do not smoke cigarettes.

D

To assess the patency of a newly placed arteriovenous graft for dialysis, the nurse should Select all that apply a. monitor the BP in the affected arm b. irrigate the graft daily with low-dose heparin c. palpate the area of the graft to feel a normal thrill d. listen with a stethoscope over the graft to detect a bruit e. frequently monitor the pulses and neurovascular status distal to the graft

CDE

What immunization should not be given to a child receiving chemotherapy for cancer? a. Tetanus vaccine b. Inactivated poliovirus vaccine c. Diphtheria, pertussis, tetanus (DPT) d. Measles, mumps, rubella (MMR)

D

What is the best patient to assign to a new graduate nurse on her first week of orientation? A. Patient with bacterial meningitis admitted from the emergency department today B. Patient returning from a craniotomy for a pituitary brain tumor C. Patient with head trauma with suspected epidural bleed admitted 3 hours earlier D. Patient with viral meningitis who is being discharged today

D

What is the classic chest x-ray finding in a patient with late-stage ARDS? A. Hyperinflation B. Infiltrates in the bases C. Deflated lung on one side D. White lung

D

Which statement by the patient indicates an understanding of discharge instructions given by the nurse about the newly prescribed medication levothyroxine (Synthroid)? A. "I will take a double dose to make up for the missed one." B. "I can expect improvement of my symptoms within 1 week." C. "I will stop the medication immediately if I feel pain or weakness in my muscles." D. "I will take this medication in the morning so it does not affect my sleep at night."

D

A patient who takes warfarin (Coumadin) and digoxin (Lanoxin) develops hypothyroidism and will begin taking levothyroxine (Synthroid). The nurse anticipates which potential adjustments in dosing for this patient? a. Decreased digoxin and decreased warfarin b. Decreased digoxin and increased warfarin c. Increased digoxin and decreased warfarin d. Increased digoxin and increased warfarin

C Thyroid preparations increase the effect of oral anticoagulants, so the warfarin dose may need to be decreased. Levothyroxine can decrease the effectiveness of digoxin, so this dose may need to be increased.

The nurse is preparing for removal of an endotracheal (ET) tube from a client. In assisting the health care provider in this procedure, which is the initial nursing action? 1.Deflate the cuff. 2.Suction the ET tube. 3.Turn off the ventilator. 4.Obtain a code cart, and place it at the bedside.

2

The client diagnosed with ARF has a serum potassium level of 6.8 mEq/L. Which collaborative treatment should the nurse anticipate for the client? 1.Administer a phosphate binder. 2.Type and crossmatch for whole blood. 3.Assess the client for leg cramps. 4.Prepare the client for dialysis.

4 Normal potassium level is 3.5 to5.5 mEq/L. A level of 6.8 mEq/L is life threatening and could lead to cardiac dysrhythmias. Therefore, the client may be dialyzed to decrease the potassium level quickly. This requires a health-care provider order, so it is a collaborative intervention.

A patient is admitted to the hospital with CKD. The nurse understands that this condition is characterized by a. progressive irreversible destruction of the kidneys b. a rapid decrease in urine output with an elevated BUN c. an increasing creatinine clearance with a decrease in urine output d. prostration, somnolence, and confusion with coma and imminent death

A

Secretes three hormones essential for proper regulation of metabolism: thyroxine (T₄), triiodothyronine (T₃), calcitonin A. Thyroid Gland B. Parathyroid gland

A

The client has end-stage renal disease. He had undergone kidney transplant 5 days ago. Which of the following is the most important intervention for the client to prevent infection? a) observe asepsis b) increase fluid intake c) avoid clients with flu d) avoid crowded places

A

The nurse is caring for a 65-yr-old man with acute respiratory distress syndrome (ARDS) who is on pressure support ventilation (PSV), fraction of inspired oxygen (FIO2) at 80%, and positive end-expiratory pressure (PEEP) at 15 cm H2O. The patient weighs 72 kg. What finding would indicate that treatment is effective? a. PaO2 of 60 mm Hg b. Tidal volume of 700 mL c. Cardiac output of 2.7 L/min d. Inspiration to expiration ratio of 1:2

A

The nurse witnesses a patient with a seizure disorder as the patient suddenly jerks the arms and legs, falls to the floor, and regains consciousness immediately. It will be most important for the nurse to a. assess the patient for a possible head injury. b. give the scheduled dose of divalproex (Depakote). c. document the timing and description of the seizure. d. notify the patient's health care provider about the seizure.

A

The patient is diagnosed with a brain tumor. Which option is the correct understanding of the preferred treatment? A. Surgical removal is preferred, even if the tumor is not malignant. B. Chemotherapy is a common and effective treatment. C. Stereotactic radiosurgery is the preferred treatment. D. A large dose of intravenous steroid therapy is preferred.

A

The nurse is teaching the 47-year-old female client about recommended screening practices for breast cancer. Which statement by the client indicates understanding of the nurse's instructions? A. "My mother and grandmother had breast cancer, so I am at risk." B. "I get a mammography every 2 years since I turned 30." C. "A clinical breast examination is performed every month since I turned 40." D. "A CT scan will be done every year after I turn 50."

A A strong family history of breast cancer indicates a risk for breast cancer. Annual screening may be indicated for a strong family history. The client may perform a self-breast examination monthly; a clinical examination by a health care provider is indicated annually. An annual mammography is performed after age 40 or in younger clients with a strong family history.

After administering somatropin (Genotropin) to a patient, the nurse would assess for potential adverse effects of this medication by monitoring which laboratory test result? A. Glucose B. Platelets C. Potassium D. Magnesium

A Hyperglycemia and hypoglycemia are potential adverse effects of somatropin therapy

The nurse is educating a client who recently had a kidney transplant about the dietary changes that will be necessary. Which of the following statements could the nurse make to the client? Select all that apply. a. additional calcium may be needed b. carbohydrates may be restricted c. extra protein may be needed d. fats may be limited e. protein may be restricted f. sodium may be restricted

ABCDEF

A nurse is assessing a client who has end-stage kidney disease. Which of the following findings should the nurse expect? Select all that apply a. anuria b. marked azotemia c. crackles in the lungs d. increased calcium level e. proteinuria

ABCE

A patient is receiving mechanical ventilation with PEEP. The patient had developed a tension pneumothorax. Select ALL the signs and symptoms that can present with this condition: A. Hypotension B. Jugular Venous Distention C. Bradycardia D. Tracheal deviation E. Hyperemia F. Tachypnea

ABDF

The nurse is reviewing the adverse effects of antithyroid medications for a patient prescribed propylthiouracil (PTU). What potential serious adverse effects should the nurse discuss with the patient during discharge teaching? (Select all that apply.) A. Joint pain B. Liver toxicity C. Kidney damage D. Increased urination E. Bone marrow toxicity

ABE

A patient is on mechanical ventilation with PEEP (positive end-expiratory pressure). Which finding below indicates the patient is developing a complication related to their therapy and requires immediate treatment? A. HCO3 26 mmHg B. Blood pressure 70/45 C. PaO2 80 mmHg D. PaCO2 38 mmHg

B

The nurse is admitting a 67-year-old patient with new-onset steatorrhea. Which question is most important for the nurse to ask? a. "How much milk do you usually drink?" b. "Have you noticed a recent weight loss?" c. "What time of day do your bowels move?" d. "Do you eat meat or other animal products?"

B Although all of the questions provide useful information, it is most important to determine if the patient has an imbalance in nutrition because of the steatorrhea.

A nurse is reviewing the plan of care for a client who is receiving mechanical ventilation. Which of the following ventilator modes will increase the client's work of breathing? Select all that apply a. assist-control b. synchronized intermittent mandatory ventilation c. continuous positive airway pressure d. pressure support ventilation e. independent lung ventilation

BCD

The nurse teaches a client with new-onset atrial fibrillation that risk factors for this dysrhythmia may include which? Select all that apply. a. Use of beta-adrenergic blockers b. Excessive alcohol use c. Advancing age d. High blood pressure e. Palpitations

BCD

Which statement about metabolic side effects of ART is true (select all that apply)? a. These are annoying symptoms that are ultimately harmless. b. ART-related body changes include central fat accumulation and peripheral wasting. c. Lipid abnormalities include increases in triglycerides and decreases in high-density cholesterol. d. Insulin resistance and hyperlipidemia can be treated with drugs to control glucose and cholesterol. e. Compared to uninfected people, insulin resistance and hyperlipidemia are more difficult

BCD Some HIV-infected patients, especially those who have been infected and have received ART for a long time, develop a set of metabolic disorders that include changes in body shape (e.g., fat deposits in the abdomen, upper back, and breasts along with fat loss in the arms, legs, and face) as a result of lipodystrophy, hyperlipidemia (i.e., elevated triglyceride levels and decreases in high-density lipoprotein levels), insulin resistance and hyperglycemia, bone disease (e.g., osteoporosis, osteopenia, avascular necrosis), lactic acidosis, and cardiovascular disease.

A client with chronic renal failure who receives hemodialysis 3 times a week is experiencing severe nausea. What should the nurse advise the client to do to manage the nausea? Select all that apply a. drink fluids before eating solid foods b. have limited amounts of fluids only when thirsty c. limit activity d. keep all dialysis appointments e. eat smaller, more frequent meals

BDE

A nurse is securing a patient's endotracheal tube with tape and observes that the tube depth changed during the retaping. Which action would be appropriate related to this incident? a) Instruct assistant to notify the primary care provider. b) Assess the patient's vital signs. c) Remove the tape, adjust the depth to ordered depth and reapply the tape. d) No action is required as depth will adjust automatically.

C

A patient has Cushing's syndrome. The nurse expects which drug to be used to inhibit the function of the adrenal cortex in the treatment of this syndrome? A. Fludrocortisone (Florinef) B. Dexamethasone (generic) C. Aminoglutethimide (Cytadren) D. Hydrocortisone (Solu-Cortef)

C

A patient returns from the OR after having a tracheostomy. While assessing the patient, which observations made by the nurse warrant immediate notification of the provider? A. patient is alert but unable to speak and has difficulty communicating his needs B. small amount of bleeding present at incision C. skin is puffy at the neck area with a crackling sensation D. respirations are audible and noisy with increased RR

C

A 72-year-old client recovering from lung cancer surgery asks the nurse to explain how she developed cancer when she has never smoked. Which factor may explain the possible cause? A. A diagnosis of diabetes treated with insulin and diet B. An exercise regimen of jogging 3 miles 4x/wk C. A history of cardiac disease D. Advancing age

D Advancing age is the single most important risk factor for cancer. As a person ages, immune protection decreases.

A thoracentesis is performed on a chest-injured client, and no fluid or air is found. Blood and fluids is administered intravenously (IV), but the client's vital signs do not improve. A central venous pressure line is inserted, and the initial reading is 20 cm H^O. The most likely cause of these findings is which of the following? A. Spontaneous pneumothorax B. Ruptured diaphragm C. Hemothorax D. Pericardial tamponade

D The reading of CVP of 20 means that there increased venous pressure backing up because the heart is not pumping effective. This would indicate the presence of cardiac tamponade.

A

Which changes on the patient's electrocardiogram (ECG) reflect hyperkalemia? a. Tall peaked T waves b. Narrow QRS complex c. Tall P waves d. Normal P-R interval

A patient on a medical unit has a potassium level of 6.8 mEq/L. What is the priority action thatthe nurse should take? a. Place the patient on a cardiac monitor. b. Check the patient's blood pressure (BP). c. Instruct the patient to avoid high-potassium foods. d. Call the lab and request a redraw of the lab to verify results.

A Dysrhythmias may occur with an elevated potassium level and are potentially lethal. Monitor the rhythm while contacting the physician or calling the rapid response team. Vital signs should be checked. Depending on the patient's history and cause of increased potassium, instruct the patient about dietary sources of potassium; however, this would not help at this point. The nurse may want to recheck the value but until then the heart rhythm needs to be monitored.

The nurse will anticipate preparing a 71-year-old female patient who is vomiting "coffee-ground" emesis for a. endoscopy. b. angiography. c. barium studies. d. gastric analysis.

A Endoscopy is the primary tool for visualization and diagnosis of upper gastrointestinal (GI) bleeding. Angiography is used only when endoscopy cannot be done because it is more invasive and has more possible complications. Barium studies are helpful in determining the presence of gastric lesions, but not whether the lesions are actively bleeding. Gastric analysis testing may help with determining the cause of gastric irritation, but it is not used for acute GI bleeding.

A nurse is collecting data regarding a client after a thyroidectomy and notes that the client has developed hoarseness and a weak voice. Which nursing action is appropriate? 1. Check for signs of bleeding. 2. Administer calcium gluconate. 3. Notify the registered nurse immediately. 4. Reassure the client that this is usually a temporary condition.

4 Weakness and hoarseness of the voice can occur as a result of trauma of the laryngeal nerve. If this develops, the client should be reassured that the problem will subside in a few days. Unnecessary talking should be discouraged. It is not necessary to notify the registered nurse immediately. These signs do not indicate bleeding or the need to administer calcium gluconate.

The nurse is assisting a health care provider with the removal of a chest tube. The nurse should instruct the client to take which action? 1.Exhale slowly. 2.Stay very still. 3.Inhale and exhale quickly. 4.Perform the Valsalva maneuver.

4 When the chest tube is removed, the client is asked to perform the Valsalva maneuver (take a deep breath, exhale, and bear down). The tube is quickly withdrawn, and an airtight dressing is taped in place. An alternative instruction is to ask the client to take a deep breath and hold the breath while the tube is removed.

The family of a patient with third-degree burns wants to know why the "scabs are being cut off" of the patient's leg. What is the most appropriate response by the nurse to this family? 1. "The scabs are really old burned tissue and need to be removed to promote healing." 2. "I'll ask the doctor to come and talk with you about the treatment plan." 3. "The patient asked for the scabs to be removed." 4. "The scabs are removed to check for blood flow to the burned area."

1 Rationale: The patient's family is describing eschar, which is the hard crust of burned necrotic tissue. Eschar needs to be removed to promote wound healing. Option 2 does not answer the family's question. Option 3 incorrectly restates the family's concern. Scabs are not removed to check for blood flow.

The nurse is caring for a client diagnosed with ARDS. Which interventions should the nurse implement? Select All that Apply 1. Assess the client's level of consciousness 2. Monitor urine output every shift 3. Turn the client every 2 hours 4. Maintain intravenous fluids as ordered 5. Place the client in the Fowler's position

1345

A 60-year-old female is diagnosed with hypothyroidism. The nurse should assess the client for: 1. tachycardia. 2. weight gain. 3. diarrhea. 4. nausea.

2

During the acute stage of meningitis, a 3-year-old child is restless and irritable. Which of the following would be most appropriate to institute? 1. Limiting conversation with the child 2. Keeping extraneous noise to a minimum 3. Allowing the child to play in the bathtub 4. Performing treatments quickly

2

A client with hypothyroidism (myxedema) is receiving levothyroxine (Synthroid), 25 mcg P.O. daily. Which finding should the nurse recognize as an adverse reaction to the drug? 1. Dysuria 2. Leg cramps 3. Tachycardia 4. Blurred vision

3 Levothyroxine, a synthetic thyroid hormone, is given to a client with hypothyroidism to simulate the effects of thyroxine. Adverse reactions to this agent include tachycardia. The other options aren't associated with levothyroxine.

The nurse has completed client teaching with the hemodialysis client about self-monitoring between hemodialysis treatments. The nurse determines that the client best understands the information if the client states to record daily the: a. Amount of activity. b. Pulse and respiratory rate. c. Intake and output and weight. d. Blood urea nitrogen and creatinine levels.

A If the client experiences air embolus during hemodialysis, the nurse should terminate dialysis immediately, notify the physician, and administer oxygen as needed.

Client with diagnosis of SIADH. The nurse would expect to see what laboratory finding? A.Serum sodium 125. B.Serum potassium 2.7 C.Serum glucose 250 D.Serum chloride 110

A

which diagnostic study best differentiates the various types of CMP? a. echocardiography b. arterial blood gases c. cardiac cath d. endomyocaridal biopsy

A

D

A nurse caring for a client who has been receiving IV diuretics suspects that the client is experiencing a fluid volume deficit. Which assessment finding would the nurse note in a client with this condition? a. Lung congestion b. Decreased hematocrit c. Increased blood pressure d. Decreased central venous pressure (CVP)

What is included in the description of positive pressure ventilation? Select all that apply a. peak inspiratory pressure predetermined b. consistent volume delivered with each breath c. increased risk for hyperventilation and hypoventilation d. preset volume of gas delivered with variable pressure based on compliance e. volume delivered varies based on selected pressure and patient lung compliance

ACE

Of the following assessment findings, the one that would most likely be seen in a child with leukemia is a. weakness of the eye muscle b. bruising, nosebleeds, pallor, and fatigue c. wheezing and shortness of breath d. abdominal swelling

B

The nurse would recognize which of the following as a potential side effect of the glucocorticoid therapy in young children? A. Arthritis B. Growth suppression C. Constipation D. Iron-deficiency anemia

B

What ventilation setting delivers preset tidal volume whenever the patient exerts inspiration, and still ensures that the patient recieves a breath if they do not spontaneously trigger the ventilator? A. Controlled Mandatory Ventilation B. Assisted Control C. Synchronized Intermittent Mandatory Ventilation D. Continuous Positive Airway Pressure

B

the nurse has completed discharge instructions for a client with application of a halo device. which action indicates that the client needs further clarification of the instructions? A. uses a draw for drinking B. drives only during the daytime C. uses caution because the device alters balance D. washing the skin daily under the lambs wool liner of the vest

B

When evaluating a male client for complications of acute pancreatitis, the nurse would observe for: a. increased intracranial pressure. b. decreased urine output. c. bradycardia. d. hypertension."

B Acute pancreatitis can cause decreased urine output, which results from the renal failure that sometimes accompanies this condition. Intracranial pressure neither increases nor decreases in a client with pancreatitis. Tachycardia, not bradycardia, usually is associated with pulmonary or hypovolemic complications of pancreatitis. Hypotension can be caused by a hypovolemic complication, but hypertension usually isn't related to acute pancreatitis."

The nurse would question a prescription for somatropin (Genotropin) in a patient with which condition? A. Dwarfism B. Acromegaly C. Growth failure D. Hypopituitarism

B Somatropin is a synthetic form of human growth hormone. Acromegaly is caused by excessive growth hormone, and thus this drug would be contraindicated

Which nursing action could the registered nurse (RN) working in a skilled care hospital unit delegate to an experienced licensed practical/vocational nurse (LPN/LVN) caring for a patient with a permanent tracheostomy? a. Assess the patient's risk for aspiration. b. Suction the tracheostomy when needed. c. Teach the patient about self-care of the tracheostomy. d. Determine the need for replacement of the tracheostomy tube.

B Suctioning of a stable patient can be delegated to LPNs/LVNs. Patient assessment and patient teaching should be done by the RN.

The nurse is assessing a patient 4 hours after a kidney transplant. Which information is most important to communicate to the health care provider? a. The urine output is 900 to 1100 mL/hr. b. The patient's central venous pressure (CVP) is decreased. c. The patient has a level 7 (0 to 10 point scale) incisional pain. d. The blood urea nitrogen (BUN) and creatinine levels are elevated.

B The decrease in CVP suggests hypovolemia, which must be rapidly corrected to prevent renal hypoperfusion and acute tubular necrosis. The other information is not unusual in a patient after a transplant

The nurse anticipates administering which medication to treat hyperuricemia associated with tumor lysis syndrome (TLS)? A.Recombinant erythropoietin (Procrit) B. Allopurinol (Zyloprim) C. Potassium chloride D. Radioactive iodine 131

B Tumor lysis syndrome results in hyperuricemia, Allopurinol decreases uric acid production and is indicated in TLS.

A client has been admitted to the hospital for urinary tract infection and dehydration. The nurse determines that the client has received adequate volume replacement if the blood urea nitrogen level drops to: a) 3 mg/dL b) 15 mg/dL c) 29 mg/dL d) 35 mg/dL

B the normal blood urea nitrogen level is 8 to 25 mg/dL

A client requires oxygen received via a face mask but wants to remain as mobile as possible once discharged home. Which intervention by the home health nurse best provides the client with maximal mobility? a. Arrange a consultation with pulmonary rehabilitation to decrease oxygen needs. b. Encourage the client to remove the mask occasionally to assess tolerance. c. Add extra connecting pieces of tubing to the client's existing oxygen setup. d. Change the face mask to a nasal cannula occasionally, such as at mealtimes.

C

The nurse has a prescription for a patient to receive prednisone (Deltasone) to treat contact dermatitis. The nurse would question this prescription for this patient with what condition? A. Asthma B. Multiple sclerosis C. Acquired immune deficiency syndrome (AIDS) D. Chronic obstructive pulmonary disease

C

The nurse is instructing the client with chronic renal failure to maintain adequate nutritional intake. Which diet would be most appropriate? a. high-carbohydrate, high-protein b. high-calcium, high-potassium, high-protein c. low-protein, low-sodium, low-potassium d. low-protein, high-potassium

C

Which activity should the nurse encourage the patient to avoid when there is a risk for intracranial pressure(ICP)? A. deep breathing B. turning C. coughing D. passive range-of-motion exercises

C

Which complementary health approach may be specifically tailored to assist in the identification of the warning signs of​ seizures? A.Behavior modification B.Massage C.Biofeedback D.Meditation

C

You have just finished assisting the physician in removing Mr. H's chest tube. While securing the dressing, you notice that Mr. H is having difficulty breathing. Upon examination, you note that his trachea has shifted to the left side of his throat. You suspect that Mr. H has developed: A) early acute respiratory failure. B) aspiration of secretions. C) a pneumothorax. D) a normal finding after chest tube removal.

C

Your recent admission has acute cholecystitis. The patient is awaiting a cholecystostomy. What signs and symptoms are associated with this condition? Select all that apply: A. Right lower quadrant pain with rebound tenderness B. Negative Murphy's Sign C. Epigastric pain that radiates to the right scapula D. Pain and fullness that increases after a greasy or spicy meal E. Fever F. Tachycardia G. Nausea

CDEFG

A client is brought into the emergency department after aspirating on pureed foods at the​ long-term care facility. The nurse knows that which physiologic change can trigger acute respiratory distress​ syndrome? A. Intracellular edema B. Increased surfactant production C. Destruction of extracellular platelets D. Release of chemical mediators

D

A patient complains of leg cramps during hemodialysis. The nurse should first a. reposition the patient. b. massage the patients legs. c. give acetaminophen (Tylenol). d. infuse a bolus of normal saline.

D

Which of the following nursing diagnoses is most appropriate for a patient experiencing myelosuppression secondary to chemotherapy for cancer treatment? A. Acute pain B. Hypothermia C. Powerlessness D. Risk for infection

D

a nurse is caring for a client who has a C4 spinal cord injury. the nurse should recognize the client is at greatest risk for which of the following complications? A. neurogenic shock B. paralytic ileus C. stress ulcer D. respiratory compromise

D

The nurse should monitor for increases in which laboratory value in a patient being treated with dexamethasone? a. Sodium b. Calcium c. Potassium d. Blood glucose

D Hyperglycemia, or increased blood glucose level, is an adverse effect of corticosteroid therapy. Sodium, calcium, and potassium levels are not affected directly by dexamethasone.

Which assessment finding would the nurse need to report most quickly to the health care provider regarding a patient with acute pancreatitis? a. Nausea and vomiting b. Hypotonic bowel sounds c. Abdominal tenderness and guarding d. Muscle twitching and finger numbness

D Muscle twitching and finger numbness indicate hypocalcemia,

The nurse is giving a group presentation on cancer prevention and recognition. Which statement by an older adult client indicates understanding of the nurse's instructions? A. "Cigarette smoking always causes lung cancer." B. "Taking multivitamins will prevent me from developing cancer." C. "If I have only one shot of whiskey a day, I probably will not develop cancer." D. "I need to report the pain going down my legs to my health care provider."

D Pain in the back of the legs could indicate prostate cancer in an older man.

B

Hyperkalemia can cause severe problems in which body system? a. Neuromuscular b. Cardiovascular c. Intestinal d. Respiratory

ABDFH

The nurse is assessing the patient at risk for fluid volume excess. Which findings indicate that the patient has fluid volume excess? (Select all that apply) a. Increased, bounding pulse b. Jugular venous distention c. Diminished peripheral pulses d. Presence of crackles e. Excessive thirst f. Elevated blood pressure g. Orthostatic hypotension h. Skin pale and cool to touch

B

The nurse is assessing the patient's urine specific gravity. The value is 1.035. How does the nurse interpret this result? a. Overhydration b. Dehydration c. Normal value for an adult d. Renal disease

D

The nurse is teaching the patient about foods high in potassium. Which food item does the nurse use as the best example? a. Bread b. Eggs c. Cereal grains d. Meat

C

Trauma patient with crushed extremities puts them at risk for? A. Hypokalemia B. Hyponatremia C. Hyperkalemia D. Hypernatremia

The hospital administrator had undergone percutaneous transhepatic cholangiography. which assessment finding indicates complication after the operation? A. Fever and chills B. Hypertension C. Bradycardia D. Nausea and diarrhea

A Septicemia is a common complication after a percutaneous transhepatic cholangiography. Evidence of fever and chills, possibly indicative of septicemia, is important. Hypotension, not hypertension, is associated with septicemia. Tachycardia, not bradycardia, is most likely to occur. Nausea and diarrhea may occur but are not classic signs of sepsis

D

the nurse is caring for several patients at risk for fluid and electrolyte imbalances. Which patient problem or condition can result in a relative hypernatremia? a. Use of salt substitute b. Diarrhea c. Drinking too much water d. NPO status for a prolonged period

The peak pressure alarm is sounding on the ventilator of the client with a recent tracheostomy. What intervention should be done first? A. Assess the client's respiratory status B. Decrease the sensitivity of the alarm C. Ensure that the connecting tubing is not kinked D. Suction the client

A The client must always be assessed before attention is turned to equipment.

A 50-year-old man has been taking prednisone (Deltasone) as part of treatment for bronchitis. He notices that the dosage of the medication decreases. During a follow-up office visit, he asks the nurse why he must continue the medication and why he cannot just stop taking it now that the feels better. What is the rationale behind tapering the doses? A. Sudden discontinuation of the medication may result in adrenal insufficiency. B. The patient would experience withdrawal symptoms if the drug were discontinued abruptly. C. Cushing's syndrome may develop as a reaction to a sudden drop in serum cortisone levels. D. When the symptoms have started to disappear, lower dosages are needed.

A

A 78-yr-old patient has stage 3 CKD and is being taught about a low-potassium diet. The nurse knows the patient understands the diet when the patient selects which foods to eat? a. Apple, green beans, and a roast beef sandwich b. Granola made with dried fruits, nuts, and seeds c. Watermelon and ice cream with chocolate sauce d. Bran cereal with ½ banana and milk and orange juice

A

A client has been admitted with chest trauma after a motor vehicle crash and has undergone intubation. The nurse checks on the client when the high pressure alarm sounds, and notes that the client has absence of breath sounds in the right upper lobe of the lung. The nurse immediately assesses for other signs of which condition? a. right pneumothorax b. pulmonary embolism c. displaced endotracheal tube d. acute respiratory distress syndrome

A

A client has been newly diagnosed with hypothyroidism and will take levothyroxine (Synthroid) 50 mcg/day by mouth. As part of the teaching plan, the nurse emphasizes that this medication: A) Should be taken in the morning B) May decrease the client's energy level C) Must be stored in a dark container D) Will decrease the client's heart rate

A

A client is receiving continuous ambulatory peritoneal dialysis (CAPD). The nurse should assess the client for which sign of peritoneal infection? a. cloudy dialysate fluid b. swelling in the legs c. poor drainage of the dialysate fluid d. redness at the catheter insertion site

A

A client is receiving parenteral nutrition (PN) suddenly is having a fever. A nurse notifies the physician and the physician initially prescribes that the solution and tubing be changed. The nurse should do which of the following with the discontinued materials? A. Send them to the laboratory for culture. B. Save them for a return to the manufacturer. C. Return them to the hospital pharmacy. D. Discard them in the unit trash.

A

A nurse collaborates with an unlicensed assistive personnel (UAP) to provide care for a client who is prescribed a 24-hour urine specimen collection. Which statement should the nurse include when delegating this activity to the UAP? a. Note the time of the client's first void and collect urine for 24 hours. b. Add the preservative to the container at the end of the test. c. Start the collection by saving the first urine of the morning. d. It is okay if one urine sample during the 24 hours is not collected.

A

A nurse walks in to a client who is in respiratory distress. The client has tracheal deviation to the right side. The nurse knows to prepare for which of the following emergent procedures? a. Chest tube insertion on the left side. b. Chest tube insertion on the right side. c. Intubation d. Tracheostomy

A

A patient in acute respiratory failure is receiving ACV with a positive end-expiratory pressure (PEEP) of 10 cm H20. What sign alerts the nurse to undesirable effects of increased airway and thoracic pressure? a. decreased BP b. decreased PaO2 c. increased crackles d. decreased spontaneous respirations

A

A patient who has vague symptoms of fatigue, headaches, and a positive test for human immunodeficiency virus (HIV) antibodies using an enzyme immunoassay (EIA) test. What instructions should the nurse give to this patient? a. "The EIA test will need to be repeated to verify the results." b. "A viral culture will be done to determine the progression of the disease." c. "It will probably be 10 or more years before you develop acquired immunodeficiency syndrome (AIDS)." d. "The Western blot test will be done to determine whether acquired immunodeficiency syndrome (AIDS) has developed."

A

A patient who is being admitted to the emergency department with intermittent chest pain gives the following list of medications to the nurse. Which medication has the most immediate implications for the patient's care? a. Sildenafil (Viagra) b. Furosemide (Lasix) c. Captopril (Capoten) d. Warfarin (Coumadin)

A

A patient with a severe acute asthma exacerbation presents to the emergency department. Over the next hour, the patient remains in respiratory distress, but the respirations have slowed. What is the best explanation? A. The patient is developing respiratory muscle fatigue. B. The respirations are exchanging oxygen and carbon dioxide more efficiently. C. The patient's anxiety level is lessening. D. The body has compensated by retaining sodium bicarbonate.

A

A patient with a suspected closed head injury has bloody nasal drainage. You suspect that this patient has a cerebrospinal fluid (CSF) leak when observing which of the following? A. A halo sign on the nasal drip pad B. Decreased blood pressure and urinary output C. A positive reading for glucose on a Test-tape strip D. Clear nasal drainage along with the bloody discharge

A

In what ventilation setting does the vent deliver tidal volume at a preset rate because the patient is not breathing spontaneously? A. Controlled Mandatory Ventilation B. Assisted Control C. Synchronized Intermittent Mandatory Ventilation D. Continuous Positive Airway Pressure

A

Nurse Lucia is providing breast cancer education at a community facility. The American Cancer Society recommends that women get mammograms: a. yearly after age 40. b. after the birth of the first child and every 2 years thereafter. c. after the first menstrual period and annually thereafter. d. every 3 years between ages 20 and 40 and annually thereafter.

A

Synthetic thyroid hormone T₄ (most common): A. Levothyroxine B. Liothyronine C. Liotrix D. Thyroid

A

The client has undergone transurethral resection of the prostate (TURP). Which intervention will the nurse incorporate in this client's postoperative care? A. Administer antispasmodic medications. B. Encourage the client to urinate around the catheter if pressure is felt. C. Perform intermittent urinary catheterization every 4 to 6 hours. D. Place the client in a supine position, with his knees flexed.

A

The client is receiving chemotherapy treatment for breast cancer and asks for additional support for managing the associated nausea and vomiting. Which complementary therapy will the nurse suggest? A. Ginger B. Journaling C. Meditation D. Yoga

A

What would the nurse assess when monitoring for the therapeutic effectiveness of vasopressin? 1. Fluid balance 2. Patient's pain scale 3. serum albumin levels 4. Adrenocorticotropic hormone (ACTH) levels

1

The nurse is administering a dose of IV vincristine when the child suddenly starts complaining of difficulty breathing and is markedly flushed. The nurse's highest priority is to a. administer an oral histamine b. stop the infusion immediately c. slow down the infusion rate d. call the practitioner

A

A patient is admitted to the emergency department with deep partial-thickness burns over 35 % of the body. What IV solution will be started initially? 1. warmed lactated Ringer's solution 2. dextrose 5% with saline solution 3. dextrose 5% with water 4. normal saline solution 5. 0.45% saline solution

1 Rationale: Warmed lactated Ringer's solution is the IV solution of choice because it most closely approximates the body's extracellular fluid composition. It is warmed to prevent hypothermia.

a nurse is doing preop teaching for a client who will have a cholecystectomy. the nurse teaches that special care must be taken to prevent what postop complication? 1. hypostatic pneumonia 2. thrombosis 3. hemorrhage 4. paralytic ileus

1 the incision is located below the ribcage; therefore the coughing and deep breathing necessary to prevent pneumonia are especially difficult post op

Which signs/symptoms should make the nurse suspect the client is experiencing a thyroid storm? 1. Obstipation and hypoactive bowel sounds. 2. Hyperpyrexia and extreme tachycardia. 3. Hypotension and bradycardia. 4. Decreased respirations and hypoxia.

2 Hyperpyrexia (high fever) and heart rate above 130 beats per minute are signs of thyroid storm, a severely exaggerated hyperthyroidism.

A 34-year-old female is diagnosed with hypothyroidism. What should the nurse assess the client for? Select all that apply. 1. rapid pulse 2. decreased energy and fatigue 3. weight gain of 10 lb (4.5 kg) 4. fine, thin hair with hair loss 5. constipation 6. menorrhagia

2356

C

A nurse caring for a client with hypocalcemia would expect to note which of the following changes on the electrocardiogram? a. Widened T wave b. Prominent U wave c. Prolonged QT interval d. Shortened ST segment

D

A nurse caring for a group of clients reviews the electrolyte laboratory results and notes a potassium level of 5.5 mEq/L on one client's laboratory report. The nurse understands that which client is at highest risk for the development of a potassium value at this level? a. The client with colitis b. The client with Cushing's syndrome c. The client who has been overusing laxatives d. The client who has sustained a traumatic burn

B

A nurse is assigned to care for a group of clients. On review of the clients' medical records, the nurse determines that which client is at risk for fluid volume excess? a. The client taking diuretics b. The client with renal failure c. The client with an ileostomy d. The client who requires gastrointestinal suctioning

A client with increased ICP is prescribed the following tests. The nurse would clarify which test with the physician? A. MRI B. LP C. CT Scan. D. Cerebral angiography

B

How should you most accurately assess the position sense of a patient with a recent traumatic brain injury? A. Ask the patient to close his or her eyes and slowly bring the tips of the index fingers together. B. Ask the patient to maintain balance while standing with his or her feet together and eyes closed. C. Ask the patient to close his or her eyes and identify the presence of a common object on the forearm. D. Place the two points of a calibrated compass on the tips of the fingers and toes, and ask the patient to discriminate the points.

B

D

Based on the factors of age, gender, and body type, which patient has the smallest percentage of total body water? a. Thin 78-year-old adult man b. Obese 35-year-old man c. Thin 25-year-old woman d. Obese 68-year-old woman

During the administration of the thrombolytic agent to a patient with an acute myocardial infarction (AMI), the nurse should stop the drug infusion if the patient experiences a. bleeding from the gums. b. increase in blood pressure. c. a decrease in level of consciousness. d. a nonsustained episode of ventricular tachycardia.

C

Synthetic thyroid hormone T₃-T4₄ combined: A. Levothyroxine B. Liothyronine C. Liotrix D. Thyroid

C

Which dietary guideline would be appropriate for the older adult homebound client with advanced respiratory disease who informs the nurse that she has no energy to eat? a) Eat one large meal at noon. b) Snack on high-carbohydrate foods frequently. c) Eat smaller meals that are high in protein. d) Contact the physician for nutrition shake.

C

The older adult client with degenerative arthritis is admitted for tracheostomy surgery. What is the best communication method for this client during the postoperative period? A. Computer keyboard B. Magic slate C. Picture board D. Pen and paper

C A picture board does not require very much dexterity for communication. Dexterity can be limited to the extent the client finds comfortable.

A

Chronic obstructive pulmonary disease (COPD) managed with prednisone can make you at risk for? A. Hypokalemia B. Hyponatremia C. Hyperkalemia D. Hypernatremia

A

Congestive heart failure managed with loop diuretics puts you at risk for? A. Hypokalemia B. Hyponatremia C. Hyperkalemia D. Hypernatremia

A​ client's mother asks the nurse if there is anything non-pharmacologic that her daughter can do to help with intractable seizures. Which response by the nurse is​ correct? A.​"Taking megadoses of vitamins might be worth a​ try." B.​"Eating a vegetarian diet has been proven to be​ successful." C.​"Taking in extra sugar on a regular basis could be​ helpful." D.​"Eating a ketogenic diet can be​ helpful."

D

How does the nurse recognize that atropine has produced a positive outcome for the client with bradycardia? a. Client states he is dizzy and weak. b. The nurse notes dyspnea. c. The client has a heart rate of 42. d. The monitor shows sinus rhythm.

D Sinus rhythm presents with heart rates from 60 to 100 beats/min; by definition, the bradydysrhythmia has resolved.

C

Hypertension managed with Aldactone puts you at risk for? A. Hypokalemia B. Hyponatremia C. Hyperkalemia D. Hypernatremia

C

Multiple transfusions of packed red blood cells can put you at risk for? A. Hypokalemia B. Hyponatremia C. Hyperkalemia D. Hypernatremia

C

The UAP reports to the nurse that the patient being evaluated for kidney problems has produced a large amount of pale yellow urine. What does the nurse do next? a. Instruct the UAP to measure the amount carefully and then discard the urine. b. Instruct the UAP to save the urine in a large bottle for a 24-hour urine specimen. c. Assess the patient for signs of fluid imbalance and check the specific gravity of the urine. d. Compare the amount of urine output to the fluid intake for the previous 8 hours.

C

The client who has undergone an exploratory laparotomy and subsequent removal of a large intestinal tumor has a nasogastric tube (NGT) in place an an IV running at 150 mL/hr via an IV pump. Which data should be reported to the HCP? a. The pump keeps sounding an alarm indicating the high pressure has been reached. b. Intake is 1800 mL, NGT output is 550 mL, and Foley output is 950 mL c. On auscultation, crackles and rhonchi in all lung fields are noted d. Client has negative pedal edema and an increasing level of consciousness

B

The client who is post-thyroidectomy complains of numbness and tingling around the mouth and the tips of the fingers. Which intervention should the nurse implement first? a. Notify the health-care provider immediately b. Tap the cheek about two cm anterior to the earlobe c. Check the serum calcium and magnesium levels d. Prepare to administer calcium gluconate IVP

A

The emergency department (ED) nurse is caring for the patient who was brought in for significant alcohol intoxication and minor trauma to the wrist. What will serial hematocrits for this patient likely show? a. Hemoconcentration b. Normal and stable hematocrits c. Progressively lower hematocrits d. Decreasing osmolality

D

The nurse is caring for the older adult patient whose serum sodium level is 150 mEq/L. The nurse assesses the patient for which common manifestation associated with this sodium result? a. Gastrointestinal disorders b. Altered urinary elimination c. Impaired skin integrity d. Altered cerebral functioning

A

The patient who has undergone which surgical procedure is at risk for hypocalcemia? a. Thyroidectomy b. Adrenalectomy c. Pancreatectomy d. Gastrectomy

A

The telemetry monitor technician notifies the nurse of the morning telemetry readings. Which client should the nurse assess first? a. The client in normal sinus rhythm with a peaked T wave b. The client diagnosed with atrial fibrillation with a rate of 100 c. The client diagnosed with a myocardial infarction who has occasional PVCs d. The client with a first-degree atrioventricular block and a rate of 92

C

Uncontrolled diabetes mellitus puts you at risk for? A. Hypokalemia B. Hyponatremia C. Hyperkalemia D. Hypernatremia

A

What is a typical nursing assessment finding for the patient with hypocalcemia? a. Paresthesias and tingling followed by numbness b. Shortened ST segment, tachycardia, and hypertension c. Constipation and hypoactive bowel sounds d. Severe muscle weakness

A client who is intubated and receiving mechanical ventilation has a problem of risk for infection. The nurse should include which measures in the care of this client? Select all that apply. 1.Monitor the client's temperature. 2.Use sterile technique when suctioning. 3.Use the closed-system method of suctioning. 4.Monitor sputum characteristics and amounts. 5.Drain water from the ventilator tubing into the humidifier bottle.

1234

The nurse is caring for a client who begins to experience seizure activity while in bed. Which actions should the nurse take?Select all that apply. 1. Loosening restrictive clothing 2. Restraining the client's limbs 3. Removing the pillow and raising padded side rails 4. Positioning the client to the side, if possible, with the head flexed forward 5. Keeping the curtain around the client and the room door open so when help arrives they can quickly enter to assist

134

A client is being treated for hypothyroidism. The nurse knows that thyroid replacement therapy has been inadequate when she notes which findings? (SATA) 1. Prolonged QT interval on electrocardiogram 2. Tachycardia 3. Low body temperature 4. Nervousness 5. Bradycardia 6. Dry mouth

135 In hypothyroidism, the body is in a hypometabolic state. Therefore, a prolonged QT interval with bradycardia and subnormal body temperature would indicate that replacement therapy was inadequate. Tachycardia, nervousness, and dry mouth are symptoms of an excessive level of thyroid hormone; these findings would indicate that the client has received an excessive dose of thyroid hormone.

The nurse should expect a client with hypothyroidism to report which health concern(s)? 1. Increased appetite and weight loss 2. Puffiness of the face and hands 3. Nervousness and tremors 4. Thyroid gland swelling

2 Hypothyroidism (myxedema) causes facial puffiness, extremity edema, and weight gain. Signs and symptoms of hyperthyroidism (Graves' disease) include an increased appetite, weight loss, nervousness, tremors, and thyroid gland enlargement (goiter).

A patient who had a mitral valve replacement with a prosthetic mechanical valve and is ready for discharge home. Which information should the healthcare provider include in the discharge teaching for this patient? Select all that apply. 1. "Your valve will need to be replaced after 10 years." 2. "If you plan to become pregnant, be sure to consult your healthcare provider." 3. "You may need to take an antibiotic before certain medical or dental procedures." 4. "You will need to come in regularly for coagulation studies." 5. "Call our office immediately if you experience an infection of any kind."

2345

A patient has severe aortic stenosis. When assessing the patient, which of these findings would be expected?Please choose from one of the following options. 1.Hypertension 2. Bounding pulses 3. Narrowed pulse pressure 4. Diastolic murmur

3

A nurse caring for a client scheduled for a transsphenoidal hypophysectomy to remove a tumor in the pituitary gland assists to develop a plan of care for the client. The nurse suggests including which specific information in the preoperative teaching plan? 1. Hair will need to be shaved. 2. Deep breathing and coughing will be needed after surgery. 3. Toothbrushing will not be permitted for at least 2 weeks following surgery. 4. Spinal anesthesia is used.

3 Based on the location of the surgical procedure, spinal anesthesia would not be used. In addition, the hair would not be shaved. Although coughing and deep breathing are important, specific to this procedure is avoiding toothbrushing to prevent disruption of the surgical site. Also, coughing may disrupt the surgical site.

A nurse instructs a client about continuous ambulatory peritoneal dialysis (CAPD). Which of the following statements if made by the client indicates an accurate understanding of CAPD? 1. A portable hemodialysis machine is used so that I will be able to ambulate during the treatment. 2. A cycling machine is used so the risk for infection is minimized. 3. No machinery is involved, and I can pursue my usual activities. 4. The drainage system can be used once during the day and a cycling machine for 3 cycles at night.

3 CAPD closely approximates normal renal function, and the client will need to infuse and drain the dialysis solution several times a day. No machinery is used, and CAPD is a manual procedure.

The client who has had a myocardial infarction is admitted to the telementry unit from intensive care. Which referral would be most appropriate for the client? 1. Social worker 2. Physical therapy 3. Cardiac rehabilitation 4. Occupation therapy

3 Cardiac rehabilitation is the most appropriate referral. The client can start rehabilitation in the hospital and then attend an outpatient cardiac rehabilitation clinic, which includes progressive exercises, diet teaching, and classes on modifying risk factors.

A nurse is caring for a client with a dysfunctional thyroid gland and is concerned that the client will exhibit a sign of thyroid storm. Which is an early indicator of this complication? 1. Bradycardia 2. Constipation 3. Hyperreflexia 4. Low-grade temperature

3 Clinical manifestations of thyroid storm include a fever as high as 106°F, hyperreflexia, abdominal pain, diarrhea, dehydration rapidly progressing to coma, severe tachycardia, extreme vasodilation, hypotension, atrial fibrillation, and cardiovascular collapse.

The nurse caring for a client who is mechanically ventilated is monitoring for complications of mechanical ventilation. Which assessment finding, if noted by the nurse, indicates the need for follow-up? 1.Muscle weakness in the arms and legs 2.A temperature of 98.6° F decreased from 99.0° F 3.A blood pressure of 90/60 mm Hg decreased from 112/78 mm Hg 4.A heart rate of 80 beats per minute decreased from 85 beats per minute

3 Complications of mechanical ventilation include the following: (1) hypotension caused by application of positive pressure, which increases intrathoracic pressure and inhibits blood return to the heart; (2) pneumothorax or subcutaneous emphysema as a result of positive pressure; (3) gastrointestinal alterations such as stress ulcers; (4) malnutrition if nutrition is not maintained; (5) infections; (6) muscular deconditioning; and (7) ventilator-dependence or inability to wean. Some muscle weakness is expected. Options 1, 2, and 4 present normal assessment findings.

A client with a chest injury has suffered flail chest. The nurse assesses the client for which most distinctive sign of flail chest? 1.Cyanosis 2.Hypotension 3.Paradoxical chest movement 4.Dyspnea, especially on exhalation

3 Flail chest results from multiple rib fractures. This results in a "floating" section of ribs. Because this section is unattached to the rest of the bony rib cage, this segment results in paradoxical chest movement. This means that the force of inspiration pulls the fractured segment inward, while the rest of the chest expands. Similarly, during exhalation, the segment balloons outward while the rest of the chest moves inward. This is a characteristic sign of flail chest.

A nurse assists in developing a plan of care for a client with hyperparathyroidism receiving calcitonin-human (Cibacalcin). Which outcome has the highest priority regarding this medication? 1. Relief of pain 2. Absence of side effects 3. Reaching normal serum calcium levels 4. Verbalization of appropriate medication knowledge

3 Hypercalcemia can occur in clients with hyperparathyroidism, and calcitonin is used to lower plasma calcium level. The highest priority outcome in this client situation would be a reduction in serum calcium level. Option 1 is unrelated to this medication. Although options 2 and 4 are expected outcomes, they are not the highest priority for administering this medication.

To evaluate a client's condition following cardiac catheterization, the nurse will palpate the pulse: 1.In all extremities 2.At the insertion site 3.Distal to the catheter insertion 4.Above the catheter insertion

3 Palpating pulses distal to the insertion site is important to evaluate for thrombophlebitis and vessel occlusion. They should be bilateral and strong.

A nurse is assessing a client who has had cranial surgery and is at risk for development of diabetes insipidus. The nurse would assess for which signs or symptoms that could indicate development of this complication? 1. Diarrhea 2. Infection 3. Polydipsia 4. Weight gain

3 Polydipsia and polyuria are classic symptoms of diabetes insipidus. The urine is pale, and the specific gravity is low. Diarrhea is not indicative of the complication. Infection is not associated with diabetes insipidus. Anorexia and weight loss also may occur.

A 57-year-old client with a history of asthma is prescribed propanolol (Inderal) to control hypertension. Before administered propranolol, which of the following actions should the nurse take first? 1.Monitor the apical pulse rate 2.Instruct the client to take medication with food 3.Question the physician about the order 4.Caution the client to rise slowly when standing.

3 Propranolol and other beta-adrenergic blockers are contraindicated in a client with asthma, so the nurse should question the physician before giving the dose. The other responses are appropriate actions for a client receiving propranolol, but questioning the physician takes priority. The client's apical pulse should always be checked before giving propranolol; if the pulse rate is extremely low, the nurse should withhold the drug and notify the physician

The charge nurse of an intensive care unit is making assignments for the night shift.Which client should be assigned to the most experienced intensive care nurse? 1. The client diagnosed with respiratory failure who is on a ventilator and requiresfrequent sedation. 2. The client diagnosed with lung cancer and iatrogenic Cushing's disease with ABGsof pH 7.35, PaO2 88, PaCO2 44, and HCO3 22. 3. The client diagnosed with Addison's disease who is lethargic and has a BP of 80/45,P 124, and R 28. 4. The client diagnosed with hyperthyroidism who has undergone a thyroidectomytwo (2) days ago and has a negative Trousseau's sign.

3 This client has a low blood pressure and tachycardia. This client could be about to go into an Addisonian crisis, a potentially life-threatening condition. The most experienced nurse should care for this client.

The client diagnosed with Cushing's disease has undergone a unilateral adrenalectomy. Which discharge instructions should the nurse teach? 1. Instruct the client to take the glucocorticosteroid and mineralcorticosteroid medications as prescribed. 2. Teach the client regarding sexual functioning and androgen replacement therapy. 3. Explain the signs and symptoms of infection and when to call the health-care provider. 4. Demonstrate turn, cough, and deep-breathing exercises that the client should perform every (2) hours.

3 This is information given to all surgical clients on discharge.

The nurse is preparing to administer the following medications. Which medication should the nurse question administering? 1. The thyroid hormone to the client who does not have a T3, T4 level. 2. The regular insulin to the client with a blood glucose level of 210 mg/dL. 3. The loop diuretic to the client with a potassium level of 3.3 mEq/L. 4. The cardiac glycoside to the client who has a digoxin level of 1.4 mg/dL.

3 This potassium level is below normal,which is 3.5 to 5.5 mEq/L. Therefore,the nurse should question administering this medication because loop diuretics cause potassium loss in the urine.

Which medication order should the nurse question in the client diagnosed with untreated hypothyroidism? 1. Thyroid hormones. 2. Oxygen. 3. Sedatives. 4. Laxatives.

3 Untreated hypothyroidism is characterized by an increased susceptibility to the effects of most hypnotic and sedative agents; therefore, the nurse should question this medication.

a 38 y/o client has chronic cirrhosis and is jaundiced. to decrease pruritus from the jaundice, the nurse would: 1. use wool blankets 2. keep room temp above 75.8 3. keep the room temp below 70.8 4. use lots of soap in the bathwater

3 a decrease in sweating decreases pruritus

The nurse is preparing a client with a new diagnosis of hypothyroidism for discharge. The nurse determines that the client understands discharge instructions if the client states that which symptoms are associated with this diagnosis? Select all that apply. 1.Tremors 2.Weight loss 3.Feeling cold 4.Loss of body hair 5.Persistent lethargy 6.Puffiness of the face

3456

A client being mechanically ventilated after experiencing a fat embolism is visibly anxious. What is the best nursing action? 1.Ask a family member to stay with the client at all times. 2.Ask the health care provider for a prescription for succinylcholine. 3.Encourage the client to sleep until arterial blood gas results improve. 4.Provide reassurance to the client and give small doses of morphine sulfate intravenously as prescribed.

4

A client has an appendectomy and develops peritonitis. The nurse should asses the client for an elevated temperature and which additional clinical indication commonly associated with peritonitis? 1. hyperactivity 2. extreme hunger 3. urinary retention 4. local muscular rigidity

4

IV heparin therapy is ordered for a client. While implementing this order, a nurse ensures that which of the following medications is available on the nursing unit? 1.Vitamin K 2.Aminocaporic acid 3.Potassium chloride 4.Protamine sulfate

4

The charge nurse is making client assignments on a medical floor. Which client should the charge nurse assign to the LPN? 1. The client with pneumonia who has a pulse oximeter reading of 91%. 2. The client with a hemothorax who has Hgb of 9 mg/dL and Hct of 20%. 3. The client with chest tubes who has jugular vein distention and BP of 96/60. 4. The client who is two (2) hours post-bronchoscopy procedure.

4

The nurse caring for a client with a pneumothorax and who has had a chest tube inserted notes continuous gentle bubbling in the suction control chamber. What action is most appropriate? 1.Do nothing, because this is an expected finding. 2.Check for an air leak because the bubbling should be intermittent. 3.Increase the suction pressure so that the bubbling becomes vigorous. 4.Immediately clamp the chest tube and notify the health care provider.

4

The nurse is caring for a client diagnosed with diabetes insipidus (DI). Which nursing intervention should be implemented? 1. Monitor blood glucoses before meals and at bedtime. 2. Restrict caffeinated beverages. 3. Check urine ketones if blood glucose is 250. 4. Assess tissue turgor every four (4) hours.

4 The client is excreting large amounts of dilute urine. If the client is unable to take in enough fluids, the client will quickly become dehydrated, so tissue turgor should be assessed frequently.

A client who is diagnosed with acute respiratory distress syndrome​ (ARDS) requires mechanical ventilation. Which ventilator mode should the nurse expect to implement to promote pressure throughout the respiratory​ cycle? A. Positive​ end-expiratory pressure​ (PEEP) B. Sensitivity C. Flow rate D. Tidal volume​ (TV)

A

A patient was admitted after a motor vehicle accident with multiple fractured ribs. Respiratory assessment includes signs/symptoms of secondary pneumothorax, which includes which of the following? A: Sharp pleuritic pain that worsens on inspiration B: Crackles over lung bases of affected lung C: Tracheal deviation toward the affected lung D: Increased diaphragmatic excursion on side of rib fractures

A

Over secretion of the adrenocortical hormones leads to: A. Cushing's syndrome B. Addison's disease

A

When caring for a client with cachexia, the nurse expects to note which symptom? A. Weight loss B. Anemia C. Bleeding tendencies D. Motor deficits

A

Which diagnostic procedure measures lung size and airway patency, producing graphic representations of lung volumes and flows? a) Pulmonary function tests b) Chest x-ray c) Skin tests d) Bronchoscopy

A

Which is a classic finding for a patient with ARDS? A. Hypoxemia despite increased oxygen administration B. Bronchodilators ordered to relieve airway spasms C. Development of Kussmaul respirations D. Development of Cheyne-Stokes respirations

A

Which of the following is the most common symptom of myocardial infarction (MI)? A) Chest pain B) Dyspnea C) Edema D) Palpitations

A

Which of the following statements is true regarding oxygen toxicity? A) It can occur in patients who inhale greater than 50% oxygen for more than 24 hours. B) It causes destruction of oxygen-free radicals. C) The most common presenting symptom is respiratory depression. D) Chest radiography is a useful tool for early diagnosis.

A

Which of the following would confirm a diagnosis of appendicitis? a. The pain is localized at a position halfway between the umbilicus and the right iliac crest. b. Mr. Liu describes the pain as occurring 2 hours after eating c. The pain subsides after eating d. The pain is in the left lower quadrant"

A

A nurse is delivering 3 L/min oxygen to a patient via nasal cannula. What percentage of delivered oxygen is the patient receiving? a) 32% b) 28% c) 47% d) 23%

A A nasal cannula is used to deliver from 1 L/minute to 6 L/minute of oxygen. 1 L/minute = 24%, 2 L/minute = 28%, 3 L/minute = 32%, 4 L/minute = 36%, 5 L/minute = 40%, and 6 L/minute = 44%

A newly admitted client has deep partial-thickness burns. The nurse expects to see which clinical manifestations? A. Painful red and white wounds B. Painless, brownish yellow eschar C. Painful reddened blisters D. Painless black skin with eschar

A A painful red and white wound bed characterizes deep partial-thickness burns; blisters are rare. Painless, brownish yellow eschar characterizes a full-thickness burn. A painful reddened blister is seen with a superficial partial-thickness burn. Painless black skin with eschar is seen in a deep full-thickness burn.

The nurse is caring for a client who has a compromised cardiopulmonary system and needs to assess the client's tissue oxygenation. The nurse would use which appropriate method to assess this client's oxygenation? a) Arterial blood gas b) Hemoglobin levels c) Hematocrit values d) Pulmonary function

A Arterial blood gases include the levels of oxygen, carbon dioxide, bicarbonate, and pH. Blood gases determine the adequacy of alveolar gas exchange and the ability of the lungs and kidneys to maintain the acid-base balance of body fluids.

A client with chronic renal failure has completed a hemodialysis treatment. The nurse would use which of the following standard indicators to evaluate the client's status after dialysis? a. Vital signs and weight. b. Potassium level and weight. c. Vital signs and BUN. d. BUN and creatinine levels.

A Following dialysis, the client's vital signs are monitored to determine whether the client is remaining hemodynamically stable. Weight is measured and compared with the client's predialysis weight to determine effectiveness of fluid extraction. Laboratory studies are done as per protocol but are not necessarily done after the hemodialysis treatment has ended.

Metabolic acidosis occurs in the oliguric phase of AKI as a result of impairment of a. ammonia synthesis. b. excretion of sodium. c. excretion of bicarbonate. d. conservation of potassium.

A Metabolic acidosis occurs in AKI because the kidneys cannot synthesize ammonia or excrete acid products of metabolism, resulting in an increased acid load. Sodium is lost in urine because the kidneys cannot conserve sodium. Impaired excretion of potassium results in hyperkalemia. Bicarbonate is normally generated and reabsorbed by the functioning kidney to maintain acid base balance.

The nurse is planning care for a 48-year-old woman with acute severe pancreatitis. The highest priority patient outcome is a. maintaining normal respiratory function. b. expressing satisfaction with pain control. c. developing no ongoing pancreatic disease. d. having adequate fluid and electrolyte balance.

A Respiratory failure can occur as a complication of acute pancreatitis, and maintenance of adequate respiratory function is the priority goal. The other outcomes would also be appropriate for the patient.

A client with hyperaldosteronism is being treated with spironolactone (Aldactone) before surgery. Which precautions does the nurse teach this client? a. Read the label before using salt substitutes. b. Do not add salt to your food when you eat. c. Avoid exposure to sunlight. d. Take Tylenol instead of aspirin for pain.

A Spironolactone is a potassium-sparing diuretic used to control potassium levels. Its use can lead to hyperkalemia. Although the goal is to increase the client's potassium, unknowingly adding potassium can cause complications. Some salt substitutes are composed of potassium chloride and should be avoided by clients on spironolactone therapy. Depending on the client, he or she may benefit from a low-sodium diet before surgery, but this may not be necessary. Avoiding sunlight and Tylenol is not necessary.

A female patient is awaiting surgery for acute peritonitis. Which action will the nurse include in the plan of care? a. Position patient with the knees flexed. b. Avoid use of opioids or sedative drugs. c. Offer frequent small sips of clear liquids. d. Assist patient to breathe deeply and cough.

A There is less peritoneal irritation with the knees flexed, which will help decrease pain. Opioids and sedatives are typically given to control pain and anxiety. Preoperative patients with peritonitis are given IV fluids for hydration. Deep breathing and coughing will increase the patient's discomfort.

In the administration of a drug such as levothyroxine (Synthroid), the nurse must teach the client: (Select all that apply.) a.) Therapy could take three weeks or longer. b.) Periodic lab tests for T4 levels are required. c.) Report weight loss, anxiety, insomnia, and palpitations. d.) Jaundice

ABC

Patients with a tracheostomy or endotracheal tube need suctioning. Which nursing interventions apply to proper suctioning technique? Select all that apply A. preoxygenate the patient for at least 30 seconds before suctioning B. Instruct the patient that he or she is going to be suctioned C. quickly insert the suction catheter until resistance is met D. suction the patient for at least 30 seconds to remove secretions E. repeat suctioning as needed for to five total suction passes

ABC

The nurse is caring for a patient who exhibits labored breathing and uses accessory muscles. The patient has crackles in both lung bases and diminished breath sounds. Which would be priority assessments for the nurse to perform? (Select all that apply.) A: SpO2 levels B: Amount of sputum production C: Change in respiratory rate and pattern D: Pain in lower calf area

ABC

A nurse is planning care for a client who will undergo PD. Which of the following actions should the nurse take? Select all that apply a. monitor blood glucose levels b. report cloudy dialysate return c. warm the dialysate in a microwave oven d. assess for SOB e. check the access site dressing for wetness

ABDE

A patient with a trach who receives unnecessary suctioning can experience which complications? Select all that apply A. bronchospasm B. mucosal damage C. impaired gag reflex D. bronchodialation E. bleeding

ABE

The nurse is assessing an older adult client with acute respiratory distress syndrome​ (ARDS). Which assessment finding indicates an early sign of hypoxemia for this​ client?(Select all that​ apply.) A. Agitation B. Confusion C. Tachypnea D. Dyspnea E. Anxiety

ABE

Patient with head injury. Which of the following are manifestations of Increased ICP? SELECT ALL THAT APPLY A.Headache. B.Tachycardia C.Hypotension D.Pupillary changes. E.Abnormal posturing.

ADE

A client is receiving peritoneal dialysis. While the dialysis solution is dwelling in the client's abdomen, the nurse should a. assess for urticaria b. observe respiratory status c. check capillary refill time d. monitor electrolyte status

B

A patient admitted for infective endocarditis has developed sharp left flank pain and hematuria. The nurse notifies the physician, recognizing that these symptoms mayindicate: a. Colonization of the microorganisms in the kidneys b. Vegetative embolization to the kidneys c. Septicemia resulting in decreased glomerular blood flow d. Hemolysis of red blood cells by hemolytic microorganisms

B

A patient is admitted to the emergency department with an open stab wound to the left chest. What is the first action that the nurse should take? a. Position the patient so that the left chest is dependent. b. Tape a nonporous dressing on three sides over the chest wound. c. Cover the sucking chest wound firmly with an occlusive dressing. d. Keep the head of the patient's bed at no more than 30 degrees elevation.

B

A patient with cardiogenic shock receives a nursing diagnosis of decreased cardiac output. With the appropriate interventions, the anticipated outcome is for the patient to achieve: a.) Baseline activity level. b.) Baseline cardiac function. c.) Reduced anxiety. d.) Decreased afterload.

B

Discharge teaching for a patient receiving glucocorticoids would include the use of which medication for pain management? A. Aspirin (Acetylsalicylic acid) B. Acetaminophen (Tylenol) C. Ibuprofen D. Naprosyn (Naproxen)

B

The blood test first used to identify a response to HIV infection is: a. Western blot b. ELISA test c. CD4+ T-cell count d. CBC

B

The home care nurse visits a 34-yr-old woman receiving peritoneal dialysis. Which statement indicates a need for immediate follow-up by the nurse? a. "Drain time is faster if I rub my abdomen." b. "The fluid draining from the catheter is cloudy." c. "The drainage is bloody when I have my period." d. "I wash around the catheter with soap and water."

B

The nurse completes a nursing history with a patient with infective endocarditis. A significant finding that constitutes a risk factor for infective endocarditis is: a. A recent myocardial infarction b. Recent cardiac catheterization c. Hypertension d. Recent streptococcal pharyngitis

B

The nurse is assigned to provide nursing care for a patient receiving mechanical ventilation. Which action should the nurse delegate to an experienced unlicensed assistive personnel (UAP)? a. Assessing the patient's respiratory status every 4 hours b. Taking vital signs and pulse oximetry readings every 4 hours c. Checking the ventilator settings to make sure they are as prescribed d. Observing whether the patient's tube needs suctioning every 2 hours

B

The nurse is positioning the female client with increased intracranial pressure. Which of the following positions would the nurse avoid? a. Head mildline b. Head turned to the side c. Neck in neutral position d. Head of bed elevated 30 to 45 degrees

B

Which disease is characterized by increased body metabolism, tachycardia, increased body temperature, and anxiety, and treated with Prophylthiouracil (PTU)? a.) Hashimoto's thyroiditis b.) Graves' disease c.) Addison's disease d.) Cushing's syndrome

B

Which is a proper nursing action for a patient in acute respiratory failure? A. Administer 100% oxygen to an intubated patient until the pathology has resolved. B. Provide chest physical therapy for patients who produce more than 30 mL of sputum per day. C. Use continuous positive airway pressure (CPAP) if the patient has weak or absent respirations. D. Administer packed red blood cells to maintain the hemoglobin level at 7 g/dL or higher.

B

A patient in the emergency department has just been diagnosed with peritonitis caused by a ruptured diverticulum. Which prescribed intervention will the nurse implement first? a. Insert a urinary catheter to drainage. b. Infuse metronidazole (Flagyl) 500 mg IV. c. Send the patient for a computerized tomography scan. d. Place a nasogastric (NG) tube to intermittent low suction.

B Because peritonitis can be fatal if treatment is delayed, the initial action should be to start antibiotic therapy (after any ordered cultures are obtained). The other actions can be done after antibiotic therapy is initiated.

The newly hired nurse is caring for a client who had a tracheostomy four hours ago. Which action by the nurse, if noted by the charge nurse, would cause the charge nurse to intervene? a) The newly hired nurse adjusts the bed to a comfortable working position. b) The newly hired nurse delegates care of the tracheostomy to a licensed practical/vocational nurse (LPN/LVN). c) The newly hired nurse explains what she is doing and the reason to the client, even though the client does not appear to be alert. d) The newly hired nurse assesses the client's pain and administers pain medication.

B Care of a tracheostomy tube in a stable situation, such as long-term care and other community-based care settings, may be delegated to licensed practical/vocational nurses (LPN/LVN); not in an acute instance.

For Rico who has chronic pancreatitis, which nursing intervention would be most helpful? A. Allowing liberalized fluid intake B. Counseling to stop alcohol consumption C. Encouraging daily exercise D. Modifying dietary protein

B Chronic pancreatitis typically results from repeated episodes of acute pancreatitis. More than half of chronic pancreatitis cases are associated with alcoholism. Counseling to stop alcohol consumption would be the most helpful for the client. Dietary protein modification is not necessary for chronic pancreatitis. Daily exercise and liberalizing fluid intake would be helpful but not the most beneficial intervention.

Which statement made to the nurse by a health care worker assigned to care for the client with HIV indicates a breach of confidentiality and requires further education by the nurse? A) ''I told the family members they needed to wash their hands when they enter and leave the room.'' B) ''The other health care worker and I were out in the hallway discussing how we were concerned about getting HIV from our client, so no one could hear us in the client's room.'' C) ''Yes, I understand the reasons why I have to wear gloves when I bathe my client.'' D) ''The client's spouse told me she got HIV from a blood transfusion.

B Discussing this client's illness outside the client's room is a breach of confidentiality.

After the first injection of an immunotherapy program, the nurse notices a large, red wheal on the client's arm, coughing, and expiratory wheezing. Which intervention should the nurse implement first? A. Notifying the health care provider immediately B. Administering I.M. epinephrine per protocol C. Beginning oxygen by way of nasal cannula D. Starting an I.V. line for medication administration

B Immediately on noticing the client's sign and symptoms, the nurse would determine that the client is experiencing anaphylaxis to the injection. The first action is to give 0.2 to 0.5 ml of 1:1,000 epinephrine I.M. Notifying the health care provider, beginning oxygen administration, and starting an I.V. line follow after the initial injection of epinephrine is administered.

A female client has a tracheostomy but doesn't require continuous mechanical ventilation. When weaning the client from the tracheostomy tube, the nurse initially should plug the opening in the tube for: a. 15 to 60 seconds. b. 5 to 20 minutes. c. 30 to 40 minutes. d. 45 to 60 minutes.

B Initially, the nurse should plug the opening in the tracheostomy tube for 5 to 20 minutes, and then gradually lengthen this interval according to the client's respiratory status. A client who doesn't require continuous mechanical ventilation already is breathing without assistance, at least for short periods; therefore, plugging the opening of the tube for only 15 to 60 seconds wouldn't be long enough to reveal the client's true tolerance to the procedure. Plugging the opening for more than 20 minutes would increase the risk of acute respiratory distress because the client requires an adjustment period to start breathing normally.

A client is receiving parenteral nutrition (PN) in the home setting has a weight gain of 5 lb in 1 week. The nurse next assesses the client to identify the presence of which of the following? A. Hypotension. B. Crackles upon auscultation of the lungs. C. Thirst. D. Polyuria.

B Normally, the weight gain of a client receiving PN is about 1-2 pound a week. A weight gain of 5 pounds over a week indicates a client is experiencing fluid retention that can result to hypervolemia. Signs of hypervolemia includes weight gain more than desired, headache, jugular vein distention, bounding pulse, and crackles on lung auscultation. Option A: Hypertension, not hypotension is expected. Options C and D are associated with hyperglycemia.

A client is diagnosed with oral candidiasis. Nurse Tina knows that this condition in AIDS is treated with: A. Trimethoprim + sulfamethoxazole B. Fluconazole C. Acyclovir D. Zidovudine

B Oral candidiasis usually responds to topical treatments such as clotrimazole troches and nystatin suspension (nystatin "swish and swallow"). Systemic antifungal medication such as fluconazole or itraconazole may be necessary for oropharyngeal infections that do not respond to these treatments.

A male client has been admitted with chest trauma after a motor vehicle accident and has undergone subsequent intubation. A nurse checks the client when the high-pressure alarm on the ventilator sounds, and notes that the client has absence of breathe sounds in right upper lobe of the lung. The nurse immediately assesses for other signs of: A. Pulmonary Embolism B. Right pneumothorax C. Displaced endotracheal tube D. Acute respiratory distress syndrome

B Pneumothorax is characterized by restlessness, tachycardia, dyspnea, pain with respiration, asymmetrical chest expansion, and diminished or absent breath sounds on the affected side. Pneumothorax can cause increased airway pressure because of resistance to lung inflation. Acute respiratory distress syndrome and pulmonary embolism are not characterized by absent breath sounds. An endotracheal tube that is inserted too far can cause absent breath sounds, but the lack of breath sounds most likely would be on the left side because of the degree of curvature of the right and left main stem bronchi.

The client who is concerned about getting a tracheostomy says, "I will be ugly, with a hole in my neck." What is the nurse's best response? A. "But you know you need this to breathe, right?" B. Do you have a pretty scarf or a large loose collar that you could place over it?" C. "Your family and friends probably won't even care." D. "It won't take you long to learn to manage."

B Suggesting strategies to cover the tracheostomy recognizes client concerns and explores options for dealing with the effects of the procedure.

The nurse would increase the comfort of the patient with appendicitis by: a. Having the patient lie prone b. Flexing the patient's right knee c. Sitting the patient upright in a chair d. Turning the patient onto his or her left side

B The patient with appendicitis usually prefers to lie still, often with the right leg flexed to decrease pain.

A nurse is working with a patient who was diagnosed with HIV several months earlier. The nurse should recognize that a patient with HIV is considered to have AIDS at the point when the CD4+ T-lymphocyte cell count drops below what threshold? A) 75 cells/mm3 of blood B) 200 cells/mm3 of blood C) 325 cells/mm3 of blood D) 450 cells/mm3 of blood

B When CD4+ T-cell levels drop below 200 cells/mm3 of blood, the person is said to have AIDS.

A client with atrial fibrillation with rapid ventricular response has received medication to slow the ventricular rate. The pulse is now 88. For which additional therapy does the nurse plan? a. Synchronized cardioversion b. Electrophysiology studies (EPS) c. Anticoagulation d. Radiofrequency ablation therapy

C

Nurse April is teaching a client who suspects that she has a lump in her breast. The nurse instructs the client that a diagnosis of breast cancer is confirmed by: a. breast self-examination. b. mammography. c. fine needle aspiration. d. chest X-ray.

C

The doctor ordered for a complete blood count. After the test, Nurse Ray received the result from the laboratory. Which laboratory values will confirm the diagnosis of appendicitis? a. RBC 5.5 x 106/mm3 b. Hct 44 % c. WBC 13, 000/mm3 d. Hgb 15 g/dL"

C

The earliest signs of increased ICP the nurse should assess for include a. Cushing's triad b. unexpected vomiting c. decreasing level of consciousness (LOC) d. dilated pupil with sluggish response to light

C

The low-pressure alarm sounds on a ventilator. The nurse assesses the client and then attempts to determine the cause of the alarm. If unsuccessful in determining the cause of the alarm, the nurse should take what initial action? a. administer oxygen b. check the client's vital signs c. ventilate the client manually d. start CPR

C

The nurse is providing care for a patient who has been living with HIV for several years. Which assessment finding most clearly indicates an acute exacerbation of the disease? A. A new onset of polycythemia B. Presence of mononucleosis-like symptoms C. A sharp decrease in the patient's CD4+ count D. A sudden increase in the patient's WBC count

C

The nurses monitors the patient with positive pressure mechanical ventilation for a. paralytic ileus because pressure on the abdominal contents affects bowel motility b. diuresis and sodium depletion because of increased release of atrial natriuretic peptide c. signs of cardiovascular insufficiency because pressure in the chest impedes venous return d. respiratory acidosis in a patient with COPD because of alveolar hyperventilation and increased PaO2 levels

C

The nursing management of a patient with an artificial airway includes a. maintaining the ET tube cuff pressure at 30 cm H20 b. routine suctioning of the tube at least every 2 hours c. observing for cardiac dysrhythmias during suctioning d. preventing tube dislodgment by limiting mouth care to lubrication of the lips

C

When planning care for a patient on a mechanical ventilator, the nurse understands that the application of positive end-expiratory pressure (PEEP) to the ventilator settings has which therapeutic effect? a. Increased inflation of the lungs b. Prevention of barotrauma to the lung tissue c. Prevention of alveolar collapse during expiration d. Increased fraction of inspired oxygen concentration (FIO2) administration

C

Which drug used in the treatment of seizures requires careful monitoring of renal function? A. Lamotrigine (Lamictal) B. Primidone (Mysoline) C. Carbamazepine (Tegretol) D. Valproic acid (Depakene)

C

Which nursing intervention is appropriate for preventing atelectasis in the postoperative patient? A: Postural drainage B: Chest percussion C: Incentive spirometer D: Suctioning

C

Which nursing intervention will be most effective when assisting the patient with coronary artery disease (CAD) to make appropriate dietary changes? a. Give the patient a list of low-sodium, low-cholesterol foods that should be included in the diet. b. Emphasize the increased risk for heart problems unless the patient makes the dietary changes. c. Help the patient modify favorite high-fat recipes by using monosaturated oils when possible. d. Inform the patient that a diet containing no saturated fat and minimal salt will be necessary.

C

Which of the following information obtained by the nurse who is caring for a patient with end-stage renal disease (ESRD) indicates the nurse should consult with the health care provider before giving the prescribed epoetin alfa (Procrit)? a. Creatinine 1.2 mg/dL b. Oxygen saturation 89% c. Hemoglobin level 13 g/dL d. Blood pressure 98/56 mm Hg

C

Which of the following symptoms might a client with right-sided heart failure exhibit? a.) Adequate urine output b.) Polyuria c.) Oliguria d.) Polydipsia

C

Which statement made by the client allows the nurse to recognize whether the client who is receiving brachytherapy for ovarian cancer understands the treatment plan? A. "I may lose my hair during this treatment." B. "I must be positioned in the same way during each treatment." C. "I will have a radioactive device in my body for a short time." D. "I will be placed in a semiprivate room for company."

C

While assessing the patient, the nurse observes constant bubbling in the water-seal chamber of the patients closed chest-drainage system. What should the nurse conclude? A) The system is functioning normally. B) The patient has a pneumothorax. C) The system has an air leak. D) The chest tube is obstructed.

C

You are providing discharge teaching to a patient being discharged home after hospitalization with pericarditis. The physician has ordered the patient to take Colchicine. Which of the following statements indicates the patient did NOT understand the education you provided? A. "I can take this medication with or without food." B. "I will notify the doctor immediately if I start experiencing nausea, vomiting, or stomach pain while taking this medication." C. "I like to take all my medications in the morning with grapefruit juice." D. "This medication is also used to treat patients with gout."

C

You are supervising a student nurse who is performing tracheostomy care for a patient. Which action by the student would cause you to intervene? A. suctioning the trach tube before performing trach care B. removing old dressings and cleaning off excess secretions C. removing the inner cannula and cleaning using standard precautions D. replacing the inner cannula and cleaning the stoma site

C

You're precepting a nursing student who is helping you provide T-Tube drain care. You explain to the nursing student that the t-shaped part of the drain is located in what part of the biliary tract? A. Cystic duct B. Common hepatic duct C. Common bile duct D. Pancreatic duct

C

After teaching a client with acromegaly who is scheduled for a hypophysectomy, the nurse assesses the client's understanding. Which statement made by the client indicates a need for additional teaching? a. I will no longer need to limit my fluid intake after surgery. b. I am glad no visible incision will result from this surgery. c. I hope I can go back to wearing size 8 shoes instead of size 12. d. I will wear slip-on shoes after surgery to limit bending over.

C Although removal of the tissue that is oversecreting hormones can relieve many symptoms of hyperpituitarism, skeletal changes and organ enlargement are not reversible. It will be appropriate for the client to drink as needed postoperatively and avoid bending over. The client can be reassured that the incision will not be visible.

A client is hospitalized with a possible disorder of the adrenal cortex. Which nursing activity is best for the charge nurse to delegate to an experienced nursing assistant? A. Ask about risk factors for adrenocortical problems. B. Assess the client's response to physiologic stressors. C. Check the client's blood glucose levels every 4 hours. D. Teach the client how to do a 24-hour urine collection.

C Blood glucose monitoring is within the nursing assistant's scope of practice if the nursing assistant has received education and evaluation in the skill. Assessing risk factors for adrenocortical problems is not part of a nursing assistant's education. Assessing the client's response to physiologic stressors requires the more complex skill set of licensed nursing staff. Teaching the proper method for a 24-hour urine collection is a multi-step process; this task should not be delegated.

Four hours after a bowel resection, a 74-year-old male patient with a nasogastric tube to suction complains of nausea and abdominal distention. The first action by the nurse should be to a. auscultate for hypotonic bowel sounds. b. notify the patient's health care provider. c. reposition the tube and check for placement. d. remove the tube and replace it with a new one.

C Repositioning the tube will frequently facilitate drainage. Because this is a common occurrence, it is not appropriate to notify the health care provider unless other interventions do not resolve the problem. Information about the presence or absence of bowel sounds will not be helpful in improving drainage. Removing the tube and replacing it are unnecessarily traumatic to the patient, so that would only be done if the tube was completely occluded.

A 25-year-old patient with a group A streptococcal pharyngitis does not want to take the antibiotics prescribed. What should the nurse tell the patient to encourage the patient to take the medications and avoid complications of the infection? a. "The complications of this infection will affect the skin, hair, and balance." b. "You will not feel well if you do not take the medicine and get over this infection." c. "Without treatment, you could get rheumatic fever, which can lead to rheumatic heart disease." d. "You may not want to take the antibiotics for this infection, but you will be sorry if you do not."

C Rheumatic fever (RF) is not common because of effective use of antibiotics to treat streptococcal infections. Without treatment, RF can occur and lead to rheumatic heart disease, especially in young adults. The complications do not include hair or balance. Saying that the patient will not feel well or that the patient will be sorry if the antibiotics are not taken is threatening to the patient and inappropriate for the nurse to say.

To prepare a 56-year-old male patient with ascites for paracentesis, the nurse a. places the patient on NPO status. b. assists the patient to lie flat in bed. c. asks the patient to empty the bladder. d. positions the patient on the right side.q

C The patient should empty the bladder to decrease the risk of bladder perforation during the procedure. The patient would be positioned in Fowler's position and would not be able to lie flat without compromising breathing. Because no sedation is required for paracentesis, the patient does not need to be NPO.

Treatment of status epilepticus requires initiation of a rapid-acting antiseizure drug that can be given intravenously. You would anticipate which drugs to be administered (select all that apply)? A. phenytoin (Dilantin) B. phenobarbital C. lorazepam (Ativan) D. diazepam (Valium) E. carbemazepine (Tegretol)

CD

As the nurse you know that acute respiratory distress syndrome (ARDS) can be caused by direct or indirect lung injury. Select below all the INDIRECT causes of ARDS: A. Drowning B. Aspiration C. Sepsis D. Blood transfusion E. Pneumonia F. Pancreatitis

CDF

Nursing management of a patient with a brain tumor includes (select all that apply) A. discussing with the patient methods to control inappropriate behavior. B. using diversion techniques to keep the patient stimulated and motivated. C. assisting and supporting the family in understanding changes in behavior. D. limiting self-care activities until the patient has regained maximum physical functioning. E. planning for seizure precautions and teaching the patient and caregiver about antiseizure drugs.

CE

A child with leukemia is receiving chemotherapy and is complaining of nausea. The nurse has been giving the scheduled antiemetic. Which of the following should the nurse do when the child is nauseated? a. encourage low-protein foods b. encourage low-caloric foods c. offer the child's favorite foods d. offer cool, clear liquids

D

A client has an arteriovenous (AV) fistula in place in the right upper extremity for hemodialysis treatments. When planning care for this client, which of the following measures should the nurse implement to promote client safely? a. take blood pressures only on the right arm to ensure accuracy b. use the fistula for all venipunctures and intravenous infusions c. ensure that small clamps are attached to the AV fistula dressing d. assess the fistula for the presence of a bruit and thrill every 4 hours

D

A client who has just been discharged from the hospital after a modified radical mastectomy is referred to a home health agency. Which nursing action will be most appropriate to delegate to an experienced home health aide? A. Assessing the safety of the home environment B. Developing a plan to decrease lymphedema risk C. Monitoring pain level and analgesic effectiveness D. Reinforcing the guidelines for hand and arm care

D

The nurse needs to quickly estimate the heart rate for a patient with a regular heart rhythm. Which method will be best to use? a.Count the number of large squares in the R-R interval and divide by 300. b.Print a 1-minute electrocardiogram (ECG) strip and count the number of QRS complexes. c.Calculate the number of small squares between one QRS complex and the next and divide into 1500. d.Use the 3-second markers to count the number of QRS complexes in 6 seconds and multiply by 10.

D

The nurse observes precaution in caring for Mr. X as HIV is most easily transmitted in: a. Vaginal secretions and urine b. Breast milk and tears c. Feces and saliva d. Blood and semen

D

The nurse will ask a 64-year-old patient being admitted with acute pancreatitis specifically about a history of a. diabetes mellitus. b. high-protein diet. c. cigarette smoking. d. alcohol consumption.

D

What is the most appropriate snack for the nurse to offer a patient with stage 4 CKD? a. raisins b. ice cream c. dill pickles d. hard candy

D

When developing a teaching plan for a 61-year-old man with the following risk factors for coronary artery disease (CAD), the nurse should focus on the a. family history of coronary artery disease. b. increased risk associated with the patient's gender. c. increased risk of cardiovascular disease as people age. d. elevation of the patient's low-density lipoprotein (LDL) level.

D

Which intervention should be implemented after a bone marrow aspiration? a. ask the child to remain in a supine position b. place the child in an upright position for 4 hours c. keep the child NPO for 6 hours d. administer analgesics as needed for pain

D

Which of the following ABG values would the nurse expect to see on the patient with Acute Respiratory Failure? A) pH 7.35 B) O2 72 C) HCO3 26 D) PCO2 55

D

Which assessment finding is of most concern for a 46-year-old woman with acute pancreatitis? a. Absent bowel sounds b. Abdominal tenderness c. Left upper quadrant pain d. Palpable abdominal mass

D A palpable abdominal mass may indicate the presence of a pancreatic abscess, which will require rapid surgical drainage to prevent sepsis. Absent bowel sounds, abdominal tenderness, and left upper quadrant pain are common in acute pancreatitis and do not require rapid action to prevent further complications.

Which of the following items would be most beneficial when providing oral care to a patient with metastatic cancer who is at risk for oral tissue injury secondary to chemotherapy? A. Firm-bristle toothbrush B. Hydrogen peroxide rinse C. Alcohol-based mouthwash D. 1 tsp salt in 1 L water mouth rinse

D A salt-water mouth rinse will not cause further irritation to oral tissue that is fragile because of mucositis, which is a side effect of chemotherapy.

Acute tubular necrosis (ATN) is the most common cause of intrarenal AKI. Which patient is most likely to develop ATN? a. Patient with diabetes mellitus b. Patient with hypertensive crisis c. Patient who tried to overdose on acetaminophen d. Patient with major surgery who required a blood transfusion

D Acute tubular necrosis (ATN) is primarily the result of ischemia, nephrotoxins, or sepsis. Major surgery is most likely to cause severe kidney ischemia in the patient requiring a blood transfusion. A blood transfusion hemolytic reaction produces nephrotoxic injury if it occurs.Diabetes mellitus, hypertension, and acetaminophen overdose will not contribute to ATN.

A

Ileostomy puts you at risk for? A. Hypokalemia B. Hyponatremia C. Hyperkalemia D. Hypernatremia

Sodium

Normal plasma value is 135 to 145 mEq

Potassium

Normal plasma value is 3.5 to 5.0 mEq/L

Magnesium

Normal value is 1.3 to 2.1 mg/dL

Phosphorus

Normal value is 3.0 to 4.5 mg/dL

Calcium

Normal value is 9.0 - 10.5 mg/dL

Chloride

Normal value is 98 to 106 mEq/L

C

Respiratory acidosis puts you at risk for? A. Hypokalemia B. Hyponatremia C. Hyperkalemia D. Hypernatremia

C

The nurse instructs the UAP to use precautions with moving and use of a lifting sheet for which patient with an electrolyte imbalance? a. Young diabetic woman with hyperkalemia b. Psychiatric patient with hyponatremia c. Older woman with hypocalcemia d. Child with severe diarrhea and hypomagnesemia

B

The nurse is caring for the psychiatric patient who is continuously drinking water. The nurse monitors for which complication related to potential hyponatremia? a. Proteinuria/prerenal failure b. Change in mental status/increased intracranial pressure c. Pitting edema/circulatory failure d. Possible stool for occult blood/gastrointestinal bleeding

B

The nurse is evaluating the hydration status of the older adult patient. If the patient is dehydrated, the nurse expects to observe which type of cardiovascular change? a. Hypertension with bounding peripheral pulses b. Tachycardia with weak peripheral pulses c. Bradycardia and distended neck veins d. Increase in pulse pressure and systolic pressure

B

The patient's blood osmolality is 302 mOsm/L. What manifestation does the nurse expect to see in the patient? a. Increased urine output b. Thirst c. Peripheral edema d. Nausea

ABDEF

The patients with which conditions are at great risk for deficient fluid volume? (Select all that apply) a. Fever of 103 degrees F b. Extensive burns c. Thyroid crisis d. Water intoxication e. Continuous fistula drainage f. Diabetes insipidus

B

What is the minimum amount of urine per day needed to excrete toxic waste products? a. 200 to 300 mL b. 400 to 600 mL c. 500 to 1000 mL d. 1000 to 1500 mL

ACEG

Which nursing interventions apply to patients with hypercalcemia? (Select all that apply) a. Administer IV normal saline (0.9% sodium chloride) b. Assess the patient for a positive Homan's sign. c. Measure the abdominal girth. d. Massage calves to encourage blood return to the heart. e. Monitor for ECG changes f. Provide adequate intake of vitamin D g. During treatment, monitor for tetany

C

Which precaution or intervention does the nurse teach the patient at continued risk for hypernatremia? a. Avoid salt substitutes b. Avoid aspirin and aspirin-containing products c. Read labels on canned or packaged foods to determine sodium content d. Increase daily intake of caffeine-containing foods and beverages

D

Which serum laboratory value indicates the patient has hypernatremia? a. Chloride greater than 95 mEq/L b. Sodium greater than 135 mEq/L c. Chloride greater than 103 mEq/L d. Sodium greater than 145 mEq/L

A

Which serum value does the nurse expect to see in the patient with hyponatremia? a. Sodium less than 136 mEq/L b. Chloride less than 95 mEq/L c. Sodium less than 145 mEq/L d. Chloride less than 103 mEq/L

A

Whish statement best explains how antidiuretic hormone (ADH) affects urine output? a. It increases permeability to water in the tubules causing a decrease in urine output. b. It increases urine output as a result of water being absorbed by the tubules. c. Urine output is reduced as the posterior pituitary decreases ADH production. d. Increased urine output results from increased osmolarity and fluid in the extracellular space.

The nurse is preparing to administer a dose of PhosLo to a patient with chronic kidney disease. This medication should have a beneficial effect on which laboratory value? a. Sodium b. Potassium c. Magnesium d. Phosphorus

D

The nurse is caring for a patient after thoracentesis. Which actions can be delegated from the nurse to the unlicensed assistive personnel (UAP)? Select all that apply. 1.) Assess puncture site and dressing for leakage. 2.)Check vital signs every 15 minutes for 1 hour. 3.) Auscultate for absent or reduced lung sounds. 4.)Remind the patient to take deep breaths. 5.)Take the specimens to the laboratory. 6.) Teach the patient symptoms of pneumothorax.

245

The nurse is administering a saturated solution of potassium iodide (SSKI). The nurse should: 1. pour the solution over ice chips. 2. mix the solution with an antacid. 3. dilute the solution with water, milk, or fruit juice and have the client drink it with a straw. 4. disguise the solution in a pureed fruit or vegetable.

3

The client diagnosed with hypothyroidism is prescribed the thyroid hormone levothyroxine (Synthroid). Which assessment data indicate the medication has been effective? 1. The client has a three (3)-pound weight gain. 2. The client has a decreased pulse rate. 3. The client's temperature is WNL. 4. The client denies any diaphoresis.

3 The client with hypothyroidism frequently has a subnormal temperature,so a temperature WNL indicates the medication is effective.

Which characteristic of a patient's recent seizure indicates a partial seizure? A. The patient lost consciousness during the seizure. B. The seizure involved lipsmacking and repetitive movements. C. The patient fell to the ground and became stiff for 20 seconds. D. The etiology of the seizure involved both sides of the patient's brain.

B

Which option is most indicative of a skull fracture after blunt head trauma? A. Facial edema B. Epitasis C. Otorrhea positive for glucose D. Laceration oozing blood

C

The issue that is often foremost in the minds of men who have been diagnosed with prostate cancer and must be addressed by the nurse is the alteration of which factor? A. Comfort because of surgical pain. B. Mobility because of treatment. C. Nutrition because of radiation treatment. D. Sexual function after treatment.

D

The medical nurse is creating the care plan of an adult patient requiring mechanical ventilation. What nursing action is most appropriate? A) Keep the patient in a low Fowlers position. B) Perform tracheostomy care at least once per day. C) Maintain continuous bedrest. D) Monitor cuff pressure every 8 hours.

D

A 35 years old client has been receiving chemotherapy to treat cancer. Which assessment finding suggests that the client has developed stomatitis (inflammation of the mouth)? a. White, cottage cheese-like patches on the tongue b. Yellow tooth discoloration c. Red, open sores on the oral mucosa d. Rust-colored sputum

C The tissue-destructive effects of cancer chemotherapy typically cause stomatitis, resulting in ulcers on the oral mucosa that appear as red, open sores. White, cottage cheese-like patches on the tongue suggest a candidal infection, another common adverse effect of chemotherapy. Yellow tooth discoloration may result from antibiotic therapy, not cancer chemotherapy. Rust-colored sputum suggests a respiratory disorder, such as pneumonia.

The nurse is caring for a patient who has been admitted for the treatment of AIDS. In the morning, the patient tells the nurse that he experienced night sweats and recently "coughed up some blood." What is the nurse's most appropriate action? A) Assess the patient for additional signs and symptoms of Kaposi's sarcoma. B) Review the patient's most recent viral load and CD4+ count. C) Place the patient on respiratory isolation and inform the physician. D) Perform oral suctioning to reduce the patient's risk for aspiration.

C These signs and symptoms are suggestive of tuberculosis, not Kaposi's sarcoma; prompt assessment and treatment is necessary. There is no indication of a need for oral suctioning and the patient's blood work will not reflect the onset of this opportunistic infection.

A nurse in a clinic is reviewing the laboratory reports of a client who has suspected cholelithiasis. Which of the following is an expected finding? A. Serum albumin 4.1 g/dL B. WBC 9,511/uL C. Direct bilirubin 2.1 mg/dL D. Serum cholesterol 171 mg/dL

C This finding is outside the expected reference range and is increased in the client who has cholelithiasis

The nurse giving discharge teaching for a client receiving carbamazepine (Tegretol) should include: A. Monitor blood glucose, and report decreased levels. B. Expect a discoloration of the contact lenses. C. Expect an orange discoloration of urine. D. Report unusual bleeding or bruises to the health care provider immediately.

D

A nurse who is caring for patient with a tracheostomy tube in place has just auscultated rhonchi bilaterally. If the patient is unsuccessful in coughing up secretions, what action should the nurse take? a. Encourage increased incentive spirometer use. b. Encourage the patient to increase oral fluid intake. c. Put on sterile gloves and use a sterile catheter to suction. d. Preoxygenate the patient for 3 minutes before suctioning.

C This patient needs suctioning now to secure a patent airway. Sterile gloves and a sterile catheter are used when suctioning a tracheostomy. Preoxygenation for 3 minutes is not necessary. Incentive spirometer (IS) use opens alveoli and can induce coughing, which can mobilize secretions. However, the patient with a tracheostomy may not be able to use an incentive spirometer. Increasing oral fluid intake would not moisten and help mobilize secretions in a timely manner.

The nurse is assessing a client with multiple trauma who is at risk for developing acute respiratory distress syndrome. The nurse should assess for which earliest sign of acute respiratory distress sysndrome? a. Bilateral wheezing b. Inspiratory crackles c. Intercostal retractions d. Increased respiratory rate

D

The nurse is assessing a patient who has a chest tube in place for the treatment of a pneumothorax. The nurse observes that the water level in the water seal rises and falls in rhythm with the patients respirations. How should the nurse best respond to this assessment finding? A) Gently reinsert the chest tube 1 to 2 cm and observe if the water level stabilizes. B) Inform the physician promptly that there is in imminent leak in the drainage system. C) Encourage the patient to do deep breathing and coughing exercises. D) Document that the chest drainage system is operating as it is intended.

D

During change-of-shift report, the nurse learns about the following four patients. Which patient requires assessment first? a. 40-year-old with chronic pancreatitis who has gnawing abdominal pain b. 58-year-old who has compensated cirrhosis and is complaining of anorexia c. 55-year-old with cirrhosis and ascites who has an oral temperature of 102° F (38.8° C) d. 36-year-old recovering from a laparoscopic cholecystectomy who has severe shoulder pain

C This patient's history and fever suggest possible spontaneous bacterial peritonitis, which would require rapid assessment and interventions such as antibiotic therapy. The clinical manifestations for the other patients are consistent with their diagnoses and do not indicate complications are occurring.

The nurse is assessing the respiratory status of a client who has suffered a fractured rib. The nurse should expect to note which finding? a. Slow, deep respirations b. Rapid, deep respirations c. Paradoxical respirations d. Pain, especially with inspiration

D

A patient's peripheral parenteral nutrition (PN) bag is nearly empty and a new PN bag has not arrived yet from the pharmacy. Which intervention is the priority? a. Monitor the patient's capillary blood glucose until a new PN bag is hung. b. Flush the peripheral line with saline and wait until the new PN bag is available. c. Infuse 5% dextrose in water until the new PN bag is delivered from the pharmacy. d. Decrease the rate of the current PN infusion to 10 mL/hr until the new bag arrives.

C To prevent hypoglycemia, the nurse should infuse a 5% dextrose solution until the next PN bag can be started. Decreasing the rate of the ordered PN infusion is beyond the nurse's scope of practice. Flushing the line and then waiting for the next bag may lead to hypoglycemia. Monitoring the capillary blood glucose is appropriate but is not the priority.

Which of the following blood tests is most indicative of cardiac damage? A) Lactate dehydrogenase B) Complete blood count (CBC) C) Troponin I D) Creatine kinase (CK)

C Troponin I levels rise rapidly and are detectable within 1 hour of myocardial injury. Troponin I levels aren't detectable in people without cardiac injury. Lactate dehydrogenase (LDH) is present in almost all body tissues and not specific to heart muscle. LDH isoenzymes are useful in diagnosing cardiac injury. CBC is obtained to review blood counts, and a complete chemistry is obtained to review electrolytes. Because CK levels may rise with skeletal muscle injury, CK isoenzymes are required to detect cardiac injury.

The nurse is caring for a patient with diabetes insipidus (DI) who is receiving vasopressin (Pitressin). What therapeutic effect does the nurse expect from this drug? A. Increase in thirst B. Improved skin turgor C. Decrease in urine output D. Normal serum albumin level

C Vasopressin increases the water reabsorption in the kidneys, thus decreasing urine output. It is used to treat DI, which presents with polydipsia, polyuria, and dehydration.

A 22-year-old female patient with an exacerbation of ulcerative colitis is having 15 to 20 stools daily and has excoriated perianal skin. Which patient behavior indicates that teaching regarding maintenance of skin integrity has been effective? a. The patient uses incontinence briefs to contain loose stools. b. The patient asks for antidiarrheal medication after each stool. c. The patient uses witch hazel compresses to decrease irritation. d. The patient cleans the perianal area with soap after each stool.

C Witch hazel compresses are suggested to reduce anal irritation and discomfort. Incontinence briefs may trap diarrhea and increase the incidence of skin breakdown. Antidiarrheal medications are not given 15 to 20 times a day. The perianal area should be washed with plain water after each stool.

Which of the following is a LATE sign of the development of a tension pneumothorax? A. Hypotension B. Tachycardia C. Tracheal deviation D. Dyspnea

C With a tension pneumothorax, you will quickly see hypotension, tachycardia, and dyspnea as the mediastinum shifts from the extra pressure in the intrapleural space on the affected side. A late sign of a tension pneumothorax is that the trachea will eventually shift to the unaffected side.

Pierre who is diagnosed with acute pancreatitis is under the care of Nurse Bryan. Which intervention should the nurse include in the care plan for the client? A. Administration of vasopressin and insertion of a balloon tamponade B. Preparation for a paracentesis and administration of diuretics C. Maintenance of nothing-by-mouth status and insertion of nasogastric (NG) tube with low intermittent suction D. Dietary plan of a low-fat diet and increased fluid intake to 2,000 ml/day

C With acute pancreatitis, the client is kept on nothing-by-mouth status to inhibit pancreatic stimulation and secretion of pancreatic enzymes. NG intubation with low intermittent suction is used to relieve nausea and vomiting, decrease painful abdominal distention, and remove hydrochloric acid. Vasopressin would be appropriate for a client diagnosed with bleeding esophageal varices. Paracentesis and diuretics would be appropriate for a client diagnosed with portal hypertension and ascites. A low-fat diet and increased fluid intake would further aggravate the pancreatitis.

The nurse is caring for a client who has been using mechanical ventilation for several months after an episode of sepsis and acute respiratory distress syndrome​ (ARDS). Which ventilator setting should the nurse anticipate the healthcare provider ordering for​ weaning? A. Positive​ end-expiratory pressure​ (PEEP) B. Bilevel ventilation​ (BIPAP) C. Assist-control mode ventilation​ (ACMV) D. Synchronized intermittent mandatory ventilation​ (SIMV)

D

The nurse is caring for a client with orders for oxygen at 5 L/min. Approximately how much FiO2 is the client receiving? a. 24% b. 28% c. 36% d. 40%

D

The nurse is caring for a client with respiratory acidosis secondary to​ end-stage acute respiratory distress syndrome​ (ARDS). Which result should the nurse anticipate on the arterial blood​ gas? A. High PaO2 and high PaCO2 B. Low PaO2 and low PaCO2 C. High PaO2 and low PaCO2 D. Low PaO2 and high PaCO2

D

The nurse is caring for a patient who was hospitalized 2 days earlier with aspiration pneumonia. Which assessment information is most important to communicate to the health care provider? a. Cough that is productive of blood-tinged sputum b. Scattered crackles throughout the posterior lung bases c. Temperature of 101.5° F (38.6° C) after 2 days of IV antibiotic therapy d. Oxygen saturation (SpO2) has dropped to 90% with administration of 100% O2 by non-rebreather mask.

D

The woman is afraid she may get HIV from her bisexual husband. What should the nurse include when teaching her about preexposure prophylaxis (select all that apply)? A. Take fluconazole (Diflucan). B. Take amphotericin B (Fungizone). C. Use condoms for risk-reducing sexual relations. D. Take emtricitabine and tenofovir (Truvada) regularly. E. Have regular HIV testing for herself and her husband.

CDE Using male or female condoms, having monthly HIV testing for the patient and her husband, and the woman taking emtricitabine and tenofovir regularly has shown to decrease the infection of heterosexual women having sex with a partner who participates in high-risk behavior. Fluconazole and amphotericin B are taken for Candida albicans, Coccidioides immitis, and Cryptococcosus neoformans, which are all opportunistic diseases associate with HIV infection

The nurse is discharging a child who has just received chemotherapy for neuroblastoma. Which of the following statements made by the child's parent indicates a need for further teaching? a. "I will inspect the skin often for any lesions" b. "I will do mouth care daily and monitor for any mouth sores" c. "I will wash my hands before caring for my child" d. "I will take a rectal temperature daily and report a temperature greater than 101 immediately"

D

The nurse is discussing activity management with a patient who is postoperative following thoracotomy. What instructions should the nurse give to the patient regarding activity immediately following discharge? A) Walk 1 mile 3 to 4 times a week. B) Use weights daily to increase arm strength. C) Walk on a treadmill 30 minutes daily. D) Perform shoulder exercises five times daily.

D

The nurse is interviewing a male client about his past medical history. Which preexisting condition may lead the nurse to suspect that a client has colorectal cancer? a. Duodenal ulcers b. Hemorrhoids c. Weight gain d. Polyps

D

The nurse is performing an assessment on a client who has returned from the dialysis unit following HD. The client is complaining of headache and nausea and is extremely restless. Which is the priority nursing action? a. monitor the client b. elevate the HOB c. assess the fistula site and dressing d. notify the HCP

D

Which interventions does the home health nurse teach to family members to reduce confusion in the client diagnosed with AIDS dementia? (Select all that apply.) A) Report any behavior changes. B) Use the Glasgow Coma Scale on a daily basis. C) Change the decorations in the home according to the season. D) Put the bed close to the window. E) Write out all instructions and have the client read them over before performing a task. F) Ask the client when he or she wants to shower or bathe. G) Mark off the days of the calendar, leaving open the current date. H) For continuity, the primary caregiver should be the only person reorienting the client.

CDFG Seasonal decorations in the home helps with maintaining orientation. This allows the client to visualize seasonal and weather changes and assists in orientation. Involving the client in planning the daily schedule helps with orientation. Using calendars and crossing off past dates helps with orientation.

The patient with CKD is considering whether to use PD or HD. What are advantages of PD when compared to HD? Select all that apply a. less protein loss b. rapid fluid removal c. less cardiovascular stress d. decreased hyperlipidemia e. requires fewer dietary restrictions

CE

An 83-year-old female patient was found lying on the bathroom floor. She said she fell 2 days ago and has not beenable to take her heart medicine or eat or drink anything since then. What conditions could be causing prerenal AKI inthis patient (select all that apply)? a. Anaphylaxis b. Renal calculi c. Hypovolemia d. Nephrotoxic drugs e. Decreased cardiac output

CE Because the patient has had nothing to eat or drink for 2 days, she is probably dehydrated and hypovolemic. Decreased cardiac output (CO) is most likely because she is older and takes heart medicine, which is probably for heart failure or hypertension. Both hypovolemia and decreased CO cause prerenal AKI. Anaphylaxis is also a cause of prerenal AKI but is not likely in this situation. Nephrotoxic drugs would contribute to intrarenal causes of AKI and renal calculi would be a postrenal cause of AKI.

The patient with chronic kidney disease is considering whether to use peritoneal dialysis (PD) or hemodialysis (HD).What are advantages of PD when compared to HD (select all that apply)? a. Less protein loss b. Rapid fluid removal c. Less cardiovascular stress d. Decreased hyperlipidemia e. Requires fewer dietary restrictions

CE Peritoneal dialysis is less stressful for the cardiovascular system and requires fewer dietary restrictions. Peritoneal dialysis actually contributes to more protein loss and increased hyperlipidemia. The fluid and creatinine removal are slower with peritoneal dialysis than hemodialysis.

A 52-year-old female patient was exposed to human immunodeficiency virus (HIV) 2 weeks ago through sharing needles with other substance users. What symptoms will the nurse teach the patient to report that would indicate the patient has developed an acute HIV infection? a. Cough, diarrhea, headaches, blurred vision, muscle fatigue b. Night sweats, fatigue, fever, and persistent generalized lymphadenopathy c. Oropharyngeal candidiasis or thrush, vaginal candidal infection, or oral or genital herpes d. Flu-like symptoms such as fever, sore throat, swollen lymph glands, nausea, or diarrhea

D

A 60-year-old man who is hospitalized with an abdominal wound infection has only been eating about 50% of meals and states, "Nothing on the menu sounds good." Which action by the nurse will be most effective in improving the patient's oral intake? a. Order six small meals daily. b. Make a referral to the dietitian. c. Teach the patient about high-calorie foods. d. Have family members bring in favorite foods.

D

A client has a chest tube in place following a left lower lobectomy done after a stab wound to the chest. When repositioning the client, the nurse notices 200 cc of dark, red fluid flows into the collection chamber of the chest drain. What is the MOST appropriate nursing action? a. Clamp the chest tube b. Call the surgeon immediately c. Prepare for blood transfusion d. Continue to monitor the rate of drainage

D

A client, age 41, visits the gynecologist. After examining her, the physician suspects cervical cancer. The nurse reviews the client's history for risk factors for this disease. Which history finding is a risk factor for cervical cancer? a. Onset of sporadic sexual activity at age 17 b. Spontaneous abortion at age 19 c. Pregnancy complicated with eclampsia at age 27 d. Human papillomavirus infection at age 32

D

A female client is receiving chemotherapy to treat breast cancer. Which assessment finding indicates a fluid and electrolyte imbalance induced by chemotherapy? a. Urine output of 400 ml in 8 hours b. Serum potassium level of 3.6 mEq/L c. Blood pressure of 120/64 to 130/72 mm Hg d. Dry oral mucous membranes and cracked lips

D

A male client with Guillain-Barré syndrome develops respiratory acidosis as a result of reduced alveolar ventilation. Which combination of arterial blood gas (ABG) values confirms respiratory acidosis? a. pH, 5.0; PaCO2 30 mm Hg b. pH, 7.40; PaCO2 35 mm Hg c. pH, 7.35; PaCO2 40 mm Hg d. pH, 7.25; PaCO2 50 mm Hg

D

A client enters the ER complaining of chest pressure and severe epigastric distress. His VS are 158/90, 94, 24, and 99*F. The doctor orders cardiac enzymes. If the client were diagnosed with an MI, the nurse would expect which cardiac enzyme to rise within the next 3 to 8 hours? 1.Creatine kinase (CK or CPK) 2.Lactic dehydrogenase (LDH) 3.LDH-1 4.LDH-2

1

A client has a serum calcium level of 7.2 mg/dl. During the physical examination, the nurse expects to assess: 1. Trousseau's sign. 2. Homans' sign. 3. Hegar's sign. 4. Goodell's sign.

1

A client has been admitted with chest trauma after a motor vehicle crash and has undergone subsequent intubation. The nurse checks the client when the high-pressure alarm on the ventilator sounds, and notes that the client has absence of breath sounds in the right upper lobe of the lung. The nurse immediately assesses for other signs of which condition? 1.Right pneumothorax 2.Pulmonary embolism 3.Displaced endotracheal tube 4.Acute respiratory distress syndrome

1

A client with subdural hematoma was given mannitol to decrease intracranial pressure (ICP). Which of the following results would best show the mannitol was effective? 1. Urine output increases 2. Pupils are 8 mm and nonreactive 3. Systolic blood pressure remains at 150 mm Hg 4. BUN and creatinine levels return to normal

1

A health care provider writes a prescription to begin to wean the client from the mechanical ventilator by use of intermittent mandatory ventilation/synchronized intermittent mandatory ventilation (IMV/SIMV). The nurse determines that the process of weaning will occur by which mechanism? 1.Gradually decreasing the respiratory rate until the client can assume the work of breathing without ventilatory assistance 2.Attaching a T-piece to the ventilator and providing supplemental oxygen at a concentration that is 10% higher than the ventilator setting 3.Providing pressure support to decrease the workload of breathing and increase the client's ability to initiate spontaneous breathing efforts 4.Removing the ventilator from the client and closely monitoring the client's ability to breathe spontaneously for a predetermined amount of time

1

Following a treadmill test and cardiac catheterization, the client is found to have coronary artery disease, which is inoperative. He is referred to the cardiac rehabilitation unit. During his first visit to the unit he says that he doesn't understand why he needs to be there because there is nothing that can be done to make him better. The best nursing response is: 1."Cardiac rehabilitation is not a cure but can help restore you to many of your former activities." 2."Here we teach you to gradually change your lifestyle to accommodate your heart disease." 3."You are probably right but we can gradually increase your activities so that you can live a more active life." 4."Do you feel that you will have to make some changes in your life now?"

1

Propylthiouracil (PTU) is prescribed for a client with Graves' disease. The nurse should teach the client to immediately report: 1. Sore throat. 2. painful, excessive menstruation. 3. constipation. 4. increased urine output.

1

The client diagnosed with ARDS is on a ventilator and the high alarm indicates an increase in the peak airway pressure. Which intervention should the nurse implement first? 1. Check the tubing for any kinks 2. Suction the airway for secretion 3. Assess the lip line of the ET tube 4. Sedate the client with a muscle relaxant

1

The client diagnosed with hyperthyroidism is complaining of being hot and cannot sit still. Which should the nurse do based on the assessment? 1. Continue to monitor the client. 2. Have the UAP take the client's vital signs. 3. Request an order for a sedative. 4. Insist the client lie down and rest.

1

The client is diagnosed with hypothyroidism. Which signs/symptoms should the nurse expect the client to exhibit? 1. Complaints of extreme fatigue and hair loss. 2. Exophthalmos and complaints of nervousness. 3. Complaints of profuse sweating and flushed skin. 4. Tetany and complaints of stiffness of the hands.

1

The health-care provider ordered STAT arterial blood gases for the client diagnosed with ARDS. The ABG results are: pH 7.38, PaO2 92, PaCO2 38, HCO3 24. Which action should the nurse implement? 1. Continue to monitor the client without taking any action 2. Encourage the client to take deep breaths and cough 3. Administer one ampule of sodium bicarbonate IVP 4. Notify the respiratory therapist of the results

1

The nurse is admitting a client diagnosed with syndrome of inappropriate antidiuretic hormone (SIADH). Which clinical manifestations should be reported to the health-care provider? 1. Serum sodium of 112 mEq/L and a headache. 2. Serum potassium of 5.0 mEq/L and a heightened awareness. 3. Serum calcium of 10 mg/dL and tented tissue turgor. 4. Serum magnesium of 1.2 mg/dL and large urinary output.

1

The nurse is performing discharge teaching for a client diagnosed with Cushing's disease. Which statement by the client demonstrates an understanding of the instructions? 1. "I will be sure to notify my health-care provider if I start to run a fever." 2. "Before I stop taking the prednisone, I will be taught how to taper it off." 3. "If I get weak and shaky, I need to eat some hard candy or drink some juice." 4. "It is fine if I continue to participate in weekend games of tackle football."

1

The nurse is planning the care of a client diagnosed with Addison's disease. Which intervention should be included? 1. Administer steroid medications. 2. Place the client on fluid restriction. 3. Provide frequent stimulation. 4. Consult physical therapy for gait training.

1

The nurse is providing care to a patient following a non-accidental traumatic brain injury. The patient has developed diabetes insipidus due to the injury. What medication is most often used in the management of diabetes insipidus? 1. desmopressin (DDAVP) 2. corticotrophin (Acthar) 3. octreotide (Sandostatin) 4. somatropin (Humatrope)

1

The nurse is providing immediate postprocedure care to a client who had a thoracentesis to relieve a tension pneumothorax that resulted from rib fractures. The goal is that the client will exhibit normal respiratory functioning, and the nurse provides instructions to assist the client toward this goal. Which client statement indicates that further instruction is needed? 1."I will lie on the affected side for an hour." 2."I can expect a chest x-ray exam to be done shortly." 3."I will let you know at once if I have trouble breathing." 4."I will notify you if I feel a crackling sensation in my chest."

1

The nurse is providing instructions to a client with a seizure disorder who will be taking phenytoin (Dilantin). Which statement, if made by the client, would indicate an understanding of the information about this medication? 1."I need to perform good oral hygiene, including flossing and brushing my teeth." 2."I should try to avoid alcohol, but if I'm not able to, I can drink alcohol in moderation." 3."I should take my medication before coming to the laboratory to have a blood level drawn." 4."I should monitor for side effects and adjust my medication dose depending on how severe the side effects are."

1

The nurse suspects the client may be developing ARDS. Which assessment data confirm the diagnosis of ARDS? 1. Low arterial oxygen when administering high concentration of oxygen 2. The client has dyspnea and tachycardia and feels anxious 3. Bilateral breath sounds clear and pulse oximeter reading is 95% 4. The client has JVD and frothy sputum

1

The nursing instructor is observing a nursing student suctioning a client through a tracheostomy tube. Which observation by the nursing instructor indicates an action by the student requiring the need for further instruction? 1.Suctioning the client every hour 2.Applying suction only during withdrawal of the catheter 3.Hyperventilating the client with 100% oxygen before suctioning 4.Applying suction intermittently during withdrawal of the catheter

1

Which action should the nurse implement for the client with a hemothorax who has a right-sided chest tube and there is excessive bubbling in the water-seal compartment? 1. Check the amount of wall suction being applied. 2. Assess the tubing for any blood clots. 3. Milk the tubing proximal to distal. 4. Encourage the client to cough forcefully.

1

Which complication should the nurse assess for in the elderly client newly diagnosed with hypothyroidism who has been prescribed levothyroxine? 1. Cardiac dysrhythmias. 2. Respiratory depression. 3. Paralytic ileus. 4. Thyroid storm.

1

Which instruction is priority for the nurse to discuss with the client diagnosed with ARDS who is being discharged from the hospital? 1. Avoid smoking and exposure to smoke 2. Do not receive flu or pneumonia vaccines 3. Avoid any type of alcohol intake 4. It will take about one month to recuperate

1

Which intervention should the nurse implement for a male client who has had a left- sided chest tube for six (6) hours and refuses to take deep breaths because it hurts too much? 1. Medicate the client and have the client take deep breaths. 2. Encourage the client to take shallow breaths to help with the pain. 3. Explain that deep breaths do not have to be taken at this time. 4. Tell the client that if he doesn't take deep breaths, he could die.

1

Which laboratory data indicate the client's pancreatitis is improving? 1. The amylase and lipase serum levels are decreased. 2. The white blood cell count (WBC) is decreased. 3. The conjugated and unconjugated bilirubin levels are decreased. 4. The blood urea nitrogen (BUN) serum level is decreased.

1

Which laboratory data make the nurse suspect the client with primary hyperparathyroidism is experiencing a complication? 1. A serum creatinine level of 2.8 mg/dL. 2. A calcium level of 9.2 mg/dL. 3. A serum triglyceride level of 130 mg/dL. 4. A sodium level of 135 mEq/L.

1

Which of the following assessment data indicated nuchal rigidity? 1. Positive Kernig's sign 2. Negative Brudzinski's sign 3. Positive homan's sign 4. Negative Kernig's sign

1

Which of the following symptoms should the nurse teach the client with unstable angina to report immediately to her physician? 1.A change in the pattern of her pain 2.Pain during sex 3.Pain during an argument with her husband 4.Pain during or after an activity such as lawnmowing

1

Which of the following would lead the nurse to suspect that a child with meningitis has developed disseminated intravascular coagulation? 1. Hemorrhagic skin rash 2. Edema 3. Cyanosis 4. Dyspnea on exertion

1

A nurse is reviewing discharge teaching with a client who has Cushing's syndrome. Which statement by the client indicates that the instructions related to dietary management were understood? 1. "I can eat foods that contain potassium." 2. "I will need to limit the amount of protein in my diet." 3. "I am fortunate that I can eat all the salty foods I enjoy." 4. "I am fortunate that I do not need to follow any special diet."

1 A diet that is low in calories, carbohydrates, and sodium but ample in protein and potassium content is encouraged for a client with Cushing's syndrome. Such a diet promotes weight loss, the reduction of edema and hypertension, the control of hypokalemia, and the rebuilding of wasted tissue.

Which of the following instructions should be included in the discharge teaching for a patient discharged with a transdermal nitroglycerin patch? 1."Apply the patch to a nonhairy, nonfatty area of the upper torso or arms." 2."Apply the patch to the same site each day to maintain consistent drug absorption." 3."If you get a headache, remove the patch for 4 hours and then reapply." 4."If you get chest pain, apply a second patch right next to the first patch."

1 A nitroglycerin patch should be applied to a nonhairy, nonfatty area for the best and most consistent absorption rates. Sites should be rotated to prevent skin irritation, and the drug should be continued if headache occurs because tolerance will develop. Sublingual nitroglycerin should be used to treat chest pain.

The nurse is admitting a client diagnosed with syndrome of inappropriate antidiuretic hormone (SIADH). Which clinical manifestations should be reported to the healthcare provider? 1. Serum sodium of 112 mEq/L and a headache. 2. Serum potassium of 5.0 mEq/L and a heightened awareness. 3. Serum calcium of 10 mg/dL and tented tissue turgor. 4. Serum magnesium of 1.2 mg/dL and large urinary output.

1 A serum sodium level of 112 mEq/L is dangerously low, and the client is at risk for seizures. A headache is a symptom of a low sodium level.`

Which nursing diagnosis is most appropriate for a client with Addison's disease? 1. Risk for infection 2. Excessive fluid volume 3. Urinary retention 4. Hypothermia

1 Addison's disease decreases the production of all adrenal hormones, compromising the body's normal stress response and increasing the risk of infection. Other appropriate nursing diagnoses for a client with Addison's disease include Deficient fluid volume and Hyperthermia. Urinary retention isn't appropriate because Addison's disease causes polyuria.

For a client in addisonian crisis, it would be very risky for a nurse to administer: 1. potassium chloride. 2. normal saline solution. 3. hydrocortisone. 4. fludrocortisone.

1 Addisonian crisis results in hyperkalemia; therefore, administering potassium chloride is contraindicated. Because the client will be hyponatremic, normal saline solution is indicated. Hydrocortisone and fludrocortisone are both useful in replacing deficient adrenal cortex hormones.

A client has abnormal amounts of circulating thyronine (T3) and thyroxine (T4). While obtaining the health history, the nurse asks the client about dietary intake. Lack of which dietary element is most likely the cause? 1. Iodine 2. Calcium 3. Phosphorus 4. Magnesium

1 Adequate dietary iodine is needed to produce T3 and T4. The other requirements for adequate T3 and T4 production are an intact thyroid gland and a functional hypothalamus-pituitary-thyroid feedback system. The remaining options are not responsible for the abnormal amounts of circulating T3 and T4.

A nurse is caring for an agitated and anxious patient who was intubated 6 hours ago and is now on mechanical ventilation. Communication efforts to calm the patient have failed, and the nurse is now turning to pharmacological intervention. Which medication does the nurse anticipate administering? 1) Lorazepam 2) Morphine sulfate 3) Pancuronium 4) Fentanyl

1 Although Pancuronium (a neuromuscular blocking agent) CAN be used, it is best to try a sedative first. If satisfactory oxygen levels still cannot be maintained, then a neuromuscular blocking agent (WITH PAIN MEDICATION AND SEDATION!) can be used.

A 33-year-old female client is admitted to the hospital with a tentative diagnosis of Graves' disease. Which symptom related to the menstrual cycle would the client be most likely to report during the initial assessment? 1. Amenorrhea 2. Menorrhagia 3. Metrorrhagia 4. Dysmenorrhea

1 Amenorrhea or a decreased menstrual flow occurs in the client with Graves' disease. Menorrhagia, metrorrhagia, and dysmenorrhea are also disorders related to the female reproductive system; however, they are not typical manifestations of Graves' disease.

A client has just been admitted with a diagnosis of myxedema coma. If all of the following interventions were prescribed, the nurse would place highest priority on completing which of the following first? 1. Administering oxygen 2. Administering thyroid hormone 3. Warming the client 4. Giving fluid replacement

1 As part of maintaining a patent airway, oxygen would be administered first. This would be quickly followed by fluid replacement, keeping the client warm, monitoring vital signs, and administering thyroid hormones.

Which intervention should the nurse implement first for the client diagnosed with a hemothorax who has had a right-sided chest tube for three (3) days and has no fluctu- ation (tidaling) in the water compartment? 1. Assess the client's bilateral lung sounds. 2. Obtain an order for a STAT chest x-ray. 3. Notify the health-care provider as soon as possible. 4. Document the findings in the client's chart.

1 Assessment of the lung sounds could indi- cate that the client's lung has reexpanded because it has been three (3) days since the chest tube has been inserted.

The nurse is monitoring a client diagnosed with appendicitis who is scheduled for surgery in 2 hours. The client begins to complain of increased abdominal pain and begns to vomit. On assessment, the nurse notes that the abdomen is distended and bowel sounds are diminished. Which is the appropriate nursing intervention? 1. Notify the Physician 2. Administer the prescribed pain medication 3. Call and ask the operating room team to perform the surgery as soon as possible 4. Reposition the client and apply a heating pad on warm setting to the client's abdomen

1 Based on the assessment information the nurse should suspect peritonitis, a complication that is associated with appendicitis, and notify the physician.

The nurse is monitoring a client diagnosed with appendicitis who is scheduled for surgery in 2 hours. The client begins to complain of increased abdominal pain and begns to vomit. On assessment, the nurse notes that the abdomen is distended and bowel sounds are diminished. Which is the appropriate nursing intervention? 1. Notify the Physician 2. Administer the prescribed pain medication 3. Call and ask the operating room team to perform the surgery as soon as possible 4. Reposition the client and apply a heating pad on warm setting to the client's abdomen

1 Based on the assessment information the nurse should suspect peritonitis, a complication that is associated with appendicitis, and notify the physician.

When caring for a client with diabetes insipidus, the nurse expects to administer: 1. vasopressin (Pitressin Synthetic). 2. furosemide (Lasix). 3. regular insulin. 4. 10% dextrose.

1 Because diabetes insipidus results from decreased antidiuretic hormone (vasopressin) production, the nurse should expect to administer synthetic vasopressin for hormone replacement therapy. Furosemide, a diuretic, is contraindicated because a client with diabetes insipidus experiences polyuria. Insulin and dextrose are used to treat diabetes mellitus and its complications, not diabetes insipidus.

Which of the following would indicate that a client has developed water intoxication secondary to treatment for diabetes insipidus? 1. Confusion and seizures 2. Sunken eyeballs and spasticity 3. Flaccidity and thirst 4. Tetany and increased blood urea nitrogen (BUN) levels

1 Classic signs of water intoxication include confusion and seizures, both of which are caused by cerebral edema. Weight gain will also occur. Sunken eyeballs, thirst, and increased BUN levels indicate fluid volume deficit. Spasticity, flaccidity, and tetany are unrelated to water intoxication.

The nurse is planning the care of a client diagnosed with Addison's disease. Whichinterventions should be included? 1. Administer steroid medications. 2. Place the client on fluid restriction. 3. Provide frequent stimulation. 4. Consult physical therapy for gait training.

1 Clients diagnosed with Addison's disease have adrenal gland hypofunction. The hormones normally produced by the gland must be replaced. Steroids and androgens are produced by the adrenal gland

Before discharge, what should a client with Addison's disease be instructed to do when exposed to periods of stress? 1. Administer hydrocortisone I.M. 2. Drink 8 oz of fluids. 3. Perform capillary blood glucose monitoring four times daily. 4. Continue to take his usual dose of hydrocortisone.

1 Clients with Addison's disease and their family members should know how to administer I.M. hydrocortisone during periods of stress. It's important to keep well hydrated during stress, but the critical component in this situation is to know how and when to use I.M. hydrocortisone. Capillary blood glucose monitoring isn't indicated in this situation because the client doesn't have diabetes mellitus. Hydrocortisone replacement doesn't cause insulin resistance.

When teaching a client with Cushing's syndrome about dietary changes, the nurse should instruct the client to increase intake of: 1. fresh fruits. 2. dairy products. 3. processed meats. 4. cereals and grains.

1 Cushing's syndrome causes sodium retention, which increases urinary potassium loss. Therefore, the nurse should advise the client to increase intake of potassium-rich foods, such as fresh fruit. The client should restrict consumption of dairy products, processed meats, cereals, and grains because they contain significant amounts of sodium.

The nurse is performing discharge teaching for a client diagnosed with Cushing'sdisease. Which statement made by the client demonstrates an understanding of theinstructions? 1. "I will be sure to notify my health-care provider if I start to run a fever." 2. "Before I stop taking the prednisone, I will be taught how to taper it off." 3. "If I get weak and shaky, I need to eat some hard candy or drink some juice." 4. "It is fine if I continue to participate in weekend games of tackle football."

1 Cushing's syndrome/disease predisposes the client to develop infections as a result of the immunosuppressive nature of the disease.

Which outcome indicates that treatment of a client with diabetes insipidus has been effective? 1. Fluid intake is less than 2,500 ml/day. 2. Urine output measures more than 200 ml/hr. 3. Blood pressure is 90/50 mm Hg. 4. Heart rate is 126 beats/min.

1 Diabetes insipidus is characterized by polyuria (up to 8 L/day), constant thirst, and an unusually high oral intake of fluids. Treatment with the appropriate drug should decrease both oral fluid intake and urine output. A urine output of 200 ml/hr indicates continuing polyuria. A blood pressure of 90/50 mm Hg and a heart rate of 126 beats/min indicate compensation for the continued fluid deficit, suggesting that treatment hasn't been effective.

A nurse notes 2+ bilateral edema in the lower extremities of a client with myocardial infarction who was admitted 2 days ago. The nurse would plan to do which of the following next? 1.Review the intake and output records for the last 2 days 2.Change the time of diuretic administration from morning to evening 3.Request a sodium restriction of 1 g/day from the physician. 4.Order daily weights starting the following morning.

1 Edema, the accumulation of excess fluid in the interstitial spaces, can be measured by intake greater than output and by a sudden increase in weight. Diuretics should be given in the morning whenever possible to avoid nocturia. Strict sodium restrictions are reserved for clients with severe symptoms.

A health care provider has prescribed propylthiouracil (PTU) for a client with hyperthyroidism, and the nurse assists in developing a plan of care for the client. A priority nursing measure to be included in the plan regarding this medication is to monitor the client for: 1. Signs and symptoms of hypothyroidism 2. Signs and symptoms of hyperglycemia 3. Relief of pain 4. Signs of renal toxicity

1 Excessive dosing with propylthiouracil may convert the client from a hyperthyroid state to a hypothyroid state. If this occurs, the dosage should be reduced. Temporary administration of thyroid hormone may be required. Propylthiouracil is not used for pain and does not cause hyperglycemia or renal toxicity.

A client with a history of hypertension is diagnosed with primary hyperaldosteronism. This diagnosis indicates that the client's hypertension is caused by excessive hormone secretion from which gland? 1. Adrenal cortex 2. Pancreas 3. Adrenal medulla 4. Parathyroid

1 Excessive secretion of aldosterone in the adrenal cortex is responsible for the client's hypertension. This hormone acts on the renal tubule, where it promotes reabsorption of sodium and excretion of potassium and hydrogen ions. The pancreas mainly secretes hormones involved in fuel metabolism. The adrenal medulla secretes the catecholamines — epinephrine and norepinephrine. The parathyroids secrete parathyroid hormone.

The nurse identifies the client problem "risk for imbalanced body temperature" fort he client diagnosed with hypothyroidism. Which intervention should be included in the plan of care? 1. Discourage the use of an electric blanket. 2. Assess the client's temperature every two (2) hours. 3. Keep the room temperature cool. 4. Space activities to promote rest.

1 External heat sources (heating pads,electric or warming blankets) should be discouraged because they increase the risk of peripheral vasodilation and vascular collapse.

A client with Addison's disease asks the nurse how a newly prescribed medication, fludrocortisone acetate (Florinef), will improve the condition. When formulating a response, the nurse should incorporate that a key action of this medication is to: 1. Help restore electrolyte balance. 2. Make the body produce more cortisol. 3. Replace insufficient circulating estrogens. 4. Alter the body's immune system functioning.

1 Fludrocortisone acetate is a long-acting oral medication with mineralocorticoid and moderate glucocorticoid activity. It is prescribed for the long-term management of Addison's disease. Mineralocorticoids cause renal reabsorption of sodium and chloride ions and the excretion of potassium and hydrogen ions. These actions help restore electrolyte balance in the body. The other options are incorrect.

A nurse is monitoring a client following a thyroidectomy for signs of hypocalcemia. Which of the following signs, if noted in the client, likely indicates the presence of hypocalcemia? 1. Tingling around the mouth 2. Negative Chvostek's sign 3. Flaccid paralysis 4. Bradycardia

1 Following a thyroidectomy, the nurse assesses the client for signs of hypocalcemia and tetany. Early signs include tingling around the mouth and fingertips, muscle twitching or spasms, palpitations or dysrhythmias, and positive Chvostek's and Trousseau's signs. Options 2, 3, and 4 are not signs of hypocalcemia.

The nurse is caring for a client immediately after removal of the endotracheal tube. The nurse should report which sign immediately if experienced by the client? 1.Stridor 2.Occasional pink-tinged sputum 3.Respiratory rate of 24 breaths/minute 4.A few basilar lung crackles on the right

1 Following removal of the endotracheal tube the nurse monitors the client for respiratory distress. The nurse reports stridor to the health care provider (HCP) immediately. This is a high-pitched, coarse sound that is heard with the stethoscope over the trachea. Stridor indicates airway edema and places the client at risk for airway obstruction. Although the findings identified in the remaining options require monitoring, they do not require immediate notification of the health care provider.

A nurse enters the room of a patient with a left-sided pneumothorax to perform an afternoon assessment. The nurse finds the patient's trachea deviated slightly to the right side compared to her morning findings, and the patient reports feeling increasingly short of breath. What is the first action the nurse should take? 1) Administer high-flow supplemental oxygen. 2) Position the patient's HOB at 30-45 degrees. 3) Call the physician. 4) Document the extent of tracheal shift in the patient's chart and reassess in 15 minutes.

1 High-flow supplemental oxygen should be administered immediately to offset the unavoidable result of hypoxemia. The nurse should also ensure a POX is applied. Adjusting the HOB will not adequately relieve the patient's shortness of breath.

A nurse is caring for a client with a diagnosis of hypoparathyroidism. The nurse reviews the laboratory results drawn on the client and notes that the calcium level is extremely low. The nurse would expect to note which of the following on data collection of the client? 1. Positive Trousseau's sign 2. Negative Chvostek's sign 3. Unresponsive pupils 4. Hyperactive bowel sounds

1 Hypoparathyroidism is related to a lack of parathyroid hormone secretion or to a decreased effectiveness of parathyroid hormone on target tissues. The end result of this disorder is hypocalcemia. When serum calcium levels are critically low, the client may exhibit positive Chvostek's and Trousseau's signs, which indicate potential tetany. Options 2, 3, and 4 are not related to the presence of hypocalcemia.

A nurse is collecting data on a client with a diagnosis of hypothyroidism. Which of these behaviors, if present in the client's history, would the nurse determine as being likely related to the manifestations of this disorder? 1. Depression 2. Nervousness 3. Irritability 4. Anxiety

1 Hypothyroid clients experience a slow metabolic rate, and its manifestation includes apathy, fatigue, sleepiness, and depression. Options 2, 3, and 4 identify the clinical manifestations of hyperthyroidism.

A client is diagnosed with the syndrome of inappropriate antidiuretic hormone (SIADH). The nurse should anticipate which laboratory test result? 1. Decreased serum sodium level 2. Decreased serum creatinine level 3. Increased hematocrit 4. Increased blood urea nitrogen (BUN) level

1 In SIADH, the posterior pituitary gland produces excess antidiuretic hormone (vasopressin), which decreases water excretion by the kidneys. This, in turn, reduces the serum sodium level, causing hyponatremia. In SIADH, the serum creatinine level isn't affected by the client's fluid status and remains within normal limits. Typically, the hematocrit and BUN level decrease.

The physician orders laboratory tests to confirm hyperthyroidism in a client with classic signs and symptoms of this disorder. Which test result would confirm the diagnosis? 1. No increase in the thyroid-stimulating hormone (TSH) level after 30 minutes during the TSH stimulation test 2. A decreased TSH level 3. An increase in the TSH level after 30 minutes during the TSH stimulation test 4. Below-normal levels of serum triiodothyronine (T3) and serum thyroxine (T4) as detected by radioimmunoassay

1 In the TSH test, failure of the TSH level to rise after 30 minutes confirms hyperthyroidism. A decreased TSH level indicates a pituitary deficiency of this hormone. Below-normal levels of T3 and T4, as detected by radioimmunoassay, signal hypothyroidism. A below-normal T4 level also occurs in malnutrition and liver disease and may result from administration of phenytoin and certain other drugs.

A client with hypoparathyroidism has hypocalcemia. The nurse avoids giving the client the prescribed vitamin and calcium supplement with which of the following liquids? 1. Milk 2. Water 3. Iced tea 4. Fruit juice

1 Milk products are high in phosphates, which should be avoided by a client with hypoparathyroidism. Otherwise, calcium products are best absorbed with milk because the vitamin D in the milk promotes calcium absorption.

Following a unilateral adrenalectomy, the nurse would assess for hyperkalemia as indicated by: 1. muscle weakness. 2. tremors. 3. diaphoresis. 4. constipation.

1 Muscle weakness, bradycardia, nausea, diarrhea, and paresthesia of the hands, feet, tongue, and face are findings associated with hyperkalemia, which is transient and occurs from transient hypoaldosteronism when the adenoma is removed. Tremors, diaphoresis, and constipation aren't seen in hyperkalemia.

Which statement by the client would cause the nurse to suspect that the thyroid test results drawn on the client this morning may be inaccurate? 1. "I had a radionuclide test done 3 days ago." 2. "When I exercise I sweat more than normal." 3. "I drank some water before the blood was drawn." 4. "That hamburger I ate before the test sure tasted good."

1 Option 1 indicates that a recent radionuclide scan had been performed. Recent radionuclide scans performed before the test can affect thyroid laboratory results. No food, fluid, or activity restrictions are required for this test, so options 2, 3, and 4 are incorrect.

A nurse is preparing to discharge a client who has had a parathyroidectomy. When teaching the client about the prescribed oral calcium supplement, what information should the nurse include? 1. Take the calcium 30 to 60 minutes following a meal. 2. Avoid sunlight because it can cause skin color change. 3. Store the calcium in the refrigerator to maintain potency. 4. Check the pulse daily and hold the dosage if it is below 60 beats per minute.

1 Oral calcium supplements can be taken 30 to 60 minutes after meals to enhance their absorption and decrease gastrointestinal irritation. All the other options are unrelated to oral calcium therapy.

The nurse is preparing to administer A.M. medications to the following clients. Which medication should the nurse question before administering? 1. Pancreatic enzymes to the client who has finished breakfast. 2. The pain medication, morphine, to the client who has a respiratory rate of 20. 3. The loop diuretic to the client who has a serum potassium level of 3.9 mEq/L. 4. The beta blocker to the client who has an apical pulse of 68 bpm.

1 Pancreatic enzymes must be administered with meals to enhance the digestion of starches and fats in the gastrointestinal tract.

The 68-year-old client diagnosed with hyperthyroidism is being treated with radio active iodine therapy. Which interventions should the nurse discuss with the client? 1. Explain it will take up to a month for symptoms of hyperthyroidism to subside. 2. Teach the iodine therapy will have to be tapered slowly over one (1) week. 3. Discuss the client will have to be hospitalized during the radioactive therapy. 4. Inform the client after therapy the client will not have to take any medication.

1 Radioactive iodine therapy is used to destroy the overactive thyroid cells. After treatment, the client is followed closely for three (3) to four (4) weeks until the euthyroid state is reached.

A patient who is being treated with topical mafenide acetate for third-degree burns is demonstrating facial and neck edema. The nurse realizes that this patient most likely 1. is developing a hypersensitivity to the medication. 2. is reacting positively to the medication. 3. needs an increase in dosage of the medication. 4. is not responding to the medication.

1 Rationale: Approximately 3%-5% of patients develop a hypersensitivity to mafenide, which can manifest as facial edema. The manifestation of facial and neck edema is considered an adverse reaction. There is inadequate information presented to assess response to the medication.

A patient has experienced a burn injury. Which of the following interventions by the nurse is of the highest priority at this time? 1. determination of the type of burn injury 2. determination of the types of home remedies attempted prior to the patient's coming to the hospital 3. assessment of past medical history 4. determination of body weight 5. determination of nutritional status

1 Rationale: Determination of the type of burn is the first step. The type of injury will dictate the interventions performed. Determining the use of home remedies, past medical history, body weight, and nutritional status must be completed, but are not of the highest priority.

A patient with third-degree burns is being treated with high-volume intravenous fluids and has a urine output of 40 cc per hour. The nurse realizes that this urine output 1. is normal for this patient. 2. provides evidence that the patient is dehydrated. 3. provides evidence that the patient is over-hydrated. 4. is indicative of pending renal failure.

1 Rationale: Intake and output measurements indicate the adequacy of fluid resuscitation, and should range from 30 to 50 mL per hour in an adult.

A patient recovering from a major burn injury is complaining of pain. Which of the following medications will be most therapeutic to the patient? 1. morphine 4 mg IV every 5 minutes 2. morphine 10 mg IM ever 3-4 hours 3. meperidine 75 mg IM every 3-4 hours 4. meperidine 50 mg PO every 3-4 hours 5. fentanyl citrate (Duragesic) 75 mcg patch every 3 days

1 Rationale: Morphine is preferred over meperidine for the burn-injured patient. Typical dose of morphine is 3-5 mg every 5-10 minutes for an adult. The intravenous route is preferred over oral and intramuscular routes. A transdermal patch would not be used because of decreased absorption of the medication through the skin of the burn-injured patient.

When monitoring the vital signs of the patient who has experienced a major burn injury, the nurse assesses a heart rate of 112 and a temperature of 99.9° F. Which of the following best describes the findings? 1. These values are normal for the patient's post-burn injury condition. 2. The patient is demonstrating manifestations consistent with the onset of an infection. 3. The patient is demonstrating manifestations consistent with an electrolyte imbalance. 4. The patient is demonstrating manifestations consistent with renal failure. 5. The patient is demonstrating manifestations of fluid volume overload.

1 Rationale: The burn-injured patient is not considered tachycardic until the heart rate reaches 120 beats per minute. In the absence of other symptoms, the temperature does not signal the presence of an infection. It could be a response to a hypermetabolic response.

A patient is scheduled for surgery to graft a burn injury on the arm. Which of the following statements should the nurse include when instructing the patient prior to the procedure? 1. "You will begin to perform exercises to promote flexibility and reduce contractures after five days." 2. "You will need to report any itching, as it might signal infection." 3. "Performing the procedure near the end of the hospitalization will reduce the incidence of infection and improve success of the procedure." 4. "The procedure will be performed in your room." 5. "You will need to be in protective isolation for several weeks after the graft is performed."

1 Rationale: The patient will begin to perform range-of-motion exercises after five days. Itching is not a symptom of infection but an anticipated occurrence that signals cellular growth. The ideal time to perform the procedure is early in the treatment of the burn injury. The procedure is performed in a surgical suite. Patients with skin grafts do not require protective isolation.

A patient receiving treatment for severe burns over more than half of his body has an indwelling urinary catheter. When evaluating the patient's intake and output, which of the following should be taken into consideration? 1. The amount of urine will be reduced in the first 24-48 hours, and will then increase. 2. The amount of urine output will be greatest in the first 24 hours after the burn injury. 3. The amount of urine will be reduced during the first eight hours of the burn injury and will then increase as the diuresis begins. 4. The amount of urine will be elevated due to the amount of intravenous fluids administered during the initial phases of treatment. 5. The amount of urine is expected to be decreased for three to five days.

1 Rationale: The patient will have an initial reduction in urinary output. Fluid is reduced in the initial phases as the body manages the insult caused by the injury and fluids are drawn into the interstitial spaces. After the shock period passes, the patient will enter a period of diuresis. The diuresis begins between 24 and 36 hours after the burn injury.

A patient with third-degree burns to her face just learned that she will have extensive scarring once the burn heals. Which of the following nursing diagnoses would be applicable to this patient at this time? 1. Powerlessness 2. Potential for Infection 3. Fluid Volume Deficit 4. Risk for Ineffective Airway Clearance

1 Rationale: This patient can begin to experience powerlessness in that she has no control over the outcome of healing on scar formation to her face. The nurse should allow the patient to express feelings in efforts to help the patient cope with the news of potential scarring. The patient with a third-degree burn is at risk for infection, however, this question is focused on the impact of her facial scarring. There is inadequate information to determine the patient's risk for fluid volume deficit or ineffective airway clearance. Further, this is not the focus of the question.

The adrenal cortex is responsible for producing which substances? 1. Glucocorticoids and androgens 2. Catecholamines and epinephrine 3. Mineralocorticoids and catecholamines 4. Norepinephrine and epinephrine

1 The adrenal glands have two divisions, the cortex and medulla. The cortex produces three types of hormones: glucocorticoids, mineralocorticoids, and androgens. The medulla produces catecholamines — epinephrine and norepinephrine.

The client is diagnosed with diabetes insipidus. Which laboratory value should bemonitored by the nurse? 1. Serum sodium. 2. Serum calcium 3. Urine glucose. 4. Urine white blood cells.

1 The client will have an elevated sodium level as a result of low circulating blood volume. The fluid is being lost through the urine. Diabetes means "to pass through" inGreek, indicating polyuria, a symptom shared with diabetes mellitus. Diabetes insipidus is a totally separate disease process.

A nurse has reinforced dietary instructions to a client with a diagnosis of hypoparathyroidism. The nurse instructs the client to include which of the following items in the diet? 1. Vegetables 2. Meat 3. Fish 4. Cereals

1 The client with hypoparathyroidism is instructed to follow a calcium-rich diet and to restrict the amount of phosphorus in the diet. The client should limit meat, poultry, fish, eggs, cheese, and cereals. Vegetables are allowed in the diet.

An older client with a history of hyperparathyroidism and severe osteoporosis is hospitalized. The nurse caring for the client plans first to address which problem? 1. The possibility of injury 2. Constipation 3. Urinary retention 4. Need for teaching about the disorder

1 The client with severe osteoporosis as a result of hyperparathyroidism is at risk for injury as a result of pathological fractures that can occur from bone demineralization. The client may also have a risk for constipation from the disease process but this is a lesser priority than client safety. The client may or may not have urinary elimination problems, depending on other factors in the client's history. There is no information in the question to support whether the client needs teaching.

The client diagnosed with a myocardial infarction asks the nurse, "why do I have to rest and take it easy? My chest doesn't hurt anymore." Which statement would be the nurse's best response? 1. "Your heart is damaged and needs about 4 to 6 weeks to heal" 2. "There is necrotic myocardial tissue that puts you at risk for dysrhythmias" 3. "Your doctor has ordered bedrest. Therefore, you must stay in bed." 4. "Just because your chest doesn't hurt anymore doesn't mean you are out of danger"

1 The heart tissue is dead, stress or activity may cause heart failure, and it does take about 6-8 weeks for scar tissue to form

Following a transsphenoidal hypophysectomy, the nurse should assess the client carefully for which condition? 1. Hypocortisolism 2. Hypoglycemia 3. Hyperglycemia 4. Hypercalcemia

1 The nurse should assess for hypocortisolism. Abrupt withdrawal of endogenous cortisol may lead to severe adrenal insufficiency. Steroids should be given during surgery to prevent hypocortisolism from occurring. Signs of hypocortisolism include vomiting, increased weakness, dehydration and hypotension. After the corticotropin-secreting tumor is removed, the client shouldn't be at risk for hyperglycemia. Calcium imbalance shouldn't occur in this situation.

Which important instruction concerning the administration of levothyroxine (Synthroid) should the nurse teach a client? 1. "Take the drug on an empty stomach." 2. "Take the drug with meals." 3. "Take the drug in the evening." 4. "Take the drug whenever convenient."

1 The nurse should instruct the client to take levothyroxine on an empty stomach (to promote regular absorption) in the morning (to help prevent insomnia and to mimic normal hormone release).

A young woman makes an appointment to see a physician at the clinic. She complains of tiredness, weight gain, muscle aches and pains, and constipation. The physician will likely order: 1. T3 and T4 serum level laboratory tests. 2. glucose tolerance test. 3. cerebral computed tomography (CT) scan. 4. adrenocortical stimulating test.

1 These complaints are strongly suggestive of thyroid disorder; T3 and T4 laboratory tests are the most useful diagnostic tests.

the nurse is developing a plan of care for a client who has a dx of acute pancreatitis. the rationale for maintaining the client on bed rest is to: 1. reduce pancreatic and gastric secretions 2. minimize the effects of hypoglycemia 3. reduce the risk of DVT 4. decrease the likelihood of orthostatic hypotension

1 bed rest decreases body metabolism and thus reduces pancreatic and gastric secretions in the client with acute pancreatitis

the nurse is administering a pancreatic enzyme to the client dx with chronic pancreatitis. which statement best explains the rationale for administering this med? 1. it is an exogenous source of protease, amylase, and lipase 2. this enzyme increases the number of bowel movements 3. this medication breaks down in the stomach to help with digestion 4. pancreatic enzymes help break down fat in the small intestine

1 pancreatic enzymes enhance the digestion of starches (carbohydrates) in the gastrointestinal tract by supplying an exogenous (outside) source of the pancreatic enzymes protease, amylase and lipase

after a liver biopsy the nurse should place the client in which position? 1. right side 2. left side 3. high fowlers 4. trendelenburg

1 when the client is placed on the right side, the biopsy site of the liver capsule is compressed against the chest wall, and the escape of blood or bile is impeded

A 25-year-old patient is admitted with partial-thickness injuries over 20% of the total body surface area involving both lower legs. The nurse would classify this injury as being which of the following? 1. a moderate burn 2. a minor burn 3. a major burn 4. a severe burn 5. an intermediate burn

1 Rationale 1: A moderate burn is a partial-thickness injury that is between 15%-25% of total body surface area in adults.

A patient has sustained a partial-thickness injury of 28% of total body surface area (TBSA) and full-thickness injury of 30% or greater of TBSA. How should the nurse classify this burn injury? 1. major 2. moderate 3. minor 4. superficial 5. intermediate

1 Rationale 1: Partial-thickness injuries of greater than 25% of total body surface area in adults and full-thickness injuries 10% or greater of TBSA are considered major burns.

A patient with third-degree burns is prescribed gastrointestinal medication. The primary action of this drug is which of the following? 1. to prevent the onset of a Curling's ulcer 2. to treat a preexisting duodenal ulcer 3. to ensure adequate peristalsis 4. for the antiemetic properties

1 Rationale: Dysfunction of the gastrointestinal system is directly related to the size of the burn wound. This can lead to a cessation of intestinal motility, which causes gastric distention, nausea, vomiting, and hematemesis. Stress ulcers or Curling's ulcers are acute ulcerations of the stomach or duodenum that form following the burn injury. There is no evidence to support the presence of a preexisting duodenal ulcer. Although peristalsis is desired, it is not the primary area of gastrointestinal concern. There is no data presented to indicate the presence of nausea or vomiting.

A 70-year-old patient has experienced a sunburn over much of the body. What self-care technique is MOST important to emphasize to an older adult in dealing with the effects of the sunburn? 1. increasing fluid intake 2. applying mild lotions 3. taking mild analgesics 4. maintaining warmth 5. using sunscreen

1 Rationale: Older adults are especially prone to dehydration; therefore, increasing fluid intake is especially important. Other manifestations could include nausea and vomiting. All the measures help alleviate the manifestations of this minor burn which include pain, skin redness, chills, and headache. Use of sunscreen is a preventative, not a treatment measure.

The nurse notes that a patient with third-degree burns is demonstrating a reduction in his serum potassium level. The nurse realizes that this finding is consistent with which of the following? 1. the resolution of burn shock 2. the onset of burn shock 3. the onset of renal failure 4. the onset of liver failure

1 Rationale: Potassium levels are initially elevated during burn shock but will decrease after burn shock resolves as fluid shifts back to intracellular and intravascular compartments. Reduced potassium levels are not indicators of the onset of renal or liver failure.

A patient, experiencing a burn that is pale and waxy with large flat blisters, asks the nurse about the severity of the burn and how long it will take to heal. With which of the following should the nurse respond to this patient? 1. The wound is a deep partial-thickness burn, and will take more than three weeks to heal. 2. The wound is a partial-thickness burn, and could take up to two weeks to heal. 3. The wound is a superficial burn, and will take up to three weeks to heal. 4. The wound is a full-thickness burn and will take one to two weeks to heal. 5. Wound healing is individualized.

1 Rationale: The wound described is a deep partial-thickness burn. Deep partial-thickness wounds will take more than three weeks to heal. A superficial burn is bright red and moist, and might appear glistening with blister formation. The healing time for this type of wound is within 21 days. A full thickness burn involves all layers of the skin and may extend into the underlying tissue. These burns take many weeks to heal. Stating that wound healing is individualized does not answer the patient's question about the severity of the burn.

A patient is being evaluated after experiencing severe burns to his torso and upper extremities. The nurse notes edema at the burned areas. Which of the following best describes the underlying cause for this assessment finding? 1. inability of the damaged capillaries to maintain fluids in the cell walls 2. reduced vascular permeability at the site of the burned area 3. decreased osmotic pressure in the burned tissue 4. increased fluids in the extracellular compartment 5. the IV fluid being administered too quickly

1 Rationale: Burn shock occurs during the first 24-36 hours after the injury. During this period, there is an increase in microvascular permeability at the burn site. The osmotic pressure is increased, causing fluid accumulation. There is a reduction of fluids in the extracellular body compartments. Manifestations of fluid volume overload would be systemic, not localized to the burn areas.

A nurse sees a patient get struck by lightning during a thunder storm on a golf course. What should be the FIRST action by the nurse? 1. Check breathing and circulation. 2. Look for entrance and exit wounds. 3. Cover the patient to prevent heat loss. 4. Move the patient indoors to a dry place. 5. Get the patient up off the ground.

1 Rationale: Cardiopulmonary arrest is the most common cause of death from lightening. Respiratory and cardiac status should be assessed immediately to determine if CPR is necessary. All other actions are secondary.

When evaluating the laboratory values of the burn-injured patient, which of the following can be anticipated? 1. decreased hemoglobin and elevated hematocrit levels 2. elevated hemoglobin and elevated hematocrit levels 3. elevated hemoglobin and decreased hematocrit levels 4. decreased hemoglobin and decreased hematocrit levels 5. hemoglobin and hematocrit levels within normal ranges

1 Rationale: Hemoglobin levels are reduced in response to the hemolysis of red blood cells. Hematocrit levels are elevated secondary to hemoconcentration, and fluid shifts from the intravascular compartment.

Following surgical debridement, a patient with third-degree burns does not bleed. The nurse realizes that this patient 1. will need to have the procedure repeated. 2. will no longer need this procedure. 3. will need to be premedicated prior to the next procedure. 4. should have an escharotomy instead.

1 Rationale: Surgical debridement is the process of excising the burn wound by removing thin slices of the wound to the level of viable tissue. If bleeding does not occur after the procedure, it will be repeated. It is an assumption that patients having debridement will all require premedication. An escharotomy involves removal of the hardened crust covering the burned area.

The client diagnosed with appendicitis has undergone an appendectomy. At two hours postoperative, the nurse takes the vital signs and notes T 102.6 F, P 132, R 26, and BP 92/46. Which interventions should the nurse implement? List in order of priority. 1. Increase the IV rate. 2. Notify the health care provider. 3. Elevate the foot of the bed. 4. Check the abdominal dressing. 5. Determine if the IV antibiotics have been administered.

1, 3, 4, 5, 2 1. The nurse should increase the IV rate to maintain the circulatory system function until further orders can be obtained.3. The foot of the bed should be elevated to help treat shock, the symptoms of which include elevated pulse and decreased BP. Those signs and an elevated temperature indicate an infection may be present and the client could be developing septicemia.4. The dressing should be assessed to determine if bleeding is occurring.5. The nurse should administer any IV antibiotics ordered after addressing hypovolemia. The nurse will need this information when reporting to the HCP.2. The HCP should be notified when the nurse has the needed information.

A patient has a scald burn on the arm that is bright red, moist, and has several blisters. The nurse would classify this burn as which of the following? Select all that apply. 1. a superficial partial-thickness burn 2. a thermal burn 3. a superficial burn 4. a deep partial-thickness burn 5. a full-thickness burn

12 Rationale: Superficial partial-thickness burn if often bright red, has a moist, glistening appearance and blister formation. Thermal burns result from exposure to dry or moist heat. A superficial burn is reddened with possible slight edema over the area. A deep partial-thickness burn often appears waxy and pale and may be moist or dry. A full-thickness burn may appear pale, waxy, yellow, brown, mottled, charred, or non-blanching red with a dry, leathery, firm wound surface.

The nurse is reviewing the results of laboratory tests to assess the renal status of a patient who experienced a major burn event on 45% of the body 24 hours ago. Which of the following results would the nurse expect to see? (Select all that apply) 1. glomerular filtration rate (GFR) reduced 2. specific gravity elevated 3. creatinine clearance reduced 4. BUN reduced 5. uric acid decreased

12 Rationale: During the initial phases of a burn injury, blood flow to the renal system is reduced, resulting in reduction in GFR and an increase in specific gravity. During this period, BUN levels, creatinine, and uric acid are increased

A patient with a burn injury is prescribed silver nitrate. Which of the following nursing interventions should be included for the patient? Standard Text: Select all that apply. 1. Monitor daily weight. 2. Monitor the serum sodium levels. 3. Prepare to change the dressings every two hours. 4. Report black skin discolorations. 5. Push fluid intake.

12 Rationale: Silver nitrate can cause hypotonicity. Manifestations of hypotonicity include weight gain and edema, which can be monitored by the determination of daily weights. Hyponatremia and hypochloremic alkalosis are common findings in patients treated with silver nitrate so serum sodium and chloride should be monitored. Changing the dressing every two hours is too frequent for the patient. Black discolorations of the skin are anticipated for patients using silver nitrate, and do not highlight a complication of therapy. Silver sulfadiazine, not silver nitrate, administration can result in the development of sulfa crystals in the urine so pushing fluid intake is not an appropriate action for this patient.

A patient comes into the emergency department with a chemical burn from contact with lye.Assessment and treatment of this patient will be based on what knowledge regarding this type of burn? (Select all that apply) 1. This is an alkali burn. 2. This type of burn tends to be deeper. 3. This is an acid burn. 4. This type of burn will be easier to neutralize. 5. This type of burn tends to be more superficial.

12 Rationale: This is an alkali burn which is more difficult to neutralize than an acid burn and tends to have a deeper penetration and be more severe than a burn caused by an acid.

The nurse has just admitted to the nursing unit a client with a basilar skull fracture who is at risk for increased intracranial pressure. Pending specific health care provider prescriptions, the nurse should safely place the client in which positions? Select all that apply. 1.Head midline 2.Neck in neutral position 3.Head of bed elevated 30 to 45 degrees 4.Head turned to the side when flat in bed 5.Neck and jaw flexed forward when opening the mouth

123

The nurse is caring for a client who is mechanically ventilated, and the high-pressure ventilator alarm is sounding. The nurse understands that which complications may cause this alarm? Select all that apply. 1.Water or a kink in the tubing 2.Biting on the endotracheal tube 3.Increased secretions in the airway 4.Disconnection or leak in the system 5.The client stops spontaneous breathing.

123

A patient is being discharged after treatment for a scald burn that caused a superficial burn over one hand and a superficial partial-thickness burn on several fingers. What should be included in this patient's discharge instructions? (Select all that apply) 1. Report any fever to your healthcare provider. 2. Report development of purulent drainage to your healthcare provider. 3. Use only sterile dressings on the fingers. 4. Cleanse the areas every hour with alcohol to prevent infection. 5. Apply the topical antimicrobial agent as instructed.

123 Rationale: Fever or purulent drainage are indicative of development of infection and should be reported to the healthcare provider. Sterile dressings only should be used on the areas of the superficial partial-thickness burns where the skin is not intact. Cleansing is necessary no more often than daily to the intact skin areas and only soap and water should be used, not alcohol. Topical agents may be ordered by the health care provider and the patient should follow directions for applying to help prevent infection of the areas.

A patient arrives at the emergency department with an electrical burn. What assessment questions should the nurse ask in determining the possible severity of the burn injury? Select all that apply. 1. What type of current was involved? 2. How long was the patient in contact with the current? 3. How much voltage was involved? 4. Where was the patient when the burn occurred? 5. What was the point of contact with the current?

123 Rationale: The severity of electrical burns depends on the type and duration of the current and amount of voltage. Location is not important in determining possible severity. Location is not important in determining possible severity.

A nurse is teaching a class of older adults at a senior center about household cleaning agents that may cause burns. Which agents should be included in these instructions? (Select all that apply) 1. drain cleaners 2. household ammonia 3. oven cleaner 4. toiler bowl cleaner 5. lemon oil furniture polish

1234 Rationale: All of the products except for the furniture polish can cause burns since they are either alkalis or acids.

In order for the nurse to correctly classify a burn injury, which of the following does the nurse need to assess? Select all that apply. 1. the depth of the burn 2. extent of burns on the body 3. the causative agent and the duration of exposure. 4. location of burns on the body 5. the time that the burn occurred

1234 Rationale: Depth of the burn (the layers of underlying tissue affected) and extent of the burn (the percentage of body surface area involved) are used in determining the amount of tissue damage and classification of the burn.The causative agent is especially important with chemical burns such as from strong acids or alkaline agents. The location of the burns on the body is one of the important determinates of classification. For example, burns of the face and hands are always considered major burns. Time of occurrence of the burn is not necessary for classification.

During the acute phase of burn treatment, important goals of patient care include which of the following? Select all that apply. 1. providing for patient comfort 2. preventing infection 3. providing adequate nutrition for healing to occur 4. splinting, positioning, and exercising affected joints 5. assessing home maintenance management

1234 Rationale: The goals of treatment for the acute period include wound cleansing and healing; pain relief; preventing infection; promoting nutrition; and splinting, positioning, and exercising affected joints. Assessment of home maintenance management is an important goal in the rehabilitative stage, not the acute stage.

The nurse is teaching the client diagnosed with hyperthyroidism. Which information should be taught to the client? Select all that apply. 1. Notify the HCP if a three (3)-pound weight loss occurs in two (2) days. 2. Discuss ways to cope with the emotional lability. 3. Notify the HCP if taking over-the-counter medication 4. Carry a medical identification card or bracelet. 5. Teach how to take thyroid medications correctly.

1234 Weight loss indicates the medication may not be effective and will probably need to be increased. The client needs to know emotional highs and lows are secondary to hyperthyroidism. With treatment, this emotional lability will subside. Any over-the-counter medications (for example, alcohol-based medications) may negatively affect the client's hyperthyroidism or medications being used for treatment. This will help any HCP immediately know of the client's condition, especially if the client is unable to tell the HCP. The client will be on anti thyroid medication not thyroid medication

The nurse is teaching the client diagnosed with hyperthyroidism. Which informationshould be taught to the client? Select all that apply. 1. Notify the HCP if a three (3)-pound weight loss occurs in two (2) days. 2. Discuss ways to cope with the emotional lability. 3. Notify the HCP if taking over-the-counter medication. 4. Carry a medical identification card or bracelet. 5. Teach how to take antithyroid medications correctly.

12345

The client diagnosed with hyperthyroidism is prescribed an antithyroid medication. Which interventions should the nurse implement? Select all that apply. 1. Monitor the client's thyroid function tests. 2. Monitor the client's weight weekly. 3. Monitor the client for gastrointestinal distress. 4. Monitor the client's vital signs. 5. Monitor the client for activity intolerance.

12345 Thyroid function tests are used to determine the effectiveness of drug therapy. Weight gain is expected as a result of a slower metabolism. Antithyroid medication may cause nausea or vomiting. Changes in metabolic rate will be manifested as changes in blood pressure, pulse, and body temperature. Hyperthyroidism results in protein catabolism, overactivity, and increased metabolism, which lead to exhaustion; therefore, the nurse should monitor for activity intolerance.

The nurse is assessing a child diagnosed with a brain tumor. Which of the following signs and symptoms would the nurse expect the child to demonstrate? Select all that apply. 1. Head tilt 2. Vomiting 3. Polydipsia 4. Lethargy 5. Increased appetite 6. Increased pulse

124

The nurse is planning the care of a client diagnosed with syndrome of inappropriate antidiuretic hormone (SIADH). Which interventions should be implemented? Select all that apply. 1. Restrict fluids per health-care provider order. 2. Assess level of consciousness every two (2) hours. 3. Provide atmosphere of stimulation. 4. Monitor urine and serum osmolality. 5. Weigh the client every three (3) days.

124 Fluids are restricted to 500-600 mL per 24hours. Orientation to person, place, and times should be assessed every two (2) hours or more often. A safe environment, not a stimulating one, is provided. Urine and serum osmolality are monitored to determine fluid volume status. The client should be weighed daily.

An experienced LPN/LVN, under the supervision of the team leader RN, is assigned to provide nursing care for a patient with a respiratory problem. Which actions are appropriate to the scope of practice of an experienced LPN/LVN? Select all that apply. 1.) Auscultating breath sounds 2.) Administering medications via metered-dose inhaler (MDI) 3.) Completing in-depth admission assessment 4.) Checking oxygen saturation using pulse oximetry 5.)Developing the nursing care plan 6.) Evaluating the patient's technique for using MDIs

124 The experienced LPN/LVN is capable of gathering data and making observations, including noting breath sounds and performing pulse oximetry. Administering medications, such as those delivered via MDIs, is within the scope of practice of the LPN/LVN. Independently completing the admission assessment, developing the nursing care plan, and evaluating a patient's abilities require additional education and skills within the scope of practice of the professional RN.

A client who has undergone radical neck dissection for a tumor has a potential problem of obstruction related to postoperative edema, drainage, and secretions. To promote adequate respiratory function in this client, the nurse should implement which activities? Select all that apply. 1.Suctioning the client as needed 2.Encouraging coughing every 2 hours 3.Placing the bed in low Fowler's position 4.Supporting the neck incision when the client coughs 5.Monitoring the respiratory status frequently as prescribed

1245

A nurse notes in the medical record that a client with Cushing's syndrome is experiencing fluid overload. Which interventions should be included in the plan of care? Select all that apply. 1. Monitoring daily weight 2. Monitoring intake and output 3. Maintaining a low-potassium diet 4. Monitoring extremities for edema 5. Maintaining a low-sodium diet

1245 The client with Cushing's syndrome experiencing fluid overload should be maintained on a high-potassium and low-sodium diet. Decreased sodium intake decreases renal retention of sodium and water. Monitoring weight, intake, output, and extremities for edema are all appropriate interventions for such a nursing diagnosis.

A client has been diagnosed with hyperthyroidism. Which signs and symptoms may indicate thyroid storm, a complication of this disorder? Select all that apply. 1.Fever 2.Nausea 3.Lethargy 4.Tremors 5.Confusion 6.Bradycardia

1245 Thyroid storm is an acute and life-threatening condition that occurs in a client with uncontrollable hyperthyroidism. Symptoms of thyroid storm include elevated temperature (fever), nausea, and tremors. In addition, as the condition progresses, the client becomes confused. The client is restless and anxious and experiences tachycardia.

The nurse is instituting seizure precautions for a client who is being admitted from the emergency department. Which measures should the nurse include in planning for the client's safety? Select all that apply. 1. Padding the side rails of the bed 2. Placing an airway at the bedside 3. Placing the bed in the high position 4. Putting a padded tongue blade at the head of the bed 5. Placing oxygen and suction equipment at the bedside 6. Flushing the intravenous catheter to ensure that the site is patent

1256

A nurse is reviewing a plan of care for a client with Addison's disease. The nurse notes that the client is at risk for dehydration and suggests nursing interventions that will prevent this occurrence. Which nursing intervention is an appropriate component of the plan of care? Select all that apply. 1. Encouraging fluid intake of at least 3000 mL/day 2. Encouraging an intake of low-protein foods 3. Monitoring for changes in mental status 4. Monitoring intake and output 5. Maintaining a low-sodium diet

134 The client at risk for deficient fluid volume should be encouraged to eat regular meals and snacks and to increase the intake of sodium, protein, and complex carbohydrates. Oral replacement of sodium losses is necessary, and maintenance of adequate blood glucose levels is required.

The RN is supervising a nursing student who will suction a patient on a mechanical ventilator. Which actions indicate that the student has a correct understanding of this procedure? Select all that apply. 1.) The student nurse uses a sterile catheter and glove. 2.)The student nurse applies suction while inserting the catheter. 3.)The student nurse applies suction during catheter removal. 4.) The student nurses uses a twirling motion when withdrawing the catheter. 5.)The student nurse uses a no. 12 French catheter. 6.)The student nurse applies suction for at least 20 seconds.

1345 The standard size catheter for an adult is a no. 12 or 14 French. Infection is possible because each catheter pass can introduce bacteria into the trachea. In the hospital, use sterile technique for suctioning and for all suctioning equipment (e.g., suction catheters, gloves, saline or water). Apply suction only during catheter withdrawal and use a twirling motion to prevent the catheter from grabbing tracheal mucosa and leading to damage to tracheal tissue. Apply suction for no more than 10 seconds to minimize hypoxemia during suctioning.

the client with an acute exacerbation of chronic pancreatitis has a nasogastric tube. which interventions should the nurse implement? select all that apply 1. monitor the clients bowel sounds 2. monitor the clients food intake 3. assess the clients intravenous site 4. provide oral and nasal care 5. monitor the clients blood glucose

1345 the return of bowel sounds indicates the return of peristalsis, and the nasogastric suction is usually discontinued within 24 to 48 hrs thereafter. the nurse should assess for signs of infection or infiltration. fasting and the ng tube increase the clients risk for mucous membrane irritation and brk dwn. blood glucose levels are monitored because clients with chronic pancreatitis can develop diabetes mellitus

The nurse is creating a plan of care for a child who is at risk for seizures. Which interventions apply if the child has a seizure? Select all that apply. 1. Time the seizure. 2. Restrain the child. 3. Stay with the child. 4. Place the child in a prone position. 5. Move furniture away from the child. 6. Insert a padded tongue blade in the child's mouth.

135

The client diagnosed with Cushing's disease is prescribed alendronate to prevent osteoporosis. Which information should the clinic nurse teach? Select all that apply. 1. Take the medication and sit upright for 30 minutes. 2. Take the medication just before going to bed. 3. Take the medication with an antacid to alleviate gastric disturbances. 4. Take the medication at least 30 minutes before breakfast. 5. Take the medication with a full glass of water.

145

A client who suffered a brain injury after falling off a ladder has recently developed syndrome of inappropriate antidiuretic hormone (SIADH). What findings indicate that the treatment he's receiving for SIADH is effective? (SATA) 1. Decrease in body weight 2. Rise in blood pressure and drop in heart rate 3. Absence of wheezes in the lungs 4. Increase in urine output 5. Decrease in urine osmolarity

145 SIADH is an abnormality involving an abundance of diuretic hormone. The predominant feature is water retention with oliguria, edema, and weight gain. Successful treatment should result in weight reduction, increased urine output, and a decrease in the urine concentration (urine osmolarity).

The nurse is developing a plan of care for a client with Addison's disease. The nurse has identified a problem of risk for deficient fluid volume and identifies nursing interventions that will prevent this occurrence. Which nursing interventions should the nurse include in the plan of care? (SATA) 1. Monitor for changes in mentation. 2. Encourage an intake of low-protein foods. 3. Encourage an intake of low-sodium foods. 4. Encourage fluid intake of at least 3000 mL per day. 5. Monitor vital signs, skin turgor, and intake and output.

145 The client at risk for deficient fluid volume should be encouraged to eat regular meals and snacks and to increase intake of sodium, protein, and complex carbohydrates and fluids. Oral replacement of sodium losses is necessary, and maintenance of adequate blood glucose levels is required. Mentation, vital signs, skin turgor and intake and output should be monitored for signs of fluid volume deficit.

A 75-year-old patient will be undergoing a mitral valve replacement for severe mitral stenosis. When reviewing the patient's chart, which of these infectious diseases would likely be present in this patient's history? 1.Meningitis 2. Rheumatic fever 3. Varicella (chicken pox) 4. Haemophilus influenzae

2

A client is returned to the nursing unit after thoracic surgery with chest tubes in place. During the first few hours postoperatively, what type of drainage should the nurse expect? 1.Serous 2.Bloody 3.Serosanguineous 4.Bloody, with frequent small clots

2

A client with thyrotoxicosis says to the nurse, "I am so irritable. 1 am having problems at work because I lose my temper very easily." Which response by the nurse would give the client the most accurate explanation of this behavior? 1. "Your behavior is caused by temporary confusion brought on by your illness." 2. "Your behavior is your caused by the excess thyroid hormone in your system." 3. "Your behavior is caused by your worrying about the seriousness of your illness." 4. "Your behavior is caused by the stress of trying to manage a career and cope with illness."

2

A health care provider (HCP) tells the nurse that a client's chest tube is to be removed. The nurse should bring which dressing materials to the bedside for the HCP's use? 1.Telfa dressing and Neosporin ointment 2.Petrolatum gauze and sterile 4 × 4 gauze 3.Benzoin spray and a hydrocolloid dressing 4.Sterile 4 × 4 gauze, Neosporin ointment, and tape

2

A nurse is reviewing the record of a child with increased ICP and notes that the child has exhibited signs of decerebrate posturing. On assessment of the child, the nurse would expect to note which of the following if this type of posturing was present? 1. Abnormal flexion of the upper extremities and extension of the lower extremities 2. Rigid extension and pronation of the arms and legs 3. Rigid pronation of all extremities 4. Flaccid paralysis of all extremities

2

A patient receiving propylthiouracil (PTU) asks the nurse how this medication will help relieve his symptoms. What is the nurse's best response? 1. "Propylthiouracil inactivates any circulating thyroid hormone, thus decreasing signs and symptoms of hyperthyroidism." 2. "Propylthiouracil inhibits the formation of new thyroid hormone, thus gradually returning your metabolism to normal." 3. "Propylthiouracil helps your thyroid gland use iodine and synthesize hormones better." 4. "Propylthiouracil stimulates the pituitary gland to secrete thyroid-stimulating hormone (TSH), which inhibits the production of hormones by the thyroid gland."

2

One day following a subtotal thyroidectomy a client begins to have tingling in the, fingers and toes. The nurse should first 1. encourage the client to flex and extend the fingers and toes. 2. notify the healthcare provider (HCP). 3. assess the client for thrombophlebitis. 4. ask the client to speak.

2

The alert and oriented client is diagnosed with a spontaneous pneumothorax, and the physician is preparing to insert a left-sided chest tube. Which intervention should the nurse implement first? 1. Gather the needed supplies for the procedure. 2. Obtain a signed informed consent form. 3. Assist the client into a side-lying position. 4. Discuss the procedure with the client.

2

The client diagnosed with Cushing's disease is prescribed pantoprazole. Which statement is the scientific rationale for prescribing this medication? 1. Pantoprazole increases the client's ability to digest food. 2. Pantoprazole decreases the excess amounts of gastric acid. 3. Pantoprazole absorbs gastric acid and eliminates it in the bowel. 4. Pantoprazole coats the stomach and prevents ulcer formation.

2

The client diagnosed with a pituitary tumor developed syndrome of inappropriate antidiuretic hormone (SIADH). Which interventions should the nurse implement? 1. Assess for dehydration and monitor blood glucose levels. 2. Assess for nausea and vomiting and weigh daily. 3. Monitor potassium levels and encourage fluid intake. 4. Administer vasopressin IV and conduct a fluid deprivation test.

2

The client is admitted to the emergency department with chest trauma. When assess- ing the client, which signs/symptoms would the nurse expect to find that support the diagnosis of pneumothorax? 1. Bronchovesicular lung sounds and bradypnea. 2. Unequal lung expansion and dyspnea. 3. Frothy bloody sputum and consolidation. 4. Barrel chest and polycythemia.

2

The client is diagnosed with hypothyroidism. Which assessment data support this diagnosis? 1. The client's vital signs are: T 99.0, P 110, R 26, and BP 145.80. 2. The client complains of constipation and being constantly cold. 3. The client has an intake of 780 mL and output of 256 mL. 4. The client complains of a headache and has projectile vomiting.

2

The client is diagnosed with meningococcal meningitis. Which preventive measurewould the nurse expect the health-care provider to order for the significant others inthe home? 1. The Haemophilus influenzae vaccine. 2. Antimicrobial chemoprophylaxis. 3. A 10-day dose pack of corticosteroids. 4. A gamma globulin injection.

2

The nurse is admitting a client to the neurological intensive care unit who is postoperative transsphenoidal hypophysectomy. Which data warrant immediate intervention? 1. The client is alert to name but is unable to tell the nurse the location. 2. The client has an output of 2,500 mL since surgery and an intake of 1,000 mL. 3. The client's vital signs are T 97.6°F, P 88, R 20, and BP 130/80. 4. The client has a 3-cm amount of dark-red drainage on the turban dressing.

2

The nurse is caring for a patient with a chest tube. The nurse knows that the drainage system is working correctly if she Observes? 1. Continuous bubbling in the waterseal chamber. 2. Intermittent bubbling in the waterseal chamber. 3. No bubbling appears in the suction chamber. 4. Titling is absent in the waterseal chamber.

2

The nurse is caring for a patient with a tracheostomy tube. Which action, if performed by the nurse, is incorrect and requires intervention from the charge nurse? 1) The nurse suctions the patient's airway when she hears noisy respirations. 2) The nurse inflates the trach cuff to 30 cm H2O. 3) The nurse ensures that there is an obturator at the patient's bedside. 4) The nurse asks that another nurse help her while she changes the tracheostomy ties for the first time.

2

The nurse is changing the tracheostomy ties on a client with a tracheostomy and is assessing the security of the ties. Which method is used to ensure that the ties are not too tightly placed? 1.The ties leave no marks on the neck. 2.The nurse places two fingers between the tie and the neck. 3.The tracheotomy can be pulled slightly away from the neck. 4.The nurse uses a 12-inch tie that is tightly affixed with hook-and-loop closures.

2

The nurse is completing a health assessment of a 42-year-old female with suspected Graves' disease. The nurse should assess this client for: 1. anorexia. 2. tachycardia. 3. weight gain. 4. cold skin.

2

The nurse is developing a plan of care for a client at risk for acute respiratory distress syndrome (ARDS). As part of the plan, the nurse will check for which item to detect an early sign of this disorder? 1.Edema 2.Dyspnea 3.Frothy sputum 4.Diminished breath sounds

2

The nurse is evaluating a client with hyperthyroidism who is taking propylthiouracil (PTU) 100 mg/day in three divided doses for maintenance therapy. Which statement from the client indicates the drug is effective? 1. "I have excess energy throughout the day." 2. "1 am able to sleep and rest at night." 3. "I have lost weight since taking this medication." 4. "I do perspire throughout the entire day."

2

The nurse is instructing the client with hypothyroidism who takes levothyroxine 100 mco digoxin, and simvastatin. Teaching regarding the use of these medications is effective if the client will take: 1. the levothyroxine with breakfast and the other medications after breakfast. 2. the levothyroxine before breakfast and the other medications 4 hours later 3. all medications together 1 hour after eating breakfast. 4. all medications before going to bed.

2

The nurse is providing instructions to a client being discharged from the hospital following removal of a chest tube that was inserted after thoracic surgery. Which statement, if made by the client, indicates a need for further teaching? 1."I should avoid heavy lifting for at least 4 to 6 weeks." 2."I should remove the chest tube site dressing as soon as I get home." 3."If I have any difficulty in breathing, I should call the health care provider." 4."If I note any signs of infection, I should contact the health care provider (HCP)."

2

The nurse would question an order for somatrem (Protropin) in a patient with which condition? 1. Dwarfism 2. Acromegaly 3. Growth failure 4. Hypopituitarism

2

The student nurse develops a plan of care for a client after a lumbar puncture. The nursing instructor corrects the student if the student documents which incorrect intervention in the plan? 1.Maintain the client in a flat position. 2.Restrict fluid intake for a period of 2 hours. 3.Assess the client's ability to void and move the extremities. 4.Inspect the puncture site for swelling, redness, and drainage.

2

When interviewing the parents of a 2-year-old child, a history of which of the following illnesses would lead the nurse to suspect pneumococcal meningitis? 1. Bladder infection 2. Middle ear infection 3. Fractured clavicle 4. Septic arthritis

2

Which endocrine disorder should the nurse assess for in the client who has a closed head injury with increased intracranial pressure? 1. Pheochromocytoma. 2. Diabetes insipidus. 3. Hashimoto's thyroiditis. 4. Gynecomastia.

2

Which nursing instruction should the nurse discuss with the client who is receiving glucocorticoids for Addison's disease? 1. Discuss the importance of tapering medications when discontinuing medication. 2. Explain the dose may need to be increased during times of stress or infection. 3. Instruct the client to take medication on an empty stomach with a glass of water. 4. Encourage the client to wear clean white socks when wearing tennis shoes.

2

Which of the following pathologic processes is often associated with aseptic meningitis? 1. Ischemic infarction of cerebral tissue 2. Childhood diseases of viral causation such as mumps 3. Brain abscesses caused by a variety of pyogenic organisms 4. Cerebral ventricular irritation from a traumatic brain injury

2

Which should the nurse do when caring for a client with chest tubes attached to a chest drainage system? 1.Empty the drainage collection chamber every shift. 2.Ensure the water level in the water seal chamber is at the 2-cm level. 3.Maintain the drainage collection device at the level of the client's chest. 4.Clamp the chest tube before moving the client from the bed to the chair.

2

Which sign/symptom indicates to the nurse the client is experiencing hypoparathyroidism? 1. A negative Trousseau's sign. 2. A positive Chvostek's sign. 3. Nocturnal muscle cramps. 4. Tented skin turgor.

2

a 32 y/o client had a cholecystectomy 2 hrs ago. the client weighs 250 lbs and had general anesthesia. the client is c/o pain. priority nursing dx at this time: 1. alteration in comfort 2. high risk for airway impairment 3. alteration in fluid volume 4. knowledge deficit

2

he client has developed iatrogenic Cushing's disease. Which statement is the scientific rationale for the development of this diagnosis? 1. The client has an autoimmune problem causing the destruction of the adrenal cortex. 2. The client has been taking steroid medications for an extended period for another disease process. 3. The client has a pituitary gland tumor causing the adrenal glands to produce too much cortisol. 4. The client has developed an adrenal gland problem for which the health-care provider does not have an explanation.

2

the nurse identifies the client problem "excess fluid volume" for the client in liver failure. which short term goal would be most appropriate for this problem? 1. the client will not gain more than 2kg a day 2. the client will have no increase in abdominal girth 3. the clients v/s will remain WNL 4. the client will receive a low sodium diet

2

A nurse is providing discharge instructions to a client who had a unilateral adrenalectomy. Which of the following will be a component of the instructions? 1. The reason for maintaining a diabetic diet 2. Instructions about early signs of a wound infection 3. Teaching regarding proper application of an ostomy pouch 4. The need for lifelong replacement of all adrenal hormones

2 A client who is undergoing a unilateral adrenalectomy will be placed on corticosteroids temporarily to avoid a cortisol deficiency. These medications will be gradually weaned in the postoperative period until they are discontinued. Because of the anti-inflammatory properties of corticosteroids, clients who undergo an adrenalectomy are at increased risk for developing wound infections. Because of this increased risk for infection, it is important for the client to know measures to prevent infection, early signs of infection, and what to do if an infection is present. Options 1, 3, and 4 are incorrect instructions.

A nurse is caring for a client with Addison's disease. The nurse checks the vital signs and determines that the client has orthostatic hypotension. The nurse determines that this finding relates to which of the following? 1. A decrease in cortisol release 2. A decreased secretion of aldosterone 3. An increase in epinephrine secretion 4. Increased levels of androgens

2 A decreased secretion of aldosterone results in a limited reabsorption of sodium and water; therefore the client experiences fluid volume deficit. A decrease in cortisol, an increase in epinephrine, and an increase in androgen secretion do not result in orthostatic hypotension.

A businesswoman comes into the clinic with a progressively enlarging neck. The client mentions that she has been in a foreign country for the previous 3 months and that she didn't eat much while she was there because she didn't like the food. The client also mentions that she becomes dizzy when lifting her arms to do normal household chores or when dressing. What endocrine disorder would the nurse expect the physician to diagnose? 1. Diabetes mellitus 2. Goiter 3. Diabetes insipidus 4. Cushing's syndrome

2 A goiter can result from inadequate dietary intake of iodine associated with changes in foods or malnutrition. It's caused by insufficient thyroid gland production and depletion of glandular iodine. Signs and symptoms of this malfunction include enlargement of the thyroid gland, dizziness when raising the arms above the head, dysphagia, and respiratory distress. Signs and symptoms of diabetes mellitus include polydipsia, polyuria, and polyphagia. Signs and symptoms of diabetes insipidus include extreme polyuria (4 to 16 L/day) and symptoms of dehydration (poor tissue turgor, dry mucous membranes, constipation, dizziness, and hypotension). Cushing's syndrome causes buffalo hump, moon face, irritability, emotional lability, and pathologic fractures.

A nurse is caring for a client who is postoperative following a craniotomy to evacuate a subdural hematoma. The nurse notes the clients urine output is greater each hour than the previous hour: from 8-9 the urine output was 200 mL, from 9-10 it was 400 mL, and from 10-11 it was 600 mL. the nurse informs the surgeon and anticipates that the lab values that will be prescribed at this time is: A. BUN B. Blood sugar C. urine ketones D. specific gravity

D

A client has driven himself to the ER. He is 50 years old, has a history of hypertension, and informs the nurse that his father died of a heart attack at 60 years of age. The client is presently complaining of indigestion. The nurse connects him to an ECG monitor and begins administering oxygen at 2 L/minute per NC. The nurse's next action would be to: 1.Call for the doctor 2.Start an intravenous line 3.Obtain a portable chest radiograph 4.Draw blood for laboratory studies

2 Advanced cardiac life support recommends that at least one or two intravenous lines be inserted in one or both of the antecubital spaces. Calling the physician, obtaining a portable chest radiograph, and drawing blood are important but secondary to starting the intravenous line.

A client is being returned to the room after a subtotal thyroidectomy. Which piece of equipment is most important for the nurse to keep at the client's bedside? 1. Indwelling urinary catheter kit 2. Tracheostomy set 3. Cardiac monitor 4. Humidifier

2 After a subtotal thyroidectomy, swelling of the surgical site (the tracheal area) may obstruct the airway. Therefore, the nurse should keep a tracheostomy set at the client's bedside in case of a respiratory emergency. Although an indwelling urinary catheter and a cardiac monitor may be used for a client after a thyroidectomy, the tracheostomy set is more important. A humidifier isn't indicated for this client.

A client whose physical findings suggest a hyperpituitary condition undergoes an extensive diagnostic workup. Test results reveal a pituitary tumor, which necessitates a transsphenoidal hypophysectomy. The evening before the surgery, the nurse reviews preoperative and postoperative instructions given to the client earlier. Which postoperative instruction should the nurse emphasize? 1. "You must lie flat for 24 hours after surgery." 2. "You must avoid coughing, sneezing, and blowing your nose." 3. "You must restrict your fluid intake." 4. "You must report ringing in your ears immediately."

2 After a transsphenoidal hypophysectomy, the client must refrain from coughing, sneezing, and blowing the nose for several days to avoid disturbing the surgical graft used to close the wound. The head of the bed must be elevated, not kept flat, to prevent tension or pressure on the suture line. Within 24 hours after a hypophysectomy, transient diabetes insipidus commonly occurs; this calls for increased, not restricted, fluid intake. Visual, not auditory, changes are a potential complication of hypophysectomy.

When caring for a client who is having clear drainage from his nares after transsphenoidal hypophysectomy, which action by the nurse is appropriate? 1. Lower the head of the bed. 2. Test the drainage for glucose. 3. Obtain a culture of the drainage. 4. Continue to observe the drainage.

2 After hypophysectomy, the client should be monitored for rhinorrhea, which could indicate a cerebrospinal fluid (CSF) leak. If this occurs, the drainage should be collected and tested for glucose, indicating the presence of CSF. The head of the bed should not be lowered to prevent increased intracranial pressure. Clear nasal drainage would not indicate the need for a culture. Continuing to observe the drainage without taking action could result in a serious complication.

A client has returned to the nursing unit after a thyroidectomy. The nurse notes that the client is complaining of tingling sensations around the mouth, fingers, and toes. On the basis of these findings, the nurse should next assess the results of which serum laboratory study? 1. Sodium 2. Calcium 3. Potassium 4. Magnesium

2 After surgery on the thyroid gland, the client may experience a temporary calcium imbalance. This is due to transient malfunction of the parathyroid glands. The nurse also would assess for Chvostek's and Trousseau's signs. The correct treatment is administration of calcium gluconate or calcium lactate. The remaining options are unrelated to the client's complaints.

A nurse is caring for a client who is receiving mechanical ventilation via an endotracheal tube. Which of the following actions should the nurse take? a. apply a vest restraint if self-extubation is attempted b. monitor ventilator settings every 8 hours c. document tube placement in centimeters at the angle of jaw d. assess breath sounds every 4 hours

D

A nurse is caring for a client with neutropenia. Which clinical manifestation indicates that an infection is present or should be ruled out? A. Coughing and deep breathing B. Evidence of pus C. Fever of 102 deg. F or higher D. Wheezes or crackles

D

The nurse has developed a postoperative plan of care for a client who had a thyroidectomy and documents that the client is at risk for developing an ineffective breathing pattern. Which nursing intervention should the nurse include in the plan of care? 1. Maintain a supine position. 2. Monitor neck circumference every 4 hours. 3. Maintain a pressure dressing on the operative site. 4. Encourage deep-breathing exercises and vigorous coughing exercises.

2 After thyroidectomy, neck circumference is monitored every 4 hours to assess for the occurrence of postoperative edema. The client should be placed in an upright position to facilitate air exchange. A pressure dressing is not placed on the operative site because it may restrict breathing. The nurse should monitor the dressing closely and should loosen the dressing if necessary. The nurse should assist the client with deep-breathing exercises, but coughing is minimized to prevent tissue damage and stress to the incision.

Which assessment data indicate that the chest tubes have been effective in treating the client with a hemothorax who has a right-sided chest tube? 1. There is gentle bubbling in the suction compartment. 2. There is no fluctuation (tidaling) in the water-seal compartment. 3. There is 250 mL of blood in the drainage compartment 4. The client is able to deep breathe without any pain.

2 At three (3) days post-insertion, no fluctuation (tidaling) indicates the lung has reexpanded, which indicates the treatment has been effective.

A nurse is reviewing a new prescription for ursodiol (Ursodeoxycholic Acid) with a client who has cholelithiasis. Which of the following should be included in the teaching? A. This medication reduces biliary spasms. B. This medication reduces inflammation in the biliary tract. C. This medication dilates the bile duct to promote passage of bile. D. This medication dissolves gall stones.

D

The nurse is assessing a client with Cushing's syndrome. Which observation should the nurse report to the physician immediately? 1. Pitting edema of the legs 2. An irregular apical pulse 3. Dry mucous membranes 4. Frequent urination

2 Because Cushing's syndrome causes aldosterone overproduction, which increases urinary potassium loss, the disorder may lead to hypokalemia. Therefore, the nurse should immediately report signs and symptoms of hypokalemia, such as an irregular apical pulse, to the physician. Edema is an expected finding because aldosterone overproduction causes sodium and fluid retention. Dry mucous membranes and frequent urination signal dehydration, which isn't associated with Cushing's syndrome.

A nurse is reviewing the health record of a client who has a malignant brain tumor and notes the client has a positive Romberg sign. Which of the following actions should the nurse take to assess for this sign? A. Stroke the lateral aspect of the sole of the foot B. Ask the client to blink his eyes C. Observe for facial drooping D. Have the client stand erect with eyes closed

D

A nurse is suctioning the nasopharyngeal airway of a patient to maintain a patent airway. For which condition would the nurse anticipate the need for a nasal trumpet? a) The patient vomits during suctioning. b) The secretions appear to be stomach contents. c) The catheter touches an unsterile surface. d) Epistaxis is noted with continued suctioning.

D

What would the nurse anticipate being included in the plan of care for a client who has been diagnosed with Graves' disease? 1. Provide a high-fiber diet. 2. Provide a restful environment. 3. Provide three small meals per day. 4. Provide the client with extra blankets.

2 Because of the hypermetabolic state, the client with Graves' disease needs to be provided with an environment that is restful both physically and mentally. Six full meals a day that are well balanced and high in calories are required, because of the accelerated metabolic rate. Foods that increase peristalsis (e.g., high-fiber foods) need to be avoided. These clients suffer from heat intolerance and require a cool environment.

A nurse suctioning a client through a tracheostomy tube should be careful not to occlude the Y-port when inserting the suction catheter because it would cause what condition to occur? a) suctioning of carbon dioxide b) prevention of suctioning c) loss of sterile field d) trauma to the tracheal mucosa

D

The physician refers the client with unstable angina for a cardiac catherization. The nurse explains to the client that this procedure is being used in this specific case to: 1.Open and dilate the blocked coronary arteries 2.Assess the extent of arterial blockage 3.Bypass obstructed vessels 4.Assess the functional adequacy of the valves and heart muscle.

2 Cardiac catherization is done in clients with angina primarily to assess the extent and severity of the coronary artery blockage, A decision about medical management, angioplasty, or coronary artery bypass surgery will be based on the catherization results.

A nurse is caring for a client with hypothyroidism who is overweight. Which food items would the nurse suggest to include in the plan? 1. Peanut butter, avocado, and red meat 2. Skim milk, apples, whole-grain bread, and cereal 3. Organ meat, carrots, and skim milk 4. Seafood, spinach, and cream cheese

2 Clients with hypothyroidism may have a problem with being over-weight because of their decreased metabolic need. They should consume foods from all food groups, which will provide them with the necessary nutrients; however, the foods should be low in calories. Option 2 is the only option that identifies food items that are low in calories.

A nurse is caring for a client after thyroidectomy and monitoring for signs of thyroid storm. The nurse understands that which of the following is a manifestation associated with this disorder? 1. Bradycardia 2. Hypotension 3. Constipation 4. Hypothermia

2 Clinical manifestations associated with thyroid storm include a fever as high as 106° F (41.1° C), severe tachycardia, profuse diarrhea, extreme vasodilation, hypotension, atrial fibrillation, hyperreflexia, abdominal pain, diarrhea, and dehydration. With this disorder, the client's condition can rapidly progress to coma and cardiovascular collapse.

A patient being treated for infective endocarditis is complaining of very sharp radiating abdominal pain that goes to the left shoulder and back. As the nurse familiar with complications of infective endocarditis, what do you suspect is the cause of this patient finding? A. Renal embolic event B. Pulmonary embolic event C. Central nervous system embolic event D. Splenic embolic event

D

A patient had a non-ST-segment-elevation myocardial infarction (NSTEMI) 3 days ago. Which nursing intervention included in the plan of care is most appropriate for the registered nurse (RN) to delegate to an experienced licensed practical/vocational nurse (LPN/LVN)? a. Evaluation of the patient's response to walking in the hallway b. Completion of the referral form for a home health nurse follow-up c. Education of the patient about the pathophysiology of heart disease d. Reinforcement of teaching about the purpose of prescribed medications

D

A patient has a cuffed trach tube without a pressure relief valve. To prevent tissue damage of the tracheal mucosa, what does the nurse do? A. deflate the cuff every 2 to 4 hours and maintain as needed B. change the tracheostomy tube every 3 days or per hospital policy C. assess and record cuff pressure each shift using the occlusive technique D. assess and record cuff pressure each shift using minimal leak technique

D

A patient has a tonic-clonic seizure while the nurse is in the patient's room. Which action should the nurse take? a. Insert an oral airway during the seizure to maintain a patent airway. b. Restrain the patient's arms and legs to prevent injury during the seizure. c. Avoid touching the patient to prevent further nervous system stimulation. d. Time and observe and record the details of the seizure and postictal state.

D

A patient is diagnosed with a primary spontaneous pneumothorax. Which of the following is NOT a correct statement about this type of pneumothorax? A. It can be caused by the rupture of a pulmonary bleb. B. It can occur in patients who are young, tall and thin without a history of lung disease. C. Smoking increases the chances of a patient developing a spontaneous pneumothorax. D. It is most likely to occur in patients with COPD, asthma, and cystic fibrosis

D

A patient is experiencing respiratory failure due to pulmonary edema. The physician suspects ARDS but wants to rule out a cardiac cause. A pulmonary artery wedge pressure is obtained. As the nurse you know that what measurement reading obtained indicates that this type of respiratory failure is NOT cardiac related? A. >25 mmHg B. <10 mmHg C. >50 mmHg D. <18 mmHg

D

A patient rapidly progressing toward ESRD asks about the possibility of a kidney transplant. In responding to the patient, the nurse knows that what is a contraindication to kidney transplantation? a. hepatitis C infection b. coronary artery disease c. refractory hypertension d. extensive vascular disease

D

A client is transferred to a rehabilitation center after being treated in the hospital for a stroke. Because the client has a history of Cushing's syndrome (hypercortisolism) and chronic obstructive pulmonary disease (COPD), the nurse formulates a nursing diagnosis of: 1. Risk for imbalanced fluid volume related to excessive sodium loss. 2. Risk for impaired skin integrity related to tissue catabolism secondary to cortisol hypersecretion. 3. Ineffective health maintenance related to frequent hypoglycemic episodes secondary to Cushing's syndrome. 4. Decreased cardiac output related to hypotension secondary to Cushing's syndrome.

2 Cushing's syndrome causes tissue catabolism, resulting in thinning skin and connective tissue loss; along with immobility related to stroke, these factors increase this client's risk for impaired skin integrity. The exaggerated glucocorticoid activity in Cushing's syndrome causes sodium and water retention which, in turn, leads to edema and hypertension. Therefore, Risk for imbalanced fluid volume and Decreased cardiac output are inappropriate nursing diagnoses. Increased glucocorticoid activity also causes persistent hyperglycemia, eliminating Ineffective health maintenance related to frequent hypoglycemic episodes as an appropriate nursing diagnosis.

A nursing instructor asks a student to describe the pathophysiology that occurs in Cushing's disease. Which statement by the student indicates an accurate understanding of this disorder? 1. "Cushing's disease is characterized by an oversecretion of insulin." 2. "Cushing's disease is characterized by an oversecretion of glucocorticoid hormones." 3. "Cushing's disease is characterized by an undersecretion of corticotropic hormones." 4. "Cushing's disease is characterized by an undersecretion of glucocorticoid hormones."

2 Cushing's syndrome is characterized by an oversecretion of glucocorticoid hormones. Addison's disease is characterized by the failure of the adrenal cortex to produce and secrete adrenocortical hormones. Options 1 and 4 are inaccurate regarding Cushing's syndrome.

The nurse identifies the client problem "risk for imbalanced body temperature" for theclient diagnosed with hypothyroidism. Which intervention would be included in theclient problem? 1. Encourage the use of an electric blanket. 2. Protect from exposure to cold and drafts. 3. Keep the room temperature cool. 4. Space activities to promote rest.

2 Decreased metabolism causes the client to be cold frequently; therefore, protecting the client from exposure to cold will help increase comfort and decrease further heat loss.

Along with persistent, crushing chest pain, which signs/symptoms would make the nurse suspect that the client is experiencing a myocardial infarction? 1. Midepigastric pain and pyrosis 2. Diaphoresis and cool clammy skin 3. Intermittent claudication and paloor 4. Jugular vein distention and dependent edema

2 Diaphoresis is a systemic reaction to the MI. The body vasoconstricts to shunt blood from the periphery to the trunk of the body; this in turn, leads to cold, clammy skin

A client who had an appendectomy for a perforated appendix returns from surgery with a drain inserted in the incisional site. The purpose of the drain is to: 1. Provide access for wound irrigation. 2. Promote drainage of wound exudates. 3. Minimize development of scar tissue. 4. Decrease postoperative discomfort.

2 Drains are inserted postoperatively in appendectomies when an abscess was present or the appendix was perforated. The purpose is to promote drainage of exudate from the wound and facilitate healing. A drain is not used for irrigation of the wound. The drain will not minimize scar tissue development or decrease postoperative discomfort.

A nurse is caring for a postoperative parathyroidectomy client. Which of the following would require the nurse's immediate attention? 1. Incisional pain 2. Laryngeal stridor 3. Difficulty voiding 4. Abdominal cramps

2 During the postoperative period, the nurse carefully observes the client for signs of hemorrhage, which cause swelling and the compression of adjacent tissue. Laryngeal stridor is a harsh, high-pitched sound heard on inspiration and expiration that is caused by the compression of the trachea and that leads to respiratory distress. It is an acute emergency situation that requires immediate attention to avoid the complete obstruction of the airway.

The client diagnosed with a pituitary tumor has developed syndrome of inappropriateantidiuretic hormone (SIADH). Which interventions would the nurse implement? 1. Assess for dehydration and monitor blood glucose levels. 2. Assess for nausea and vomiting and weigh daily. 3. Monitor potassium levels and encourage fluid intake. 4. Administer vasopressin IV and conduct a fluid deprivation test.

2 Early signs and symptoms are nausea and vomiting. The client has a syndrome of the inappropriate secretion of the antidiuresis(against allowing the body to urinate) hormone. In other words, the client is producing a hormone that will not allow the client to urinate.

A client with a history of Addison's disease and flulike symptoms accompanied by nausea and vomiting over the past week is brought to the facility. When he awoke this morning, his wife noticed that he acted confused and was extremely weak. The client's blood pressure is 90/58 mm Hg, his pulse is 116 beats/minute, and his temperature is 101° F (38.3° C). A diagnosis of acute adrenal insufficiency is made. What would the nurse expect to administer by I.V. infusion? 1. Insulin 2. Hydrocortisone 3. Potassium 4. Hypotonic saline

2 Emergency treatment for acute adrenal insufficiency (Addisonian crisis) is I.V. infusion of hydrocortisone and saline solution. The client is usually given a dose containing hydrocortisone 100 mg I.V. in normal saline every 6 hours until the client's blood pressure returns to normal. Insulin isn't indicated in this situation because adrenal insufficiency is usually associated with hypoglycemia. Potassium isn't indicated because these clients are usually hyperkalemic. The client needs normal — not hypotonic — saline solution.

A client visits the physician's office complaining of agitation, restlessness, and weight loss. The physical examination reveals exophthalmos, a classic sign of Graves' disease. Based on history and physical findings, the nurse suspects hyperthyroidism. Exophthalmos is characterized by: 1. dry, waxy swelling and abnormal mucin deposits in the skin. 2. protruding eyes and a fixed stare. 3. a wide, staggering gait. 4. more than 10 beats/minute difference between the apical and radial pulse rates.

2 Exophthalmos is characterized by protruding eyes and a fixed stare. Dry, waxy swelling and abnormal mucin deposits in the skin typify myxedema, a condition resulting from advanced hypothyroidism. A wide, staggering gait and a differential between the apical and radial pulse rates aren't specific signs of thyroid dysfunction.

The client diagnosed with Addison's disease asks the nurse, "Why do I have to take fludrocortisone?" Which statement is the nurse's best response? 1. "It will keep you from getting high blood sugars." 2. "Fludrocortisone helps the body retain sodium." 3. "Fludrocortisone prevents muscle cramping." 4. "It stimulates the pituitary gland to secrete adrenocorticotropic hormone."

2 Fludrocortisone (Florinef) is a mineral corticosteroid. Mineral corticosteroids help the body to maintain the correct serum sodium levels. Florinef is the preferred medication for Addison's disease, primary hypoaldosteronism, and congenital adrenal hyperplasia when sodium wasting occurs.

A nurse is collecting data from a client who is being admitted to the hospital for a diagnostic workup for primary hyperparathyroidism. The nurse understands that which client complaint would be characteristic of this disorder? 1. Diarrhea 2. Polyuria 3. Polyphagia 4. Weight gain

2 Hypercalcemia is the hallmark of hyperparathyroidism. Elevated serum calcium levels produce osmotic diuresis (polyuria). This diuresis leads to dehydration and the client would lose weight. Options 1, 3, and 4 are gastrointestinal (GI) symptoms but are not associated with the common GI symptoms typical of hyperparathyroidism (nausea, vomiting, anorexia, constipation).

A patient receiving treatment for a pneumothorax calls on the call light to tell you something is wrong with their chest tube. When you arrive to the room you note that the drainage system has fallen on its side, and there is a large crack in the system. What is your next PRIORITY? A. Place the patient in supine position and clamp the tubing. B. Notify the physician immediately. C. Disconnect the drainage system and get a new one. D. Disconnect the tubing from the drainage system and insert the tubing 1 inch into a bottle of sterile water and obtain a new system.

D

A nurse is collecting data on a client with hyperparathyroidism. Which of the following questions would elicit the accurate information about this condition from the client? 1. "Do you have tremors in your hands?" 2. "Are you experiencing pain in your joints?" 3. "Have you had problems with diarrhea lately?" 4. "Do you notice swelling in your legs at night?"

2 Hyperparathyroidism causes an oversecretion of parathyroid hormone (PTH), which causes excessive osteoblast growth and activity within the bones. When bone reabsorption is increased, calcium is released from the bones into the blood, causing hypercalcemia. The bones suffer demineralization as a result of calcium loss, leading to bone and joint pain, and pathological fractures.

The nurse is performing an assessment on a client with a diagnosis of hyperthyroidism. Which assessment finding should the nurse expect to note in this client? 1. Dry skin 2. Bulging eyeballs 3. Periorbital edema 4. Coarse facial features

2 Hyperthyroidism is clinically manifested by goiter (increase in the size of the thyroid gland) and exophthalmos (bulging eyeballs). Other clinical manifestations include nervousness, fatigue, weight loss, muscle cramps, and heat intolerance. Additional signs found in this disorder include tachycardia; shortness of breath; excessive sweating; fine muscle tremors; thin, silky hair and thin skin; infrequent blinking; and a staring appearance.

The client has developed iatrogenic Cushing's disease. Which is a scientific rationalefor the development of this problem? 1. The client has an autoimmune problem that causes the destruction of the adrenalcortex. 2. The client has been taking steroid medications for an extended period for anotherdisease process. 3. The client has a pituitary gland tumor that causes the adrenal glands to produce toomuch cortisol. 4. The client has developed an adrenal gland problem for which the health-careprovider does not have an explanation.

2 Iatrogenic means that a problem has been caused by a medical treatment or procedure—in this case, treatment with steroids for another problem. Clients taking steroids over a period of time develop the clinical manifestations of Cushing's disease.Disease processes for which long-term steroids are prescribed include chronic obstructive pulmonary disease, cancer, and arthritis.

A nurse is collecting data on a client admitted to the hospital with a diagnosis of myxedema. Which data collection technique will provide data necessary to support the admitting diagnosis? 1. Auscultation of lung sounds 2. Inspection of facial features 3. Percussion of the thyroid gland 4. Palpation of the adrenal glands

2 Inspection of facial features will reveal the characteristic coarse features, presence of edema around the eyes and face, and a blank expression that are characteristic of myxedema. The techniques in the remaining options will not reveal any data that would support the diagnosis of myxedema.

A patient with chronic kidney disease (CKD) who will be undergoing long-term hemodialysis is receiving discharge instructions from the nurse in regard to the insertion of a new arteriovenous (AV) fistula in her left arm. Which statement by the patient indicates a need for further teaching? 1. "I will avoid sleeping on my left arm at night." 2. "I will continue my daily weight training exercise program to stay fit and healthy." 3. "I should have my blood pressure checked on my right arm from now on." 4. "I will avoid wearing constrictive clothing or jewelry on my left arm."

2 It is recommended that patients lift no more than 5-10 pounds with their fistula arms. Exercise for people with CKD is important because of the many health benefits, but weight lifting exercises should be discussed with a doctor first. Patients should avoid sleeping on their fistula arm because doing so can restrict blood flow distal to the fistula site. Likewise, constrictive clothing or jewelry should be avoided as well. Patients should avoid IV insertion and blood pressure monitoring on their fistula arms to prevent damage to the delicate blood vessel graft site.

The nurse is discharging a client diagnosed with diabetes insipidus. Which statementmade by the client warrants further intervention? 1. "I will keep a list of my medications in my wallet and wear a Medi bracelet." 2. "I should take my medication in the morning and leave it refrigerated at home." 3. "I should weigh myself every morning and record any weight gain." 4. "If I develop a tightness in my chest, I will call my health-care provider."

2 Medication taken for DI is usually every8-12 hours, depending on the client. The client should keep the medication close at hand.

A client is admitted to an emergency department, and a diagnosis of myxedema coma is made. Which action should the nurse prepare to carry out initially? 1. Warm the client. 2. Maintain a patent airway. 3. Administer thyroid hormone. 4. Administer fluid replacement.

2 Myxedema coma is a rare but serious disorder that results from persistently low thyroid production. Coma can be precipitated by acute illness, rapid withdrawal of thyroid medication, anesthesia and surgery, hypothermia, and the use of sedatives and opioid analgesics. In myxedema coma, the initial nursing action is to maintain a patent airway. Oxygen should be administered, followed by fluid replacement, keeping the client warm, monitoring vital signs, and administering thyroid hormones by the intravenous route.

Postoperative nursing care for a client after an appendectomy should include which of the following? 1. Administering sitz baths four times a day. 2. Noting the first bowel movement after surgery. 3. Limiting the client's activity to bathroom privileges. 4. Measuring abdominal girth every 2 hours.

2 Noting the client's first bowel movement after surgery is important because this indicates that normal peristalsis has returned. Sitz baths are used after rectal surgery, not appendectomy. Ambulation is started the day of surgery and is not confined to bathroom privileges. The abdomen should be auscultated for bowel sounds and palpated for softness, but there is no need to measure the girth every 2 hours.

A nurse is performing an admission assessment on a client with a diagnosis of pheochromocytoma. The nurse should assess for the major sign associated with pheochromocytoma by performing which action? 1. Obtaining the client's weight 2. Taking the client's blood pressure 3. Testing the client's urine for glucose 4. Palpating the skin for its temperature

2 Pheochromocytoma is a catecholamine-producing tumor. Hypertension is the major sign associated with pheochromocytoma. Taking the client's blood pressure would assess the blood pressure status. Weight loss, glycosuria, and diaphoresis are also clinical manifestations of pheochromocytoma, yet hypertension is the major sign.

A patient comes into the clinic to be seen for a burn that appears moist with blisters. The nurse realizes that this patient most likely has experienced which of the following? 1. first-degree burn 2. superficial second-degree burn 3. deep second-degree burn 4. third-degree burn

2 Rationale: Partial-thickness, or second-degree, burns can either be superficial or deep. This patient's burn, which appears moist with blisters, is consistent with a superficial second-degree burn. A first-degree burn would involve only the surface layer of skin. Redness would be expected. Deep second-degree and third-degree burns would be deeper and involve more damage to the dermis, epidermis, and underlying tissue.

Which medication should the nurse anticipate the health-care provider ordering for the client diagnosed with ARDS? 1. An aminoglycoside antibiotic 2. A synthetic surfactant 3. A potassium cation 4. A nonsteroidal anti-inflammatory drug

2 Surfactant therapy may be prescribed to reduce the surface tension in the aveoli. The surfactant helps maintain open alveoli, decreases the work of breathing, improves compliance, and helps prevent atelectasis.

A client with medullary carcinoma of the thyroid has an excess function of the C cells of the thyroid gland. When reviewing the most recent laboratory results, the nurse should expect which electrolyte abnormality? 1. Sodium 2. Calcium 3. Potassium 4. Magnesium

2 The C cells of the thyroid gland are helpful in maintaining normal plasma calcium levels. They do not affect the levels of sodium, potassium, or magnesium.

The client is admitted to rule out Cushing's syndrome. Which laboratory tests wouldthe nurse anticipate being ordered? 1. Plasma drug levels of quinidine, digoxin, and hydralazine. 2. Plasma levels of ACTH and cortisol. 3. 24-hour urine for metanephrine and catecholamine. 4. Spot urine for creatinine and white blood cells.

2 The adrenal gland secretes cortisol and the pituitary gland secretes adrenocorticotropic hormone (ACTH), a hormone used by the body to stimulate the production of cortisol.

A nurse is caring for a client diagnosed with hyperparathyroidism who is prescribed furosemide (Lasix). The nurse reinforces dietary instructions to the client. Which of the following is an appropriate instruction? 1. Increase dietary intake of calcium. 2. Drink at least 2 to 3 L of fluid daily. 3. Eat sparely when experiencing nausea. 4. Decrease dietary intake of potassium.

2 The aim of treatment in the client with hyperparathyroidism is to increase the renal excretion of calcium and decrease gastrointestinal absorption and bone resorption. This is aided by the sufficient intake of fluids. Dietary restriction of calcium may be used as a component of therapy. The parathyroid is responsible for calcium production, and the term, "hyperparathyroidism" can be indicative of an increase in calcium. The client should eat foods high in potassium, especially if the client is taking furosemide. Limiting nutrients is not advisable.

The unlicensed nursing assistant is assisting the client with a chest tube to ambulate to the bathroom. Which situation warrants immediate intervention from the nurse? 1. The client's chest tube is below the level of the chest. 2. The nursing assistant has the chest tube attached to suction. 3. The nursing assistant allowed the client out of the bed. 4. The nursing assistant uses a bedside commode for the client.

2 The chest tube system can function as a result of gravity and does not have to be attached to suction. Keeping it attached to suction could cause the client to trip and fall. Therefore, this is a safety issue and the nurse should intervene and explain this to the nursing assistant.

A client scheduled for a thyroidectomy says to the nurse, "I am so scared to get cut in my neck." Based on the client's statement, the nurse determines that the client is experiencing which problem? 1. Inadequate knowledge about the surgical procedure 2. Fear about impending surgery 3. Embarrassment about the changes in personal appearance 4. Lack of support related to the surgical procedure

2 The client is having a difficult time coping with the scheduled surgery. The client is able to express fears but is scared. No data in the question support options 1, 3, and 4.

A nurse is reinforcing home care instructions to a client with a diagnosis of Cushing's syndrome. Which statement reflects a need for further client education? 1. "Taking my medications exactly as prescribed is essential." 2. "I need to read the labels on any over-the-counter medications I purchase." 3. "My family needs to be familiar with the signs and symptoms of hypoadrenalism." 4. "I could experience the signs and symptoms of hyperadrenalism because of Cushing's."

2 The client with Cushing's syndrome should be instructed to take the medications exactly as prescribed. The nurse should emphasize the importance of continuing medications, consulting with the health care provider before purchasing any over-the-counter medications, and maintaining regular follow-up care. The nurse should also instruct the client in the signs and symptoms of both hypoadrenalism and hyperadrenalism.

A client with Cushing's disease is being admitted to the hospital after a stab wound to the abdomen. The nurse plans care and places highest priority on which potential problem? 1. Nervousness 2. Infection 3. Concern about appearance 4. Inability to care for self

2 The client with a stab wound has a break in the body's first line of defense against infection. The client with Cushing's disease is at great risk for infection because of excess cortisol secretion and subsequent impaired antibody function and decreased proliferation of lymphocytes. The client may also have a potential for the problems listed in the other options but these are not the highest priority at this time.

The nurse is assessing an adolescent who is admitted to the hospital with appendicitis. The nurse should report which of the following to the HCP? 1) change in pain rating of 7 to 8 on a 10 point scale. 2) sudden relief of sharp pain, shifting to diffuse pain. 3)shallow breathing with normal vital signs. 4) decrease of pain rating from 8 to 6 when parents visit.

2 The nurse notifies the HCP if the client has sudden relief of sharp pain and on presence of more diffuse pain. this change in the pain indicates the appendix has ruprured. The diffuse pain is typically accompanied by rigid guarding of the abdomen, progressive abdominal distension, tachycardia, pallor, chills, and irritability. The slight increase pain can be expected; the decrease in pain when parents visit may be attributed to being distracted from the pain. shallow breathing is likely due to the pain and is insignificant when other vital signs are normal

A nurse is reviewing the postoperative prescriptions for a client who had a transsphenoidal hypophysectomy. Which health care provider's prescription, if noted on the record, indicates the need for clarification? 1. Instruct the client about the need for a Medic-Alert bracelet. 2. Apply a loose dressing if any clear drainage is noted. 3. Monitor vital signs and neurological status. 4. Instruct the client to avoid blowing the nose.

2 The nurse should observe for clear nasal drainage, constant swallowing, and a severe, persistent, generalized, or frontal headache. These signs and symptoms indicate cerebrospinal fluid leak into the sinuses. If clear drainage is noted following this procedure, the health care provider needs to be notified immediately. Options 1, 3, and 4 indicate appropriate postoperative interventions.

Early this morning, a client had a subtotal thyroidectomy. During evening rounds, the nurse assesses the client, who now has nausea, a temperature of 105° F (40.5° C), tachycardia, and extreme restlessness. What is the most likely cause of these signs? 1. Diabetic ketoacidosis 2. Thyroid crisis 3. Hypoglycemia 4. Tetany

2 Thyroid crisis usually occurs in the first 12 hours after thyroidectomy and causes exaggerated signs of hyperthyroidism, such as high fever, tachycardia, and extreme restlessness. Diabetic ketoacidosis is more likely to produce polyuria, polydipsia, and polyphagia; hypoglycemia, to produce weakness, tremors, profuse perspiration, and hunger. Tetany typically causes uncontrollable muscle spasms, stridor, cyanosis, and possibly asphyxia.

Which of the following clients is at risk for developing thyrotoxicosis? 1. A client with hypothyroidism 2. A client with Graves' disease who is having surgery 3. A client with diabetes mellitus scheduled for debridement of a foot ulcer 4. A client with diabetes insipidus scheduled for an invasive diagnostic test

2 Thyrotoxicosis is usually seen in clients with Graves' disease with the symptoms precipitated by a major stressor. This complication typically occurs during periods of severe physiological or psychological stress such as trauma, sepsis, the birth process, or major surgery. It also must be recognized as a potential complication following a thyroidectomy.

A client visits the health care provider's office for a routine physical examination and reports a new onset of intolerance to cold. Since hypothyroidism is suspected, which additional information would be noted during the client's assessment? 1. Weight loss and tachycardia 2. Complaints of weakness and lethargy 3. Diaphoresis and increased hair growth 4. Increased heart rate and respiratory rate

2 Weakness and lethargy are the most common complaints associated with hypothyroidism. Other common symptoms include intolerance to cold, weight gain, bradycardia, decreased respiratory rate, dry skin, and hair loss.

a client has had gallbladder attacks in the past and is considered at risk for gallstone formation. the nurse should check that the clients diet is: 1. high in antacids and calcium 2. low in cholesterol 3. high in protein and carbohydrates 4. free of electrolytes

2 biliary calculi are high in cholesterol, therefore the cholesterol intake of the client should be limited

the client dx with acute pancreatitis is being discharged home. which statement by the client indicates the teaching has been effective? 1. i should decrease my intake of coffee, tea, and cola 2. i will eat a low fat diet and avoid spicy foods 3. i will check my amylase and lipase levels daily 4. i will return to work tomorrow but take it easy

2 high fat and spicy foods stimulate gastric and pancreatic secretions and may precipitate an acute pancreatic attack

a 38 y/o woman was admitted with a medical dx of cholecystitis. the nurse would expect the hx of her present illness to include intolerance to which nutrient? 1. carbohydrates 2. fat 3. protein 4. vitamin c

2 inflammation of the gallbladder, presence of gallstones, or both interfere with movement of the bile from the liver or gallbladder into the small intestine, where it promotes the digestion of fats

a client with pancreatitis has a hx of alcohol abuse. the nurse will observe the client for agitation, nausea, vomiting, delirium tremens, and visual, auditory and tactile hallucinations. these are indications of: 1. possible cirrhosis of the liver 2. alcohol withdrawal 3. depression 4. suicidal thoughts

2 the airway is of highest priority in postop clients after general anesthesia

A 56-year-old female client is being discharged after undergoing a thyroidectomy. Which discharge instructions would be appropriate for this client? 1. "Report signs and symptoms of hypoglycemia." 2. "Take thyroid replacement medication as ordered." 3. "Watch for changes in body functioning, such as lethargy, restlessness, sensitivity to cold, and dry skin, and report these changes to the physician." 4. "Recognize the signs of dehydration." 5. "Carry injectable dexamethasone at all times."

23

The nurse is caring for a client diagnosed with a myocardial infarction who is experiencing chest pain. Which interventions should the nurse implement first? Select All that Apply 1. Administer morphine sulfate Intramuscularly 2. Administer an aspirin orally 3. Apply oxygen via nasal cannula 4. Place the client in a supine position 5. Administer nitroglycerin subcutaneously

23

A nurse is monitoring a client with Graves' disease for signs of thyrotoxic crisis (thyroid storm). Which of the following signs and symptoms, if noted in the client, will alert the nurse to the presence of this crisis? Select all that apply. 1. Bradycardia 2. Fever 3. Sweating 4. Agitation 5. Pallor

234 Thyrotoxic crisis (thyroid storm) is an acute, potentially life-threatening state of extreme thyroid activity that represents a breakdown in the body's tolerance to a chronic excess of thyroid hormones. The clinical manifestations include fever greater than 100° F, severe tachycardia, flushing and sweating, and marked agitation and restlessness. Delirium and coma can occur.

The nurse monitoring a client receiving peritoneal dialysis notes that the clietn's outflow is less than the inflow. Select all nursing actions in the situation that apply. 1. Contact the physician. 2. Check the level of the drainage bag. 3. Reposition the client to his or her side. 4. Place the client in good body alignment. 5. Check the peritoneal dialysis system for kinks. 6. Increase the flow rate of the peritoneal dialysis solution.

2345 If outflow drainage is inadequate, the nurse attempts to stimulate outflow by changing the client's position. Turning the client to the other side or making sure that the client is in good body alignment may assist with outflow drainage. The drainage bag needs to be lower than the client's abdomen to enhance gravity drainage. The connecting tubing and peritoneal dialysis system are also checked for kinks or twisting and the clamps on the system are checked to ensure that they are open. There is no reason to contact the physician. Increasing the flow rate is an inappropriate action and is not associated with the amount of outflow solution.

The client is admitted to the emergency department, and the nurse suspects a cardiac problem. Which assessment interventions should the nurse implement? Select All that Apply 1. Obtain a midstream urine specimen 2. Attach telemetry monitor to the client 3. Start a saline lock in the right arm 4. Draw a baseline metabolic panel (BMP) 5. Request an order for a STAT 12-lead ECG

235 Anytime a nurse suspects cardiac problems, the electrical conductivity of the heart should be assessed. Emergency medications for heart problems are primarily administered intravenously, so starting a saline lock in the right arm is appropriate. A 12-lead ECG evaluates the electrical conductivity of the heart from all planes.

The nurse is caring for a client with a right-sided chest tube secondary to a pneumothorax. Which interventions should the nurse implement when caring for this client? Select all that apply. 1. Place the client in a low-Fowler's position. 2. Assess chest tube drainage system frequently. 3. Maintain strict bed rest for the client. 4. Secure a loop of drainage tubing to the sheet. 5. Observe the site for subcutaneous emphysema.

245

A patient with longstanding valve disease requires a valve replacement. When discussing the differences between biological and mechanical valves, what information should the healthcare provider include?Select all that apply. 1. Patients using mechanical valves experience less hemolysis of their red blood cells. 2. Biologic valves are commonly used for older patients because these valves are less durable. 3. Patients who have a biologic valve are at increased risk for thromboembolism. 4. Mechanical valves do not last as long as biologic valves. 5. Patients who have mechanical valves require long-term anticoagulant therapy. 6. The hemodynamics produced by biologic valves is superior to mechanical valves.

256

A client enters the ER complaining of severe chest pain. A myocardial infarction is suspected. A 12 lead ECG appears normal, but the doctor admits the client for further testing until cardiac enzyme studies are returned. All of the following will be included in the nursing care plan. Which activity has the highest priority? 1.Monitoring vital signs 2.Completing a physical assessment 3.Maintaining cardiac monitoring 4.Maintaining at least one IV access site

3

A client with hyperthyroidism is hospitalized to have a thyroidectomy. The healthcare provider (HCP) has prescribed propranolol. In reviewing the client's history, the nurse notes that the client has asthma. The nurse should next: 1. take the client's pulse and withhold the propranolol if the pulse is <100 beats per minute. 2. count the client's respirations and withhold the propranolol if the respirations are <20 breaths per minute. 3. Contact the HCP and discuss the prescription for propranolol because of the client's history of having asthma. 4. instruct the client to make position change slowly.

3

A nurse is caring for a client on a mechanical ventilator. The high-pressure alarm on the ventilator sounds. The nurse suspects that the most likely cause of the alarm is which finding? 1.A disconnection of the ventilator tubing 2.An exaggerated client inspiratory effort 3.Accumulation of respiratory secretions 4.Generation of extreme negative pressure by the client

3

A nurse who is participating in a client care conference with other members of the health care team is discussing the condition of a client with acute respiratory distress syndrome (ARDS). The health care provider states that as a result of fluid in the alveoli, surfactant production is falling. The nurse understands that which is the natural consequence of insufficient surfactant? 1.Atelectasis and viral infection 2.Bronchoconstriction and stridor 3.Collapse of alveoli and decreased compliance 4.Decreased ciliary action and retained secretions

3

A nursing student is developing a plan of care for a client with a chest tube that is attached to a Pleur-Evac drainage system. Which intervention in the care plan indicates the need for further teaching for the student? 1.Position the client in semi-Fowler's position. 2.Add water to the suction chamber as it evaporates. 3.Instruct the client to avoid coughing and deep breathing. 4.Tape the connection sites between the chest tube and the drainage system.

3

A patient admits to the E.D. with fractures of 3 lower ribs. Which of the following is the priority concern of the nurse caring for this patient? 1) infection risk 2) pain 3) hemorrhage risk 4) airway maintenance

3

After treatment with radioactive iodine (RAI, 1-131) I, the nurse should teach the client to: 1. monitor for signs and symptoms of hyperthyroidism. 2. rest for 1 week to prevent complications of the medication. 3. take thyroxine replacement for the remainder of the client's life. 4. assess for hypertension and tachycardia resulting from altered thyroid activity

3

Following a subtotal thyroidectomy, the nurse asks the client to speak immediately upon regaining consciousness. The nurse does this to monitor for signs of: 1. internal hemorrhage. 2. decreasing level of consciousness. 3. laryngeal nerve damage. 4. upper airway obstruction.

3

In order to prevent the development of tolerance, the nurse instructs the patient to: 1.Apply the nitroglycerin patch every other day 2.Switch to sublingual nitroglycerin when the patient's systolic blood pressure elevates to >140 mm Hg 3.Apply the nitroglycerin patch for 14 hours each and remove for 10 hours at night 4.Use the nitroglycerin patch for acute episodes of angina only

3

One hour after administering IV furosemide (Lasix) to a client with heart failure, a short burst of ventricular tachycardia appears on the cardiac monitor. Which of the following electrolyte imbalances should the nurse suspect? 1.Hypocalcemia 2.Hypermagnesemia 3.Hypokalemia 4.Hypernatremia

3

The client diagnosed with ARDS is transferred to the intensive care department and placed on a ventilator. Which intervention should the nurse implement first? 1. Confirm that they ventilator settings are correct 2. Verify that the ventilator alarms are functioning properly 3. Assess the respiratory status and pulse oximeter reading. 4. Monitor the client's arterial blood gas results

3

The client diagnosed with Cushing's disease has undergone a unilateral adrenalectomy. Which discharge instructions should the nurse discuss with the client? 1. Instruct the client to take the glucocorticoid and mineralocorticoid medications as prescribed. 2. Teach the client regarding sexual functioning and androgen replacement therapy. 3. Explain the signs and symptoms of infection and when to call the health-care provider. 4. Demonstrate turn, cough, and deep-breathing exercises the client should perform every two (2) hours.

3

The client has had a liver biopsy. which postprocedure intervention should the nurse implement? 1. instruct the client to void immediatly 2. keep the client NPO for 8 hours 3. place the client on the right side 4. monitor BUN and creatinine level

3

The client is being admitted to the outpatient department prior to an endoscopicretrograde cholangiopancreatogram (ERCP) to rule out cancer of the pancreas. Whichpre-procedure instruction should the nurse teach? 1. Prepare to be admitted to the hospital after the procedure for observation. 2. If something happens during the procedure, then emergency surgery will be done. 3. Do not eat or drink anything after midnight the night before the test. 4. If done correctly, this procedure will correct the blockage of the stomach.

3

The client who smokes two packs of cigarettes a day develops ARDS after a near-drowning. The client asks the nurse, "What is happening to me? Why did I get this?" Which statement by the nurse is most appropriate? 1. Most people who almost drown end up developing ARDS 2. Platelets and fluid enter the alveoli due to permeability instability. 3. Your lungs are filling up with fluid, causing breathing problems 4. Smoking has caused your lungs to become weakened, so you got ARDS

3

The nurse has assisted the health care provider and the anesthesiologist with placement of an endotracheal (ET) tube for a client in respiratory distress. What is the initial nursing action to evaluate proper ET tube placement? 1.Tape the ET tube in place, and note the centimeter marking at the lip line. 2.Ask the radiology department to obtain a stat portable radiograph at the client's bedside. 3.Use an Ambu (resuscitation) bag to ventilate the client and assess for bilateral breath sounds. 4.Attach the ET tube to the ventilator and determine whether the client is able to tolerate the tidal volume prescribed.

3

The nurse instructs the patient is scheduled to have a radioactive iodine uptake test to: 1. watch for any signs of bleeding or swelling from the biopsy site. 2. avoid contact with others until notified otherwise. 3. wash hands with soap and water after each urination for 24 hours after the test. 4. this test demonstrates the effectiveness of the pituitary gland on the thyroid gland.

3

The nurse is caring for a client with a tracheostomy tube who is receiving mechanical ventilation. The nurse is monitoring for complications related to the tracheostomy and suspects tracheoesophageal fistula when which occurs? 1.Suctioning is required frequently. 2.The client's skin and mucous membranes are light pink. 3.Aspiration of gastric contents occurs during suctioning. 4.Excessive secretions are suctioned from the tube and stoma.

3

The nurse is caring for clients on a medical floor. Which client should be assessed first? 1. The client diagnosed with syndrome of inappropriate antidiuretic hormone (SIADH) who has a weight gain of 1.5 pounds since yesterday. 2. The client diagnosed with a pituitary tumor who has developed diabetes insipidus (DI) and has an intake of 1,500 mL and an output of 1,600 mL in the last 8 hours. 3. The client diagnosed with syndrome of inappropriate antidiuretic hormone (SIADH) who is having muscle twitching. 4. The client diagnosed with diabetes insipidus (DI) who is complaining of feeling tired after having to get up at night.

3

The nurse is preparing to care for a client after a lumbar puncture. The nurse should plan to place the client in which best position immediately after the procedure? 1.Prone in semi-Fowler's position 2.Supine in semi-Fowler's position 3.Prone with a small pillow under the abdomen 4.Lateral with the head slightly lower than the rest of the body

3

The nurse is presenting a class on chest tubes. Which statement describes a tension pneumothorax? 1. A tension pneumothorax develops when an air-filled bleb on the surface of the lung ruptures. 2. When a tension pneumothorax occurs, the air moves freely between the pleural space and the atmosphere. 3. The injury allows air into the pleural space but prevents it from escaping from the pleural space. 4. A tension pneumothorax results from a puncture of the pleura during a central line placement.

3

The nurse writes a problem of "altered body image" for a 34-year-old client diagnosed with Cushing's disease. Which intervention should be implemented? 1. Monitor blood glucose levels prior to meals and at bedtime. 2. Perform a head-to-toe assessment on the client every shift. 3. Use therapeutic communication to allow the client to discuss feelings. 4. Assess bowel sounds and temperature every four (4) hours.

3

The patient asks about his lab test, which showed a high level of TSH and a low level of T4. You explain: 1. "It means that you have an inconsistency in your thyroid tests, and you will need more testing." 2. "I am sorry. You will have to ask your doctor about your lab results. We are not allowed to discuss them." 3. "The TSH is sending a message to your thyroid gland to increase production, but your thyroid isn't doing that." 4. "That means that you will have to go on hormone therapy for the rest of your life."

3

The wife of the client diagnosed with septic meningitis asks the nurse, "I am soscared. What is meningitis?" Which statement would be the most appropriateresponse by the nurse? 1. "There is bleeding into his brain causing irritation of the meninges." 2. "A virus has infected the brain and meninges, causing inflammation." 3. "This is a bacterial infection of the tissues that cover the brain and spinal cord." 4. "This is an inflammation of the brain parenchyma caused by a mosquito bite."

3

Which assessment data indicate to the nurse the client diagnosed with ARDS has experienced a complication secondary to the ventilator? 1. The client's urine output is 100 mL in four hours 2. The pulse oximeter reading is greater than 95% 3. The client has asymetrical chest expansion 4. THe telemetry reading shows sinus tachycardia

3

Which signs/symptoms indicate the client with hypothyroidism is not taking enough thyroid hormone? 1. Complaints of weight loss and fine tremors. 2. Complaints of excessive thirst and urination. 3. Complaints of constipation and being cold. 4. Complaints of delayed wound healing and belching.

3

the client dx with acute pancreatitis is in pain. which position should the nurse assist the client to assume to help decrease the pain? 1. recommend lying in the prone position with legs extended 2. maintain a tripod position over the bedside table 3. place in side lying position with knees flexed 4. encourage a supine position with a pillow under the knees

3

When caring for a client who's being treated for hyperthyroidism, it's important to: 1. provide extra blankets and clothing to keep the client warm. 2. monitor the client for signs of restlessness, sweating, and excessive weight loss during thyroid replacement therapy. 3. balance the client's periods of activity and rest. 4. encourage the client to be active to prevent constipation.

3 A client with hyperthyroidism needs to be encouraged to balance periods of activity and rest. Many clients with hyperthyroidism are hyperactive and complain of feeling very warm. Consequently, it's important to keep the environment cool and to teach the client how to manage his physical reactions to heat. Clients with hypothyroidism — not hyperthyroidism — complain of being cold and need warm clothing and blankets to maintain a comfortable temperature. They also receive thyroid replacement therapy, often feel lethargic and sluggish, and are prone to constipation. The nurse should encourage clients with hypothyroidism to be more active to prevent constipation.

Which of the following laboratory test results would suggest to the nurse that a client has a corticotropin-secreting pituitary adenoma? 1. High corticotropin and low cortisol levels 2. Low corticotropin and high cortisol levels 3. High corticotropin and high cortisol levels 4. Low corticotropin and low cortisol levels

3 A corticotropin-secreting pituitary tumor would cause high corticotropin and high cortisol levels. A high corticotropin level with a low cortisol level and a low corticotropin level with a low cortisol level would be associated with hypocortisolism. Low corticotropin and high cortisol levels would be seen if there was a primary defect in the adrenal glands.

A nurse is preparing to provide instructions to a client with Addison's disease regarding diet therapy. The nurse understands that which of the following diets would likely be prescribed for this client? 1. Low-protein diet 2. Low-sodium diet 3. High-sodium diet 4. Low-carbohydrate diet

3 A high-sodium, high-complex carbohydrate, and high-protein diet will be prescribed for the client with Addison's disease. To prevent excess fluid and sodium loss, the client is instructed to maintain an adequate salt intake of up to 8 g of sodium daily and to increase salt intake during hot weather, before strenuous exercise, and in response to fever, vomiting, or diarrhea.

The nurse is monitoring a client for signs of hypocalcemia after thyroidectomy. Which sign or symptom, if noted in the client, would most likely indicate the presence of hypocalcemia? 1. Bradycardia 2. Flaccid paralysis 3. Tingling around the mouth 4. Absence of Chvostek's sign

3 After thyroidectomy the nurse assesses the client for signs of hypocalcemia and tetany. Early signs include tingling around the mouth and in the fingertips, muscle twitching or spasms, palpitations or arrhythmias, and Chvostek's and Trousseau's signs. Bradycardia, flaccid paralysis, and absence of Chvostek's sign are not signs of hypocalcemia.

The client diagnosed with a myocardial infarction is six hours post-right femoral percutanous transluminal coronary angioplasty (PTCA), also known as balloon surgery. Which assessment data would require immediate intervention by the nurse? 1. The client is keeping the affected extremity straight 2. The pressure dressing to the right femoral area is intact 3. The client is complaining of numbness in the right foot 4. The client's right pedal pulse is +3 and bounding

3 Any neurovascular assessment data that is abnormal requires intervention by the nurse; numbness may indicate decreased blood flow to the right foot

A patient recovering from thoracic surgery is on long-term mechanical ventilation and becomes very frustrated when he tries to communicate. What intervention should the nurse perform to assist the patient? A) Assure the patient that everything will be all right and that remaining calm is the best strategy. B) Ask a family member to interpret what the patient is trying to communicate. C) Ask the physician to wean the patient off the mechanical ventilator to allow the patient to speak freely. D) Express empathy and then encourage the patient to write, use a picture board, or spell words with an alphabet board.

D

A patient who has been taking corticosteroids has developed a "moon face" and facial redness, and has many bruises on her arms. Which of these is the most appropriate nursing diagnosis? A. Risk for infection B. Imbalanced nutrition: Less than body requirements C. Deficient fluid volume D. Disturbed body image

D

A patient with ICP monitoring has pressure of 12 mm Hg. The nurse understand that this pressure reflects a. a severe decrease in cerebral perfusion pressure b. an alteration in the production of CSF c. the loss of autoregulatory control of ICP d. a normal balance between brain tissue, blood, and CSF

D

A patient with a permanent trach is interested in developing an exercise regime. Which activity does the nurse advise the patient to avoid? A. aerobics B. Tennis C. golf D. swimming

D

A 19 year-old patient being administered PEEP begins to have copious amounts of secretions that she says she "just cannot cough up." Which of the following nursing actions is most appropriate at this time? 1) Assess the patient further and utilize bedside suction equipment. 2) Assess O2 sats and continue to monitor patient if results are 95% or above. 3) Obtain respiratory therapy consult. 4) Obtain an order for a mucolytic agent from the physician.

3 At this time, the nurse should obtain a respiratory therapy consult. Any break in the closed ventilator system causes the loss of PEEP, so respiratory therapy needs to be consulted to add in-line suctioning.

The nurse is assessing a client with possible Cushing's syndrome. In a client with Cushing's syndrome, the nurse would expect to find: 1. hypotension. 2. thick, coarse skin. 3. deposits of adipose tissue in the trunk and dorsocervical area. 4. weight gain in arms and legs.

3 Because of changes in fat distribution, adipose tissue accumulates in the trunk, face (moonface), and dorsocervical areas (buffalo hump). Hypertension is caused by fluid retention. Skin becomes thin and bruises easily because of a loss of collagen. Muscle wasting causes muscle atrophy and thin extremities.

During the first 24 hours after a client is diagnosed with Addisonian crisis, which intervention should the nurse perform frequently? 1. Weigh the client. 2. Test urine for ketones. 3. Assess vital signs. 4. Administer oral hydrocortisone.

3 Because the client in Addisonian crisis is unstable, vital signs and fluid and electrolyte balance should be assessed every 30 minutes until he's stable. Daily weights are sufficient when assessing the client's condition. The client shouldn't have ketones in his urine, so there is no need to assess the urine for their presence. Oral hydrocortisone isn't administered during the first 24 hours in severe adrenal insufficiency.

The nurse is admitting a client diagnosed with primary adrenal cortex insufficiency(Addison's disease). When assessing the client, which clinical manifestations would thenurse expect to find? 1. Moon face, buffalo hump, and hyperglycemia. 2. Hirsutism, fever, and irritability. 3. Bronze pigmentation, hypotension, and anorexia. 4. Tachycardia, bulging eyes, and goiter.

3 Bronze pigmentation of the skin, particularly of the knuckles and other areas of skincreases, occurs in Addison's disease.Hypotension and anorexia also occur withAddison's.

A client with primary diabetes insipidus is ready for discharge on desmopressin (DDAVP). Which instruction should the nurse provide? 1. "Administer desmopressin while the suspension is cold." 2. "Your condition isn't chronic, so you won't need to wear a medical identification bracelet." 3. "You may not be able to use desmopressin nasally if you have nasal discharge or blockage." 4. "You won't need to monitor your fluid intake and output after you start taking desmopressin."

3 Desmopressin may not be absorbed if the intranasal route is compromised. Although diabetes insipidus is treatable, the client should wear medical identification and carry medication at all times to alert medical personnel in an emergency and ensure proper treatment. The client must continue to monitor fluid intake and output and get adequate fluid replacement.

A patient with a possible pulmonary embolism complains of chest pain and difficulty breathing. The nurse finds a heart rate of 142 beats/minute, blood pressure of 100/60 mmHg, and respirations of 42 breaths/minute. Which action should the nurse take first? a. Administer anticoagulant drug therapy. b. Notify the patient's health care provider. c. Prepare patient for a spiral computed tomography (CT). d. Elevate the head of the bed to a semi-Fowler's position.

D

A nurse reviews a plan of care for a postoperative client following a thyroidectomy and notes that the client is at risk for breathing difficulty. Which of the following nursing interventions will the nurse suggest to include in the plan of care? 1. Maintain a supine position. 2. Encourage coughing and deep breathing exercises. 3. Monitor neck circumference frequently. 4. Maintain a pressure dressing on the operative site.

3 Following a thyroidectomy, the client should be placed in an upright position to facilitate air exchange. The nurse should assist the client with deep breathing exercises, but coughing is minimized to prevent tissue damage and stress to the incision. A pressure dressing is not placed on the operative site because it could affect breathing. The nurse should monitor the dressing closely and should loosen the dressing if necessary. Neck circumference is monitored at least every 4 hours to assess for postoperative edema.

Following hypophysectomy, a client complains of being very thirsty and having to urinate frequently. The initial nursing action is to: 1. Document the complaints. 2. Increase fluid intake. 3. Check the urine specific gravity. 4. Check for urinary glucose.

3 Following hypophysectomy, diabetes insipidus can occur temporarily because of antidiuretic hormone deficiency. This deficiency is related to surgical manipulation. The nurse should check the urine for specific gravity and report the results if they are less than 1.005. Urinary glucose and diabetes mellitus is not a concern here. In this situation, increasing fluid intake would require a health care provider's prescription. The client's complaint would be documented but not as an initial action.

A preoperative client is scheduled for adrenalectomy to remove a pheochromocytoma. The nurse would most closely monitor which of the following items in the preoperative period? 1. Intake and output 2. Blood urea nitrogen (BUN) 3. Vital signs 4. Urine glucose and ketones

3 Hypertension is the hallmark of pheochromocytoma. Severe hypertension can precipitate a stroke or sudden blindness. Although all the items are appropriate nursing assessments for the client with pheochromocytoma, the priority is to monitor the vital signs, especially the blood pressure.

The nurse is instructing a client with Cushing's syndrome on follow-up care. Which of these client statements would indicate a need for further instruction? 1. "I should avoid contact sports." 2. "I should check my ankles for swelling." 3. "I need to avoid foods high in potassium." 4. "I need to check my blood glucose regularly."

3 Hypokalemia is a common characteristic of Cushing's syndrome, and the client is instructed to consume foods high in potassium. Clients with this condition experience activity intolerance, osteoporosis, and frequent bruising. Fluid volume excess results from water and sodium retention. Hyperglycemia is caused by an increased cortisol secretion.

The nurse is monitoring the respiratory status of a client after creation of a tracheostomy. The nurse understands that oxygen saturation measurements obtained by pulse oximetry may be inaccurate if the client has which coexisting problem? 1.Fever 2.Epilepsy 3.Hypotension 4.Respiratory failure

3 Hypotension, shock, or the use of peripheral vasoconstricting medications may result in inaccurate pulse oximetry readings as a result of impaired peripheral perfusion. Fever and epilepsy would not affect the accuracy of measurement. Respiratory failure also would not affect the accuracy of measurement, although the readings may be abnormally low.

A patient with a subarachnoid hemorrhage is intubated and placed on a mechanical ventilator. When monitoring the patient, the nurse will need to notify the health care provider if the patient develops a. oxygen saturation of 94%. b. respirations of 18 breaths/min. c. green nasogastric tube drainage. d. increased jugular vein distention (JVD).

D

The nurse is caring for clients on a medical floor. Which client should be assessed first? 1. The client diagnosed with syndrome of inappropriate antidiuretic hormone(SIADH) who has a weight gain of 1.5 pounds since yesterday. 2. The client diagnosed with a pituitary tumor who has developed diabetes insipidus(DI) and has an intake of 1500 mL and an output of 1600 mL in the last 8 hours. 3. The client diagnosed with syndrome of inappropriate antidiuretic hormone(SIADH) who is having muscle twitching. 4. The client diagnosed with diabetes insipidus (DI) who is complaining of feelingtired after having to get up at night.

3 Muscle twitching is a sign of early sodium imbalance. If an immediate intervention is not made, the client could begin to seize.

A patient with a trach is being discharged to home. In patient teaching, what does the nurse instruct the patient to do? A. use sterile technique when suctioning B. instill tap water into the artificial airway C. clean the trach tube with soap and water D. Increase the humidity in the home

D

A client receiving thyroid replacement therapy develops the flu and forgets to take her thyroid replacement medicine. The nurse understands that skipping this medication will put the client at risk for developing which life-threatening complication? 1. Exophthalmos 2. Thyroid storm 3. Myxedema coma 4. Tibial myxedema

3 Myxedema coma, severe hypothyroidism, is a life-threatening condition that may develop if thyroid replacement medication isn't taken. Exophthalmos, protrusion of the eyeballs, is seen with hyperthyroidism. Thyroid storm is life-threatening but is caused by severe hyperthyroidism. Tibial myxedema, peripheral mucinous edema involving the lower leg, is associated with hypothyroidism but isn't life-threatening.

Which preprocedure information should be taught to the female client having an exercise stress test in the morning? 1. Wear open-toed shoes to the stress test 2. Inform the client not to wear a bra 3. Do not eat anything for 4 hours 4. Take the beta blocker one hour before the test

3 NPO decreases the chance of aspiration in case of emergency. In addition, if the client has just had a meal, the blood supply will be shunted to the stomach for digestion and away from the heart, perhaps leading to an inaccurate test result.

During the previous few months, a 56-year-old woman felt brief twinges of chest pain while working in her garden and has had frequent episodes of indigestion. She comes to the hospital after experiencing severe anterior chest pain while raking leaves. Her evaluation confirms a diagnosis of stable angina pectoris. After stabilization and treatment, the client is discharged from the hospital. At her follow-up appointment, she is discouraged because she is experiencing pain with increasing frequency. She states that she is visiting an invalid friend twice a week and now cannot walk up the second flight of steps to the friend's apartment without pain. Which of the following measures that the nurse could suggest would most likely help the client deal with this problem? 1.Visit her friend earlier in the day. 2.Rest for at least an hour before climbing the stairs. 3.Take a nitroglycerin tablet before climbing the stairs. 4.Lie down once she reaches the friend's apartment.

3 Nitroglycerin may be used prophylactically before stressful physical activities such as stair climbing to help the client remain pain free. Visiting her friend early in the day would have no impact on decreasing pain episodes. Resting before or after an activity is not as likely to help prevent an activity-related pain episode.

A patient comes into the physician's office after sustaining chemical burns to the left side of his face and right wrist. The nurse realizes that this patient needs to be treated 1. in the outpatient ambulatory clinic. 2. in the emergency department. 3. in a burn center. 4. in the doctor's office and then at home.

3 Rationale: Adult patients who should be treated at burn centers include those with burns that involve the hands, feet, face, eyes, ears, or perineum. Patients having small or noninvasive burns may be managed at an outpatient clinic are mild in nature. The emergency department is a location for evaluation of a burn. The physician's office like the ambulatory clinic can manage mild burns.

An incoherent client with a history of hypothyroidism is brought to the emergency department by the rescue squad. Physical and laboratory findings reveal hypothermia, hypoventilation, respiratory acidosis, bradycardia, hypotension, and nonpitting edema of the face and pretibial area. Knowing that these findings suggest severe hypothyroidism, the nurse prepares to take emergency action to prevent the potential complication of: 1. thyroid storm. 2. cretinism. 3. myxedema coma. 4. Hashimoto's thyroiditis.

3 Severe hypothyroidism may result in myxedema coma, in which a drastic drop in the metabolic rate causes decreased vital signs, hypoventilation (possibly leading to respiratory acidosis), and nonpitting edema. Thyroid storm is an acute complication of hyperthyroidism. Cretinism is a form of hypothyroidism that occurs in infants. Hashimoto's thyroiditis is a common chronic inflammatory disease of the thyroid gland in which autoimmune factors play a prominent role.

The nurse is completing discharge teaching to the client diagnosed with acute pancreatitis. Which instruction should the nurse discuss with the client? 1. Instruct the client to decrease alcohol intake. 2. Explain the need to avoid all stress. 3. Discuss the importance of stopping smoking. 4. Teach the correct way to take pancreatic enzymes

3 Smoking stimulates the pancreas to release pancreatic enzymes and should be stopped.

The client has just returned from a cardiac catherization. Which assessment data would warrant immediate intervention from the nurse? 1. The client's BP is 110/70 and pulse is 90 2. The client's groin dressing is dry and intact 3. The client refuses to keep the leg straight 4. The client denies any numbness and tingling

3 The client bends the legs, it could cause insertion site bleeding. This is arterial blood and the client could bleed to death very quickly, so this requires immediate intervention.

The client is admitted to the medical unit with a diagnosis of rule out diabetes insipidus(DI). Which instructions should the nurse teach regarding a fluid deprivation test? 1. The client will be asked to drink 100 mL of fluid as rapidly as possible and then will not be allowed fluid for 24 hours. 2. The client will be given an injection of antidiuretic hormone, and urine output will be measured for four (4) to six (6) hours. 3. The client will be NPO, and vital signs and weights will be done hourly until theend of the test. 4. An IV will be started with normal saline, and the client will be asked to try and hold the urine in the bladder until a sonogram can be done.

3 The client is deprived of all fluids, and if the client has DI the urine production will not diminish. Vital signs and weights are taken every hour to determine circulatory status. If a marked decrease in weight or vital signs occurs, the test is immediately terminated.

A client with Cushing's syndrome verbalizes concern to the nurse regarding the appearance of the buffalo hump that has developed. Which response by the nurse is appropriate? 1. "Don't be concerned, this problem can be covered with clothing." 2. "This is permanent, but looks are deceiving and not that important." 3. "Usually, these physical changes slowly improve following treatment." 4. "Try not to worry about it. There are other things to be concerned about."

3 The client with Cushing's syndrome should be reassured that most physical changes resolve with treatment. Options 1, 2, and 4 are not therapeutic responses.

A client with hyperthyroidism is about to receive radioactive iodine as an outpatient. What safety measures should the nurse teach the client to protect his family while he undergoes treatment? 1. Good hand washing 2. How to isolate himself in one room of the house 3. Use of disposable eating utensils 4. Not worrying about precautions

3 The client with hyperthyroidism can receive radioactive iodine as an outpatient with some precautions, such as using disposable eating utensils, and avoiding kissing, sexual intercourse, and holding babies. Good hand washing is always necessary to prevent the spread of infection; however, it provides no protection against radioactive iodine therapy. Isolation isn't necessary, but radiation precautions are

A client has been diagnosed with pheochromocytoma. Which clinical manifestation is most indicative of this condition? 1. Water loss 2. Bradycardia 3. Hypertension 4. Decreased cardiac output

3 The client with pheochromocytoma has a benign or malignant tumor in the adrenal medulla. Because the medulla secretes epinephrine and norepinephrine, the client will exhibit signs related to excesses of these catecholamines, including tachycardia, increased cardiac output, and increased blood pressure. Vasoconstriction of the renal arteries triggers the renin-angiotensin system, resulting in water reabsorption and retention.

The unlicensed nursing assistant complains to the nurse that she has filled the water pitcher four (4) times during the shift for a client diagnosed with a closed head injury and the client has asked for the pitcher to be filled again. Which intervention should the nurse do first? 1. Tell the unlicensed nursing assistant to fill the pitcher again. 2. Instruct the unlicensed nursing assistant to start measuring I & O. 3. Assess the client for polyuria and polydipsia. 4. Check the client's BUN and creatinine levels.

3 The first action should be to determine if the client is experiencing polyuria and polydipsia as a result of developing diabetes insipidus, a complication of the head trauma.

A patient with a tracheostomy without a tube in place develops increased coughing, inability to expectorate secretions, and difficulty breathing. What are these assessment findings related to? A. overinflation of the pilot balloon B. tracheoesophageal fistula C. cuff leak and rupture D. tracheal stenosis

D

A patient with severe pericarditis has developed a large pericardial effusion. The patient is symptomatic. The physician orders what type of procedure to help treat this condition? A. Pericardiectomy B. Heart catheterization C. Thoracotomy D. Pericardiocentesis

D

A client with suspected Cushing's syndrome is scheduled for adrenal venography. A nurse has provided instructions to the client regarding the test. Which statement by the client indicates a need for further instruction? 1. "I need to sign an informed consent." 2. "The insertion site will be locally anesthetized." 3. "I will be placed in a high-sitting position for the test." 4. "I may feel a burning sensation after the dye is injected."

3 The test aids in determining whether signs and symptoms are caused by abnormalities in the adrenal gland. The nurse assesses the client for allergies to iodine before the test. The client is informed that the supine position is necessary to access the femoral vein. An informed consent form is required, the insertion site will be locally anesthetized, and the client will experience a transient burning sensation after the dye is injected.

The male client diagnosed with syndrome of inappropriate antidiuretic hormone(SIADH) secondary to cancer of the lung tells the nurse that he would like to discontinue the fluid restriction and does not care if he dies. Which action by the nurse would be an example of the ethical principle of autonomy? 1. Discuss the information the client told the nurse with the health-care provider and significant other. 2. Explain that it is possible that the client would seize if he drank fluid beyond the restrictions. 3. Notify the health-care provider of the client's wishes and give the client fluids as desired. 4. Allow the client an extra drink of water and explain that the nurse could get into trouble if the client tells the health-care provider.

3 This is an example of autonomy (the client has the right to decide for himself).

A nurse is caring for a 29 year-old patient on a med-surg unit with 3 lower rib fractures. Which of the following findings, if noted by the nurse, is most concerning? 1) Patient rates pain 8/10. 2) Patient reports feeling muscle spasms over the fracture area when he coughs. 3) Patient's temperature is 99.8F. 4) The nurse feels a crackling, grating sensation over the lower ribs.

3 This patient has spiked a fever which, even though it is slight, could be indicative of pneumonia or atelectasis. This needs to be further investigated. Crepitus and muscle spasms over the area are expected. Extreme pain is also expected, and would be the nurse's immediate concern after addressing the patient's elevated temperature.

the client dx with end stage renal failure and ascites is scheduled for a paracentesis. which client teaching should the nurse discuss with the client? 1. explain the procedure will be done in the operating room 2. instruct the client a foley cath will have to be inserted 3. tell the client v/s will be taken frequently after the procedure 4. provide instructions on holding the breath when the hcp inserts the cath

3 hot water increases pruritus and soap will cause dry skin, which increases pruritus, therefore, the nurse should discuss this with the UAP

the male client dx with chronic pancreatitis calls and reports to the clinic nurse he has been having a lot of gas along with frothy and very foul smelling stools. which intervention should the nurse implement? 1. explain this is common for chronic pancreatitis 2. ask the client to bring in a stool specimen to the clinic 3. arrange an appointment with the hcp for today 4. discuss the need to decrease fat in the diet so this wont happen

3 steatorrhea (fatty, frothy, foul smelling stool) is caused by a decrease in pancreatic enzyme secretion and indicates impaired digestion and possibly an increase in the severity of the pancreatitis. the client should see the hcp

A physician has just performed a thoracostomy for a pleural effusion. The nurse handed the patient tubing from the drain to the physician, who attached it to the chest tube. The drain is properly filled with water and placed in an upright position below the patient's chest. The physician orders suction to the chest drain system. With a dry suction control chamber (as is present in the Atrium Oasis), how should the nurse adjust the vacuum source? A. Adjust the vacuum source until the dial on the vacuum regulator reads -20mmHg B. Adjust the vacuum source until constant, gentle bubbling just begins in the suction control chamber C. Adjust the vacuum source until the bellows indicator is all the way to the right of the indicator window D. Adjust the vacuum source until the bellows indicator reaches the arrow mark in the indicator window

D

The nurse should include which interventions in the plan of care for a client with hyperthyroidism? (SATA) 1. Provide a warm environment for the client. 2. Instruct the client to consume a low-fat diet. 3. A thyroid-releasing inhibitor will be prescribed. 4. Encourage the client to consume a well-balanced diet. 5. Instruct the client that thyroid replacement therapy will be needed. 6. Instruct the client that episodes of chest pain are expected to occur.

34 The clinical manifestations of hyperthyroidism are the result of increased metabolism caused by high levels of thyroid hormone. Interventions are aimed at reduction of the hormones and measures to support the signs and symptoms related to an increased metabolism. The client often has heat intolerance and requires a cool environment. The nurse encourages the client to consume a well-balanced diet because clients with this condition experience increased appetite. Iodine preparations are used to treat hyperthyroidism. Iodine preparations decrease blood flow through the thyroid gland and reduce the production and release of thyroid hormone. Thyroid replacement is needed for hypothyroidism. The client would notify the health care provider if chest pain occurs because it could be an indication of an excessive medication dose.

A physician has prescribed propylthiouracil (PTU) for a client with hyperthyroidism and the nurse develops a plan of care for the client. A priority nursing assessment to be included in the plan regarding this medication is to assess for: a) relief of pain b) signs of renal toxicity c) signs and symptoms of hyperglycemia d) signs and symptoms of hypothyroidism

D

A week after kidney transplantation, a client develops a temperature of 101, the BP is elevated, and there is tenderness over the transplanted kidney. The serum creatinine is rising and urine output is decreased. The x-ray indicates that the transplanted kidney is enlarged. Based on these assessment findings, the nurse anticipates which treatment? a. antibiotic therapy b. peritoneal dialysis c. removal of the transplanted kidney d. increased immunosuppression therapy

D

After bone marrow aspiration is performed on a child, the nurse should a. apply an adhesive bandage b. place the child in the Trendelenburg position c. ask the child to remain in the supine position d. apply a pressure bandage

D

After change-of-shift report, which patient should the nurse assess first? a. 72-year-old with cor pulmonale who has 4+ bilateral edema in his legs and feet b. 28-year-old with a history of a lung transplant and a temperature of 101° F (38.3° C) c. 40-year-old with a pleural effusion who is complaining of severe stabbing chest pain d. 64-year-old with lung cancer and tracheal deviation after subclavian catheter insertion

D

After receiving change-of-shift report about the following four patients, which patient should the nurse assess first? a. 39-year-old with pericarditis who is complaining of sharp, stabbing chest pain b. 56-year-old with variant angina who is to receive a dose of nifedipine (Procardia) c. 65-year-old who had a myocardial infarction (MI) 4 days ago and is anxious about the planned discharge d. 59-year-old with unstable angina who has just returned to the unit after having a percutaneous coronary intervention (PCI)

D

After receiving change-of-shift report, which patient will the nurse assess first? a. A patient with cystic fibrosis who has thick, green-colored sputum b. A patient with pneumonia who has coarse crackles in both lung bases c. A patient with emphysema who has an oxygen saturation of 91% to 92% d. A patient with septicemia who has intercostal and suprasternal retractions

D

An elementary teacher who has just been diagnosed with epilepsy after having a generalized tonic-clonic seizure tells the nurse, "I cannot teach anymore, it will be too upsetting if I have a seizure at work." Which response by the nurse is best? a. "You may want to contact the Epilepsy Foundation for assistance." b. "You might benefit from some psychologic counseling at this time." c. "The Department of Vocational Rehabilitation can help with work retraining." d. "Most patients with epilepsy are well controlled with antiseizure medications."

D

A client with end-stage renal disease is receiving continuous ambulatory peritoneal dialysis. The nurse is monitoring the client for signs of complications associated with peritoneal dialysis. Select all that apply. 1. Pruritus 2. Oliguria 3. Tachycardia 4. Cloudy outflow 5. Abdominal pain

345 Tachycardia can be caused by peritonitis, a complication of peritoneal dialysis; the heart rate increases to meet the metabolic demands associated with infection. Cloudy or opaque dialysate outflow (effluent) is the earliest sign of peritonitis; it is caused by the constituents associated with an infectious process. Abdominal pain is associated with peritonitis, a complication of peritoneal dialysis; pain results from peritoneal inflammation, abdominal distention, and involuntary muscle spasms. Severe itching (pruritus) is caused by metabolic waste products that are deposited in the skin; dialysis removes metabolic waste products, preventing this adaptation associated with kidney failure. The production of abnormally small amounts of urine (oliguria) is a sign of kidney failure, not a complication of peritoneal dialysis.

The nurse is assessing the functioning of a chest tube drainage system in a client who has just returned from the recovery room following a thoracotomy with wedge resection. Which are the expected assessment findings? Select all that apply. 1.Excessive bubbling in the water seal chamber 2.Vigorous bubbling in the suction control chamber 3.Drainage system maintained below the client's chest 4.50 mL of drainage in the drainage collection chamber 5.Occlusive dressing in place over the chest tube insertion site 6.Fluctuation of water in the tube in the water seal chamber during inhalation and exhalation

3456

A client with Addison's disease is scheduled for discharge after being hospitalized for an adrenal crisis. Which statements by the client would indicate that client teaching has been effective? (SATA) 1. "I have to take my steroids for 10 days." 2. "I need to weigh myself daily to be sure I don't eat too many calories." 3. "I need to call my doctor to discuss my steroid needs before I have dental work." 4. "I will call the doctor if I suddenly feel profoundly weak or dizzy." 5. "If I feel like I have the flu, I'll carry on as usual because this is an expected response." 6. "I need to obtain and wear a Medic Alert bracelet."

346 Dental work can be a cause of physical stress; therefore, the client's physician needs to be informed about the dental work and an adjusted dosage of steroids may be necessary. Fatigue, weakness, and dizziness are symptoms of inadequate dosing of steroid therapy; the physician should be notified if these symptoms occur. A Medic Alert bracelet allows health care providers to access the client's history of Addison's disease if the client is unable to communicate this information. A client with Addison's disease doesn't produce enough steroids, so routine administration of steroids is a lifetime treatment. Daily weights should be monitored to monitor changes in fluid balance, not calorie intake. Influenza is an added physical stressor and the client may require an increased dosage of steroids. The client shouldn't "carry on as usual."

The nurse should include which interventions in the plan of care for a client with hypothyroidism? Select all that apply. 1.Provide a cool environment for the client. 2.Instruct the client to consume a high-fat diet. 3.Instruct the client about thyroid replacement therapy. 4.Encourage the client to consume fluids and high-fiber foods in the diet. 5.Inform the client that iodine preparations will be prescribed to treat the disorder. 6.Instruct the client to contact the health care provider (HCP) if episodes of chest pain occur

346 The clinical manifestations of hypothyroidism are the result of decreased metabolism from low levels of thyroid hormone. Interventions are aimed at replacement of the hormone and providing measures to support the signs and symptoms related to decreased metabolism. The client often has cold intolerance and requires a warm environment. The nurse encourages the client to consume a well-balanced diet that is low in fat for weight reduction and high in fluids and high-fiber foods to prevent constipation. Iodine preparations may be used to treat hyperthyroidism. Iodine preparations decrease blood flow through the thyroid gland and reduce the production and release of thyroid hormone; they are not used to treat hypothyroidism. The client is instructed to notify the HCP if chest pain occurs because it could be an indication of over replacement of thyroid hormone.

A nurse has reinforced instructions to the client with hyperparathyroidism regarding home care measures related to exercise. Which statement by the client indicates a need for further instruction? Select all that apply. 1. "I enjoy exercising but I need to be careful." 2. "I need to pace my activities throughout the day." 3. "I need to limit playing football to only the weekends." 4. "I should gauge my activity level by my energy level." 5. "I should exercise in the evening to encourage a good sleep pattern."

35 The client should be instructed to avoid high-impact activity or contact sports such as football. Exercising late in the evening may interfere with restful sleep. The client with hyperparathyroidism should pace activities throughout the day and plan for periods of uninterrupted rest. The client should plan for at least 30 minutes of walking each day to support calcium movement into the bones. The client should be instructed to use energy level as a guide to activity.

A client is seen in the clinic with a possible parathormone deficiency. Diagnosis of this condition includes the analysis of serum electrolytes. Which electrolytes would the nurse expect to be abnormal? (SATA) 1. Sodium 2. Potassium 3. Calcium 4. Chloride 5. Glucose 6. Phosphorous

36 A client with a parathormone deficiency has abnormal calcium and phosphorous values because parathormone regulates these two electrolytes. Potassium, chloride, sodium, and glucose aren't affected by a parathormone deficiency.

A client has a chest tube attached to a water seal drainage system. As part of routine nursing care, the nurse should ensure that which intervention is implemented? 1.The water seal chamber has continuous bubbling, and assessment for crepitus is done once a shift. 2.The amount of drainage into the chest tube is noted and recorded every 24 hours in the client's record. 3.The suction control chamber has sterile water added every shift, and the system is kept below waist level. 4.The connection between the chest tube and the drainage system is taped, and an occlusive dressing is maintained at the insertion site.

4

A client is scheduled for a cardiac catherization using a radiopaque dye. Which of the following assessments is most critical before the procedure? 1.Intake and output 2.Baseline peripheral pulse rates 3.Height and weight 4.Allergy to iodine or shellfish

4

A client who has undergone a subtotal thy- roidectomy is subject to complications in the first 48 hours after surgery. The nurse should obtain and keep at the bedside equipment to: 1. begin total parenteral nutrition. 2. initiate defibrillation. 3. administer tube feedings. 4. perform a tracheotomy.

4

A nurse knows that which of the following is the most important item required at the bedside of a patient with an endotracheal tube at all times? 1) A 4x4 piece of sterile gauze and a 100 mL container of sterile water. 2) A portable chest X-ray machine with a lead vest. 3) A soft-bristled toothbrush and chlorahexadine-based oral care supplies. 4) An Ambu bag

4

After administering somatropin (Serostim), the nurse would assess for adverse effects by monitoring which parameters? 1. Serum potassium levels 2. Mental status 3. Respiratory rate 4. Serum glucose levels

4

The client has a right-sided chest tube. As the client is getting out of the bed it is acci- dentally pulled out of the pleural space. Which action should the nurse implement first? 1. Notify the health-care provider to have chest tubes reinserted STAT. 2. Instruct the client to take slow shallow breaths until the tube is reinserted. 3. Take no action and assess the client's respiratory status every 15 minutes. 4. Tape a petroleum jelly occlusive dressing on three (3) sides to the insertion site.

4

The client is three (3) days postoperative unilateral adrenalectomy. Which discharge instructions should the nurse teach? 1. Discuss the need for lifelong steroid replacement. 2. Instruct the client on administration of vasopressin. 3. Teach the client to care for the suprapubic Foley catheter. 4. Tell the client to notify the HCP if the incision is inflamed.

4

The client with ARDS is on a mechanical ventilator. Which intervention should be included in the nursing care plan addressing the endotracheal tube care? 1. Do not move or touch the ET tube 2. Obtain a chest x-ray daily 3. Determine if the ET cuff is deflated 4. Ensure that the ET tube is secured

4

The low-exhaled volume alarm sounds on a mechanical ventilator of a client with an endotracheal tube. The nurse determines that the cause for alarm activation may be which complication? 1.Excessive secretions 2.Kinks in the ventilator tubing 3.The presence of a mucous plug 4.Displacement of the endotracheal tube

4

The nurse caring for a client with a closed chest drainage system notes that the fluctuation (tidaling) in the water-seal compartment has stopped. On the basis of this assessment finding, the nurse would suspect which occurrence? 1.The system needs changing. 2.Suction needs to be increased. 3.Suction needs to be decreased. 4.The chest tubes are obstructed.

4

The nurse creates a plan of care for a child at risk for tonic-clonic seizures. In the plan of care, the nurse identifies seizure precautions and documents that which item(s) need to be placed at the child's bedside? 1. Emergency cart 2. Tracheotomy set 3. Padded tongue blade 4. Suctioning equipment and oxygen

4

The nurse is administering a calcium channel blocker to the client diagnosed with a myocardial infarction. Which assessment data would cause the nurse to question administering this medication? 1. The client's apical pulse is 64 2. The client's calcium level is elevated 3. The client's telemetry shows occasional PVCs 4. The client's blood pressure is 90/62

4

The nurse is admitting a client to rule out aldosteronism. Which assessment data support the client's diagnosis? 1. Temperature. 2. Pulse. 3. Respirations. 4. Blood pressure.

4

The nurse is assessing a client with multiple trauma who is at risk for developing acute respiratory distress syndrome. The nurse should assess for which earliest sign of acute respiratory distress syndrome? 1.Bilateral wheezing 2.Inspiratory crackles 3.Intercostal retractions 4.Increased respiratory rate

4

When discussing recent onset of feelings of sadness and depression in a client with hypothyroidism who has just started to take thyroid hormone replacement, the nurse should inform the client that these feelings are: 1. the effects of thyroid hormone replacement therapy and will diminish over time. 2. related to thyroid hormone replacement therapy and will not diminish over time. 3. a normal part of having a chronic illness. 4. most likely related to low thyroid hormone levels and will improve with treatment.

4

Which nursing intervention should be included in the plan of care for the client diagnosed with hyperthyroidism? 1. Increase the amount of fiber in the diet. 2. Encourage a low-calorie, low-protein diet. 3. Decrease the client's fluid intake to 1,000 mL/day. 4. Provide six (6) small, w

4

Which sign/symptom should the nurse expect in the client diagnosed with syndrome of inappropriate antidiuretic hormone (SIADH)? 1. Excessive thirst. 2. Orthopnea. 3. Ascites. 4. Concentrated urine output.

4

Which type of precautions should the nurse implement for the client diagnosed withseptic meningitis? 1. Standard Precautions. 2. Airborne Precautions. 3. Contact Precautions. 4. Droplet Precautions.

4

a client has been given a dx of acute pancreatitis. the nurse will assess this client for: 1. hyperkalemia 2. metabolic acidosis 3. hypocalcemia 4. hyperglycemia

4

the client dx with liver failure is experiencing pruritus secondary to severe jaundice. which action by the unlicensed assistive personnel warrants intervention by the nurse? 1. the UAP is assisting the client to take a hot soapy shower 2. the UAP applies an emollient to the clients legs and back 3. the UAP puts mittens on both hands of the client 4. the cUAP pats the clients skin dry with a clean towel

4

The nurse is caring for a client with Addison's disease. The client asks the nurse about the risks associated with this disease, specifically about addisonian crisis. Regarding prevention of this complication, how should the nurse inform the client? 1. "You can take either hydrocortisone or fludrocortisone for replacement." 2. "You need to take your fludrocortisone 3 times a day to prevent a crisis." 3. "You need to increase salt in your diet, particularly during stressful situations." 4. "You need to decrease your dosages of glucocorticoids and mineralocorticoids during stressful situations."

4 Addison's disease is a result of adrenocortical insufficiency, and management is focused on treating the underlying cause. Hormone therapy is used for replacement. Hydrocortisone has both glucocorticoid and mineralocorticoid properties and needs to be taken 3 times daily, with two thirds of the daily dose taken on awakening. Fludrocortisone is taken once daily in the morning. Salt additives are necessary, particularly during times of stress, to compensate for excess heat or humidity as a result of the condition. There needs to be an increased dose of cortisol given for stressful situations such as surgery or hospitalization.

The nurse is caring for a client with a diagnosis of Addison's disease and is monitoring the client for signs of addisonian crisis. The nurse should assess the client for which manifestation that would be associated with this crisis? 1. Agitation 2. Diaphoresis 3. Restlessness 4. Severe abdominal pain

4 Addisonian crisis is a serious life-threatening response to acute adrenal insufficiency that most commonly is precipitated by a major stressor. The client in addisonian crisis may demonstrate any of the signs and symptoms of Addison's disease, but the primary problems are sudden profound weakness; severe abdominal, back, and leg pain; hyperpyrexia followed by hypothermia; peripheral vascular collapse; coma; and renal failure. The remaining options do not identify clinical manifestations associated with addisonian crisis.

The nurse is providing an in-service on thyroid disorders. One of the attendees asks the nurse, "Why don't the people in the United States get goiters as often?" Which statement by the nurse is the best response? 1. "It is because of the screening techniques used in the United States." 2. "It is a genetic predisposition rare in North Americans." 3. "The medications available in the United States decrease goiters." 4. "Iodized salt helps prevent the development of goiters in the United States."

4 Almost all of the iodine entering the body is retained in the thyroid gland. A deficiency in iodine will cause the thyroid gland to work hard and enlarge, which is called a goiter. Goiters are commonly seen in geographical regions having an iodine deficiency. Most table salt in the United States has iodine added.

A nurse is assessing the blood pressure of a client diagnosed with primary hypertension. The nurse ensures accurate measurement by avoiding which of the following? 1.Seating the client with arm bared, supported, and at heart level. 2.Measuring the blood pressure after the client has been seated quietly for 5 minutes. 3.Using a cuff with a rubber bladder that encircles at least 80% of the limb. 4.Taking a blood pressure within 15 minutes after nicotine or caffeine ingestion.

4 BP should be taken with the client seated with the arm bared, positioned with support and at heart level. The client should sit with the legs on the floor, feet uncrossed, and not speak during the recording. The client should not have smoked tobacco or taken in caffeine in the 30 minutes preceding the measurement. The client should rest quietly for 5 minutes before the reading is taken. The cuff bladder should encircle at least 80% of the limb being measured. Gauges other than a mercury sphygmomanometer should be calibrated every 6 months to ensure accuracy

A client with Graves' disease has exophthalmos and is experiencing photophobia. Which intervention would best assist the client with this problem? 1. Administering methimazole (Tapazole) every 8 hours 2. Lubricating the eyes with tap water every 2 to 4 hours 3. Instructing the client to avoid straining or heavy lifting 4. Obtaining dark glasses for the client

4 Because photophobia (light intolerance) accompanies this disorder, dark glasses are helpful in alleviating the symptom. Medical therapy for Graves' disease does not help alleviate the clinical manifestation of exophthalmos. Other interventions may be used to relieve the drying that occurs from not being able to completely close the eyes; however, the question is asking what the nurse can do for photophobia. Tap water, which is hypotonic, could actually cause more swelling to the eye because it could pull fluid into the interstitial space. In addition, the client is at risk for developing an eye infection because the solution is not sterile. There is no need to prevent straining with exophthalmos.

A nurse is caring for a client experiencing thyroid storm. Which of the following would be a priority concern for this client? 1. Inability to cope with the treatment plan 2. Lack of sexual drive 3. Self-consciousness about body appearance 4. Potential for cardiac disturbances

4 Clients in thyroid storm are experiencing a life-threatening event, which is associated with uncontrolled hyperthyroidism. It is characterized by high fever, severe tachycardia, delirium, dehydration, and extreme irritability. The signs and symptoms of the disorder develop quickly, and therefore emergency measures must be taken to prevent death. These measures include maintaining hemodynamic status and patency of airway as well as providing adequate ventilation. Options 1, 2, and 3 are not a priority in the care of the client in thyroid storm.

The client is 3 hours post myocardial infarction. Which data would warrant immediate intervention by the nurse? 1. Bilateral peripheral pulses 2+ 2. The pulse oximeter reading is 96% 3. The urine output is 240 mL in the last 4 hours 4. Cool, clammy, diaphoretic skin

4 Cold, clammy skin is an indicator of cardiogenic shock, which is a complication of MI and warrants immediate intervention.

Which clinical manifestation should the nurse expect to note when assessing a client with Addison's disease? 1. Edema 2. Obesity 3. Hirsutism 4. Hypotension

4 Common manifestations of Addison's disease include postural hypotension from fluid loss, syncope, muscle weakness, anorexia, nausea, vomiting, abdominal cramps, weight loss, depression, and irritability. The manifestations in options 1, 2, and 3 are not associated with Addison's disease.

A client with acute appendicitis develops a fever, tachycardia, and hypotension. Based on these assessment findings, the nurse should further assess the client for which of the following complications? 1. Deficient fluid volume. 2. Intestinal obstruction. 3. Bowel ischemia. 4. Peritonitis.

4 Complications of acute appendicitis are perforation, peritonitis, and abscess development. Signs of the development of peritonitis include abdominal pain and distention, tachycardia, tachypnea, nausea, vomiting, and fever. Because peritonitis can cause hypovolemic shock, hypotension can develop. Deficient fluid volume would not cause a fever. Intestinal obstruction would cause abdominal distention, diminished or absent bowel sounds, and abdominal pain. Bowel ischemia has signs and symptoms similar to those found with intestinal obstruction.

A nursing student notes in the medical record that a client with Cushing's syndrome is experiencing body image disturbances. The need for additional education regarding this problem is identified when the nursing student suggests which nursing intervention? 1. Encouraging the client's expression of feelings 2. Evaluating the client's understanding of the disease process 3. Encouraging family members to share their feelings about the disease process 4. Evaluating the client's understanding that the body changes need to be dealt with

4 Evaluating the client's understanding that the body changes that occur in this disorder need to be dealt with is an inappropriate nursing intervention. This option does not address the client's feelings. Options 1, 2, and 3 are appropriate because they address the client and family feelings regarding the disorder.

The nurse is developing a plan of care for the client diagnosed with acquired immunodeficiency syndrome (AIDS) who has developed an infection in the adrenal gland.Which problem would have the highest priority? 1. Altered body image. 2. Activity intolerance. 3. Impaired coping. 4. Fluid volume deficit.

4 Fluid volume deficit (dehydration) can leadto circulatory impairment and hyperkalemia.

A nurse is caring for a postoperative adrenalectomy client. Which of the following does the nurse specifically monitor for in this client? 1. Peripheral edema 2. Bilateral exophthalmos 3. Signs and symptoms of hypocalcemia 4. Signs and symptoms of hypovolemia

4 Following adrenalectomy, the client is at risk for hypovolemia. Aldosterone, secreted by the adrenal cortex, plays a major role in fluid volume balance by retaining sodium and water. A deficiency of adrenocortical hormones does not cause the clinical manifestations noted in options 1, 2, and 3.

For a client with Graves' disease, which nursing intervention promotes comfort? 1. Restricting intake of oral fluids 2. Placing extra blankets on the client's bed 3. Limiting intake of high-carbohydrate foods 4. Maintaining room temperature in the low-normal range

4 Graves' disease causes signs and symptoms of hypermetabolism, such as heat intolerance, diaphoresis, excessive thirst and appetite, and weight loss. To reduce heat intolerance and diaphoresis, the nurse should keep the client's room temperature in the low-normal range. To replace fluids lost via diaphoresis, the nurse should encourage, not restrict, intake of oral fluids. Placing extra blankets on the bed of a client with heat intolerance would cause discomfort. To provide needed energy and calories, the nurse should encourage the client to eat high-carbohydrate foods

The nurse caring for a client who has had a subtotal thyroidectomy reviews the plan of care and determines which problem is the priority for this client in the immediate postoperative period? 1. Dehydration 2. Infection 3. Urinary retention 4. Bleeding

4 Hemorrhage is one of the most severe complications that can occur following thyroidectomy. The nurse must frequently check the neck dressing for bleeding and monitor vital signs to detect early signs of hemorrhage, which could lead to shock. T3 and T4 do not regulate fluid volumes in the body. Infection is a concern for any postoperative client but is not the priority in the immediate postoperative period. Urinary retention can occur in postoperative clients as a result of medication and anesthesia but is not the priority from the options provided.

A nurse is caring for a client following a thyroidectomy. The client tells the nurse that she is concerned because of voice hoarseness. The client asks the nurse whether the hoarseness will subside. The nurse appropriately tells the client that the hoarseness: 1. Indicates nerve damage 2. Is harmless but permanent 3. Will worsen before it subsides 4. Is normal and will gradually subside

4 Hoarseness that develops in the postoperative period is usually the result of laryngeal pressure or edema and will resolve within a few days. The client should be reassured that the effects are transitory. Options 1, 2, and 3 are incorrect.

A nurse working on an endocrine nursing unit understands that which correct concept is used in planning care? 1. Clients with Cushing's syndrome are likely to experience episodic hypotension. 2. Clients with hyperthyroidism must be monitored for weight gain. 3. Clients who have diabetes insipidus should be assessed for fluid excess. 4. Clients who have hyperparathyroidism should be protected against falls.

4 Hyperparathyroidism is a disease that involves excess secretion of parathyroid hormone (PTH). Elevation of PTH causes excess calcium to be removed from the bones. There is a decline in bone mass, which may cause a fracture if a fall occurs. Cushing's syndrome is likely to cause hypertension. Clients with hypothyroidism must be monitored for weight gain and clients with hyperthyroidism must be monitored for weight loss. Clients who have diabetes insipidus should be assessed for fluid deficit.

A client with Addison's disease comes to the clinic for a follow-up visit. When assessing this client, the nurse should stay alert for signs and symptoms of: 1. calcium and phosphorus abnormalities. 2. chloride and magnesium abnormalities. 3. sodium and chloride abnormalities. 4. sodium and potassium abnormalities.

4 In Addison's disease, a form of adrenocortical hypofunction, aldosterone secretion is reduced. Aldosterone promotes sodium conservation and potassium excretion. Therefore, aldosterone deficiency increases sodium excretion, predisposing the client to hyponatremia, and inhibits potassium excretion, predisposing the client to hyperkalemia. Because aldosterone doesn't regulate calcium, phosphorus, chloride, or magnesium, an aldosterone deficiency doesn't affect levels of these electrolytes directly.

A client with severe head trauma sustained in a car accident is admitted to the intensive care unit. Thirty-six hours later, the client's urine output suddenly rises above 200 ml/hour, leading the nurse to suspect diabetes insipidus. Which laboratory findings support the nurse's suspicion of diabetes insipidus? 1. Above-normal urine and serum osmolality levels 2. Below-normal urine and serum osmolality levels 3. Above-normal urine osmolality level, below-normal serum osmolality level 4. Below-normal urine osmolality level, above-normal serum osmolality level

4 In diabetes insipidus, excessive polyuria causes dilute urine, resulting in a below-normal urine osmolality level. At the same time, polyuria depletes the body of water, causing dehydration that leads to an above-normal serum osmolality level. For the same reasons, diabetes insipidus doesn't cause above-normal urine osmolality or below-normal serum osmolality levels.

Which nursing diagnosis takes highest priority for a client with hyperthyroidism? 1. Risk for imbalanced nutrition: More than body requirements related to thyroid hormone excess 2. Risk for impaired skin integrity related to edema, skin fragility, and poor wound healing 3. Disturbed body image related to weight gain and edema 4. Imbalanced nutrition: Less than body requirements related to thyroid hormone excess

4 In the client with hyperthyroidism, excessive thyroid hormone production leads to hypermetabolism and increased nutrient metabolism. These conditions may result in a negative nitrogen balance, increased protein synthesis and breakdown, decreased glucose tolerance, and fat mobilization and depletion. This puts the client at risk for marked nutrient and calorie deficiency, making Imbalanced nutrition: Less than body requirements the most important nursing diagnosis. Options 2 and 3 may be appropriate for a client with hypothyroidism, which slows the metabolic rate.

A client newly diagnosed with renal failure has just been started on peritoneal dialysis. During the infusion of the dialysate, the client complains of abdominal pain. Which action by the nurse is appropriate? 1. Stop the dialysis. 2. Slow the infusion. 3. Decrease the amount to be infused. 4. Explain that the pain will subside after the first few exchanges.

4 Pain during the inflow of dialysate is common during the first few exchanges because of peritoneal irritation; however, the pain usually disappears after 1 to 2 weeks of treatment. The infusion amount should not be decreased, and the infusion should not be slowed or stopped.

When assessing a client with pheochromocytoma, a tumor of the adrenal medulla that secretes excessive catecholamine, the nurse is most likely to detect: 1. a blood pressure of 130/70 mm Hg. 2. a blood glucose level of 130 mg/dl. 3. bradycardia. 4. a blood pressure of 176/88 mm Hg.

4 Pheochromocytoma, a tumor of the adrenal medulla that secretes excessive catecholamine, causes hypertension, tachycardia, hyperglycemia, hypermetabolism, and weight loss. It isn't associated with the other options.

Before undergoing a subtotal thyroidectomy, a client receives potassium iodide (Lugol's solution) and propylthiouracil (PTU). The nurse would expect the client's symptoms to subside: 1. in a few days. 2. in 3 to 4 months. 3. immediately. 4. in 1 to 2 weeks.

4 Potassium iodide reduces the vascularity of the thyroid gland and is used to prepare the gland for surgery. Potassium iodide reaches its maximum effect in 1 to 2 weeks. PTU blocks the conversion of thyroxine to triiodothyronine, the more biologically active thyroid hormone. PTU effects are also seen in 1 to 2 weeks. To relieve symptoms of hyperthyroidism in the interim, clients are usually given a beta-adrenergic blocker such as propranolol.

A client has begun medication therapy with propylthiouracil. The nurse should assess the client for which condition as an adverse effect of this medication? 1. Joint pain 2. Renal toxicity 3. Hyperglycemia 4. Hypothyroidism

4 Propylthiouracil is prescribed for the treatment of hyperthyroidism. Excessive dosing with this agent may convert a hyperthyroid state to a hypothyroid state. If this occurs, the dosage should be reduced. Temporary administration of thyroid hormone may be required to treat the hypothyroid state. Propylthiouracil is not used for relief of joint pain. It does not cause renal toxicity or hyperglycemia.

A female patient comes into the clinic complaining of nausea and vomiting after spending the weekend at a seaside resort. Which of the following should be the most important assessment for the nurse? 1. normal rest and sleep pattern 2. typical meal pattern 3. if the patient had to change time zones when traveling to the resort 4. if the patient has been sunburned

4 Rationale: Sunburns result from exposure to ultraviolet light. Because the skin remains intact, the manifestations in most cases are mild and are limited to pain, nausea, vomiting, skin redness, chills, and headache. The patient has not reported concerns which will support issues with sleep pattern, diet, and travel.

The nursing care for the client in addisonian crisis should include which intervention? 1. Encouraging independence with activities of daily living (ADLs) 2. Allowing ambulation as tolerated 3. Offering extra blankets and raising the heat in the room to keep the client warm 4. Placing the client in a private room

4 The client in addisonian crisis has a reduced ability to cope with stress due to an inability to produce corticosteroids. Compared to a multibed room, a private room is easier to keep quiet, dimly lit, and temperature controlled. Also, visitors can be limited to reduce noise, promote rest, and decrease the risk of infection. The client should be kept on bed rest, receiving total assistance with ADLs to avoid stress as much as possible. Because extremes of temperature should be avoided, measures to raise the body temperature, such as extra blankets and turning up the heat, should be avoided.

The client diagnosed with hyperthyroidism is prescribed propylthiouracil. Which laboratory data should the nurse monitor? 1. The client's arterial blood gases (ABGs). 2. The client's serum potassium level. 3. The client's red blood cell (RBC) count. 4. The client's white blood cell (WBC) count.

4 The client receiving PTU, a hyperthyroid treatment, is at risk for agranulocytosis; therefore, the client's WBC count should be checked periodically. Because agranulocytosis puts the client at greater risk for infection, efforts to control invasion of microbes should be strictly observed.

A client has been diagnosed with Cushing's syndrome. The nurse should assess the client for which expected manifestations of this disorder? 1. Dizziness 2. Weight loss 3. Hypoglycemia 4. Truncal obesity

4 The client with Cushing's syndrome may exhibit a number of different manifestations. These may include moon face, truncal obesity, and a "buffalo hump" fat pad. Other signs include hyperglycemia, hypernatremia, hypocalcemia, peripheral edema, hypertension, increased appetite, and weight gain. Dizziness is not part of the clinical picture for this disorder.

Which nursing intervention should be included in the plan of care for the client diagnosed with hyperthyroidism? 1. Increase the amount of fiber in the diet. 2. Encourage a low-calorie, low-protein diet. 3. Decrease the client's fluid intake to 1,000 mL/day. 4. Provide six (6) small, well-balanced meals a day.

4 The client with hyperthyroidism has an increased appetite; therefore, well-balanced meals served several times throughout the day will help with the client's constant hunger

Which nursing measure would be effective in preventing complications in a client with Addison's disease? 1. Restricting fluid intake 2. Offering foods high in potassium 3. Checking family support systems 4. Monitoring the blood glucose

4 The decrease in cortisol secretion that characterizes Addison's disease can result in hypoglycemia. Therefore monitoring the blood glucose would detect the presence of hypoglycemia so that it can be treated early to prevent complications. Fluid intake should be encouraged to compensate for dehydration. Potassium intake should be restricted because of hyperkalemia. Option 3 is not a priority for this client.

A nurse is providing wound care to a client 1 day after the client underwent an appendectomy. A drain was inserted into the incisional site during surgery. Which action should the nurse perform when providing wound care? 1. Remove the dressing and leave the incision open to air. 2. Remove the drain if wound drainage is minimal. 3. Gently irrigate the drain to remove exudate. 4. Clean the area around the drain moving away from the drain.

4 The nurse should gently clean the area around the drain by moving in a circular motion away from the drain. Doing so prevents the introduction of microorganisms to the wound and drain site. The incision cannot be left open to air as long as the drain is intact. The nurse should note the amount and character of wound drainage, but the surgeon will determine when the drain should be removed. Surgical wound drains are not irrigated.

A client is scheduled for a transsphenoidal hypophysectomy to remove a pituitary tumor. Preoperatively, the nurse should assess for potential complications by: 1. testing for ketones in the urine. 2. testing urine specific gravity. 3. checking temperature every 4 hours. 4. performing capillary glucose testing every 4 hours.

4 The nurse should perform capillary glucose testing every 4 hours because excess cortisol may cause insulin resistance, placing the client at risk for hyperglycemia. Urine ketone testing isn't indicated because the client does secrete insulin and, therefore, isn't at risk for ketosis. Urine specific gravity isn't indicated because although fluid balance can be compromised, it usually isn't dangerously imbalanced. Temperature regulation may be affected by excess cortisol and isn't an accurate indicator of infection.

Which statement made by the client makes the nurse suspect the client is experiencing hyperthyroidism? 1. "I just don't seem to have any appetite anymore." 2. "I have a bowel movement about every 3 to 4 days." 3. "My skin is really becoming dry and coarse." 4. "I have noticed all my collars are getting tighter."

4 The thyroid gland (in the neck) en-larges as a result of the increased need for thyroid hormone production; an enlarged gland is called a goiter.

The nurse explains to a client with thyroid disease that the thyroid gland normally produces: 1. iodine and thyroid-stimulating hormone (TSH). 2. thyrotropin-releasing hormone (TRH) and TSH. 3. TSH, T3, and calcitonin. 4. T3, T4, and calcitonin.

4 The thyroid gland normally produces thyroid hormone (T3 and T4) and calcitonin. TSH is produced by the pituitary gland to regulate the thyroid gland. TRH is produced by the hypothalamus gland to regulate the pituitary gland.

During preoperative teaching for a client who will undergo subtotal thyroidectomy, the nurse should include which statement? 1. "The head of your bed must remain flat for 24 hours after surgery." 2. "You should avoid deep breathing and coughing after surgery." 3. "You won't be able to swallow for the first day or two." 4. "You must avoid hyperextending your neck after surgery."

4 To prevent undue pressure on the surgical incision after subtotal thyroidectomy, the nurse should advise the client to avoid hyperextending the neck. The client may elevate the head of the bed as desired and should perform deep breathing and coughing to help prevent pneumonia. Subtotal thyroidectomy doesn't affect swallowing.

A client with myxedema has changes in intellectual function such as impaired memory, decreased attention span, and lethargy. The client's husband is upset and shares his concerns with the nurse. Which statement by the nurse is helpful to the client's husband? 1. "Would you like me to ask the health care provider for a prescription for a stimulant?" 2. "Give it time. I've seen dozens of clients with this problem that fully recover." 3. "I don't blame you for being frustrated, because the symptoms will only get worse." 4. "It's obvious that you are concerned about your wife's condition, but the symptoms may improve with continued therapy."

4 Using therapeutic communication techniques, the nurse acknowledges the husband's concerns and conveys that the client's symptoms are common with myxedema. With thyroid hormone therapy, these symptoms should decrease, and cognitive function often returns to normal. Option 1 is not helpful, and it blocks further communication. Option 3 is pessimistic and untrue. Option 2 is not appropriate and offers false reassurance.

The nurse is performing an assessment on a client with a diagnosis of Cushing's syndrome. Which should the nurse expect to note on assessment of the client? 1. Skin atrophy 2. The presence of sunken eyes 3. Drooping on 1 side of the face 4. A rounded "moonlike" appearance to the face

4 With excessive secretion of adrenocorticotropic hormone (ACTH) and chronic corticosteroid use, the person with Cushing's syndrome develops a rounded moonlike face; prominent jowls; red cheeks; and hirsutism on the upper lip, lower cheek, and chin. The remaining options are not associated with the assessment findings in Cushing's syndrome.

the client is dx with acute pancreatitis. which health care providers admitting order should the nurse question? 1. bedrest with bathroom privleges. 2. initiate iv therapy of D5W at 125 mL/hr 3. weigh client daily 4. low fat, low carb diet

4 the client will be NPO, which will decrease stimulation of the pancreatic enzymes, resulting in decreased autodigestion of the pancreas, therefore decreasing pain

The nurse is caring for a client with a diagnosis of Cushing's syndrome. Which expected signs and symptoms should the nurse monitor for? Select all that apply. 1. Anorexia 2. Dizziness 3. Weight loss 4. Moon face 5. Hypertension6. Truncal obesity

456 A client with Cushing's syndrome may exhibit a number of different manifestations. These could include moon face, truncal obesity, and a buffalo hump fat pad. Other signs include hypokalemia, peripheral edema, hypertension, increased appetite, and weight gain. Dizziness is not part of the clinical picture for this disorder.

A 30 year old female is being treated for infective endocarditis with IV antibiotics. At the beginning of the hospitalization, the patient's symptoms were severe and sudden with a high fever but are now controlled. She has no significant health history other than 2 cesarean sections in the past. She is being prepped for a central line placement so she can be discharged home with home health to continue the 4 week antibiotic regime. What is type of infective endocarditis this classified as based on the information listed? A. Acute Infective Endocarditis B. Subacute Infective Endocarditis C. Non-infective Endocarditis D. Pericarditis

A

A 6-year-old boy is admitted to the pediatric unit with chills and a fever of 104°F (40°C). What physiological process explains why the child is at risk for developing dyspnea? A: Fever increases metabolic demands, requiring increased oxygen need. B: Blood glucose stores are depleted, and the cells do not have energy to use oxygen. C: Carbon dioxide production increases as result of hyperventilation. D: Carbon dioxide production decreases as a result of hypoventilation.

A

A client asks the nurse what might trigger a seizure. Which situation should the nurse include in the​ response? A.Exposure to toxins B.Decreased intracranial pressure C.Low body temperature D.Low blood pressure

A

A client with acquired immunodeficiency syndrome (AIDS) is experiencing shortness of breath related to Pneumocystis jiroveci pneumonia. Which measure should the nurse include in the plan of care to assist the client in performing activities of daily living? a) provide supportive care with hygiene needs b) provide meals and snacks with high-protein, high calorie, and high-nutritional value c) provide small, frequent meals

A

A client's electrocardiogram strip shows atrial and ventricular rates of 80 complexes per minute. The PR interval is 0.14 second, and the QRS complex measures 0.08 second. The nurse interprets this rhythm is: A) Normal sinus rhythm B) Sinus bradycardia C) Sinus tachycardia D) Sinus dysrhythmia

A

A major advantage of peritoneal dialysis is a. the diet is less restricted and dialysis can be performed at home b. the dialysate is biocompatible and causes no long-term consequences c. high glucose concentrations of the dialysate causes a reduction in appetite, promoting weight loss d. no medications are required because of the enhances efficiency of the peritoneal membrane in removing toxins

A

A man with ESRD is scheduled for HD following healing of an arteriovenous fistula. What should the nurse explain to him that will occur during dialysis? a. he will be able to visit, read, sleep, or watch TV while reclining in a chair b. he will be placed on a cardiac monitor to detect any adverse effects that may occur c. the dialyzer will remove and hold part of his blood for 20-30 minutes to remove the waste products d. a large catheter with two lumens will be inserted into the fistula to send blood to and return it from the dialyzer

A

A nurse is inserting an oropharyngeal airway for a patient who vomits when it is inserted. Which action would be the first that should be taken by the nurse related to this occurrence? a) Quickly position the patient on his or her side. b) Put on disposable gloves and remove the oral airway. c) Check that the airway is the appropriate size for the patient. d) Put on sterile gloves and suction the airway.

A

A patient has ICP monitoring with an intraventricular catheter. A priority nursing intervention for the patient is a. aseptic technique to prevent infection b. constant monitoring of ICP waveforms c. removal of CSF to maintain normal ICP d. sampling CSF to determine abnormalities

A

A patient has a nursing diagnosis of risk for ineffective cerebral tissue perfusion related to cerebral edema. An appropriate nursing intervention for the patient is a. avoiding positioning the patient with neck and hip flexion b. maintaining hyperventilation to a PaCO2 of 15 to 20 mm Hg c. clustering nursing activities to provide periods of uninterrupted rest d. routine suctioning to prevent accumulation of respiratory secretions

A

A patient has a temporary trach following surgery to the neck area to remove a benign tumor. Which nursing intervention is performed to prevent obstruction of the tracheostomy tube? A. Provide tracheal suctioning when there are noisy respirations B. Provide oxygenation to maintain pulse oximeter readings C. inflate the cuff to maximum pressure and check it once per shift D. suction regularly and as needed (prn) with an oral suction device

A

A patient has been discharged home after thoracic surgery. The home care nurse performs the initial visit and finds the patient discouraged and saddened. The client states, I am recovering so slowly. I really thought I would be better by now. What nursing action should the nurse prioritize? A) Provide emotional support to the patient and family. B) Schedule a visit to the patients primary physician within 24 hours. C) Notify the physician that the patient needs a referral to a psychiatrist. D) Place a referral for a social worker to visit the patient.

A

A patient in asystole is likely to receive which of the following drug treatments? a. Atropine and epinephrine b. Lidocaine and amiodarone c. Digoxin and procainamide d. β-Adrenergic blockers and dopamine

A

A patient is being transferred to your unit with acute cholecystitis. In report the transferring nurse tells you that the patient has a positive Murphy's Sign. You know that this means: A. The patient stops breathing in when the examiner palpates under the ribs on the right upper side of the abdomen at the midclavicular line. B. The patient stops breathing out when the examiner palpates under the ribs on the right upper side of the abdomen at the midclavicular line. C. The patient verbalizes pain when the lower right quadrant is palpated. D. The patient reports pain when pressure is applied to the right lower quadrant but then reports an increase in pain intensity when the pressure is released.

A

A patient is brought to the emergency department unconscious following a barbiturate overdose. Which potential complication will the nurse include when developing the plan of care? a. Hypercapnic respiratory failure related to decreased ventilatory effort b. Hypoxemic respiratory failure related to diffusion limitations c. Hypoxemic respiratory failure related to shunting of blood d. Hypercapnic respiratory failure related to inc

A

A patient is exhibiting signs of a pneumothorax following tracheostomy. The surgeon inserts a chest tube into the anterior chest wall. What should the nurse tell the family is the primary purpose of this chest tube? A) To remove air from the pleural space B) To drain copious sputum secretions C) To monitor bleeding around the lungs D) To assist with mechanical ventilation

A

A patient is receiving preoperative teaching for a partial laryngectomy and will have a tracheostomy postoperatively. How does the nurse define a tracheostomy to the patient? A. opening in the trachea that enables breathing B. temporary procedure that will be reversed at a later date C. technique using positive pressure to improve gas exchange D. Procedure that holds open the upper airways

A

A patient received a kidney transplant last month. Because of the effects of immunosuppressive drugs and CKD, what complication of transplantation should the nurse be assessing the patient for? a. infection b. rejection c. malignancy d. cardiovascular disease

A

A patient who has tested positive for the human immunodeficiency virus (HIV) arrives at the clinic with a report of fever, nonproductive cough, and fatigue. The patient's CD4 count is 184 cells/mcL. How should the healthcare provider interpret these findings?Please choose from one of the following options. A. The patient is diagnosed with acquired immunodeficiency syndrome (AIDS). B.The patient is now in the latent stages of HIV infection C.These findings provide evidence that the patient has seroconverted. D. This is an expected finding because the patient has tested positive for HIV.

A

A patient, who has recovered from cholecystitis, is being discharged home. What meal options below are best for this patient? A. Baked chicken with steamed carrots and rice B. Broccoli and cheese casserole with gravy and mashed potatoes C. Cheeseburger with fries D. Fried chicken with a baked potato

A

A pediatric client is not responding to treatment for acute respiratory distress syndrome​ (ARDS) and requires intubation. The mother is anxious that the procedure will hurt. Which is the best response by the​ nurse? A. "We will administer medication to help him sleep through the intubation and as needed while he is on the​ ventilator." B. ​"He may feel a little​ nervous, but he will get used to the ventilator soon​ enough." C. "He will actually feel better and may even like the ventilator because he will be able to breathe more​ comfortably." D. "Don't worry; intubation and using a ventilator​ doesn't hurt at​ all."

A

A physician has ordered an arterial blood gas test for a client with a respiratory disorder. What is the most common role of the nurse in performing the arterial blood gas test? a) Implement measures to prevent complications after arterial puncture. b) Measure the partial pressure of oxygen dissolved in plasma. c) Measure the percentage of hemoglobin saturated with oxygen. d) Perform the arterial puncture to obtain the specimen.

A

An unconscious patient with multiple injuries arrives in the emergency department. Which nursing intervention receives the highest priority? A. establishing an airway B. replacing blood loss C. stopping bleeding from open wounds D. checking for a neck fracture

A

During admission of a patient with a severe head injury to the ED, the nurse places highest priority on assessment for a. patency of of airway b. presence of a neck injury c. neurologic status with Glascow Coma Scale d. CSF leakage from ears and nose

A

During the past 6 months, a client diagnosed with acquired immunodeficiency syndrome has had chronic diarrhea and has lost 18 pounds. Additional assessment findings include tented skin turgor, dry mucous membranes, and listleness. Which nursing diagnosis focuses attention on the client's most immediate problem? A. Deficient fluid volume related to diarrhea and abnormal fluid loss B. Imbalanced nutrition: less than body requirements related to nausea and vomiting C. Disturbed thought processes related to central nervous system effects of disease D. Diarrhea related to the disease process and acute infection

A

For a male client who has a chest tube connected to a closed water-seal drainage system, the nurse should include which action in the plan of care? a. Measuring and documenting the drainage in the collection chamber b. Maintaining continuous bubbling in the water-seal chamber c. Keeping the collection chamber at chest level d. Stripping the chest tube every hour

A

Gina, a home health nurse is visiting a home care client with advanced lung cancer. Upon assessing the client, the nurse discovers wheezing, bradycardia, and a respiratory rate of 10 breaths/minute. These signs are associated with which condition? a. Hypoxia b. Delirium c. Hyperventilation d. Semiconsciousness

A

In preparing a patient in the ICU for oral ET intubation, what should the nurse do that is most important for successful intubation? a. place the patient supine with the head extended and the neck flexed b. tell the patient that the tongue must be extruded while the tube is inserted c. position the patient supine with the head hanging over the edge of the bed to align the mouth and trachea d. inform the patient that while it will not be possible to talk during insertion of the tube, speech will be possible after it is correctly placed

A

In teaching parents how to minimize or prevent bleeding episodes when the child is myelosuppressed, the nurse includes what information? a. Meticulous mouth care is essential to avoid mucositis. b. Rectal temperatures are necessary to monitor for infection. c. Intramuscular injections are preferred to intravenous ones. d. Platelet transfusions are given to maintain a count greater than 50,000/mm3.

A

Medication has been ineffective in controlling a​ client's seizures. Which treatment option should the nurse suspect will be discussed with the​ client? A.Surgical resection B.Herbal remedies C.More sleep D.Head massage

A

Mr. G requires neuromuscular blockade to facilitate mechanical ventilation. Which is not a true statement when providing nursing care to paralyzed patients? A) Because patients under neuromuscular blockade are unable to react to the environment, special safety precautions are not needed. B) Pain medication is required because neuromuscular blocking agents do not have an analgesic effect. C) Patients under the influence of neuromuscular blocking agents are fully aware of activity around them. D) The nurse must be alert for complications of immobility such as deep vein thrombosis, pressure ulcers, and atelectasis.

A

The OR nurse is setting up a water-seal chest drainage system for a patient who has just had a thoracotomy. The nurse knows that the amount of suction in the system is determined by the water level. At what suction level should the nurse set the system? A) 20 cm H2O B) 15 cm H2O C) 10 cm H2O D) 5 cm H2O

A

The advantage of continuous replacement therapy over hemodialysis is its ability to a. remove fluid without the use of a dialysate. b. remove fluid in less than 24 hours. c. allow the patient to receive the therapy at the work site. d. be administered through a peripheral line.

A

The client is struggling with body image after breast cancer surgery. Which behavior indicates to the nurse that the client is maladaptive? A. Avoiding eye contact with staff B. Saying, "I feel like less of a woman" C. Requesting a temporary prosthesis immediately D. Saying, "This is the ugliest scar ever"

A

The client with continuous ambulatory peritoneal dialysis (CAPD) has cloudy dialysate. Which of the following is the best initial nursing action? a) send fluid to the laboratory for culture b) administer antibiotic c) do nothing, this is expected d) stop drainage of fluid

A

The critical care charge nurse is responsible for the care of four patients receiving mechanical ventilation. Which patient is most at risk for failure to wean and ventilator dependence? a. A 68-year-old patient with a history of smoking and emphysema b. A 57-year-old patient who experienced a cardiac arrest c. A 49-year-old postoperative patient who had a colectomy d. A 29-year-old patient who is recovering from flail chest

A

The critical care nurse and the other members of the care team are assessing the patient to see if he is ready to be weaned from the ventilator. What are the most important predictors of successful weaning that the nurse should identify? A) Stable vital signs and ABGs B) Pulse oximetry above 80% and stable vital signs C) Stable nutritional status and ABGs D) Normal orientation and level of consciousness

A

The nurse cares for a patient who has just had a thoracentesis. Which assessment information obtained by the nurse is a priority to communicate to the health care provider? a. Oxygen saturation is 88%. b. Blood pressure is 145/90 mm Hg. c. Respiratory rate is 22 breaths/minute when lying flat. d. Pain level is 5 (on 0 to 10 scale) with a deep breath.

A

The nurse goes to assess a new patient and finds him lying supine in bed. The patient tells the nurse that he feels short of breath. Which nursing action should the nurse perform first? A: Raise the head of the bed to 45 degrees. B: Take his oxygen saturation with a pulse oximeter. C: Take his blood pressure and respiratory rate. D: Notify the health care provider of his shortness of breath

A

The nurse has instructed a patient who is receiving hemodialysis about appropriate dietary choices. Which menu choice by the patient indicates that the teaching has been successful? a. Scrambled eggs, English muffin, and apple juice b. Oatmeal with cream, half a banana, and herbal tea c. Split-pea soup, whole-wheat toast, and nonfat milk d. Cheese sandwich, tomato soup, and cranberry juice

A

The nurse is assessing the patency of a client's left arm AV fistula prior to initiating HD. Which findings indicate that the fistula is patent? a. palpation of a thrill over the fistula b. presence of a radial pulse in the left wrist c. visualization of enlarged blood vessels at the fistula site d. capillary refill <3 seconds in the nail beds of the fingers on the left hand

A

The nurse is caring for a client with a seizure disorder currently controlled with antiseizure medication. The client​ states, "A friend recommended an herbal supplement for my depression. Can I take​ it?" Which response by the nurse is​ correct? A.​"St. John's wort has been known to decrease the effectiveness of your antiseizure​ medication." B.​"You can take valerian along with your antiseizure medication to help you​ sleep." C.​"You should avoid​ garlic, because it can decrease the effectiveness of your antiseizure​ medication." D.​"Essential oils would be a better option with your antiseizure​ medication."

A

The nurse is caring for a patient who had kidney transplantation several years ago. Which assessment finding may indicate that the patient is experiencing adverse effects to the prescribed corticosteroid? a. Joint pain b. Tachycardia c. Postural hypotension d. Increase in creatinine level

A

The nurse is caring for a patient who is ready to be weaned from the ventilator. In preparing to assist in the collaborative process of weaning the patient from a ventilator, the nurse is aware that the weaning of the patient will progress in what order? A) Removal from the ventilator, tube, and then oxygen B) Removal from oxygen, ventilator, and then tube C) Removal of the tube, oxygen, and then ventilator D) Removal from oxygen, tube, and then ventilator

A

The nurse is caring for a patient with a tracheostomy who has recently been transferred from the ICU. but he has no unusual occurrences related to the tracheostomy or his oxygenation status. What does the routine care for this patient include? A. thorough respiratory assessment at least every 2 hours B. maintaining the cuff pressure between 50 and 100 mm Hg C. Suctioning as needed; maximum suction time of 20 seconds D. changing the tracheostomy dressing once a day

A

The nurse is caring for a postoperative client who has a prescription for meperidine (Demerol) 7 5mg intramuscularly (IM) every 4 hours as needed for pain. Before and after administering Demerol, the nurse would assess which most important sign? a) Respiratory rate and depth b) Urinary intake and output c) Orthostatic blood pressure d) Apical pulse

A

The nurse is informed while receiving a nursing report that the client has been hypoxic during the evening shift. Which assessment finding is consistent with hypoxia? a) Confusion b) Decreased blood pressure c) Decreased respiratory rate d) Hyperactivity

A

The nurse is teaching a client about maintaining a proper diet to prevent an endocrine disorder. Which food does the nurse suggest after the client indicates a dislike of fish? A. Iodized salt for cooking B. More red meat C. More green vegetables D. Salt substitute for cooking

A

The nurse is teaching postmastectomy exercises to the client. Which statement made by the client indicates that teaching has been effective? A. "For the pulley exercise, I'll drape a 6-foot-long rope over a shower curtain rod or over the top of a door." B. "In rope turning, I'll hold the rope with my arms flexed." C. "In rope turning, I'll start by making large circles." D. "With hand wall climbing, I'll walk my hands up the wall and back down until they are at waist level."

A

The nurse needs to apply oxygen to a patient who has a precise oxygen level prescribed. Which of the following oxygen-delivery systems should the nurse select to administer the oxygen to the patient? A: Nasal cannula B: Venturi mask C: Simple face mask without inflated reservoir bag D: Plastic face mask with inflated reservoir bag

A

The nurse notes that a patients endotracheal tube (ET), which was at the 21-cm mark, is now at the 24-cm mark and the patient appears anxious and restless. Which action should the nurse take first? a. Listen to the patients lungs. b. Offer reassurance to the patient. c. Bag the patient at an FIO2 of 100%. d. Notify the patients health care provider.

A

The nurse should question a prescription for aminoglutethimide (Cytadren) in a patient with which condition? A. Addison's disease B. Adrenal malignancy C. Cushing's syndrome D. Metastatic breast cancer

A

The nurse should question the use of barbiturates for the treatment of seizure activity if prescribed for which of the following clients? A. 30-year-old pregnant female B. 24-year-old male with new diagnosis of seizures C. 55-year-old female with history of diabetes mellitus D. 45-year-old male with history of hypertension

A

The nurse teaches the client that the major difference between angina and pain associated with myocardial infarction (MI) is that: A) Angina is relieved with nitroglycerin and rest. B) Angina can be fatal. C) MI pain always radiates to the left arm or jaw. D) MI pain cannot be treated.

A

The nurse will monitor for clinical manifestations of hypercapnia when a patient in the emergency department has a. chest trauma and multiple rib fractures. b. carbon monoxide poisoning after a house fire. c. left-sided ventricular failure and acute pulmonary edema. d. tachypnea and acute respiratory distress syndrome (ARDS).

A

The nursing intervention that would be most helpful for the child who has stomatitis from cancer chemotherapy would be a. an anesthetic preparation without alcohol b. viscous lidocaine c. lemon glycerin swabs d. a mild sedative

A

The patient is admitted with a diagnosis of bacterial meningitis. The patient has a temperature of 101° F and a headache rated as an 8. Which prescription has a priority for you to administer? A. IV cefuroxime (Ceftin) B. Vital signs C. PO acetaminophen (Tylenol) D. Neurologic check

A

The patient reports falling when he his foot got "stuck" on a crack in the sidewalk, hitting his head when he fell, and "passing out". The paramedics found the patient walking at the scene and talking before transporting the patient to the hospital. In the emergency department, the patient starts to lose consciousness. This is a classic scenario for which complication? A. Epidural hematoma B. Subdural hematoma C. Subarachnoid bleed D. Diffuse axial inju

A

The patient with type 1 diabetes mellitus with hypoglycemia is having a seizure. Which medication should the nurse anticipate administering to stop the seizure? A. IV dextrose solution B. IV diazepam (Valium) C. IV phenytoin (Dilantin) D. Oral carbamazepine (Tegretol)

A

The unlicensed assistive personnel (UAP) is assisting with feeding for a patient with severe end-stage chronic obstructive pulmonary disease (COPD). Which instruction will the nurse provide the UAP? A.) Encourage the patient to eat foods that are high in calories and protein. B.) Feed the patient as quickly as possible to prevent early satiety. C.) Offer lots of fluids between bites of food. D.) Try to get the patient to eat everything on the tray

A

Thyroid drugs for hypo-thyroidism: A. Levothyroxine, Liothyronine, Liotrix, Thyroid B. Methimazole (Tapazole), Propylthiouracil (PTU) C. All of the above

A

Toxicity from which of the following medications may cause a client to see a green-yellow halo around lights? A) Digoxin B) Furosemide (Lasix) C) Metoprolol (Lopressor) D) Enalapril (Vasotec)

A

What action does the nurse perform to follow safe technique when using a portable oxygen cylinder? a) Checking the amount of oxygen in the cylinder before using it b) Using a cylinder for a patient transfer that indicates available oxygen is 500 psi c) Placing the oxygen cylinder on the stretcher next to the patient d) Discontinuing oxygen flow by turning cylinder key counterclockwise until tight

A

What is the standard to evaluate the degree of impaired consciousness for a patient with an acute head trauma? A. Best eye opening, verbal response, and motor response B. National Institutes of Health (NIH) Stroke Scale C. Romberg test D. Widening pulse pressure, bradycardia, and respirations

A

What nursing action will limit hypoxia when suctioning a client's airway? A. apply suction only after catheter is inserted B. limit suctioning with catheter to half a minute C. Lubricate the catheter with saline before insertion D. Use a sterile suction catheter for each suctioning episode

A

What pathophysiologic condition can result in ARDS? A. Damage to the alveolar-capillary membrane B. Copious exudates production C. Airway spasms and vasoconstriction D. Change in the inspiratory-to-expiratory ratio

A

What should a male client over age 52 do to help ensure early identification of prostate cancer? a. Have a digital rectal examination and prostate-specific antigen (PSA) test done yearly. b. Have a transrectal ultrasound every 5 years. c. Perform monthly testicular self-examinations, especially after age 50. d. Have a complete blood count (CBC) and blood urea nitrogen (BUN) and creatinine levels checked yearly.

A

When a patient is admitted to the emergency department following a head injury, the nurse's first priority in management of the patient once a patent airway is confirmed is a. maintaining cervical spine precautions b. determining the presence of increased ICP c. monitoring for changes in neurologic status d. establishing IV access with a large-bore catheter

A

When assessing a patient with a traumatic brain injury, you notice uncoordinated movement of the extremities. How would you document this? A. Ataxia B. Apraxia C. Anisocoria D. Anosognosia

A

When assessing for potential adverse effects of fludrocortisone (Florinef), the nurse monitors for signs and symptoms of which condition? A. Hypokalemia B. Hypovolemia C. Hyponatremia . Hypercalcemia

A

When caring for a patient who has just arrived on the medical-surgical unit after having cardiac catheterization, which nursing intervention should the nurse delegate to a licensed practical/vocational nurse (LPN/LVN)? a. Give the scheduled aspirin and lipid-lowering medication. b. Perform the initial assessment of the catheter insertion site. c. Teach the patient about the usual postprocedure plan of care. d. Titrate the heparin infusion according to the agency protocol.

A

When caring for a patient with a left arm arteriovenous fistula, which action will the nurse include in the plan of care to maintain the patency of the fistula? a. Check the fistula site for a bruit and thrill. b. Assess the rate and quality of the left radial pulse. c. Compare blood pressures in the left and right arms. d. Irrigate the fistula site with saline every 8 to 12 hours.

A

When is it beneficial to clamp a patient's chest tube? A. When ordered by a physician to simulate tube removal and assess the patient's response B. Whenever a patient leaves the nursing unit and cannot be monitored C. When ambulating a postoperative patient with a chest tube D. It is never beneficial to clamp a patient's chest tube

A

When the nurse is taking a history for a patient who is a possible candidate for a kidney transplant, which information about the patient indicates that the patient is not an appropriate candidate for transplantation? a. The patient has metastatic lung cancer. b. The patient has poorly controlled type 1 diabetes. c. The patient has a history of chronic hepatitis C infection. d. The patient is infected with the human immunodeficiency virus.

A

Which action will the nurse take when evaluating a patient who is taking phenytoin (Dilantin) for adverse effects of the medication? a. Inspect the oral mucosa. b. Listen to the lung sounds. c. Auscultate the bowel tones. d. Check pupil reaction to light.

A

Which antiepileptic drug is effective for almost all forms of seizures? A. Valproic acid (Depakene) B. Carbamazepine (Tegretol) C. Phenobarbital (Luminal) D. Phenytoin (Dilantin)

A

Which assessment information obtained by the nurse when caring for a patient receiving mechanical ventilation indicates the need for suctioning? a. The respiratory rate is 32 breaths/min. b. The pulse oximeter shows a SpO2 of 93%. c. The patient has not been suctioned for the last 6 hours. d. The lungs have occasional audible expiratory wheezes.

A

Which complication is the result of constant pressure exerted by a tracheostomy cuff causing tracheal dilation and erosion of cartilage? A. Tracheomalacia B. Tracheal stenosis C. Tracheoesophageal fistula D. Trachea-innominate artery fistula

A

Which complication of CKD is treated with erythropoietin? a. anemia b. hypertension c. hyperkalemia d. mineral and bone disorder

A

Which nursing action should be implemented in the care of a patient who is experiencing increased ICP? A. Monitor fluid and electrolyte status astutely. B. Position the patient in a high-Fowler's position. C. Administer vasoconstrictors to maintain cerebral perfusion. D. Maintain physical restraints to prevent episodes of agitation.

A

Which of the following client responses shows a correct understanding of continuous ambulatory peritoneal dialysis (CAPD)? a) I am expected to perform the procedure at home b) the procedure lasts for one hour c) I have to sit and raise my legs during the procedure d) I have to go to the hospital for this procedure

A

Which of the following parameters should be checked before administering digoxin? a.) Apical pulse b.) Blood pressure c.) Radial pulse d.) Respiratory rate

A

Which of the following problems is expected in a client who is in end-stage renal failure? a) anemia b) thalassemia c) renal calculi d) hypotension

A

You recognize that status epilepticus is a medical emergency because A. seizures continue without a return of consciousness. B. fractures of a limb may occur. C. urinary fecal incontinence may occur. D. heart rate becomes bradycardic.

A

an unconscious client assumes a decerebrate posture in response to any noxious stimuli. when drawing a blood sample, the nurse should expect the client to: A. rigidly extend all four extremities B. internally flex the arms and extend the legs C. tightly curl into a fetal position D. internally rotate the arms and legs

A

the nurse reinforces teaching for a pt after a cholecystectomy on a low fat diet. the nurse will know that the pt understands the diet if which of the following menu items is selected? a. roast chicken rice gelatin dessert b. cream of chicken soup, milk, gelatin dessert c. meat loaf, mashed potatoes with small amount of gravy, green beans d. turkey and cheese sandwich on whole grain bread, milk

A

A client has just been admitted to the emergency department and requires high-flow oxygen therapy after suffering facial burns and smoke inhalation. Which oxygen delivery device should the nurse use initially? A. Face tent B. Venturi mask C. Nasal cannula D. Non-rebreather mask

A A client with smoke inhalation and facial burns who requires high-flow oxygen should initially be placed on a face tent because this is the only noninvasive high-flow device that will minimize painful and contaminating contact with burned facial tissue.

The nurse is reviewing the health history for an older adult client recently admitted to the burn unit with severe burns to the upper body from a house fire. The nurse plans to contact the health care provider if the client's history reveals which condition? A. Heart failure B. Diverticulitis C. Hypertension D. Emphysema

A A client's health history, including any pre-existing illnesses, must be known for appropriate management. Obtain specific information about the client's history of cardiac or kidney problems, chronic alcoholism, substance abuse, and diabetes mellitus. Any of these problems can influence fluid resuscitation. The stress of a burn injury can make a mild disease process worsen. In older clients, especially those with cardiac disease, a complicating factor in fluid resuscitation may be heart failure or myocardial infarction. Diverticulitis, hypertension and emphysema are important to be aware of in guiding treatment options. However, heart failure is the main concern when attempting to optimize this older client's fluid resuscitation.

The nurse should encourage fluids every 2 hours for older adult clients because of a decrease in which factor? A. Antidiuretic hormone (ADH) production B. General metabolism C. Glucose tolerance D. Ovarian production of estrogen

A A decrease in ADH production causes urine to be more dilute, so urine might not concentrate when fluid intake is low. The older adult is at greater risk for dehydration as a result of urine loss. A decrease in general metabolism causes decreased tolerance to cold, decreased appetite, and decreased heart rate and blood pressure; it is not related to fluid intake or hydration. A decrease in glucose tolerance does not affect fluid intake or hydration. A decrease in estrogen production causes a decrease in bone density and is not related to fluid intake and hydration.

Which assessment finding in a patient admitted with acute decompensated heart failure (ADHF) requires the most immediate action by the nurse? a.Oxygen saturation of 88% b.Weight gain of 1 kg (2.2 lb) c.Heart rate of 106 beats/minute d.Urine output of 50 mL over 2 hours

A A decrease in oxygen saturation to less than 92% indicates hypoxemia. The nurse should administer supplemental oxygen immediately to the patient. An increase in apical pulse rate, 1-kg weight gain, and decreases in urine output also indicate worsening heart failure and require nursing actions, but the low oxygen saturation rate requires the most immediate nursing action.

Based on the nurse's knowledge of glucocorticoids, what instructions should be given for this drug? A. Take the medication every evening. B. Inform the patient that the drug can be taken with coffee. C. Advise that the drug does not have to be tapered before stopped. D. Advise to take drug with milk, other dairy products, or food.

D

The nurse is providing postoperative care for a 30-year-old female patient after an appendectomy. The patient has tested positive for human immunodeficiency virus (HIV). What type of precautions should the nurse observe to prevent the transmission of this disease? a. Droplet precautions b. Contact precautions c. Airborne precautions d. Standard precautions

D

After receiving change-of-shift report about these four clients, which client does the nurse attend to first? a) Client with acute adrenal insufficiency who has a blood glucose of 36 mg/dL b) Client with diabetes insipidus who has a dose of desmopressin (DDAVP) due c) Client with hyperaldosteronism who has a serum potassium of 3.4 mEq/L d) Client with pituitary adenoma who is reporting a severe headache

A A glucose level of 36 mg/dL is considered an emergency; this client must be assessed and treated immediately. Although it is important to maintain medications on schedule, the client requiring a dose of desmopressin is not the first client who needs to be seen. A serum potassium of 3.4 mEq/L in the client with hyperaldosteronism may be considered normal (or slightly hypokalemic), based on specific hospital levels. The client reporting a severe headache needs to be evaluated as soon as possible after the client with acute adrenal insufficiency. As an initial measure, the RN could delegate obtaining vital signs to unlicensed assistive personnel.

A hospital patient is immunocompromised because of stage 3 HIV infection and the physician has ordered a chest radiograph. How should the nurse most safely facilitate the test? A) Arrange for a portable x-ray machine to be used. B) Have the patient wear a mask to the x-ray department. C) Ensure that the radiology department has been disinfected prior to the test. D) Send the patient to the x-ray department, and have the staff in the department wear masks.

A A patient who is immunocompromised is at an increased risk of contracting nosocomial infections due to suppressed immunity. The safest way the test can be facilitated is to have a portable x-ray machine in the patient's room. This confers more protection than disinfecting the radiology department or using masks

A pt with a history of peptic ulcer disease has presented to the emergency department with complaints of severe abdominal pain and a rigid, boardlike abdomen, prompting the health care team to suspect a perforated ulcer. Which of the following actions should the nurse anticipate? A) Providing IV fluids and inserting a nasogastric tube B) Administering oral bicarbonate and testing the patient's gastric pH level C) Performing a fecal occult blood test and administering IV calcium gluconate D) Starting parenteral nutrition and placing the patient in a high-Fowler's position

A A perforated peptic ulcer requires IV replacement of fluid losses and continued gastric aspiration by NG tube. Nothing is given by mouth and gastric pH testing is not a priority. Calcium gluconate is not a medication directly relevant to the patient's suspected diagnosis and parenteral nutrition is not a priority in the short term.

The nurse is teaching a patient taking an antithyroid medication to avoid food items high in iodine. Which food item should the nurse instruct the patient to avoid? A. Milk B. Eggs C. Chicken D. Seafood

D

The nurse is teaching the client with a new permanent pacemaker. Which statement by the client indicates the need for further discharge education? a. "I will be able to shower again soon." b. "I need to take my pulse every day." c. "I might trigger airport security metal detectors." d. "I no longer need my heart pills."'

D

A patient with HIV infection has begun experiencing severe diarrhea. What is the most appropriate nursing intervention to help alleviate the diarrhea? A) Administer antidiarrheal medications on a scheduled basis, as ordered. B) Encourage the patient to eat three balanced meals and a snack at bedtime. C) Increase the patient's oral fluid intake. D) Encourage the patient to increase his or her activity level.

A Administering antidiarrheal agents on a regular schedule may be more beneficial than administering them on an as-needed basis, provided the patient's diarrhea is not caused by an infectious microorganism. Increased oral fluid may exacerbate diarrhea; IV fluid replacement is often indicated. Small, more frequent meals may be beneficial, and it is unrealistic to increase activity while the patient has frequent diarrhea

Because of the risk of agranulocytosis, the nurse should teach the patient who has been prescribed PTU to report which symptom? a. Anorexia b. Bleeding gums c. Pale conjunctiva d. Sore throat

D

The nurse preparing to administer a dose of calcium acetate to a patient with chronic kidney disease (CKD). Which laboratory result will the nurse monitor to determine if the desired effect was achieved? a. Sodium b. Potassium c. Magnesium d. Phosphorus

D

Which statement is the most important for a nurse to make to a patient who is taking methimazole? A) "You need to notify your doctor if you have a sore throat and fever." B) "Another medication can be given if you experience any nausea." C) "You may experience some muscle soreness with this medicine." D) "Headache and dizziness may occur but not very frequently."

A Agranulocytosis (the absence of granulocytes to fight infection) is the most serious toxicity associated with methimazole. Sore throat and fever may be the earliest signs. Nausea, muscle soreness, and headache and dizziness are other adverse effects of methimazole that are not as serious as agranulocytosis.

A nurse cares for a client with a deficiency of aldosterone. Which assessment finding should the nurse correlate with this deficiency? a. Increased urine output b. Vasoconstriction c. Blood glucose of 98 mg/dL d. Serum sodium of 144 mEq/L

A Aldosterone, the major mineralocorticoid, maintains extracellular fluid volume. It promotes sodium and water reabsorption and potassium excretion in the kidney tubules. A client with an aldosterone deficiency will have increased urine output. Vasoconstriction is not related. These sodium and glucose levels are normal; in aldosterone deficiency, the client would have hyponatremia and hyperkalemia. Aldosterone, the major mineralocorticoid, maintains extracellular fluid volume. It promotes sodium and water reabsorption and potassium excretion in the kidney tubules. A client with an aldosterone deficiency will have increased urine output. Vasoconstriction is not related. These sodium and glucose levels are normal; in aldosterone deficiency, the client would have hyponatremia and hyperkalemia.

An emergency nurse cares for a client who is experiencing an acute adrenal crisis. Which action should the nurse take first? a. Obtain intravenous access. b. Administer hydrocortisone succinate (Solu-Cortef). c. Assess blood glucose. d. Administer insulin and dextrose.

A All actions are appropriate for the client with adrenal crisis. However, therapy is given intravenously, so the priority is to establish IV access. Solu-Cortef is the drug of choice. Blood glucose is monitored hourly and treatment is provided as needed. Insulin and dextrose are used to treat any hyperkalemia.

The nurse provides discharge teaching for a patient who has two fractured ribs from an automobile accident. Which statement, if made by the patient, would indicate that teaching has been effective? a. "I am going to buy a rib binder to wear during the day." b. "I can take shallow breaths to prevent my chest from hurting." c. "I should plan on taking the pain pills only at bedtime so I can sleep." d. "I will use the incentive spirometer every hour or two during the day."

D

The nurse is preparing a client for a CT (computed tomography) scanning of the abdomen with contrast. What question should be asked before the examination? A. "Are you allergic to iodine or seafood?" B. "Have you had anything to eat or drink within the past 12 hours?" C. "Have you finished drinking all the required fluid?" D. "Can you tolerate being tilted from side to side during the procedure?"

A Allergies to iodine or seafood can mean a cross-allergic reaction to the contrast dye used for CT scans. Clients reporting such allergies are scheduled for CT without contrast to avoid anaphylactic reactions.

A nurse is planning the care of a patient with AIDS who is admitted to the unit with Pneumocystis pneumonia (PCP). Which nursing diagnosis has the highest priority for this patient? A) Ineffective Airway Clearance B) Impaired Oral Mucous Membranes C) Imbalanced Nutrition: Less than Body Requirements D) Activity Intolerance

A Although all these nursing diagnoses are appropriate for a patient with AIDS, Ineffective Airway Clearance is the priority nursing diagnosis for the patient with Pneumocystis pneumonia (PCP). Airway and breathing take top priority over the other listed concerns.

When providing discharge teaching for the patient after a laparoscopic cholecystectomy, what information should the nurse include? A. A lower-fat diet may be better tolerated for several weeks. B. Do not return to work or normal activities for 3 weeks. C. Bile-colored drainage will probably drain from the incision. D. Keep the bandages on and the puncture site dry until it heals.

A Although the usual diet can be resumed, a low-fat diet is usually better tolerated for several weeks following surgery. Normal activities can be gradually resumed as the patient tolerates. Bile-colored drainage or pus, redness, swelling, severe pain, and fever may all indicate infection. The bandage may be removed the day after surgery, and the patient can shower.

The charge nurse on the medical-surgical unit is making client assignments for the shift. Which client is the most appropriate to assign to an LPN/LVN? A. Client with Cushing's syndrome who requires orthostatic vital signs assessments B. Client with diabetes mellitus who was admitted with a blood glucose of 45 mg/dL C. Client with exophthalmos who has many questions about endocrine function D. Client with possible pituitary adenoma who has just arrived on the nursing unit

A An LPN/LVN will be familiar with Cushing's syndrome and the method for assessment of orthostatic vital signs. The client with a blood glucose of 45 mg/dL, the client with questions about endocrine function, and the client with a possible pituitary adenoma all have complex needs that require the experience and scope of practice of an RN.

The nurse suspects the presence of an arterial epidural hematoma in the patient who experiences a. failure to regain consciousness following a head injury b. a rapid deterioration of neurologic function within 24 to 48 hours following a head injury c. nonspecific, nonlocalizing progression of alteration in LOC occurring over weeks or months d. unconsciousness at the time of a head injury with a brief period of consciousness followed by a decrease in LOC

D

The nurse teaches the client that intraperitoneal chemotherapy will be delivered where? A. Into the veins of the legs B. Into the lung C. Into the heart D. Into the abdominal cavity

D

The nurse understands that hormone treatment for prostate cancer works by which action? A. Decreases blood flow to the tumor. B. Destroys the tumor. C. Shrinks the tumor. D. Suppresses growth of the tumor.

D

The oxygen delivery system chosen for the patient in acute respiratory failure should A. always be a low-flow device, such as a nasal cannula. B. correct the PaO2 to a normal level as quickly as possible. C. administer positive-pressure ventilation to prevent CO2 narcosis. D. maintain the PaO2 at ≥60 mm Hg at the lowest O2 concentration possible.

D

The teaching plan for a client being started on long-acting nitroglycerin includes the action of this drug. The nurse teaches that this drug relieves chest pain by which action? A) Dilating just the coronary arteries B) Decreasing the blood pressure C) Increasing contractility of the heart D) Dilating arteries and veins

D

To verify the correct placement of an endotracheal tube (ET) after insertion, the best initial action by the nurse is to a. auscultate for the presence of bilateral breath sounds. b. obtain a portable chest radiograph to check tube placement. c. observe the chest for symmetrical movement with ventilation. d. use an end-tidal CO2 monitor to check for placement in the trachea.

D

Treatment of hyper-thyroidism includes: A. Radioactive iodine (I¹³³) works by destroying the thyroid gland B. Surgery to remove all or part of the thyroidgland C. Lifelong thyroid hormone replacement maybe needed D. All of the above

D

For a male client who has acquired immunodeficiency syndrome with chronic diarrhea, anorexia, a history of oral candidiasis, and weight loss, which dietary instruction would be included in the teaching plan? A. "Follow a low-protein, high-carbohydrate diet." B. "Eat three large meals per day." C. "Include unpasteurized dairy products in the diet." D. "Follow a high-protein, high-calorie diet.

D

Which client has the most urgent need for frequent nursing assessment? A. An older client who was admitted 2 hours ago with emphysema and dyspnea and has a 45-year 2-pack-per-day smoking history, and is receiving 50% oxygen through a Venturi mask B. A young client who has had a tracheostomy for 1 week, who is on room air with SpO2 in the upper 90's, who has been receiving antibiotic therapy for 16 hours, and who has foul-smelling drainage on the tracheostomy ties C. An older adult client who is anxious to go home with her new tank of oxygen and supply of nasal cannulas and is being discharged with a new prescription for home oxygen therapy D. A middle-aged client who was admitted yesterday with pneumonia and is receiving oxygen at 2 L/min through a nasal cannula

A An older adult client with a long history of smoking and chronic lung disease who is receiving high-flow oxygen delivery is at elevated risk for respiratory depression owing to the hypoxic drive of respirations countered by high levels of oxygen. This client must be assessed frequently while receiving high-flow oxygen.

A client admitted to the hospital for chest pain is diagnosed with angina. The nurse should teach the client that the most common characteristic of anginal pain is that it is: A. Relieved by rest B. Precipitated by light activity C. Described by sharp or knifelike D. Unaffected by the administration of vasodilators

A Anginal pain is commonly relieved by immediate rest because rest decreases the cardiac workload. Angina is usually precipitated by exertion, emotion, or a heavy meal. anginal pain is usually described as tightness, or heaviness. Nitroglycerin dilates coronary arteries which increases oxygen to the myocardium, decreasing the pain.

A client with iatrogenic Cushing's syndrome is a resident in a long-term care facility. Which nursing action included in the client's care would be best to delegate to unlicensed assistive personnel (UAP)? a) Assist with personal hygiene and skin care. b) Develop a plan of care to minimize risk for infection. c) Instruct the client on the reasons to avoid overeating. d) Monitor for signs and symptoms of fluid retention.

A Assisting a client with bathing and skin care is included in UAP scope of practice. It is not within their scope of practice to develop a plan of care, although they will play a very important role in following the plan of care. Client teaching requires a broad education and should not be delegated to UAP. Monitoring for signs and symptoms of fluid retention is part of client assessment, which requires a higher level of education and clinical judgment.

After the nurse gives IV atropine to a patient with symptomatic type 1, second-degree atrioventricular (AV) block, which finding indicates that the medication has been effective? a.Increase in the patient's heart rate b.Increase in strength of peripheral pulses c.Decrease in premature atrial contractions d.Decrease in premature ventricular contractions

A Atropine will increase the heart rate and conduction through the AV node. Because the medication increases electrical conduction, not cardiac contractility, the quality of the peripheral pulses is not used to evaluate the drug effectiveness. The patient does not have premature atrial or ventricular contractions.

A patient with acute respiratory distress syndrome (ARDS) who is intubated and receiving mechanical ventilation develops a pneumothorax. Which action will the nurse anticipate taking? a. Lower the positive end-expiratory pressure (PEEP). b. Increase the fraction of inspired oxygen (FIO2). c. Suction more frequently. d. Increase the tidal volume.

A Because barotrauma is associated with high airway pressures, the level of PEEP should be decreased. The other actions will not decrease the risk for pneumothorax.

The nurse is caring for a client with atrial fibrillation. In addition to an antidysrhythmic, what medication does the nurse plan to administer? a. Heparin b. Atropine c. Dobutamine d. Magnesium sulfate

A Clients with atrial fibrillation are prone to blood pooling in the atrium, clotting, then embolizing. Heparin is used to prevent thrombus development in the atrium and the consequence of embolization (i.e., stroke).

The client with chronic renal failure has an indwelling abdominal catheter for peritoneal dialysis. The client spills water on the catheter dressing while bathing. The nurse should immediately: a) change the dressing b) reinforce the dressing c) flush the peritoneal dialysis catheter d) scrub the catheter with povidine-iodine

A Clients with peritoneal dialysis catheters are at high risk for infection. A wet dressing is a conduit for bacteria to reach the catheter insertion site. The nurse ensures that the dressing is kept dry at all times. Reinforcing the dressing is not a safe practice to prevent infection in this circumstance. Flushing the catheter is not indicated. Scrubbing the catheter with povidone-iodine is done at the time of connection or disconnection of peritoneal dialysis.

Which patient statement indicates that the nurse's teaching following a gastroduodenostomy has been effective? a. "Vitamin supplements may prevent anemia." b. "Persistent heartburn is common after surgery." c. "I will try to drink more liquids with my meals." d. "I will need to choose high carbohydrate foods."

A Cobalamin deficiency may occur after partial gastrectomy, and the patient may need to receive cobalamin via injections or nasal spray. Although peptic ulcer disease may recur, persistent heartburn is not expected after surgery and the patient should call the health care provider if this occurs. Ingestion of liquids with meals is avoided to prevent dumping syndrome. Foods that have moderate fat and low carbohydrate should be chosen to prevent dumping syndrome.

A male client suffers acute respiratory distress syndrome as a consequence of shock. The client's condition deteriorates rapidly, and endotracheal (ET) intubation and mechanical ventilation are initiated. When the high-pressure alarm on the mechanical ventilator sounds, the nurse starts to check for the cause. Which condition triggers the high-pressure alarm? A. Kinking of the ventilator tubing B. disconnected ventilator tube C. An ET cuff leak D. A change in the oxygen concentration without resetting the oxygen level alarm

A Conditions that trigger the high-pressure alarm include kinking of the ventilator tubing, bronchospasm or pulmonary embolus, mucus plugging, water in the tube, coughing or biting on the ET tube, and the client's being out of breathing rhythm with the ventilator. A disconnected ventilator tube or an ET cuff leak would trigger the low-pressure alarm. Changing the oxygen concentration without resetting the oxygen level alarm would trigger the oxygen alarm.

A client admitted after using crack cocaine develops ventricular fibrillation. After determining unresponsiveness, which action should the nurse take next? a. Defibrillate at 200 J. b. Establish IV access. c. Place an oral airway and ventilate. d. Start cardiopulmonary resuscitation (CPR).

A Defibrillating is of priority before any other resuscitative measures according to Advanced Cardiac Life Support protocols.

For which of the following dysrhythmias is defibrillation primarily indicated? a. Ventricular fibrillation b. Third-degree AV block c. Uncontrolled atrial fibrillation d. Ventricular tachycardia with a pulse

A Defibrillation is always indicated in the treatment of ventricular fibrillation. Drug treatments are normally used in the treatment of uncontrolled atrial fibrillation and for ventricular tachycardia with a pulse (if the patient is stable). Otherwise, synchronized cardioversion is used (as long as the patient has a pulse). Pacemakers are the treatment of choice for third-degree heart block.

A client has been admitted to the medical intensive care unit with a diagnosis of diabetes insipidus (DI) secondary to lithium overdose. Which medication is used to treat the DI? a) Desmopressin (DDAVP) b) Dopamine hydrochloride (Intropin) c) Prednisone d) Tolvaptan (Samsca)

A Desmopressin is the drug of choice for treatment of severe DI. It may be administered orally, nasally, or by intramuscular or intravenous routes. Dopamine hydrochloride is a naturally occurring catecholamine and inotropic vasopressor; it would not be used to treat DI. Prednisone would not be used to treat DI. Tolvaptan is a selective competitive arginine vasopressin receptor 2 antagonist and is not used with DI.

A patient with HIV has a nursing diagnosis of Risk for Impaired Skin Integrity. What nursing intervention best addresses this risk? A) Utilize a pressure-reducing mattress. B) Limit the patient's physical activity. C) Apply antibiotic ointment to dependent skin surfaces. D) Avoid contact with synthetic fabrics.

A Devices such as alternating-pressure mattresses and low-air-loss beds are used to prevent skin breakdown. Activity should be promoted, not limited, and contact with synthetic fabrics does not necessary threaten skin integrity. Antibiotic ointments are not normally used unless there is a break in the skin surface

Goiter, a type of hypo-thyroidism, includes: A. Enlargement of the thyroid gland B. Results from over stimulation by elevated levels of TSH C. TSH is elevated because there is little or no thyroid hormone in circulation D. All of the above

D

In the oliguric phase of renal failure, what is the most appropriate nursing diagnosis? a) fluid volume deficit b) activity intolerance c) ineffective breathing pattern d) fluid volume excess

D

A registered nurse is instructing a new nursing graduate about hemodialysis. Which statement if made by the new nursing graduate would indicate an inaccurate understanding of the procedure for hemodialysis? a. Sterile dialysate must be used. b. Warming the dialysate increases the efficiency of diffusion. c. Heparin sodium is administered during dialysis. d. Dialysis cleanses the blood from accumulated waste products.

A Dialysate is made from clear water and chemicals and is free from any metabolic waste products or medications. Bacteria and other microorganisms are too large to pass through the membrane; therefore, the dialysate does not need to be sterile. The dialysate is warmed to approximately 100° F to increase the efficiency of diffusion and to prevent a decrease in the client's blood temperature. Heparin sodium inhibits the tendency of blood to clot when it comes in contact with foreign substances. Option 4 is the purpose of dialysis.

A client with diabetes insipidus (DI) has dry lips and mucous membranes and poor skin turgor. Which intervention does the nurse provide first? a) Encourage fluids b) Offer lip balm c) Perform a 24-hour urine test d) Withhold desmopressin acetate (DDAVP)

A Dry lips and mucous membranes and poor skin turgor are indications of dehydration, which can occur with DI. This is a serious condition that must be treated rapidly. Encouraging fluids is the initial step, provided the client is able to tolerate oral intake. Lip balm may make the client more comfortable, but does not address the problem of dehydration. A 24-hour urine test will identify loss of electrolytes and adrenal androgen metabolites, but will not correct the dehydration that this client is experiencing. Desmopressin acetate is a synthetic form of antidiuretic hormone that is given to reduce urine production; it improves DI and should not be withheld.

The nurse provides teaching for a patient receiving corticotropin. The nurse will instruct the patient to contact the provider if which condition occurs? a. Bruising b. Constipation c. Myalgia d. Nausea

A Ecchymosis is an adverse reaction to corticotropin and should be reported. Constipation and nausea are known side effects but are not serious. Myalgia is not common.

A client with burn injuries states, "I feel so helpless." Which nursing intervention is most helpful for this client? A. Encouraging participation in wound care B. Encouraging visitors C. Reassuring the client that he or she will be fine D. Telling the client that these feelings are normal

A Encouraging participation in wound care will offer the client some sense of control. Encouraging visitors may be a good distraction, but will not help the client achieve a sense of control. Reassuring the client that he or she will be fine is neither helpful nor therapeutic. Telling the client that his or her feelings are normal may be reassuring, but does not address the client's issue of feeling helpless.

A nurse cares for a client with chronic hypercortisolism. Which action should the nurse take? a. Wash hands when entering the room. b. Keep the client in airborne isolation. c. Observe the client for signs of infection. d. Assess the client's daily chest x-ray.

A Excess cortisol reduces the number of circulating lymphocytes, inhibits maturation of macrophages, reduces antibody synthesis, and inhibits production of cytokines and inflammatory chemicals. As a result, these clients are at greater risk of infection and may not have the expected inflammatory manifestations when an infection is present. The nurse needs to take precautions to decrease the client's risk. It is not necessary to keep the client in isolation. The client does not need a daily chest x-ray.

A client with partial-thickness burns of the face and chest caused by a campfire is admitted to the burn unit. The nurse plans to carry out which health care provider request first? A. Give oxygen per facemask. B. Infuse lactated Ringer's solution at 150 mL/hr. C. Give morphine sulfate 4 to 10 mg IV for pain control. D. Insert a 14 Fr retention catheter.

A Facial burns are frequently associated with upper airway inflammation. Administration of oxygen will assist in maintaining the client's tissue oxygenation at an optimal level. Although fluid hydration and pain control are important, the nurse's first priority is the client's airway. Monitoring output is important, but the nurse's first priority is the client's airway.

A nurse is preparing to hang a fat emulsion (lipids) and observes some visible fat globules at the top of the solution. The nurse ensure to do which of the following actions? A. Take another bottle of solution. B. Runs the bottle solution under a warm water. C. Rolls the bottle solution gently. D. Shake the bottle solution vigorously.

A Fat emulsions are used as dietary supplements for patients who are unable to get enough fat in their diet, usually because of certain illnesses or recent surgery. The nurse should examine the bottle of fat emulsion for separation of emulsion into layers or fat globules or the accumulation of froth. The nurse should not hang a fat emulsion if any of these observed and should return the solution to the pharmacy.

A nurse is reviewing nutrition teaching for a client who has cholecystitis. Which of the following food choices can trigger cholecystitis? A. Brownie with nuts B. Bowl of mixed fruit C. Grilled turkey D. Baked potato

A Foods that are high in fat, such as a brownie with nuts, can cause cholecystitis.

What is the best method to prevent autocontamination for a client with burns? A. Change gloves when handling wounds on different areas of the body. B. Ensure that the client is in isolation therapy. C. Restrict visitors. D. Watch for early signs of infection.

A Gloves should be changed when wounds on different areas of the body are handled and between handling old and new dressings. Isolation therapy methods and restricting visitors are used to prevent cross-contamination, not autocontamination. Watching for early signs of infection does not prevent contamination.

A clinic nurse is caring for a patient admitted with AIDS. The nurse has assessed that the patient is experiencing a progressive decline in cognitive, behavioral, and motor functions. The nurse recognizes that these symptoms are most likely related to the onset of what complication? A) HIV encephalopathy B) B-cell lymphoma C) Kaposi's sarcoma D) Wasting syndrome

A HIV encephalopathy is a clinical syndrome characterized by a progressive decline in cognitive, behavioral, and motor functions. The other listed complications do not normally have cognitive and behavioral manifestations

A client is diagnosed with human immunodeficiency virus (HIV) infection. The nurse prepares a care plan for the client, knowing that HIV is primarily a condition in which: a) immunosuppression occurs and is indicated by a T4 lymphocyte count of less than 200/mm3 b) bacterial infection occurs, causing weakness c) fungal infection occurs, causing a rash and pruritus d) protozoan infection occurs, causing a fever and nonproductive cough

A HIV infection causes immunosuppression and is indicated by a T4 lymphocyte count of less than 200/mm3. Although bacterial, fungal, and protozoal infection can occur, these occur as opportunistic infections as a result of the immunosuppression.

When teaching a patient infected with HIV regarding transmission of the virus to others, which statement made by the patient would indicate a need for further teaching? A. "I will need to isolate any tissues I use so as not to infect my family." B. "I will notify all of my sexual partners so they can get tested for HIV." C. "Unprotected sexual contact is the most common mode of transmission." D. "I do not need to worry about spreading this virus to others by sweating at the gym."

A HIV is not spread casually. The virus cannot be transmitted through hugging, dry kissing, shaking hands, sharing eating utensils, using toilet seats, or attending school with an HIV-infected person. It is not transmitted through tears, saliva, urine, emesis, sputum, feces, or sweat.

An HIV-infected patient presents at the clinic for a scheduled CD4+ count. The results of the test are 45 cells/L, and the nurse recognizes the patient's increased risk for Mycobacterium avium complex (MAC disease). The nurse should anticipate the administration of what drug? A) Azithromycin B) Vancomycin C) Levofloxacin D) Fluconazole

A HIV-infected adults and adolescents should receive chemoprophylaxis against disseminated Mycobacterium avium complex (MAC disease) if they have a CD4+ count less than 50 cells/µL. Azithromycin (Zithromax) or clarithromycin (Biaxin) are the preferred prophylactic agents. Vancomycin, levofloxacin, and fluconazole are not prophylactic agents for MAC.

A client undergoing hemodialysis is at risk for bleeding from the heparin used during the hemodialysis treatment. The nurse assesses for this occurrence by periodically checking the results of which of the following lab tests? a. Partial thromboplastin time (PTT) b. Prothrombin time (PT) c. Thrombin time (TT) d. Bleeding time

A Heparin is the anticoagulant used most often during hemodialysis. The hemodialysis nurse monitors the extent of anticoagulation by checking the PTT, which is the appropriate measure of heparin effect. The PT is used to monitor the effect of warfarin (Coumadin) therapy. Thrombin and bleeding times are not used to measure the effect of heparin therapy, although they are useful in the diagnosis of other clotting abnormalities.

Which manifestations should a nurse investigate first when monitoring a patient who is taking levothyroxine (Synthroid)? A) Tachycardia B) Tremors C) Insomnia D) Irritability

A High doses of levothyroxine may cause thyrotoxicosis, a condition of profound excessive thyroid activity. Tachycardia is the priority assessment, because it can lead to severe cardiac dysfunction. Tremors, insomnia, and irritability are other symptoms of thyrotoxicosis and should be assessed after tachycardia.

The physician orders continuous intravenous nitroglycerin infusion for the client with MI. Essential nursing actions include which of the following? A) Obtaining an infusion pump for the medication B) Monitoring BP q4h C) Monitoring urine output hourly D) Obtaining serum potassium levels daily

A IV nitro infusion requires an infusion pump for precise control of the medication. BP monitoring would be done with a continuous system, and more frequently than every 4 hours. Hourly urine outputs are not always required. Obtaining serum potassium levels is not associated with nitroglycerin infusion.

The nurse assesses a client with a new tracheotomy, and the tracheostomy tube is pulsating in synchrony with the client's heartbeat. Which is the nurse's priority action? a. Notify the health care provider immediately. b. Stabilize the tube by reapplying the ties. c. Change the inner cannula of the tube. d. Increase the inflation pressure of the cuff.

A If a tracheostomy tube is pulsating with the client's heart rate, this could indicate proximity to the innominate artery and may cause erosion of the artery if left in this position. The provider should be notified immediately. Reapplying the ties, changing the inner cannula, and increasing the inflation pressure of the cuff are all interventions that will not solve the immediate problem of proximity of the tube to the innominate artery.

What action should you take as part of care for a patient who had a craniotomy? A. Use promethazine (Phenergan) for nausea. B. Position the patient on the operative side if a bone flap was removed. C. Administer phenytoin (Dilantin) by rapid intravenous push (IVP) every 6 hours. D. Keep the head in alignment with the trunk.

D

What are included in nursing considerations related to the administration of chemotherapeutic drugs? a. Anaphylaxis cannot occur because the drugs are considered toxic to normal cells. b. Infiltration will not occur unless superficial veins are used for the intravenous infusion. c. Many chemotherapeutic agents are vesicants that can cause severe cellular damage if drug infiltrates. d. Good hand washing is essential when handling chemotherapeutic drugs, but gloves are not necessary.

D

What should the nurse recognize as a factor commonly responsible for sodium and fluid retention in the patient on mechanical ventilation? a. increased release of ADH b. increased release of atrial natriuretic factor c. increased insensible water loss via the airway d. decreased renal perfusion with release of renin

D

The client being hemodialyzed suddenly becomes short of breath and complains of chest pain. The client is tachycardic, pale, and anxious. The nurse suspects air embolism. The priority action for the nurse is to: a. Discontinue dialysis and notify the physician. b. Monitor vital signs every 15 minutes for the next hour. c. Continue dialysis at a slower rate after checking the lines for air. d. Bolus the client with 500 mL of normal saline to break up the embolus.

A If the client experiences air embolus during hemodialysis, the nurse should terminate dialysis immediately, notify the physician, and administer oxygen as needed.

When assessing a patient with chronic lung disease, the nurse finds a sudden onset of agitation and confusion. Which action should the nurse take first? a. Check pupil reaction to light. b. Notify the health care provider. c. Attempt to calm and reassure the patient. d. Assess oxygenation using pulse oximetry.

D

When assessing the body function of a patient with increased ICP, the nurse should initially assess a. corneal reflex testing b. extremity strength testing c. pupillary reaction to light d. circulatory and respiratory status

D

A 58-year-old woman has just returned to the nursing unit after an esophagogastroduodenoscopy (EGD). Which action by unlicensed assistive personnel (UAP) requires that the registered nurse (RN) intervene? a. Offering the patient a drink of water b. Positioning the patient on the right side c. Checking the vital signs every 30 minutes d. Swabbing the patient's mouth with cold water

A Immediately after EGD, the patient will have a decreased gag reflex and is at risk for aspiration. Assessment for return of the gag reflex should be done by the RN. The other actions by the UAP are appropriate.

A client admitted with angina compains of severe chest pain and suddenly becomes unresponsive. After establishing unresponsiveness, which of the following actions should the nurse take first? A) Activate the resuscitation team B) Open the client's airway C) Check for breathing D) Check for signs of circulation

A Immediately after establishing unresponsiveness, the nurse should activate the resuscitation team. The next step is to open the airway using the head-tilt, chin-lift maneuver and check for breathing (looking, listening, and feeling for no more than 10-seconds). If the client isn't breathing, give two slow breaths using a bag mask or pocket mask. Next, check for signs of circulation by palpating the carotid pulse.

When reviewing the 12-lead electrocardiograph (ECG) for a healthy 79-year-old patient who is having an annual physical examination, what will be of most concern to the nurse? a.The PR interval is 0.21 seconds. b.The QRS duration is 0.13 seconds. c.There is a right bundle-branch block. d.The heart rate (HR) is 42 beats/minute.

D

When the ventilator alarm sounds, the nurse finds the patient lying in bed holding the endotracheal tube (ET). Which action should the nurse take first? a. Offer reassurance to the patient. b. Activate the hospitals rapid response team. c. Call the health care provider to reinsert the tube. d. Manually ventilate the patient with 100% oxygen.

D

A 20-year-old has a mandatory electrocardiogram (ECG) before participating on a college soccer team and is found to have sinus bradycardia, rate 52. Blood pressure (BP) is 114/54, and the student denies any health problems. What action by the nurse is most appropriate? a.Allow the student to participate on the soccer team. b.Refer the student to a cardiologist for further diagnostic testing. c.Tell the student to stop playing immediately if any dyspnea occurs. d.Obtain more detailed information about the student's family health history.

A In an aerobically trained individual, sinus bradycardia is normal. The student's normal BP and negative health history indicate that there is no need for a cardiology referral or for more detailed information about the family's health history. Dyspnea during an aerobic activity such as soccer is normal.

Which assessment is most important for the nurse to perform for the client receiving one unit of packed red blood cells from an autologous donation? A. Temperature B. Blood pressure C. Oxygen saturation D. IV site for hives

A In an autologous blood transfusion, the client receives his or her own blood components. Therefore the chances for an incompatibility type reaction do not exist. The main problems that can come from autologous transfusion are fluid overload and infection from blood contamination during the collection, storage, or infusion processes. Fluid overload is very unlikely when only one unit is being transfused. Contamination and infection are just as likely with an autologous transfusion as they are with a transfusion of donated blood products. The most important assessment is for signs of infection, including temperature.

The nurse is caring for a patient in the emergency department with complaints of acute abdominal pain, nausea, and vomiting. When the nurse palpates the patient's left lower abdominal quadrant, the patient complains of pain in the right lower quadrant. The nurse will document this as which of the following diagnostic signs of appendicitis? a. Rovsing sign b. referred pain c. Chvostek's sign d. rebound tenderness

A In patients with suspected appendicitis, Rovsing sign may be elicited by palpation of the left lower quadrant, causing pain to be felt in the right lower quadrant.

A 50-year-old patient who underwent a gastroduodenostomy (Billroth I) earlier today complains of increasing abdominal pain. The patient has no bowel sounds and 200 mL of bright red nasogastric (NG) drainage in the last hour. The highest priority action by the nurse is to a. contact the surgeon. b. irrigate the NG tube. c. monitor the NG drainage. d. administer the prescribed morphine.

A Increased pain and 200 mL of bright red NG drainage 12 hours after surgery indicate possible postoperative hemorrhage, and immediate actions such as blood transfusion and/or return to surgery are needed. Because the NG is draining, there is no indication that irrigation is needed. Continuing to monitor the NG drainage is not an adequate response. The patient may need morphine, but this is not the highest priority action.

When admitting a patient in possible respiratory failure with a high PaCO2, which assessment information will be of most concern to the nurse? a. The patient is somnolent. b. The patient's SpO2 is 90%. c. The patient complains of weakness. d. The patient's blood pressure is 162/94.

A Increasing somnolence will decrease the patient's respiratory rate and further increase the PaCO2 and respiratory failure. Rapid action is needed to prevent respiratory arrest. An SpO2 of 90%, weakness, and elevated blood pressure all require ongoing monitoring but are not indicators of possible impending respiratory arrest.

A patient who takes the oral antidiabetic agent metformin (Glucophage) will begin taking levothyroxine (Synthroid). The nurse will teach this patient to monitor for a. hyperglycemia. b. hypoglycemia. c. hyperkalemia. d. hypokalemia.

A Insulin and oral antidiabetic drugs may need to be increased in patients taking levothyroxine. Patients shouldbe taught to monitor for hyperglycemia, because of the reduced effects of these drugs.

Which action by a patient who is using peritoneal dialysis (PD) indicates that the nurse should provide more teaching about PD? a. The patient slows the inflow rate when experiencing pain. b. The patient leaves the catheter exit site without a dressing. c. The patient plans 30 to 60 minutes for a dialysate exchange. d. The patient cleans the catheter while taking a bath every day.

D

A 26-year-old patient is admitted to the hospital in severe respiratory distress. His oxygen saturations are 80% despite supplemental oxygen provided by facemask. The physician decides to intubate the patient to help with his breathing oxygenation. Which medication would the nurse most likely administer when assisting with intubation? a. Midazolam (Versed) b. Zolpidem (Ambien) c. Phentermine (Adipex-P) d. Modafinil (Provigil)

A Intubation is most often performed by inserting a tube into the mouth and passing it into the trachea in order to provide help and support for a patients breathing. Most registered nurses do not perform endotracheal intubation, but they may assist the physician with placing the tube. The nurse may give medications to sedate the patient during the procedure, since it can be traumatic for the patient. Some medications given for sedation include midazolam, fentanyl, and etomidate

Which action for a patient receiving tube feedings through a percutaneous endoscopic gastrostomy (PEG) may be delegated to a licensed practical/vocational nurse (LPN/LVN)? a. Providing skin care to the area around the tube site b. Teaching the patient how to administer tube feedings c. Determining the need for adding water to the feedings d. Assessing the patient's nutritional status at least weekly

A LPN/LVN education and scope of practice include actions such as dressing changes and wound care. Patient teaching and complex assessments (such as patient nutrition and hydration status) require registered nurse (RN)-level education and scope of practice.

A (DNR) client has a non-rebreather oxygen mask and breathing appears to be labored. What does the nurse do first? A. Ensures that the tubing is patent and that oxygen flow is high B. Notifies the chaplain and the family member of record C. Calls the Rapid Response Team and prepares to intubate D. Comforts the client and confirms that signed DNR orders are in the chart

A Labored breathing and ultimately suffocation can occur if the reservoir bag kinks, or if the oxygen source disconnects or is not set to high flow levels.

A patient has been given instructions about levothyroxine (Synthroid). Which statement by the patient indicates understanding of these instructions? A) "I'll take this medication in the morning so as not to interfere with sleep." B) "I'll plan to double my dose if I gain more than 1 pound per day." C) "It is best to take the medication with food so I don't have any nausea." D) "I'll be glad when I don't have to take this medication in a few months."

A Levothyroxine is used to treat hypothyroidism by increasing the basal metabolism and thus wakefulness. It is administered as a once-daily dose and is a lifelong therapy. It is best taken on an empty stomach to enhance absorption.

Which assessment data should the nurse obtain when completing a health history on a client with a seizure​ disorder? A.Vital signs B.Level of consciousness C.Neurologic exam D.Presence of auras

D

Which assessment finding indicates to the nurse that the client is at high risk for a malignant breast lesion? A. 1-cm freely mobile rubbery mass discovered by the client B. Ill-defined painful rubbery lump in the outer breast quadrant C. Backache and breast fungal infection D. Nipple discharge and dimpling

D

Which client on a medical-surgical unit does the charge nurse assign to the LPN/LVN? A. 28-year-old with morbid obesity who had bariatric surgery today B. 30-year-old recently admitted with severe diarrhea and Clostridium difficile infection C. 36-year-old whose family needs instruction about how to use a gastric feeding tube D. 39-year-old with a jejunal feeding tube who needs elemental feedings administered

D

A 48-year-old man who has just been started on tube feedings of full-strength formula at 100 mL/hr has 6 diarrhea stools the first day. Which action should the nurse plan to take? a. Slow the infusion rate of the tube feeding. b. Check gastric residual volumes more frequently. c. Change the enteral feeding system and formula every 8 hours. d. Discontinue administration of water through the feeding tube.

A Loose stools indicate poor absorption of nutrients and indicate a need to slow the feeding rate or decrease the concentration of the feeding. Water should be given when patients receive enteral feedings to prevent dehydration. When a closed enteral feeding system is used, the tubing and formula are changed every 24 hours. High residual volumes do not contribute to diarrhea.

Which information given by a patient admitted with chronic stable angina will help the nurse confirm this diagnosis? a. The patient states that the pain "wakes me up at night." b. The patient rates the pain at a level 3 to 5 (0 to 10 scale). c. The patient states that the pain has increased in frequency over the last week. d. The patient states that the pain "goes away" with one sublingual nitroglycerin tablet.

D

Which intervention is appropriate for the nurse caring for a male client in severe pain receiving a continuous I.V. infusion of morphine? a. Assisting with a naloxone challenge test before therapy begins b. Discontinuing the drug immediately if signs of dependence appear c. Changing the administration route to P.O. if the client can tolerate fluids d. Obtaining baseline vital signs before administering the first dose

D

Which of the following anti-hypertensive medications is contraindicated for clients with renal insufficiency? a) beta-adrenergic blockers b) calcium-channel blockers c) direct-acting vasodilators d) angiotensin-converting enzyme inhibitors

D

A nurse is giving general instructions to a client receiving hemodialysis. Which of the following statements would be appropriate for the nurse to include? a. Several types of medications should be withheld on the day of dialysis until after the procedure. b. Medications should be double-dosed on the morning of hemodialysis to prevent loss. c. It's acceptable to exceed the fluid restriction on the day before hemodialysis. d. It's acceptable to eat whatever you want on the day before hemodialysis.

A Many medications are dialyzable, which means they are extracted from the bloodstream during dialysis. Therefore, many medications may be withheld on the day of dialysis until after the procedure. It is not typical for medications to be "double-dosed," because there is no way to be certain how much of each medication is cleared by dialysis. Clients receiving hemodialysis are not routinely taught that it is acceptable to disregard dietary and fluid restrictions.

The nurse is caring for a mechanically ventilated patient with a cuffed tracheostomy tube. Which action by the nurse would best determine if the cuff has been properly inflated? a. Use a manometer to ensure cuff pressure is at an appropriate level. b. Check the amount of cuff pressure ordered by the health care provider. c. Suction the patient first with a fenestrated inner cannula to clear secretions. d. Insert the decannulation plug before the nonfenestrated inner cannula is removed.

A Measurement of cuff pressure using a manometer to ensure that cuff pressure is 20 mm Hg or lower will avoid compression of the tracheal wall and capillaries. Never insert the decannulation plug in a tracheostomy tube until the cuff is deflated and the nonfenestrated inner cannula is removed. Otherwise, the patient's airway is occluded. A health care provider's order is not required to determine safe cuff pressure. A nonfenestrated inner cannula must be used to suction a patient to prevent tracheal damage occurring from the suction catheter passing through the fenestrated openings.

Which of the following statements is true about intrapleural (the space between the parietal and visceral or pulmonary pleurae) pressure under normal conditions? A. It is always positive B. It is negative during inhalation; positive during exhalation C. It is positive during inhalation; negative during exhalation D. It is always negative

D

A patient who has AIDS is being treated in the hospital and admits to having periods of extreme anxiety. What would be the most appropriate nursing intervention? A) Teach the patient guided imagery. B) Give the patient more control of her antiretroviral regimen. C) Increase the patient's activity level. D) Collaborate with the patient's physician to obtain an order for hydromorphone.

A Measures such as relaxation and guided imagery may be beneficial because they decrease anxiety, which contributes to weakness and fatigue. Increased activity may be of benefit, but for other patients this may exacerbate feelings of anxiety or loss. Granting the patient control has the potential to reduce anxiety, but the patient is not normally given unilateral control of the ART regimen. Hydromorphone is not used to treat anxiety.

The RN working on an oncology unit has just received report on these clients. Which client should be assessed first? A. A client with chemotherapy-induced neutropenia who has just been admitted with an elevated temperature B. A client with lymphoma who will need administration of an antiemetic before receiving chemotherapy C. A client with metastatic breast cancer who is scheduled for external beam radiation in 1 hour D. A client with xerostomia associated with laryngeal cancer who needs oral care before breakfast

A Neutropenia poses high risk for life-threatening sepsis and septic shock, which develop and progress rapidly in immune suppressed people; the nurse should see this client first.

Which of the following statements is true regarding patient movement while requiring chest drainage? (assume a physician order or protocol exists) A. Patients may go only from bed to a chair while the chest tube is connected to a chest drain B. If patient must leave nursing unit, suction tubing should be clamped shut while chest drain is disconnected from suction C. If a patient is ambulatory, the chest tube should be clamped shut while the chest drain is disconnected from suction D. Patients may walk around once the nurse disconnects the drain from suction as long as the drain remains below the chest

D

Which of the following statements made by a student nurse indicates the need for further teaching about suctioning a patient with an endotracheal tube? A: "Suctioning the patient requires sterile technique." B: "I'll apply suction while rotating and withdrawing the suction catheter." C: "I'll suction the mouth after I suction the endotracheal tube." D: "I'll instill 5 mL of normal saline into the tube before hyperoxygenating the patient."

D

An RN from the orthopedic unit has been floated to the medical unit. Which client assignment for the floated RN is best? A. The client with a resolving pulmonary embolus who is receiving oxygen at 6 L/min through a nasal cannula B. The client with chronic lung disease who is being evaluated for possible home oxygen use C. The client with a newly placed tracheostomy who is receiving oxygen through a tracheostomy collar D. The client with chronic bronchitis who is receiving oxygen at 60% through a Venturi mask

A Orthopedic nurses are familiar with pulmonary emboli and with administration of oxygen through nasal cannulas.

A patient develops increasing dyspnea and hypoxemia 2 days after having cardiac surgery. To determine whether the patient has acute respiratory distress syndrome (ARDS) or pulmonary edema caused by left ventricular failure, the nurse will anticipate assisting with a. inserting a pulmonary artery catheter. b. obtaining a ventilation-perfusion scan. c. drawing blood for arterial blood gases. d. positioning the patient for a chest radiograph.

A Pulmonary artery wedge pressures are normal in the patient with ARDS because the fluid in the alveoli is caused by increased permeability of the alveolar-capillary membrane rather than by the backup of fluid from the lungs (as occurs in cardiogenic pulmonary edema). The other tests will not help in differentiating cardiogenic from noncardiogenic pulmonary edema.

Which of the following results is the primary treatment goal for angina? A) Reversal of ischemia B) Reversal of infarction C) Reduction of stress and anxiety D) Reduction of associated risk factors

A Reversal of the ischemia is the primary goal, achieved by reducing oxygen consumption and increasing oxygen supply. An infarction is permanent and can't be reversed.

A firefighter who was involved in extinguishing a house fire is being treated for smoke inhalation. He develops severe hypoxia 48 hours after the incident, requiring intubation and mechanical ventilation. Which of the following conditions has he most likely developed? A. Acute respiratory distress syndrome (ARDS). B. Atelectasis. C. Bronchitis. D. Pneumonia

A Severe hypoxia after smoke inhalation typically is related to ARDS. The other choices aren't typically associated with smoke inhalation.

Which of these prescribed interventions will the nurse implement first for a hospitalized patient who is experiencing continuous tonic-clonic seizures? a. Give phenytoin (Dilantin) 100 mg IV. b. Monitor level of consciousness (LOC). c. Obtain computed tomography (CT) scan. d. Administer lorazepam (Ativan) 4 mg IV.

D

Which option indicates a sign of Cushing's triad, an indication of increased intracranial pressure (ICP)? A. Heart rate increases from 90 to 110 beats/minute B. Kussmaul respirations C. Temperature over 100.4° F (38° C) D. Heart rate decreases from 75 to 55 beats/minute

D

Which patient is having the most difficulty breathing? A. The patient who reports one-pillow orthopnea B. The patient with an inspiratory to expiratory ratio of 1:2 C. The patient who speaks a sentence before breathing D. The patient with paradoxic breathing

D

A female patient who is on drug therapy for hyperaldosteronism develops menstrual disorders. Which prescribed drug may be the cause of this condition? a. Spironolactone b. Amlodipine c. Dexamethasone d. Aminoglutethimide

A Spironolactone is a potassium-sparing diuretic given to patients with hyperaldosteronism to treat hyperkalemia. This drug can cause menstrual disorders in women. Amlodipine and dexamethasone both control high blood pressure. Aminoglutethimide is given to decrease aldosterone synthesis.

When teaching fire safety to parents at a school function, which advice does the school nurse offer about the placement of smoke and carbon monoxide detectors? A. "Every bedroom should have a separate smoke detector." B. "Every room in the house should have a smoke detector." C. "If you have a smoke detector, you don't need a carbon monoxide detector." D. "The kitchen and the bedrooms are the only rooms that need smoke detectors."

A Teach all people to use home smoke detectors and carbon monoxide detectors and to ensure these are in good working order. The number of detectors needed depends on the size of the home. Recommendations are that each bedroom has a separate smoke detector, there should be at least one detector in the hallway of each story, and at least one detector is needed for the kitchen, each stairwell, and each home entrance. Carbon monoxide detectors are instrumental in picking up other types of carbon monoxide gas, such as from a defective heating unit.

The nurse manager in a long-term care facility is developing a plan for primary and secondary prevention of colorectal cancer. Which tasks associated with the screening plan will be delegated to nursing assistants within the facility? A. Testing of stool specimens for occult blood B. Teaching about the importance of dietary fiber C. Referring clients for colonoscopy procedures D. Giving vitamin and mineral supplements

A Testing of stool specimens for occult blood is done according to a standardized protocol and can be delegated to nursing assistants

To determine whether there is a delay in impulse conduction through the atria, the nurse will measure the duration of the patient's a.P wave. b.Q wave. c.P-R interval. d.QRS complex.

A The P wave represents the depolarization of the atria. The P-R interval represents depolarization of the atria, atrioventricular (AV) node, bundle of His, bundle branches, and the Purkinje fibers. The QRS represents ventricular depolarization. The Q wave is the first negative deflection following the P wave and should be narrow and short.

A client is receiving oxygen via Venturi mask at 40%. On assessment the nurse finds the client cyanotic with labored respirations. Which action does the nurse perform first? a. Remove bedding from around the adaptor opening. b. Listen to lung sounds and obtain a respiratory rate. c. Call respiratory therapy to check oxygen saturation. d. Notify the provider or Rapid Response Team immediately.

A The Venturi mask works by drawing in a specific amount of air to mix with the oxygen through holes in an adaptor fitted at the bottom of the mask. Holes of different sizes allow different amounts of room air to be entrained, changing the amount of oxygen delivered. Bedding (or clothing) wrapped around those holes would effectively change the FiO2. The nurse should ensure that the holes remain unobstructed. Other options are appropriate but are not the first choice, because this simple step may be what solves the problem.

A client with pheochromocytoma is admitted for surgery. What does the nurse do for the admitting assessment? a) Avoids palpating the abdomen b) Monitors for pulmonary edema with a chest x-ray c) Obtains a 24-hour urine specimen on admission d) Places the client in a room with a roommate for distraction

A The abdomen must not be palpated in a client with pheochromocytoma because this action could cause a sudden release of catecholamines and severe hypertension. The tumor on the adrenal gland causes sympathetic hyperactivity, increasing blood pressure and heart rate, not pulmonary edema. A 24-hour urine collection will already have been completed to determine the diagnosis of pheochromocytoma. A client diagnosed with a pheochromocytoma may feel anxious as part of the disease process; providing a roommate for distraction will not reduce the client's anxiety.

The nurse is teaching a client about his fenestrated tracheostomy tube. Which statement by the client indicates an accurate understanding of the tube? a. "I'm glad I will still be able to talk with this tube in place." b. "It is great that this tube does not have to be cleaned regularly." c. "This tube will not get dislodged because it never needs suctioning." d. "Because I can't swallow, I will need another tube for eating."

A The client can speak with a fenestrated tube, which has a hole in it and allows air to flow over the vocal cords. The tube still needs to be cleaned and suctioned. The tube may become dislodged, and the client is able to swallow.

The client is being discharged home with a tracheostomy. Which statement by the client indicates the need for further teaching about tracheostomy care? A. "I can only take baths, no showers." B. "I can put normal saline in my tracheostomy to keep the secretions from getting thick." C. "I should put cotton or foam over the tracheostomy hole." D. "I will have to learn to suction myself."

A The client does not understand that he can shower with the use of a shower shield over the tracheostomy tube to prevent water from entering the airway. Additional teaching is necessary.

A nurse is making initial rounds at the beginning of the shift and notice that the parenteral nutrition (PN) bag of an assigned client is empty. Which of the following solutions readily available on the nursing unit should the nurse hang until another PN solution is mixed and delivered to the nursing unit? A. 10% dextrose in water. B. 5% dextrose in water. C. 5% dextrose in normal saline. D. 5% dextrose in lactated Ringer solution.

A The client is at risk of hypoglycemia. Hence the nurse will hang a solution that has the highest amount of glucose until the new parenteral nutrition solution becomes readily available.

The nurse is teaching a client about how to monitor therapy effectiveness for syndrome of inappropriate antidiuretic hormone. What does the nurse tell the client to look for? a) Daily weight gain of less than 2 pounds b) Dry mucous membranes c) Increasing heart rate d) Muscle spasms

A The client must monitor daily weights because this assesses the degree of fluid restriction needed. A weight gain of 2 pounds or more daily or a gradual increase over several days is cause for concern. Dry mucous membranes are a sign of dehydration and an indication that therapy is not effective. An increased heart rate indicates increased fluid retention or dehydration and hypovolemia, and either condition is an indication that therapy is not effective. Muscle spasms are associated with hyponatremia and are an indication of a change in the client's neurologic status. Untreated hyponatremia can lead to seizures and coma.

Jeovina, with advanced breast cancer is prescribed tamoxifen (Nolvadex). When teaching the client about this drug, the nurse should emphasize the importance of reporting which adverse reaction immediately? a. Vision changes b. Hearing loss c. Headache d. Anorexia

A The client must report changes in visual acuity immediately because this adverse effect may be irreversible. Tamoxifen isn't associated with hearing loss. Although the drug may cause anorexia, headache, and hot flashes, the client need not report these adverse effects immediately because they don't warrant a change in therapy.

For client safety and quality care, which technique is best for the nurse to use when suctioning the client with a tracheostomy tube? A. Hyperoxygenate before and after suctioning B. Repeat suctioning until the tube is clear C. Apply suction during insertion of the tube D. Suction for 30 seconds

A The client should be preoxygenated with 100% oxygen for 30 seconds to 3 minutes to prevent hypoxemia. After suctioning, the client should be hyperoxygenated for 1-5 minutes, or until the client's baseline heart rate and oxygen saturation are within normal limits.`

What is the nurse's best action for the client who has undergone a laparoscopic cholecystectomy and complains of "free air pain." A. Ambulate the client. B. Instruct the client to breathe deeply and cough. C. Maintain the client on bedrest with his or her legs elevated. D. Insert a rectal tube to facilitate the passage of flatus.

A The client who has undergone a laparoscopic cholecystectomy may complain of free air pain because of the retention of carbon dioxide in the abdomen. The nurse assists the client with early ambulation to promote absorption of the carbon dioxide.

A client with syndrome of inappropriate antidiuretic hormone is admitted with a serum sodium level of 105 mEq/L. Which request by the health care provider does the nurse address first? a) Administer infusion of 150 mL of 3% NaCl over 3 hours. b) Draw blood for hemoglobin and hematocrit. c) Insert retention catheter and monitor urine output. d) Weigh the client on admission and daily thereafter.

A The client with a sodium level of 105 mEq/L is at high risk for seizures and coma. The priority intervention is to increase the sodium level to a more normal range. Ideally, 3% NaCl should be infused through a central line or with a small needle through a large vein to prevent irritation. Monitoring laboratory values for fluid balance and monitoring urine output are important, but are not the top priority. Monitoring client weight will help in the assessment of fluid balance; however, this is also not the top priority.

The nurse instructs a client with renal failure who is receiving hemodialysis about dietary modifications. The nurse determines that the client understands these dietary modifications if the client selects which items from the menu? a. Cream of wheat, blueberries, coffee b. Sausage and eggs, banana, orange juice. c. Bacon, cantaloupe melon, tomato juice. d. Cured pork, grits, strawberries, orange juice.

A The diet for a client with renal failure who is receiving hemodialysis should include controlled amounts of sodium, phosphorus, calcium, potassium, and fluids. Options 2, 3, and 4 are high in sodium, phosphorus and potassium.

Which of the following interventions should be the first priority when treating a client experiencing chest pain while walking? A) Sit the client down B) Get the client back to bed C) Obtain an ECG D) Administer sublingual nitroglycerin

A The initial priority is to decrease the oxygen consumption; this would be achieved by sitting the client down. An ECG can be obtained after the client is sitting down. After the ECGm sublingual nitro would be administered. When the client's condition is stabilized, he can be returned to bed.

While suctioning the ET tube of a spontaneously breathing patient, the nurse notes that the patient develops bradycardia with PVCs. What should the nurse do first? a. stop the suctioning and assess the patient for spontaneous respirations b. attempt to resuction the patient with reduced suction pressure and pass time c. stop the suctioning and ventilate the patient with slow, small-volume breaths using a bag-valve-mask d. stop suctioning and ventilate the patient with a BVM device with 100% oxygen until the HR returns to baseline

D

After an anterior wall myocardial infarction, which of the following problems is indicated by auscultation of crackles in the lungs? a.) Left-sided heart failure b.) Pulmonic valve malfunction c.) Right-sided heart failure d.) Tricuspid valve malfunction

A The left ventricle is responsible for the most of the cardiac output. An anterior wall MI may result in a decrease in left ventricular function. When the left ventricle doesn't function properly, resulting in left-sided heart failure, fluid accumulates in the interstitial and alveolar spaces in the lungs and causes crackles. Pulmonic and tricuspid valve malfunction causes right-sided heart failure.

The standard policy on the cardiac unit states, "Notify the health care provider for mean arterial pressure (MAP) less than 70 mm Hg." The nurse will need to call the health care provider about the a.postoperative patient with a BP of 116/42. b.newly admitted patient with a BP of 150/87. c.patient with left ventricular failure who has a BP of 110/70. d.patient with a myocardial infarction who has a BP of 140/86.

A The mean arterial pressure (MAP) is calculated using the formula MAP = (systolic BP + 2 diastolic BP)/3. The MAP for the postoperative patient in answer 3 is 67. The MAP in the other three patients is higher than 70 mm Hg.

The healthcare provider prescribes Nitroglycerin ointment to be applied topically every 8 hours for a client who was admitted for chest pain and a MI. Which statement, if made by the client, would indicate understanding of the side effects of nitroglycerin ointment? "I may experience: A. A headache B. Increased BP readings C. A slow Pulse rate D. Confusion

A The most common SE of Nitro is a headache. Additional cardio SE are tachycardia, hypotension and dizziness not confusion.

To position a client's burned upper extremities appropriately, how does the nurse position the client's elbow? A. In a neutral position B. In a position of comfort C. Slightly flexed D. Slightly hyperextended

A The neutral (extended) position is the correct placement of the elbow to prevent contracture development. Placing the elbow in a position of comfort is not the best placement because the client then usually wants to flex the joint, which increases the risk for contracture development. The slightly flexed position increases the risk for contracture development. The slightly hyperextended position is not indicated and can be painful.

A nurse is assessing the patency of a client's left arm arteriovenous fistula prior to initiating hemodialysis. Which finding indicates that the fistula is patent? a. Palpation of a thrill over the fistula. b. Presence of a radial pulse in the left wrist. c. Absence of a bruit on auscultation of the fistula. d. Capillary refill less than 3 seconds in the nail beds of the fingers of the left hand.

A The nurse assesses the patency of the fistula by palpating for the presence of a thrill or auscultating for a bruit. The presence of a thrill and bruit indicate patency of the fistula. Although the presence of a radial pulse in the left wrist and capillary refill shorter than 3 seconds in the nail beds of the fingers on the left hand are normal findings, they do not assess fistula patency.

A nurse is caring for a client who has pheochromocytoma. Which of the following actions should the nurse take? a. Elevated the head of the client's bed. b. Palpate the client's abdomen. c. Monitor the client for hypotension. d. Check the client's urine specific gravity.

A The nurse should elevate the head of the client's bed to reduce BP and abdominal pressure. Palpating the abdomen can cause release of catecholamines and increase BP. Hypertension is associated with pheochromocytoma, not hypotension. Urine specific gravity is monitored for client's who have DI or SIADH.

Which intervention by a new nurse who is caring for a patient who has just had an implantable cardioverter-defibrillator (ICD) inserted indicates a need for more education about care of patients with ICDs? a.The nurse assists the patient to do active range of motion exercises for all extremities. b.The nurse assists the patient to fill out the application for obtaining a Medic Alert ID. c.The nurse gives amiodarone (Cordarone) to the patient without first consulting with the health care provider. d.The nurse teaches the patient that sexual activity usually can be resumed once the surgical incision is healed.

A The patient should avoid moving the arm on the ICD insertion site until healing has occurred in order to prevent displacement of the ICD leads. The other actions by the new nurse are appropriate for this patient.

A patient is being scheduled for endoscopic retrograde cholangiopancreatography (ERCP) as soon as possible. Which actions from the agency policy for ERCP should the nurse take first? a. Place the patient on NPO status. b. Administer sedative medications. c. Ensure the consent form is signed. d. Teach the patient about the procedure.

A The patient will need to be NPO for 8 hours before the ERCP is done, so the nurse's initial action should be to place the patient on NPO status. The other actions can be done after the patient is NPO

When prone positioning is used in the care of a patient with acute respiratory distress syndrome (ARDS), which information obtained by the nurse indicates that the positioning is effective? a. The patient's PaO2 is 90 mm Hg, and the SaO2 is 92%. b. Endotracheal suctioning results in minimal mucous return. c. Sputum and blood cultures show no growth after 24 hours. d. The skin on the patient's back is intact and without redness.

A The purpose of prone positioning is to improve the patient's oxygenation as indicated by the PaO2 and SaO2. The other information will be collected but does not indicate whether prone positioning has been effective.

You are caring for Ms. Y. She is scheduled to begin weaning from mechanical ventilation today. Which assessment would be the best indicator of her readiness to be weaned? A) Minute ventilation greater than 10 L/min B) Respiratory rate at least 30/min C) FiO2 less than 50% D) Rapid shallow breathing index (RSBI)

D

Which assessment finding by the nurse when caring for a patient with ARDS who is being treated with mechanical ventilation and high levels of positive end-expiratory pressure (PEEP) indicates that the PEEP may need to be decreased? a. The patient has subcutaneous emphysema. b. The patient has a sinus bradycardia with a rate of 52. c. The patient's PaO2 is 50 mm Hg and the SaO2 is 88%. d. The patient has bronchial breath sounds in both the lung fields.

A The subcutaneous emphysema indicates barotrauma caused by positive pressure ventilation and PEEP. Bradycardia, hypoxemia, and bronchial breath sounds are all concerns and will need to be addressed, but they are not indications that PEEP should be reduced.

The HIV-infected patient is taught health promotion activities including good nutrition; avoiding alcohol, tobacco, drug use, and exposure to infectious agents; keeping up to date with vaccines; getting adequate rest; and stress management. What is the rationale behind these interventions that the nurse knows? A. Delaying disease progression B. Preventing disease transmission C. Helping to cure the HIV infection D. Enabling an increase in self-care activities

A These health promotion activities along with mental health counseling, support groups, and a therapeutic relationship with health care providers will promote a healthy immune system, which may delay disease progression. These measures will not cure HIV infection, prevent disease transmission, or increase self-care activities

You are the charge nurse on the telemetry unit and are responsible for making client assignments. Which client would be appropriate to assign to the float RN from the medical-surgical unit? a. The 64-year-old admitted for weakness who has a first-degree heart block with a heart rate of 58 beats/min b. The 71-year-old admitted for heart failure who is short of breath and has a heart rate of 120 to 130 beats/min c. The 88-year-old admitted with an elevated troponin level who is hypotensive with a heart rate of 96 beats/min d. The 92-year-old admitted with chest pain who has premature ventricular complexes and a heart rate of 102 beats/min

A This client has a stable, asymptomatic dysrhythmia, which usually requires no treatment; this client can be managed by a nurse with less cardiac dysrhythmia training.

A client presents to the emergency department with a history of adrenal insufficiency. The following laboratory values are obtained: Na+ 130 mEq/L, K+ 5.6 mEq/L, and glucose 72 mg/dL. Which is the first request that the nurse anticipates? a) Administer insulin and dextrose in normal saline to shift potassium into cells. b) Give spironolactone (Aldactone) 100 mg orally. c) Initiate histamine2 (H2) blocker therapy with ranitidine for ulcer prophylaxis. d) Obtain arterial blood gases to assess for peaked T waves.

A This client is hyperkalemic. The nurse should anticipate a request to administer 20 to 50 units of insulin with 20 to 50 mg of dextrose in normal saline as an IV infusion to shift potassium into the cells. Spironolactone is a potassium-sparing diuretic that helps the body keep potassium, which the client does not need. Although H2 blocker therapy would be appropriate for this client, it is not the first priority. Arterial blood gases are not used to assess for peaked T waves associated with hyperkalemia; an electrocardiogram needs to be obtained instead.

Which member of the health care team demonstrates reducing the risk for infection for the client with acquired immunodeficiency syndrome (AIDS)? A) The dietary worker hands the disposable meal trays to the LPN assigned to the client. B) The social worker encourages the client to verbalize about stressors at home. C) Housekeeping thoroughly cleans and disinfects the hallways near the client's room. D) Health care provider orders vital signs including temperature every 8 hours.

A This limits the number of health care personnel entering the room. Verbalizing stressors does not reduce the risk for infection. Bathrooms, not hallways, that are cleaned at least once daily by housekeeping reduces infection. Vital signs, including temperature, should be taken every 4 hours to detect potential infection.

a nurse is caring for a client who experienced aa cervical spin injury 24 hours ago. which of the following types of prescribed medication should the nurse clarify with the provider? A. steroid B. plasma expanders C. H2 antagonists D. muscle relaxants

D

a nurse is caring for a client who has a closed had injury with ICp readings ranging from 16-22 mmHg.which of the following actions should the nurse take to decrease the potential for raising the clients ICP (select all that apply) A. suction the endotracheal tube frequently B. decrease the noise level in the clients room C. elevate the clients head on two pillow D. administer stool softener

D

a pt with acute pacreatitis is NPO and has been receiving on IV hydration. which laboratory result indicates the need to consult the dietitian for nutritional support? a. potassium 4.2 mEq/L b. sodium 130 mEq/L c. fasting glucose 82 mg/dL d. serum albumin 2.9g/dL

D

the nurse is caring for a pt with chronic pancreatitis, the nurse would expect an elevation in which of the following lab tests? a. serum bilirubin b. serum calcium c. serum albumin d. serum amylase

D

A patient enters the ED presenting with symptoms of shortness of breath, severe chest pain, and diminished heart sounds. His blood pressure is 90/70 and his heart rate is 110. You notice that the trachea appears to be deviated to the right. What is your nursing priority? A) Prepare for an emergency insertion of a needle into the second intercostal space, midclavicular line B) Hang IV fluids and prepare for chest tube insertion C) Encourage patient to breathe into a paper bag and obtain ABG's. D) Assess for allergies and administer epinephrine as ordered

A This patient is presenting with symptoms of a tension pneumothorax. In this emergent situation, a needle can be inserted at the second intercostal space, midclavicular line to immediately allow some air to flow out of the pleural space. A chest tube would then be inserted. The lung re-expansion would correct the abnormal blood pressure and heart rate, and the patient does not appear to be having an allergic reaction.

A client who had several episodes of chest pain is scheduled for an exercise ECG. Which explanation should the nurse include when teaching the client about this procedure? A. "This is a noninvasive test to check your heart's response to physical activity". B. "This test is the definitive method to identify the actual cause of your chest pain". C. "The findings of this test will be of minimal assistance in the treatment of angina". D. "The findings from this minimally invasive test will show how your body reacts to exercise".

A This test evaluates the hearts ability to meet the need for additional oxygen in response to the stress of exercising. Changes in the ECG identify dysrhythmias and ST changes indicative of myocardial ischemia. This test assists in the differential diagnosis of chest pain; the diagnosis of heart disease is made via the results of a variety of diagnostic procedures and laboratory tests. This is a valuable test that will influence the diagnosis and treatment of heart disease. This is a noninvasive test.

The nurse's plan of care for a patient with stage 3 HIV addresses the diagnosis of Risk for Impaired Skin Integrity Related to Candidiasis. What nursing intervention best addresses this risk? A) Providing thorough oral care before and after meals B) Administering prophylactic antibiotics C) Promoting nutrition and adequate fluid intake D) Applying skin emollients as needed

A Thorough mouth care has the potential to prevent or limit the severity of this infection. Antibiotics are irrelevant because of the fungal etiology. The patient requires adequate food and fluids, but these do not necessarily prevent candidiasis. Skin emollients are not appropriate because candidiasis is usually oral.

The nurse is caring for a client with a burn injury who is receiving silver sulfadiazine (Silvadene) to the burn wounds. Which best describes the goal of topical antimicrobials? A. Reduction of bacterial growth in the wound and prevention of systemic sepsis B. Prevention of cross-contamination from other clients in the unit C. Enhanced cell growth D. Reduced need for a skin graft

A Topical antimicrobials such as silver sulfadiazine are an important intervention for infection prevention in burn wounds. Topical antimicrobials such as silver sulfadiazine do not prevent cross-contamination from other clients in the unit. They do not enhance cell growth, nor do they minimize the need the need for a skin graft.

A client is being weaned off from parenteral nutrition (PN) and is given a go-signal to take a regular diet. The ongoing solution rate has been 120ml/hr. A nurse expects that which of the following prescriptions regarding the PN solution will accompany the diet order? A. Decrease the PN rate to 60ml/hr. B. Start 0.9% normal saline at 30 ml/hr. C. Maintain the present infusion rate. D. Discontinue the PN.

A When a client begins eating a regular diet after a period of receiving PN, the PN is decreased slowly. PN that is terminated abruptly will cause hypoglycemia. Gradually decreasing the infusion rate allows the client to remain sufficiently nourished during the transition to a normal diet and prevents an episode of hypoglycemia.

The unlicensed assistive personnel (UAP) tells the nurse that a patient who is receiving oxygen at a flow rate of 6 L/min by nasal cannula is reporting nasal passage discomfort. What intervention should the nurse suggest to the UAP to improve the patient's comfort for this problem? A.) Humidify the patient's oxygen. B.) Use a simple face mask instead of a nasal cannula. C.) Provide the patient with an extra pillow. D.) Have the patient sit up in a chair at the bedside.

A When the oxygen flow rate is higher than 4 L/min, the mucous membranes can be dried out. The best treatment is to add humidification to the oxygen delivery system. Applying water-soluble jelly to the nares can also help decrease mucosal irritation. None of the other options will treat the problem.

A man with end-stage kidney disease is scheduled for hemodialysis following healing of an arteriovenous fistula(AVF). What should the nurse explain to him that will occur during dialysis? a. He will be able to visit, read, sleep, or watch TV while reclining in a chair. b. He will be placed on a cardiac monitor to detect any adverse effects that might occur. c. The dialyzer will remove and hold part of his blood for 20 to 30 minutes to remove the waste products. d. A large catheter with two lumens will be inserted into the fistula to send blood to and return it from the dialyzer.

A While patients are undergoing hemodialysis, they can perform quiet activities that do not require the limb that has the vascular access. Blood pressure is monitored frequently and the dialyzer monitors dialysis function but cardiac monitoring is not usually indicated. The hemodialysis machine continuously circulates both the blood and the dialysate past the semipermeable membrane in the machine.Graft and fistula access involve the insertion of two needles into the site: one to remove blood from and the other to return blood to the dialyzer.

Which of the following should be considered in the diet of the client with end-stage-renal-disease (ESRD)? a) limit fluid intake during anuric phase b) limit phosphorus and vitamin D-rich food c) limit calcium-rich food d) limit carbohydrates

A during ESRD, fluid intake of the client should be limited during anuric phase to prevent fluid overload. Fluid overload increases renal workload, pulmonary edema, and congestive heart failure.

The nurse is caring for a patient following an appendectomy. The patienttakes a deep breath, coughs, and then winces in pain. Which of thefollowing statements, if made by the nurse to the patient, is BEST?" A.) "Take three deep breaths, hold your incision, and then cough." B.) "That was good. Do that again and soon it won't hurt as much." C.) "It won't hurt as much if you hold your incision when you cough." D.) "Take another deep breath, hold it, and then cough deeply.

A most effective way of deep breathing and coughing, dilates airway and expands lung surface area

A

A nurse is assigned to care for a group of clients. On review of the client's medical records, the nurse determines that which client is at risk for a fluid volume deficit? a. A client with a colostomy b. A client with congestive heart failure c. A client on long-term corticosteroid therapy d. A client receiving frequent wound irrigations

C

A nurse is caring for a client with acute congestive heart failure who is receiving high doses of a diuretic. On assessment, the nurse notes tha tthe client has flat neck veins, generalized muscle weakness, and diminished deep tendon reflexes. The nurse suspects hyponatremia. What additional signs would the nurse expect to note in a client with hyponatremia? a. Exstreme thirst b. Decreased urinary output c. Hyperactive bowel sounds d. Increased specific gravity of the urine

D

A nurse is reviewing laboratory results and notes that a client's serum sodium level is 150 mEq/L. The nurse reports the serum sodium level to the physician and the physician prescribes dietary instructions based on the sodium level. Which food item does the nurse instruct the client to avoid? a. Peas b. Nuts c. Cauliflower d. Processed oat cereals

A

A nurse reviews a client's electrolyte laboratory report and notes that the potassium level is 3.2 mEq/L. Which of the following would the nurse note on the electrocardiogram as a result of the laboratory value? a. U waves b. Absent P waves c. Elevated T waves d. Elevated ST segment

A

A nurse reviews a client's laboratory report and notes that the client's serum phosphorus level is 2 mg/dL. Which condition most likely caused this serum phosphorus level? a. Alcoholism b. Renal insufficiency c. Hypoparathyroidism d. Tumor lysis syndrome

D

A nurse reviews the electrolyte results of an assigned client and notes that the potassium level is 5.4 mEq/L. Which of the following would the nurse expect to note on the electrocardiogram as a result of the laboratory value? a. ST depression b. Inverted T wave c. Prominent U wave d. Tall peaked T waves

Although his oxygen saturation is above 92%, an orally intubated, mechanically ventilated patient is restless and very anxious. What interventions will most likely decrease the risk of accidental extubation? Select all that apply a. administer sedatives b. have a caregiver stay with the patient c. obtain an order and apply soft wrist restraints d. remind the patient that he needs the tube inserted to breathe e. move the patient to an area close to the nurse's station for closer observation

AB

A nurse is completing nutritional teaching for a client who has pancreatitis. Which of the following statements by the client indicates an understanding of the teaching? (SATA) A. I plan to eat small, frequent meals. B. I will eat easy-to-digest foods with limited spice C. I will use skim milk when cooking D. I plan to drink regular cola E. I will limit alcohol intake to two drinkers per day

ABC

When monitoring a patient who is taking corticosteroids, the nurse observes for which adverse effects? (Select all that apply) A. Fragile skin B. Hyperglycemia C. Nervousness D. Hypotension E. Weight loss F. Drowsiness

ABC

Which statements accurately describe HIV infection (select all that apply)? a. Untreated HIV infection has a predictable pattern of progression. b. Late chronic HIV infection is called acquired immunodeficiency syndrome (AIDS). c. Untreated HIV infection can remain in the early chronic stage for a decade or more. d. Untreated HIV infection usually remains in the early chronic stage for 1 year or less. e. Opportunistic diseases occur more often when the CD4+ T cell count is high and the viral load is low

ABC

a nurse is reviewing trigger factos that can cause seizures with a client who has na new diagnosis of generalized seizures. which of the following information should the nurse include in this review? (select all that apply) A. avoid overwhelming fatigue B. remove caffeinated products from the diet C. limit looking at flashing lights D. perform aerobic exercise E. limit episodes of hypoventilation F. use of aerosol hairspray is recommended

ABC

The client with benign prostatic hyperplasia (BPH) is being discharged with alpha-adrenergic blockers. Which information is important for the nurse to include when teaching the client about this type of pharmacologic management? Select all that apply. A. Avoid drugs used to treat erection problems. B. Be careful when changing positions. C. Keep all appointments for follow-up laboratory testing. D. Hearing tests will need to be conducted periodically. E. Take the medication in the afternoon.

ABC Drugs used to treat erectile dysfunction can worsen side effects, such as hypotension. Alpha-adrenergic blockers may cause orthostatic hypotension, can cause liver damage, do not affect hearing and should be taken in the evening to decrease the risk of problems related to hypotension.

A nurse is caring for a client with a history of renal insufficiency who is scheduled for a cardiac catheterization. Which actions should the nurse take prior to the catheterization? (Select all that apply.) a. Assess for allergies to iodine. b. Administer intravenous fluids. c. Assess blood urea nitrogen (BUN) and creatinine results. d. Insert a Foley catheter. e. Administer a prophylactic antibiotic. f. Insert a central venous catheter.

ABC If the client has kidney disease (as indicated by BUN and creatinine results), fluids and Mucomyst may be given 12 to 24 hours before the procedure for renal protection. The client should be assessed for allergies to iodine, including shellfish; the contrast medium used during the catheterization contains iodine. A Foley catheter and central venous catheter are not required for the procedure and would only increase the client's risk for infection. Prophylactic antibiotics are not administered prior to a cardiac catheterization.

When caring for a patient with infective endocarditis, the nurse will assess the patient for which vascular manifestations (select all that apply)? a. Osler's nodes b. Janeway's lesions c. Splinter hemorrhages d. Subcutaneous nodules e. Erythema marginatum lesions

ABC Osler's nodes, Janeway's lesions, and splinter hemorrhages are all vascular manifestations of infective endocarditis. Subcutaneous nodules and erythema marginatum lesions occur with rheumatic fever.

A patient who has recovered from ARDS in the ICU is now malnourished and has lost a significant amount of weight. The physician orders TPN to add nutrition for the patient, who subsequently develops re-feeding syndrome. Which of the following signs or symptoms would the nurse expect to see with re-feeding syndrome? Select all that apply. a. Impaired mental status b. Insulin resistance c. Seizures d. Persistent weight loss e.Constipation

ABC Re-feeding syndrome occurs as the body's response when a person is fed or given nutrients after a period of starvation. When a patient is extremely malnourished and then receives TPN with its significant nutrient formulation, the body has to adjust to receiving nutrients again, which causes shifts in electrolytes in the body. As a result, the patient may experience significant complications associated with electrolyte imbalance as the body responds to the increased nutrient intake. The patient may suffer from confusion and impaired mental status, insulin resistance, seizures, and paralysis.

A nurse is planning postprocedure care for a client who received hemodialysis. Which of the following interventions should the nurse include in the plan of care? Select all that apply a. check BUN and blood creatinine b. administer medications the nurse withheld prior to dialysis c. observe for findings of hypovolemia d. assess the access site for bleeding e. evaluate BP on the arm with AV access

ABCD

Which of the following would the nurse anticipate being ordered for the patient with pulmonary contusion? SATA: A) IV fluids B) Intubation/mechanical ventilation C) Opioids D) Antibiotics E) Albumin

ABCD IV fluids would be necessary to prevent hypovolemia because of the fluid that is leaving the vascular spaces into the lungs/pleural spaces. This must be administered judiciously to prevent fluid volume overload or worsening lung function. Intubation or mechanical ventilation may be ordered, if pulmonary contusion is severe. Opioids are often used for pain relief. Antibiotics would be administered prophylactically to prevent infection from arising. Albumin would not be given in this disorder.

a nurse is assessing a client who has seizure disorder. the client reports he thinks he is out to have a seizure. which of the following actions should the nurse implement (select all that apply) A. provide privacy B. ease the client to the floor if standing C. move furniture away from the client D. loosen the clients clothing E. protect the clients head with padding F. restrain the client

ABCDE

The nurse is caring for a patient who receives levothyroxine. Which OTC drugs should not be administered within hours of admin of levythyroxine? Select all that apply. a. Aluminum hydroxide b. Calcium carbonate c. Cimetidine d. Ferrous sulfate e. Milk of magnesia

ABCDE Do not eat or take anything within hours of levothyroxine. Food decreases absorption!

A nurse is completing an assessment of client who has increased intracranial pressure. Which of the following are expected findings? (Select all that apply) A. Disoriented to time and place B. Restlessness and irritability C. Unequal pupils D. ICP 15 mm/Hg E. Headahe

ABCE

A nurse is performing the admission assessment of a patient who has AIDS. What components should the nurse include in this comprehensive assessment? Select all that apply. A) Current medication regimen B) Identification of patient's support system C) Immune system function D) Genetic risk factors for HIV E) History of sexual practices

ABCE

A nurse is planning postoperative care for a client following a kidney transplant. Which of the following actions should the nurse include? Select all that apply a. obtain daily weights b. assess dressings for bloody drainage c. replace hourly urine output with IV fluids d. expect oliguria in the first 4 hours e. monitor blood electrolytes

ABCE

Assessment of an IV cocaine user with infective endocarditis should focus on which signs and symptoms (select all that apply) a. retinal hemorrhages b. splinter hemorrhages c. presence of osler's nodes d. painless nodules over bony prominences e. painless erythematous macules on palms and soles

ABCE

A​ 50-year-old client with a newly diagnosed seizure disorder is depressed because they are not allowed to drive and have lost their independence. Which question should the nurse ask to support the​ client? (Select all that​ apply.) A.​"What does being able to drive mean to​ you?" B.​"How is not being able to drive affecting​ you?" C.​"Who is supporting you during this​ transition?" D.​"Do you have someone who can drive you to​ appointments?" E.​"What kind of alternate transportation are you​ using?"

ABCE

A client is diagnosed with acute respiratory distress syndrome​ (ARDS). The​ client's spouse asks the nurse what caused ARDS. Which etiology of indirect injury to the lungs should the nurse include in the​ response? (Select all that​ apply.) A. Multiple blood transfusions B. Systemic sepsis C. Fat embolism D. Pancreatitis E. Smoke inhalation

ABD

A nurse educator is reviewing the indications for chest drainage systems with a group of medical nurses. What indications should the nurses identify? Select all that apply. A) Post thoracotomy B) Spontaneous pneumothorax C) Need for postural drainage D) Chest trauma resulting in pneumothorax E) Pleurisy

ABD

Patients with CKD experience an increase incidence of cardiovascular disease related to Select all that apply a. hypertension b. vascular calcifications c. a genetic predisposition d. hyperinsulinemia causing dyslipidemia e. increased high-density lipoprotein levels

ABD

The nurse is caring for a patient who was just placed on mechanical ventilation and is observing the patient's vital signs because positive-pressure ventilation can lead to: (Select all that apply.) A) decreased cardiac output. B) decreased venous return. C) increased renal function. D) decreased intracranial pressure. E) increased hepatic function.

ABD

Which signs and symptoms differentiate hypoxemic respiratory failure from hypercapnic respiratory failure (select all that apply)? A. Cyanosis B. Tachypnea C. Morning headache D. Paradoxic breathing E. Pursed-lip breathing

ABD Clinical manifestations that occur with hypoxemic respiratory failure include cyanosis, tachypnea, and paradoxic chest or abdominal wall movement with the respiratory cycle. Clinical manifestations of hypercapnic respiratory failure include morning headache, pursed-lip breathing, and decreased or increase respiratory rate with shallow breathing.

When teaching a patient the adverse effects of desmopressin (DDAVP), the nurse will instruct the patient to monitor for which potential adverse effects? (Select all that apply.) A. Headache B. Weight gain C. Hypotension D. Nasal irritation E. Hyperglycemia

ABD Desmopressin works to decrease urine output; thus, the patient could retain fluid and gain weight. Other common adverse effects include increased blood pressure, fever, headache, abdominal cramps, and nausea. Desmopressin does not affect serum glucose levels. Because it is administered intranasally, it can be irritating; thus, nostrils should be rotated.

Which of the following are possible complications following a Percutaneous Transluminal Coronary Angioplasty (PTCA)? Select all that apply: A) Hematoma formation B) Acute Renal Failure C) Hypothermia D) Cardiac Tamponade E) Bradycardia

ABD Hematoma formation at the site of sheath insertion is a common complication following a PCTA and should be monitored for postoperatively. Acute renal failure may also occur d/t the use of dye during the procedure. Cardiac Tamponade may occur if a coronary vessel ruptures following the procedure.

A nurse assesses clients with potential endocrine disorders. Which clients are at high risk for adrenal insufficiency? (SATA) a. A 22-year-old female with metastatic cancer b. A 43-year-old male with tuberculosis c. A 51-year-old female with asthma d. A 65-year-old male with gram-negative sepsis e. A 70-year-old female with hypertension

ABD Metastatic cancer, tuberculosis, and gram-negative sepsis are primary causes of adrenal insufficiency. Active tuberculosis is a contributing factor for syndrome of inappropriate antidiuretic hormone. Hypertension is a key manifestation of Cushing's disease. These are not risk factors for adrenal insufficiency.

The patient with CKD is receiving dialysis, and the nurse observes excoriations on the patient's skin. Whatpathophysiologic changes in CKD can contribute to this finding (select all that apply)? a. Dry skin b. Sensory neuropathy c. Vascular calcifications d. Calcium-phosphate skin deposits e. Uremic crystallization from high BUN

ABD Pruritus is common in patients receiving dialysis.It causes scratching from dry skin, sensory neuropathy, and calcium-phosphate deposition in the skin. Vascular calcifications contribute to cardiovascular disease, not to itching skin. Uremic frost rarely occurs without BUN levels greater than 200 mg/dL, which should not occur in a patient on dialysis; urea crystallizes on the skin and also causes pruritis.

A nurse is preparing to initiate hemodialysis for a client who has AKI. Which of the following actions should the nurse take? Select all that apply a. review the medications the client currently takes b. assess the AV fistula for a bruit c. calculate the client's hourly urine output d. measure the client's weight e. check blood electrolytes

ABDE

A patient was intubated for acute respiratory failure, and there is an endotracheal tube in place. Which nursing interventions are appropriate for this patient? Select all that apply. A. ensure that the oxygen is warmed and humidified B. suction the airway, then the mouth, and give oral care C. Suction the airway with the oral suction equiptment D. Position the tubing so it does not pull on the airway E. apply suction only when withdrawing the suction catheter

ABDE

The client performs self PD. What should the nurse teach the client about preventing peritonitis? Select all that apply a. broad-spectrum antibiotics may be administered to prevent infection b. antibiotics may be added to the dialysate to treat peritonitis c. clean technique is permissible for prevention of peritonitis d. peritonitis is characterized by cloudy dialysate drainage and abdominal discomfort e. peritonitis is the most common and serious complication of peritoneal dialysis

ABDE

The nurse is monitoring a patient for increased ICP following a head injury. Which of the following manifestations indicate an increased ICP (select all that apply) a. fever b. oriented to name only c. narrowing pulse pressure d. dilated right pupil > left pupil e. decorticate posturing to painful stimulus

ABDE

The nurse is planning discharge teaching for a​ 30-year-old female client who was newly diagnosed with​ tonic-clonic seizures. Which information should the nurse include in this teaching​ plan? (Select all that​ apply.) A.Wearing a bracelet that provides health information B.Avoiding driving while taking antiseizure medication C.Keeping a padded tongue blade at home in case of a seizure D.Taking showers rather than tub baths E.Monitoring the menstrual cycle

ABDE

The nurse monitoring the client receiving PD notes that the client's outflow is less than the inflow. Which actions should the nurse take? Select all that apply a. check the level of the drainage bag b. reposition the client to their side c. contact the HCP d. place the client in good body alignment e. check the PD system for kinks

ABDE

Which assessments and interventions should the nurse consider when performing tracheal suctioning? Select all that apply. a) Closely assess the patient before, during, and after the procedure. b) Hyperoxygenate the patient before and after suctioning. c) Limit the application of suction to 20 to 30 seconds. d) Monitor the patient's pulse frequently to detect potential effects of hypoxia and stimulation of the vagus nerve. e) Use an appropriate suction pressure (80-150 mm Hg). f) Insert the suction catheter no further than 1 cm past the length of the tracheal or endotracheal tube.

ABDE Close assessment of the patient before, during, and after the procedure is necessary to limit negative effects. Risks include hypoxia, infection, tracheal tissue damage, dysrhythmias, and atelectasis. The nurse should hyperoxygenate the patient before and after suctioning and limit the application of suction to 10 to 20 seconds. The nurse should also take the patient's pulse frequently to detect potential effects of hypoxia and stimulation of the vagus nerve. Using an appropriate suction pressure (80-150 mm Hg) will help prevent atelectasis related to the use of high negative pressure. Research suggests that insertion of the suction catheter should be limited to a predetermined length (no further than 1 cm past the length of the tracheal or endotracheal tube) to avoid tracheal mucosal damage, including epithelial denudement, loss of cilia, edema, and fibrosis

A nurse is completing a nutritional status of a patient who has been admitted with AIDS-related complications. What components should the nurse include in this assessment? Select all that apply. A) Serum albumin level B) Weight history C) White blood cell count D) Body mass index E) Blood urea nitrogen (BUN) level

ABDE Nutritional status is assessed by obtaining a dietary history and identifying factors that may interfere with oral intake, such as anorexia, nausea, vomiting, oral pain, or difficulty swallowing. In addition, the patient's ability to purchase and prepare food is assessed. Weight history (i.e., changes over time); anthropometric measurements; and blood urea nitrogen (BUN), serum protein, albumin, and transferrin levels provide objective measurements of nutritional status. White cell count is not a typical component of a nutritional assessment.

A nurse is planning care for a client who has acute gastritis. Which of the following nursinginterventions should the nurse include in the plan of care? (Select all that apply.) A. Evaluate intake and output. B. Monitor laboratory reports of electrolytes. C. Provide three large meals a day. D. Administer ibuprofen for pain. E. Observe stool characteristics.

ABE

The client with chronic renal failure is on chronic hemodialysis. Which of the following indicate improvement of the client's condition due to hemodialysis? Select all that apply a) the client's BP is 130/90 b) the client's serum potassium is 4.8 mEq/L c) the client's hemoglobin level is 10 g/dL d) the client's serum calcium is 7.7 mg/dL e) the client's serum sodium is 140 mEg/L f) the client's serum magnesium is 4 mEq/L g) the client's weight has increased from 60 kg to 63 kg

ABE

The nurse is caring for a client who is in the process of weaning off of mechanical ventilation. Which assessment finding should the nurse report to the healthcare​ provider?(Select all that​ apply.) A. Agitation B. Pallor C. Oxygen saturation level of​ 98% D. Respiratory rate of 18​ beats/min E. Abdominal breathing

ABE

The nurse is caring for a client with suspected acute respiratory distress syndrome​ (ARDS). Which symptom of ARDS should the nurse anticipate will appear within 24 to 48 hours after the initial​ insult?(Select all that​ apply.) A. Shortness of breath B. Rapid breathing C. Arterial blood gases varying from normal limits D. Fluid imbalance E. Chest​ x-ray clear of infiltrates

ABE

When caring for a client with suspected SIADH, the nurse reviews the medical record to uncover which signs and symptoms consistent with this syndrome? (select all that apply) A. Hyponatremia B. Mental status changes C. Azotemia D. Bradycardia E. Weakness

ABE ADH is secreted or produced ectopically, resulting in water retention and sodium dilution which causes confusion and changes in mental status and weakness. Tachycardia may result from fluid volume excess.

The nurse is caring for a client who has developed a bradycardia. Which possible causes should the nurse investigate? Select all that apply a. Bearing down for a bowel movement b. Possible inferior wall myocardial infarction (MI) c. Client stating that he just had a cup of coffee d. Client becoming emotional when visitors arrived e. Diltiazem (Cardizem) administered an hour ago

ABE The Valsalva maneuver stimulates the vagus nerve, causing bradycardia. Inferior wall MI is a cause of bradycardia and heart blocks. Calcium channel blockers such as diltiazem may cause bradycardia.

the nurse is planning to institute seizure precautions for a client who is being admitted from the emergency department. which measures should the nurse include in planning care for the client's safety? Select all that apply A. padding the side rails of the bed B. placing an airway at the bedside C. placing the bed in the high position D. putting a padded tongue blade at the head of the bed E. placing oxygen and suction equipment at the bed side F. having intravenous equipment ready for insertion of an intravenous catheter

ABEF

Indications for:Levothyroxine, Liothyronine. Liotrix, Thyroid (SATA) A. Used to treat all three forms of hypo-thyroidism. B. Used to treat hyper-thyroidism and to prevent the surge in thyroid hormones that occurs after the surgical treatment or during radioactive iodine treatment for hyperthyroidism C. Used for thyroid replacement in patients whose thyroid glands have been surgically removed or destroyed by radioactive iodine in the treatment of thyroid cancer or hyperthyroidism

AC

A nurse is teaching a client who is scheduled for a kidney transplant about organ rejection. Which of the following statements should the nurse include? Select all that apply a. "expect an immediate removal of the donor kidney for a hyperacute rejection" b. "you might need to begin dialysis to monitor your kidney function for a hyperacute reaction" c. "a fever is a manifestation of an acute rejection" d. "fluid retention is a manifestation of an acute rejection" e. "your provider will increase your immunosuppressive medications for a chronic rejection"

ACD

The nurse is admitting a client with a history of frequent​ tonic-clonic seizures. Which information would be most valuable for the nurse to obtain when performing the health history​ assessment? (Select all that​ apply.) A.Presence of auras B.Incontinence during seizure C.Triggers for seizures D.Age of seizure onset E.Duration of seizures

ACD

When preparing to admit a patient who has been treated for status epilepticus in the emergency department, which equipment should the nurse have available in the room (select all that apply)? a. Siderail pads b. Tongue blade c. Oxygen mask d. Suction tubing e. Nasogastric tube

ACD

You are providing care for a patient who has been admitted to the hospital with a head injury who requires regular neurologic vital signs. Which assessments are components of the patient's score on the Glasgow Coma Scale (select all that apply)? A. Eye opening B. Abstract reasoning C. Best verbal response D. Best motor response E. Cranial nerve function

ACD

a nurse is assessing for the presence of brudzinski's sign in a client who has suspected meningitis. which of the following actions should the nurse take when performing this technique? (select all that apply) A. place client in supine position B. flex clients hip and knee C. place hands behind the clients neck D. bend clients head toward chest E. straighten the clients flexed leg at the knee

ACD

A patient undergoing hemodialysis starts to become hypotensive and is experiencing muscle cramps. What are the correct nursing actions that should be taken? (Select all that apply) A. Decrease the blood flow into the dialyzer B. Give a hypertonic saline solution C. Administer a hypertonic glucose solution D. Start an infusion of 0.9% normal saline

ACD Hypotension and muscle cramps occurring during hemodialysis usually occur because of rapid removal of vascular volume; therefore, slowing down the blood flow in the dialyzer will help reverse the hypotension and cramping. It is not recommended to give a hypertonic saline solution for treatment of hypotension and muscle cramps during hemodialysis because the high sodium load can be more problematic. Administering a hypertonic glucose solution helps with muscle cramps when undergoing hemodialysis. Starting an infusion of 0.9% normal saline is part of the usual treatment for hypotension and muscle cramps associated with hemodialysis.

A nurse must position the patient prone after his diagnosis of acute respiratory distress syndrome (ARDS). Which of the following is a benefit of using this position? Select all that apply. A. Decreased atelectasis B. Reduced need for endotracheal intubation C. Mobilization of secretions D. Decreased fluid accumulation E. Increased response to corticosteroid therapy

ACD Prone positioning, or placing the patient face down with the head turned to the side, helps with pulmonary function in the patient diagnosed with ARDS. Studies have shown that patients who are positioned prone and who have respiratory conditions often have improved outcomes of decreased lung atelectasis, mobilization of secretions to enhance suctioning, and decreased fluid accumulation in the lung tissue.

The nurse knows that which of the following people groups are more likely to experience nontraditional signs and symptoms of a heart attack/angina? A) Elderly B) Patients with hx of heart failure C) Patients with hx of diabetes D) Women E) Patients taking a beta blocker

ACD The elderly, there are often no symptoms that present (silent CAD) and sometimes may have weakness or changes in mental status as signs of an MI. Patients with a history of diabetes also can have silent heart attacks. Women often experience atypical signs and symptoms of a heart attack such as fatigue, shoulder blade discomfort, and SOB.

A patient has a recent tracheostomy. What necessary equipment does the nurse ensure is kept at the bedside? Select all that apply. A. ambu bag B. pair of wire cutters C. oxygen tubing D. suction equipment E. tracheostomy tube with obturator

ACDE

Which potential side effects should be included in the teaching plan for a client undergoing radiation therapy for laryngeal cancer? select all that apply. A. Fatigue B. Changes in color of hair C. Change in taste D. Changes in skin of the neck E. Difficulty swallowing

ACDE

Which statements made by a client who has diabetes insipidus indicate to the nurse that more teaching is needed? (Select all that apply.) a. If I gain more than 2 lbs. (1 kg) in a day, I will limit my fluid intake. b. If I become thirstier, I will take another dose of the drug. c. I will avoid aspirin and aspirin-containing substances. d. I will stop taking the drug for 24 hours before I have any dental work performed. e. I will limit my intake of salt and sodium to no more than 2 g daily. f. I will wear my medical alert bracelet at all times.

ACDE

A nurse assesses clients with potential endocrine disorders. Which clients are at high risk for hypopituitarism? (SATA) a. A 20-year-old female with benign pituitary tumors b. A 32-year-old male with diplopia c. A 41-year-old female with anorexia nervosa d. A 55-year-old male with hypertension e. A 60-year-old female who is experiencing shock f. A 68-year-old male who has gained weight recently

ACDE Pituitary tumors, anorexia nervosa, hypertension, and shock are all conditions that can cause hypopituitarism. Diplopia is a manifestation of hypopituitarism, and weight gain is a manifestation of Cushing's disease and syndrome of inappropriate antidiuretic hormone. They are not risk factors for hypopituitarism.

What are possible complications that can occur with suctioning from an artificial airway? Select all that apply A. infection B. coughing C. hypoxia D. tissue (mucosa) trauma E. Vagal stimulation F. bronchospasm

ACDEF

A nurse assesses a client with anterior pituitary hyperfunction. Which clinical manifestations should the nurse expect? (SATA) a. Protrusion of the lower jaw b. High-pitched voice c. Enlarged hands and feet d. Kyphosis e. Barrel-shaped chest f. Excessive sweating

ACDEF Anterior pituitary hyperfunction typically will cause protrusion of the lower jaw, deepening of the voice, enlarged hands and feet, kyphosis, barrel-shaped chest, and excessive sweating.

The nurse has received in report that the client receiving chemotherapy has severe neutropenia. Which of the following does the nurse plan to implement? Select all that apply. A. Assess for fever. B. Observe for bleeding. C. Administer pegfilgrastim (Neulasta). D. Do not permit fresh flowers or plants in the room. E. Do not allow his 16-year-old son to visit. F. Teach the client to omit raw fruits and vegetables from his diet.

ACDF Any temperature elevation in a client with neutropenia is considered a sign of infection and should be reported immediately. Administration of biological response modifiers, such as filgrastim (Neupogen) and pegfilgrastim (Neulasta), is indicated in neutropenia to prevent infection and sepsis. All fruits and vegetables should be cooked well; raw fruits and vegetables may harbor organisms, as well as Flowers and plants. Thrombocytopenia cause bleeding, not low neutrophils.The client is at risk for infection, not the visitors, if they are well. However, very small children, who may get frequent colds and viral infections, may pose a risk.

A nurse is caring for a client who is having surgery for the removal of an encapsulated acoustic tumor. Which of the following potential complications should the nurse monitor for postoperatively? (Select all that apply) A. Increased intracranial pressure B. Hemorrhagic shock C. Hydrocephalus D. Hypoglycemia E. Seizures

ACE

A nurse is reviewing a prescription for dexamethasone (Decadron) with a client who has an expanding brain tumor. Which of the following are appropriate statements by the nurse? (Select all that apply) A. It is given to reduce swelling of the brain B. You will need to monitor for low blood sugar C. You may notice weight gain D. Tumor growth will be delayed E. It can cause you to retain fluids

ACE

The nurse presents a seminar on HIV testing to a group of seniors and their caregivers in an assisted living facility. Which responses fit the Centers for Disease Control and Prevention's (CDC's) recommendations for HIV testing? (Select all that apply.) A) ''I am 78 years old and I was treated and cured of syphilis many years ago.'' B) ''In 1986, I received a transfusion of platelets.'' C) ''Seven years ago, I was released from a penitentiary.'' D) ''I used to smoke marijuana 30 years ago, but I have not done any drugs since.'' E) ''I had sex with a man with a disreputable past from New York back in the late 1960s, but I have been happily married since 1971.'' F) ''At 68, I am going to get married for the fourth time.''

ACF

A nurse assesses a client who potentially has hyperaldosteronism. Which serum laboratory values should the nurse associate with this disorder? (SATA) a. Sodium: 150 mEq/L b. Sodium: 130 mEq/L c. Potassium: 2.5 mEq/L d. Potassium: 5.0 mEq/L e. pH: 7.28 f. pH: 7.50

ACF Aldosterone increases reabsorption of sodium and excretion of potassium. Hyperaldosteronism causes hypernatremia, hypokalemia, and metabolic alkalosis. Hyponatremia, hyperkalemia, and acidosis are manifestations of adrenal insufficiency.

Select-all-that-apply: What are the typical signs and symptoms of infective endocarditis? A. Hyperthermia B. S4 gallop C. Enlarged Spleen D. Hyperkalemia E. Substernal pain that radiates to the back F. Heart failure G. Cardiac Murmur

ACFG

The parents of a child with Hodgkin disease ask how the physician will know what type of cancer their child has. Which of the following definitive signs and symptoms should the nurse describe? Select all that apply a. the most common finding is enlarged, firm, nontender movable nodes in the supraclavicular or cervical area b. tests include complete blood count, prothrombin time and G6PD, erythropoetin, and sedimentation rate c. generally a bone marrow biopsy is done to look for the presence of blast cells d. the presence of Sternberg-Reed cells is considered diagnostic of Hodgkin disease e. the presence of a white reflection as opposed to the normal red pupillary reflex in the pupil is a classic sign

AD

What precautions should the nurse take during mouth care and repositioning of an oral ET tube to prevent and detect tube dislodgment? Select all that apply a. confirm bilateral breath sounds after care b. use suction pressures less than 120 mm Hg c. use humidified inspired gas to help thin secretions d. one staff member holds the tube and one performs care e. move secretions into larger airways with turning every 2 hours

AD

A nurse assesses a client with Cushing's disease. Which assessment findings should the nurse correlate with this disorder? (SATA) a. Moon face b. Weight loss c. Hypotension d. Petechiae e. Muscle atrophy

ADE

A nurse is assessing a client for HIV. Which of the following are risk factors associated with this virus?(Select all that apply.) A. Perinatal exposure B. Pregnancy C. Monogamous sex partner D. Older adult woman E. Occupational exposure

ADE

A patient was admitted following a motor vehicle accident with multiple fractured ribs. Respiratory assessment includes signs/symptoms of secondary pneumothorax. Which are the most common assessment findings associated with a pneumothorax? (Select all that apply). A. Sharp pleuritic pain that worsens on inspiration B. Crackles over lung bases of affected lung C. Tracheal deviation toward the affected lung D. Worsening dyspnea E. Absent lung sounds to auscultation on affected side

ADE

Which of the following actions is an important part of oral care for an intubated patient? (Select all that apply.) A) Brushing the teeth and tongue with a soft-bristled toothbrush B) Using lemon-glycerin swabs on the patient's lips and gums C) Using alcohol-based mouthwash every 2 hours D) Using a tonsil suction to keep secretions cleared out of the mouth E) Providing lip moisturizer as needed

ADE

a nurse is planning care for a client who has meningitis and is at risk for increased intracranial pressure (ICP). which of the following actions should the nurses plan to take? (select all that apply) A. implement seizure precaution B. perform neurological checks four times a day C. administer morphine for the report of neck and generalized pain D. turn off room lights and television E. monitor for impaired extra ocular movement F. encourage the client to cough frequently

ADE

Which factors indicate that a client's burn wounds are becoming infected? (Select all that apply.) A. Dry, crusty granulation tissue B. Elevated blood pressure C. Hypoglycemia D. Edema of the skin around the wound E. Tachycardia

ADE Pale, boggy, dry, or crusted granulation tissue is a sign of infection, as is swelling or edema of the skin around the wound. Tachycardia is a systemic sign of infection. Hypotension, not elevated blood pressure, and hyperglycemia, not hypoglycemia, are systemic signs of infection.

Which interventions help to prevent aspiration during eating for a client with a tracheostomy? (Select all that apply.) a. Provide close supervision for the client during eating and drinking. b. Add liquids to foods to make them thinner and easier to swallow. c. Inflate the tracheostomy cuff tube to maximum pressure before starting. d. Let the client indicate readiness for another bite when being fed. e. Have the client tuck the chin down and forward while swallowing. f. Instruct the client to dry swallow to clear food particles from the throat. g. Place the client in a semi-Fowler's position for an hour after eating.

ADEF The client with a tracheostomy will require close supervision, even if the client is feeding himself or herself. Do not rush the client. Allow him or her to indicate when ready for another bite. Teaching interventions should include instructing the client to tuck the chin down and forward while swallowing to encourage food to move down smoothly. Dry swallowing helps remove food residue. Food may actually become easier to aspirate if it is thinner in texture. The nurse should not initiate adding air to inflate the cuff of a tracheostomy tube further without a physician's order; if possible, the cuff should be deflated during eating. Placing the client in a semi-Fowler's position after the meal will not prevent aspiration.

Nursing management of the patient with acute pancreatitis includes: (SATA) A. Check for signs of hypocalcemia B. Provide a diet low in carbohydrates C. Giving insulin based on sliding scale D. Observing stools for signs of steatorrhea E. Monitoring for infection, particularly respiratory tract infection

AE During the acute phase, it is important to monitor vital signs. Hemodynamic stability may be compromised by hypotension, fever, and tachypnea. Injection fluids are ordered, and the response to therapy is monitored. Fluid and electrolyte balances are closely monitored. Frequent vomiting, along with gastric suction, may result in decreased levels of chloride, sodium, and potassium. Because hypocalcemia can occur in acute pancreatitis, the nurse should observe for symptoms of tetany, such as jerking, irritability, and muscular twitching. Numbness or tingling around the lips and in the fingers is an early indicator of hypocalcemia. The patient should be assessed for Chvostek's sign or Trousseau's sign. A patient with acute pancreatitis should be observed for fever and other manifestations of infection. Respiratory infections are common because the retroperitoneal fluid raises the diaphragm, which causes the patient to take shallow, guarded abdominal breaths.

A 24-yr-old woman donated a kidney via a laparoscopic donor nephrectomy to a nonrelated recipient. The patient is experiencing significant pain and refuses to get up to walk. How should the nurse respond? a. Have the transplant psychologist convince her to walk. b. Encourage even a short walk to avoid complications of surgery. c. Tell the patient that no other patients have ever refused to walk. d. Tell the patient she is lucky she did not have an open nephrectomy.

B

A 25-year-old male patient has been diagnosed with HIV. The patient does not want to take more than one antiretroviral drug. What reasons can the nurse tell the patient about for taking more than one drug? A. Together they will cure HIV. B. Viral replication will be inhibited. C. They will decrease CD4+ T cell counts. D. It will prevent interaction with other drugs

B

A 34-year-old female client is requesting information about mammograms and breast cancer. She isn't considered at high risk for breast cancer. What should the nurse tell this client? a. She should have had a baseline mammogram before age 30. b. She should eat a low-fat diet to further decrease her risk of breast cancer. c. She should perform breast self-examination during the first 5 days of each menstrual cycle. d. When she begins having yearly mammograms, breast self-examinations will no longer be necessary.

B

A client admitted for uncontrolled hypertension and chest pain was started on a daily diuretic two days ago upon admission, with prescriptions for a daily basic metabolic panel. The clients potassium level this morning is 2.7 mEq/L. Which action should the nurse take next? A. Notify the healthcare provider that the potassium level is above normal B. Notify the healthcare provider that the potassium level is below normal C. No action is required because the potassium level is within normal limits D. Hold the clients morning diuretic dose

B

A client arrives at the emergency room complaining of chest pain and dizziness. The client has a history of angina. The healthcare provider prescribes an ECG and lab tests. A change in which component of the ECG tracing should the nurse recognize as the client actively having a myocardial infarction? A. QRS complex B. ST segment C. P wave D. R wave

B

A client receiving parenteral nutrition (PN) complains of a headache. A nurse notes that the client has an increased blood pressure, bounding pulse, jugular distension, and crackles bilaterally. The nurse determines that the client is experiencing which complication of PN therapy? A. Air embolism. B. Hypervolemia. C. Hyperglycemia. D. Sepsis.

B

A client reports that they usually have a seizure on the first day of their period. Which response by the nurse is​ correct? A.​"Having your period has no relationship to your​ seizures." B.​"Menstruation is a common trigger for​ seizures." C.​"What makes you think having your period is​ related?" D.​"Females who have a lot of menstrual cramps often have seizure​ activity."

B

A client who had a MI receives 15 mg of morphine sulfate for chest pain. 15 minutes after receiving the drug, The client complains of feeling dizzy. What action should the nurse take? A. Determine if this is an allergic reaction B. Place the client in the supine position and take the vital signs C. Elevate the clients head and keep the extremities warm D. Tell the client that this is not a typical sensation after receiving morphine sulfate

B

A client who is taking levothyroxine (Synthroid) begins to develop weight loss, diarrhea, and intolerance. The nurse should be aware that this might be an indication of what hormonal condition? a.) Addison's disease b.) Hyperthyroidism c.) Cushing's syndrome d.) Development of acromegaly

B

A client with chronic renal failure has asked to be evaluated for a home continuous ambulatory PD program (CAPD). The nurse should explain that the major advantage of this approach is that it a. is relatively low in cost b. allows the client to be more independent c. is faster and more efficient than standard PD d. has fewer potential complications than does standard PD

B

A male client is in isolation after receiving an internal radioactive implant to treat cancer. Two hours later, the nurse discovers the implant in the bed linens. What should the nurse do first? a. Stand as far away from the implant as possible and call for help. b. Pick up the implant with long-handled forceps and place it in a lead-lined container. c. Leave the room and notify the radiation therapy department immediately. d. Put the implant back in place, using forceps and a shield for self-protection, and call for help.

B

A newly hired nurse is performing a focused respiratory assessment. The nurse mentor will intervene if which action by the newly hired nurse is noted? a) The newly hired nurse palpates the point of maximal impulse (PMI). b) The newly hired nurse auscultates breath sounds as the client breathes through the nose. c) The newly hired nurse attaches a pulse oximetry to the client's index finger. d) The newly hired nurse explains the assessment procedure before performing it.

B

A nurse is assessing a client who reports severe headache and a stiff neck. The nurse's assessment reveals positive Kernig's and Brudzinski's signs. Which of the following actions should the nurse perform first? A. Administer antibiotics B. Implement droplet precautions C. Initiate IV access D. Decrease bright lights

B

A nurse is caring for a client who has just undergone a craniotomy for a suprtentorial tumor. Which of the following postoperative prescriptions should the nurse clarify with the provider? A. Dexamethasone (Decadron) 30 mg IV bolus BID B. Morphine sulfate 2 mg IV bolus PRN every 2 hours for pain C. Ondansetron (Zofran) 4 mg IV bolus PRN every 4-6 hour for nausea D. Phenytoin (Dilantin) 100 mg IV bolus TID

B

A nurse is choosing a catheter to use to suction a patient's endotracheal tube via an open system. On which variable would the nurse base the size of the catheter to use? a) The age of the patient b) The size of the endotracheal tube c) The type of secretions to be suctioned d) The height and weight of the patient

B

A nurse is monitoring the status of a client's fat emulsion (lipid) infusion and notes that the infusion is 2 hours delay. The nurse should do which of the following actions? A. Adjust the infusion rate to catch up over the next hour. B. Make sure the infusion rate is infusing at the ordered rate. C .Increase the infusion rate to catch up over the next few hours. D. Adjust the infusion rate to full blast until the solution is back on time.

B

A nurse is preparing to ambulate a client on the 3rd day after cardiac surgery. The nurse would plan to do which of the following to enable the client to best tolerate the ambulation? a.) Encourage the client to cough and deep breathe b.) Premedicate the client with an analgesic c.) Provide the client with a walker d.) Remove telemetry equipment because it weighs down the hospital gown.

B

A nurse is providing postural drainage for a patient with cystic fibrosis. In which position should the nurse place the patient to drain the right lobe of the lung? a) High Fowler's position b) Left side with pillow under chest wall c) Lying position/half on abdomen and half on side d) Trendelenberg position

B

A nurse is reading the results of a lumbar puncture performed on a client suspected too having bacterial meningitis. which of the following findings should the nurse recognize as being consistent with this diagnosis? A. elevated glucose B. elevated protein C. presence of red blood cells D. presence of D dimer

B

A nurse is reviewing the health record of a student newly admitted to a university and living in a dormitory. The health record indicates the student requires follow up immunizations. Which of the following organisms should the nurse plan to vaccinate the student against? A. Streptococcus pneumoniae B. Neisseria Meningitidis C. Bartonella Henselae D. Rickettsia ricketsii

B

A nurse is teaching a client who has a new diagnosis of pernicious anemia due to chronic gastritis.Which of the following should be included in the teaching? A. Cells producing gastric acid have been damaged. B. A monthly injection of medication is required. C. Vitamin K supplements will be administered. D. Increased production of intrinsic factor is occurring.

B

A patient has been on a non-rebreathing mask at 10 L/min for 4 days and is complaining of a dry cough, a stuffy nose, and substernal chest pain (pain score, 6 of 10) that increases with deep breathing. The chest radiograph shows no changes, and the 12-lead electrocardiography (ECG) findings are normal. The nurse suspects the patient is experiencing: A) hypercapnia. B) oxygen toxicity. C) unstable angina. D) absorption atelectasis.

B

A patient is in the primary infection stage of HIV. What is true of this patient's current health status? A) The patient's HIV antibodies are successfully, but temporarily, killing the virus. B) The patient is infected with HIV but lacks HIV-specific antibodies. C) The patient's risk for opportunistic infections is at its peak. D) The patient may or may not develop long-standing HIV infection.

B

A patient receiving propylthiouracil (PTU) asks the nurse, "How does this medication relieve symptoms?" What is the nurse's best response? A. "PTU helps your thyroid gland synthesize and use iodine, which produces hormones better." B. "PTU inhibits the formation of new thyroid hormone, thus returning your metabolism to normal." C. "PTU causes the pituitary gland to secrete thyroid-stimulating hormone, which blocks the production of hormones by the thyroid gland." D. "PTU removes thyroid hormones that are already circulating in your bloodstream, thus decreasing the adverse effects of this medication."

B

A patient sustained a serious crush injury to the neck and had a trach tube placed 3 days ago. As the nurse is performing trach care, the patient suddenly sneezes forcefully and the tube falls out onto the bed linens. What does the nurse do? A. ventilate the patient with 100% oxygen and notify the provider B. quickly and gently replace the tube with a clean cannula kept at the bedside C. quickly rinse the tube with sterile solution and gently replace it D. Give the patient oxygen; call for assistance and a new tracheostomy kit

B

A patient who underwent a percutaneous, transluminal coronary angioplasty four weeks ago has a subsequent ejection fraction of 30%. The patient returns for a follow-up visit. Examination reveals lungs that are clear to auscultation and slight pedal edema. The patient's medications are digoxin (Lanoxin), furosemide (Lasix), enalapril maleate (Vasotec), and aspirin. The patient reports a 5-lb (2.27-kg) weight gain over the past two days. The cardiac-vascular nurse's initial action is to: a. document the weight and reassess the patient at the next session. b. inquire about the patient's medication compliance. c. notify the patient's physician. d. review the patient's most recent nuclear scan.

B

A patient who underwent a percutaneous, transluminal coronary angioplasty four weeks ago has a subsequent ejection fraction of 30%. The patient returns for a followup visit. Examination reveals lungs that are clear to auscultation and slight pedal edema. The patient's medications are digoxin (Lanoxin), furosemide (Lasix), enalapril maleate (Vasotec), and aspirin. The patient reports a 5 lb (2.27-kg) weight gain over the past two days. The cardiac/vascular nurse's initial action is to: a.) Document the weight, and reassess the patient at the next session. b.) Inquire about the patient's medication compliance. c.) Notify the patient's physician. d.) Review the patient's most recent nuclear scan.

B

A patient who underwent coronary artery bypass surgery demonstrates effective understanding of discharge teaching by: a. asking whether smoking one cigarette per day is acceptable. b. clarifying when cardiac rehabilitation will begin. c. describing plans to relax in a hot tub during recovery. d. planning to resume driving his or her spouse to work next week.

B

A patient with a trach tube is currently alert and cooperative but seems to be coughing more frequently and producing more secretions than usual. The nurse determines there is a need for suctioning. Which nursing intervention does the nurse use to prevent hypoxia for this patient? A. allow the patient to breathe room air prior to suctioning B. avoid prolonged suctioning time C. suction frequently when the patient is coughing D. use the largest available catheter

B

A patient with a tracheostomy is unable to speak. He is not in acute distress, but is gesturing and trying to communicate with the nurse. Which nursing intervention is the best approach in this situation? A. reply on the family to interpret for the patient B. ask questions that can be answered with a yes or no response C. obtain an immediate consult with the speech therapist D. encourage the patient to rest rather than struggle with communication

B

A patient with hypercapnic respiratory failure has a respiratory rate of 8 and an SpO2 of 89%. The patient is increasingly lethargic. Which collaborative intervention will the nurse anticipate? a. Administration of 100% oxygen by non-rebreather mask b. Endotracheal intubation and positive pressure ventilation c. Insertion of a mini-tracheostomy with frequent suctioning d. Initiation of bilevel positive pressure ventilation (BiPAP)

B

Adverse effects for:Methimazole (Tapazole)Propylthiouracil (PTU) A. Cardiac dysrhythmia B. Liver and bone marrow toxicity; joint pain C. May also cause: tachycardia, palpitations, angina, hypertension, insomnia, tremors, headache, anxiety, nausea, diarrhea, menstrual irregularities, weight loss, appetite changes, sweating, heat intolerance

B

After completion of PD, the nurse should assess the client for a. hematuria b. weight loss c. hypertension d. increased urine output

B

After the insertion of an arteriovenous graft (AVG) in the right forearm, a patient complains of pain and coldness of the right fingers. Which action should the nurse take? a. Elevate the patients arm above the level of the heart. b. Report the patients symptoms to the health care provider. c. Remind the patient about the need to take a daily low-dose aspirin tablet. d. Educate the patient about the normal vascular response after AVG insertion.

B

After the nurse teaches the patient about the use of carvedilol (Coreg) in preventing anginal episodes, which statement by a patient indicates that the teaching has been effective? a. "Carvedilol will help my heart muscle work harder." b. "It is important not to suddenly stop taking the carvedilol." c. "I can expect to feel short of breath when taking carvedilol." d. "Carvedilol will increase the blood flow to my heart muscle."

B

An emergency department nurse is using a manual resuscitation bag (Ambu bag) to assist ventilation in a patient with lung cancer who has stopped breathing on his own. What is an appropriate step in this procedure? a) Tilt the patient's head forward. b) Hold the mask tightly over the patient's nose and mouth. c) Pull the patient's jaw backward. d) Compress the bag twice the normal respiratory rate for the patient.

B

Anti-thyroid drugs for hyper-thyroidism: A. Levothyroxine, Liothyronine, Liotrix, Thyroid B. Methimazole (Tapazole), Propylthiouracil (PTU) C. All of the above

B

Before weaning a male client from a ventilator, which assessment parameter is most important for the nurse to review? a. Fluid intake for the last 24 hours b. Baseline arterial blood gas (ABG) levels c. Prior outcomes of weaning d. Electrocardiogram (ECG) results

B

Common signs and symptoms of hypo-thyroidism: A. Bloody stool and vomiting B. Thickened skin, hair loss, constipation, lethargy, and anorexia C. Fever, weight gain, high energy D. Jaudice, thin skin, and random hair growth

B

Diltiazem (Cardizem) is ordered for a patient with newly diagnosed Prinzmetal's (variant) angina. When teaching the patient, the nurse will include the information that diltiazem will a. reduce heart palpitations. b. decrease spasm of the coronary arteries. c. increase the force of the heart contractions. d. help prevent plaque from forming in the coronary arteries.

B

During PD, the nurse observes that the flow of dialysate stops before all the solution has drained out. The nurse should a. have the client sit in a chair b. turn the client from side to side c. reposition the peritoneal catheter d. have the client walk

B

During assessment of a client with acute respiratory distress syndrome​ (ARDS), the nurse notes an oxygen saturation of​ 78% and a respiratory rate of 28​ breaths/min. The nurse notifies the healthcare provider and should prepare for intubation using which type of​ airway? A. Oropharyngeal airway B. Endotracheal tube C. Tracheostomy D. Nasopharyngeal airway

B

Four hours after mechanical ventilation is initiated for a patient with chronic obstructive pulmonary disease (COPD), the patients arterial blood gas (ABG) results include a pH of 7.50, PaO2 of 80 mm Hg, PaCO2 of 29 mm Hg, and HCO3 of 23 mEq/L (23 mmol/L). The nurse will anticipate the need to a. increase the FIO2. b. decrease the respiratory rate. c. increase the tidal volume (VT). d. leave the ventilator at the current settings.

B

In replying to a patient's questions about the seriousness of her CKD, the nurse knows that the stage of CKD is based on what? a. total daily urine output b. GFR c. degree of altered mental status d. serum creatinine and urea levels

B

Indications for:Methimazole (Tapazole)Propylthiouracil (PTU) A. Used to treat all three forms of hypo-thyroidism. B. Used to treat hyper-thyroidism and to prevent the surge in thyroid hormones that occurs after the surgical treatment or during radioactive iodine treatment for hyperthyroidism C. Used for thyroid replacement in patients whose thyroid glands have been surgically removed or destroyed by radioactive iodine in the treatment of thyroid cancer or hyperthyroidism

B

Mr. J, a 26-year-old patient with diabetes, is admitted to the unit in severe diabetic ketoacidosis. His pH is 7.29. Understanding the principles of the oxyhemoglobin dissociation curve, you would expect which finding when you measure his SaO2? A) The SaO2 may be higher than normal. B) The SaO2 may be lower than normal. C) The SaO2 is not affected because he does not have pulmonary disease. D) You must know the HCO3- before you can predict changes in the SaO2.

B

Myxedema, which includes fatigue, general weakness, and muscle cramps, is a symptom of which endocrine disorder treated with levothyroxine (Synthroid)? a. Hyperthyroidism b. Hypothyroidism c. Cushing's syndrome d. Addison's disease

B

Nurse Amy is speaking to a group of women about early detection of breast cancer. The average age of the women in the group is 47. Following the American Cancer Society guidelines, the nurse should recommend that the women: a. perform breast self-examination annually. b. have a mammogram annually. c. have a hormonal receptor assay annually. d. have a physician conduct a clinical examination every 2 years.

B

The ABCD method offers one way to assess skin lesions for possible skin cancer. What does the A stand for? a. Actinic b. Asymmetry c. Arcus d. Assessment

B

The RN is teaching an unlicensed assistive personnel (UAP) to check oxygen saturation by pulse oximetry. What will the nurse be sure to tell the UAP about patients with darker skin? A.) "Be aware that patients with darker skin usually show a 3% to 5% higher oxygen saturation compared with light-skinned patients." B.) "Usually dark-skinned patients show a 3% to 5% lower oxygen saturation by pulse oximetry than light-skinned patients." C.) "With a dark-skinned patient, you may get more accurate results by measuring pulse oximetry on the patient's toes." D.) "More accurate results may result from continuous pulse oximetry monitoring than spot checking when a patient has darker skin."

B

The client with chronic renal failure is undergoing peritoneal dialysis. He asks why the nurse monitors his blood glucose levels. Which of the following will be the most appropriate response by the nurse? a) I have to check if you have diabetes mellitus b) the dialysate contains glucose c) the procedure may lower your blood glucose levels d) it is a routine procedure for every client who undergoes the treatment

B

The client with prostate cancer asks why he must have surgery instead of radiation, even if it is the least invasive type. What is the nurse's best response? A."It is because your cancer growth is large." B. "Surgery is the most common intervention to cure the disease." C. "Surgery slows the spread of cancer." D. "The surgery is to promote urination."

B

The emergency department nurse is assessing a client who has sustained a blunt injury to the chest wall. Which finding indicates the presence of a pneumothorax in this client? a. A low respiratory rate b. Diminished breath sounds c. The presence of a barrel chest d. A sucking sound at the site of injury

B

The following would result in a loss of the water seal: A. Momentary tipping-over of the Atrium drainage system. B. Evaporation of the water in the water seal chamber below the 2 cm mark. C. Suction removed or turned off. D. The drainage chambers are full.

B

The high-pressure alarm on a patient's ventilator goes off. When the nurse enters the room to assess the patient, who has acute respiratory distress syndrome (ARDS), the oxygen saturation monitor reads 87% and the patient is struggling to sit up. Which action should the nurse take first? A.) Reassure the patient that the ventilator will do the work of breathing for him. B.) Manually ventilate the patient while assessing possible reasons for the high-pressure alarm. C.) Increase the fraction of inspired oxygen (Fio2) on the ventilator to 100% in preparation for endotracheal suctioning. D.) Insert an oral airway to prevent the patient from biting on the endotracheal tube.

B

The high-pressure alarm on a patient's ventilator goes off. When the nurse enters the room to assess the patient, who has acute respiratory distress syndrome (ARDS), the oxygen saturation monitor reads 87% and the patient is struggling to sit up. Which action should the nurse take first? a. Reassure the patient that the ventilator will do the work of breathing for him. b. Manually ventilate the patient while assessing possible reasons for the high-pressure alarm. c. Increase the fraction of inspired oxygen (Fio2) on the ventilator to 100% in preparation for endotracheal suctioning. d. Insert an oral airway to prevent the patient from biting on the endotracheal tube.

B

The nurse determines that alveolar hypoventilation is occurring in a patient on a ventilator when what happens? a. the patient develops cardiac dysrhythmias b. auscultation reveals an air leak around the ET tube cuff c. ABG results show a PaCO2 of 32 mm Hg and a pH of 7.47 d. the patient tries to breathe faster than the ventilator setting

B

The nurse is administering an intravenous chemotherapeutic agent to a child with leukemia. The child suddenly begins to wheeze and have severe urticaria. What nursing action is most appropriate to initiate? a. Recheck the rate of drug infusion. b. Stop the drug infusion immediately. c. Observe the child closely for next 10 minutes. d. Explain to the child that this is an expected side effect.

B

The nurse is assessing a client with lung cancer. Which symptom does the nurse anticipate finding? A. Easy bruising B. Dyspnea C. Night sweats D. Chest wound

B

The nurse is assessing a patient who is receiving peritoneal dialysis with 2 L inflows. Which information should be reported immediately to the health care provider? a. The patient has an outflow volume of 1800 mL. b. The patients peritoneal effluent appears cloudy. c. The patient has abdominal pain during the inflow phase. d. The patient complains of feeling bloated after the inflow.

B

The nurse is caring for a 10 year old with leukemia who is receiving chemotherapy. The child is on neutropenic precautions. Friends of the child come to the desk and ask for a vase for the flowers they have brought with them. Which of the following is the best response? a. "I will get you a special vase that we use on this unit" b. "The flowers from your garden are beautiful but should not be placed in the room at this time" c. "As soon as I can get a vase, I will put the flowers in it and bring it to the room" d. "Get rid of the flowers immediately. You could harm the child"

B

The nurse is caring for a client with acute respiratory distress syndrome​ (ARDS) who needs an artificial airway to assist in maintaining an open airway. Which airway will the nurse plan to reposition every 8 hours while providing​ care? A. Tracheostomy B. Nasopharyngeal C. Endotracheal D. Oropharyngeal

B

The nurse is caring for a client with an endotracheal tube who is on a ventilator. When assessing the client, the nurse knows to maintain what cuff pressure to maintain appropriate pressure on the tracheal wall? A) Between 10 and 15 mm Hg B) Between 15 and 20 mm Hg C) Between 20 and 25 mm Hg D) Between 25 and 30 mm Hg

B

The nurse is caring for a client with unstable angina whose cardiac monitor shows ventricular tachycardia. Which action is appropriate to implement first? a. Defibrillate using 200 J. b. Check the client for a pulse. c. Cardiovert the client at 50 J. d. Administer IV ibutilide (Corvert).

B

The nurse is caring for a male client who recently underwent a tracheostomy. The first priority when caring for a client with a tracheostomy is: a. helping him communicate. b. keeping his airway patent. c. encouraging him to perform activities of daily living. d. preventing him from developing an infection.

B

The nurse is caring for a patient receiving an initial dose of chemotherapy to treat a rapidly growing metastatic colon cancer. The nurse is aware that this patient is at risk for tumor lysis syndrome (TLS) and will monitor the patient closely for which of the following abnormalities associated with this oncologic emergency? A. Hypokalemia B. Hypocalcemia C. Hypouricemia D. Hypophosphatemia

B

The nurse is discussing the techniques of chest physiotherapy and postural drainage (respiratory treatments) to a client having expectoration problems because of chronic thick, tenacious mucus production in the lower airway. The nurse explains that after the client is positioned for postural drainage the nurse will perform which action to help loosen secretions? a. Palpation and clubbing b. Percussion and vibration c. Hyperoxygenation and suctioning d. Administer a bronchodilator and monitor peak flow

B

The nurse is evaluating and assessing a patient with a diagnosis of chronic emphysema. The patient is receiving oxygen at a flow rate of 5 L/min by nasal cannula. Which finding concerns the nurse immediately? A.) Fine bibasilar crackles B.) Respiratory rate of 8 breaths/min C.) The patient sitting up and leaning over the nightstand D.) A large barrel chest

B

The nurse is preparing to suction a client via a tracheostomy tube. The nurse should plan to limit the suctioning attempt to a maximum of which time period? a. 5 seconds b. 10 seconds c. 30 seconds d. 60 seconds

B

The nurse is preparing to suction a client via a tracheostomy tube. The nurse should plan to limit the suctioning time to a maximum of which time period? a. 5 seconds b. 10 seconds c. 30 seconds d. 60 seconds

B

The nurse is watching the cardiac monitor, and a patient's rhythm suddenly changes. There are no P waves. Instead there are fine, wavy lines between the QRS complexes. The QRS complexes measure 0.08 sec (narrow), but they occur irregularly with a rate of 120 beats/min. The nurse correctly interprets that this rhythm is which of the following? a. Sinus tachycardia b. Atrial fibrillation c. Ventricular fibrillation d. Ventricular tachycardia

B

The nurse plans care for a patient with increased ICP with the knowledge that the best way to position the patient is to a. keep the head of the bed flat b. elevate the head of the bed to 30 degrees c. maintain patient on the left side with the head supported on a pillow d. use a continuous rotation bed to continuously change patient position

B

The nurse prepares discharge teaching for a client receiving isosorbide dinitrate for treatment of angina. What information must the nurse include? A) Limit exercise to 30 minutes twice per week. B) Avoid alcohol consumption. C) Monitor intake and output. D) Report skin flushing to the physician.

B

The nurse provides information to the caregiver of a 68-year-old man with epilepsy who has tonic-clonic seizures. Which statement, if made by the caregiver, requires further teaching? A. "It is normal for a person to be sleepy after a seizure." B. "I should call 911 if breathing stops during the seizure." C. "The jerking movements may last for 30 to 40 seconds." D. "Objects should not be placed in the mouth during a seizure."

B

The nurse receives change-of-shift report on the following four patients. Which patient should the nurse assess first? a. A 23-year-old patient with cystic fibrosis who has pulmonary function testing scheduled b. A 46-year-old patient on bed rest who is complaining of sudden onset of shortness of breath c. A 77-year-old patient with tuberculosis (TB) who has four antitubercular medications due in 15 minutes d. A 35-year-old patient who was admitted the previous day with pneumonia and has a temperature of 100.2° F (37.8° C)

B

The nurse receives in report that the client with a pacemaker has experienced loss of capture. Which situation is consistent with this? a. The pacemaker spike falls on the T wave. b. Pacemaker spikes are noted, but no P wave or QRS complex follows. c. The heart rate is 42, and no pacemaker spikes are seen on the rhythm strip. d. The client demonstrates hiccups.

B

The nurse recognizes that teaching a 44-year-old woman following a laparoscopic cholecystectomy has been effective when the patient states which of the following? a. "I can expect yellow-green drainage from the incision for a few days." b. "I can remove the bandages on my incisions tomorrow and take a shower." c. "I should plan to limit my activities and not return to work for 4 to 6 weeks." d. "I will always need to maintain a low-fat diet since I no longer have a gallbladder."

B

The nurse recognizes the presence of Cushing's triad in the patient with a. Increased pulse, irregular respiration, increased BP b. decreased pulse, irregular respiration, increased pulse pressure c. increased pulse, decreased respiration, increased pulse pressure d. decreased pulse, increased respiration, decreased systolic BP

B

The nurse suspects metastasis from left breast cancer to the thoracic spine when the client has which symptom? A. Vomiting B. Back pain C. Frequent urination D. Cyanosis of the toes

B

The nurse will suspect that the patient with stable angina is experiencing a side effect of the prescribed metoprolol (Lopressor) if the a. patient is restless and agitated. b. blood pressure is 90/54 mm Hg. c. patient complains about feeling anxious. d. cardiac monitor shows a heart rate of 61 beats/minute.

B

The nurse would suspect a patient is taking too much levothyroxine (Synthroid) when the patient exhibits which adverse effect? A. Lethargy B. Irritability C. Feeling cold D. Weight gain

B

The patient is seen in the clinic due to an increase in the frequency of seizure activity. In addition to a thorough health history you should draw blood for A. anemia. B. serum drug levels. C. arterial blood gases. D. electrolytes.

B

The physician has ordered continuous positive airway pressure (CPAP) with the delivery of a patients high-flow oxygen therapy. The patient asks the nurse what the benefit of CPAP is. What would be the nurses best response? A) CPAP allows a higher percentage of oxygen to be safely used. B) CPAP allows a lower percentage of oxygen to be used with a similar effect. C) CPAP allows for greater humidification of the oxygen that is administered. D) CPAP allows for the elimination of bacterial growth in oxygen delivery systems.

B

To improve the physical activity level for a mildly obese 71-year-old patient, which action should the nurse plan to take? a. Stress that weight loss is a major benefit of increased exercise. b. Determine what kind of physical activities the patient usually enjoys. c. Tell the patient that older adults should exercise for no more than 20 minutes at a time. d. Teach the patient to include a short warm-up period at the beginning of physical activity.

B

To prevent accidental decannulation of a tracheostomy tube, what does the nurse do? A. obtain an order for continuous upper extremity restraints B. secure the tube in place using ties or fabric fastners C.allow some flexibility in motion of the tube while coughing D. instruct the patient to hold the tube with a tissue while coughing

B

To prevent the most common serious complication of PD, what is most important for the nurse to do? a. infuse the dialysate slowly b. use strict aseptic technique in the dialysis procedures c. have the patient empty the bowel before the inflow phase d. reposition the patient frequently and promote deep breathing

B

What does the nurse include in the teaching plan for a client receiving a beta blocker for treatment of angina? A) Discontinue drug if heart rate <60. B) Do not discontinue drug abruptly. C) Exercise heart rate should be 110-120. D) Monitor for hyperglycemia.

B

What is the difference between viral and bacterial meningitis? A. Viral meningitis is treated with sulfa drugs, and bacterial meningitis is treated with penicillin. B. Full recovery is expected with the viral form, but more serious residual effects are seen with the bacterial form. C. Viral meningitis is considered more ominous because serious brain involvement is experienced. D. Typical signs of viral meningitis are diplopia and fasciculations, whereas signs of bacterial meningitis are fever and nuchal rigidity.

B

What is the most serious electrolyte disorder associated with kidney disease? a. hypocalcemia b. hyperkalemia c. hyponatremia d. hypermagnesemia

B

What nursing measure is indicated to reduce the potential for seizures and increased ICP in a patient with bacterial meningitis? A. Administering codeine for relief of head and neck pain B. Controlling fever with prescribed drugs and cooling techniques C. Keeping the room darkened and quiet to minimize environmental stimulation D. Maintaining the patient on strict bed rest with the head of the bed slightly elevated

B

What plan should the nurse use when weaning a patient from a ventilator? a. decrease the delivered FIO2 concentration b. intermittent trials of spontaneous ventilation followed by ventilatory support to provide rest c. substitute ventilator support with manual resuscitation bag if the patient becomes hypoxic d. implement weaning procedures around the clock until the patient does not experience ventilator fatigue

B

What would the critical care nurse recognize as a condition that may indicate a patients need to have a tracheostomy? A) A patient has a respiratory rate of 10 breaths per minute. B) A patient requires permanent ventilation. C) A patient exhibits symptoms of dyspnea. D) A patient has respiratory acidosis.

B

When admitting a patient with a non-ST-segment-elevation myocardial infarction (NSTEMI) to the intensive care unit, which action should the nurse perform first? a. Obtain the blood pressure. b. Attach the cardiac monitor. c. Assess the peripheral pulses. d. Auscultate the breath sounds.

B

When caring for a patient who has had a head injury, which assessment information is of most concern to the nurse? a. The blood pressure increases from 120/54 to 136/62. b. The patient is more difficult to arouse. c. The patient complains of a headache at pain level 5 of a 10-point scale. d. The patient's apical pulse is slightly irregular.

B

When discussing glucocorticoids to a patient, what statement by the nurse is accurate regarding the action of these medications? A. They decrease serum sodium and glucose levels. B. They regulate carbohydrate, fat, and protein metabolism. C. They stimulate defense mechanisms to produce immunity. D. They are produced in lower amounts during times of stress.

B

When evaluating the effectiveness of preoperative teaching with a patient scheduled for coronary artery bypass graft (CABG) surgery using the internal mammary artery, the nurse determines that additional teaching is needed when the patient says which of the following? a. "They will circulate my blood with a machine during the surgery." b. "I will have small incisions in my leg where they will remove the vein." c. "They will use an artery near my heart to go around the area that is blocked." d. "I will need to take an aspirin every day after the surgery to keep the graft open."

B

When the home health RN is planning care for a patient with a seizure disorder, which nursing action can be delegated to an LPN/LVN? a. Make referrals to appropriate community agencies. b. Place medications in the home medication organizer. c. Teach the patient and family how to manage seizures. d. Assess for use of medications that may precipitate seizures.

B

When titrating IV nitroglycerin (Tridil) for a patient with a myocardial infarction (MI), which action will the nurse take to evaluate the effectiveness of the medication? a. Monitor heart rate. b. Ask about chest pain. c. Check blood pressure. d. Observe for dysrhythmias.

B

Which action can the same-day surgery charge nurse delegate to an experienced unlicensed assistive personnel (UAP) who is helping with the care of a client who is having a breast biopsy? A. Assess anxiety level about the surgery. B. Monitor the vital signs after surgery. C. Obtain data about breast cancer risk factors. D. Teach about postoperative routine care.

B

Which assessment data collected by the nurse who is admitting a patient with chest pain suggest that the pain is caused by an acute myocardial infarction (AMI)? a. The pain increases with deep breathing. b. The pain has lasted longer than 30 minutes. c. The pain is relieved after the patient takes nitroglycerin. d. The pain is reproducible when the patient raises the arms.

B

Which assessment finding by the nurse caring for a patient who has had coronary artery bypass grafting using a right radial artery graft is most important to communicate to the health care provider? a. Complaints of incisional chest pain b. Pallor and weakness of the right hand c. Fine crackles heard at both lung bases d. Redness on both sides of the sternal incision

B

Which client problem does the nurse set as the priority for the client experiencing chemotherapy-induced peripheral neuropathy? A. Potential for lack of understanding related to side effects of chemotherapy B. Risk for Injury related to sensory and motor deficits C. Potential for ineffective coping strategies related to loss of motor control D. Altered sexual function related to erectile dysfunction

B

Which measure is most effective in preventing the spread of viral meningitis? A. Have close personnel wear surgical masks. B. Avoid touching respiratory secretions. C. Obtain yearly vaccinations. D. Gargle daily with salt wate

B

Which measurement is the best indicator of how well an antiseizure medication is working? A. Serum drug levels B. Frequency and duration of seizures C. Liver enzymes D. Urinary output

B

Which nursing action for a patient who has arrived for a scheduled hemodialysis session is most appropriate for the RN to delegate to a dialysis technician? a. Educate patient about fluid restrictions. b. Check blood pressure before starting dialysis. c. Assess for reasons for increase in predialysis weight. d. Determine the ultrafiltration rate for the hemodialysis.

B

Which nursing action is important when suctioning the secretions of a client with a tracheostomy? A. use a new sterile catheter with each insertion B. initiate suction as the catheter is being withdrawn C. insert the catheter until the cough reflex is stimulated D. Remove the inner cannula before inserting the suction catheter

B

Which of the following is the best method to prevent the spread of infection to an immunosuppressed child? a. administer antibiotics prophylactically to the child b. have people wash their hands prior to contact with the child c. assign the same nurses to care for the child each day d. limit visitors to family members only

B

Which of the following landmarks is the correct one for obtaining an apical pulse? a.) Left intercostal space, midaxillary line b.) Left fifth intercostal space, midclavicular line c.) Left second intercostal space, midclavicular line d.) Left seventh intercostal space, midclavicular line

B

Which of the following situations is likely to result in an absence of fluctuations in the chest drainage tubing? A. The tubing is coiled on the bed with a straight path to the chest drain B. The tubing is blocked in some way C. The patient is receiving positive pressure ventilation D. The patient is ambulatory

B

Which of the following would be most appropriate for a male client with an arterial blood gas (ABG) of pH 7.5, PaCO2 26 mm Hg, O2 saturation 96%, HCO3 24 mEq/L, and PaO2 94 mm Hg? a. Administer a prescribed decongestant. b. Instruct the client to breathe into a paper bag. c. Offer the client fluids frequently. d. Administer prescribed supplemental oxygen.

B

Which of these does the nurse recognize as the goal of palliative surgery for the client with cancer? A. Cure of the cancer B. Relief of symptoms or improved quality of life C. Allowing other therapies to be more effective D. Prolonging the client's survival time

B

Which of these nursing actions included in the care of a mechanically ventilated patient with acute respiratory distress syndrome (ARDS) can the RN delegate to an experienced LPN/LVN working in the intensive care unit? a. Assess breath sounds b. Insert a retention catheter c. Place patient in the prone position d. Monitor pulmonary artery pressures

B

Which patient information will the nurse plan to obtain in order to determine the effectiveness of the prescribed calcium carbonate (Caltrate) for a patient with chronic kidney disease (CKD)? a. Blood pressure b. Phosphate level c. Neurologic status d. Creatinine clearance

B

Which patient is most likely going into respiratory failure? A. A patient who report that he feels short of breath while eating B. A patient with the following arterial blood gas values over the past 3 hours: pH 7.50, 7.45, and 7.40 C. A patient with an oxygen saturation value of 93% D. A patient with chronic obstructive pulmonary disease (COPD) who has distant breath sounds

B

Which should be included in the client's plan of care during dialysis therapy? a. limit the client's visitors b. monitor the client's blood pressure c. pad the side rails of the bed d. keep the client on NPO status

B

While caring for a patient who has acute pericarditis, the nurse suspects the development of cardiac tamponade based on which assessment finding? a. A pulsus paradoxus of 8 mmHg b. Jugular vein distention c. Increased systolic blood pressure with widening pulse pressure d. Hiccoughs and hoarseness

B

While caring for a patient with an endotracheal tube, the nurses recognizes that suctioning is required how often? A) Every 2 hours when the patient is awake B) When adventitious breath sounds are auscultated C) When there is a need to prevent the patient from coughing D) When the nurse needs to stimulate the cough reflex

B

You are asked to prepare Mr. W for a thoracentesis to relieve his left pleural effusion. He is very short of breath. Which position would be most appropriate for him during the procedure? A) Lying on his right side with his back flush to the edge of the bed B) Sitting on the side of the bed with his arms supported on the bedside table C) Lying prone with the head of the bed in Trendelenburg to facilitate drainage of fluids D) Lying supine with both arms place beneath the head

B

You are caring for a patient admitted with a subdural hematoma after a motor vehicle accident. Which change in vital signs would you interpret as a manifestation of increased intracranial pressure? A. Tachypnea B. Bradycardia C. Hypotension D. Narrowing pulse pressure

B

You are taking care of the patient with an ETT. The pressure in the cuff is found to be 18 cm H2O. The nurse knows that which of the following is the greatest concern? A) Risk for bleeding B) Risk for aspiration C) Risk for tissue damage D) Risk for hypoxia

B

Your patient is post-op day 3 from a cholecystectomy due to cholecystitis and has a T-Tube. Which finding during your assessment of the T-Tube requires immediate nursing intervention? A. The drainage from the T-Tube is yellowish/green in color. B. There is approximately 750 cc of drainage within the past 24 hours. C. The drainage bag and tubing is at the patient's waist. D. The patient is in the Semi-Fowler's position.

B

a nurse is assessing a client who reports severe headache and a stiff neck. the nurses assessment reveals positive kernels and brudzinskis signs. which of the following actions should the nurses perform first? A. administer antibiotics B. implement droplet precautions C. initiate IV access D. decrease bright lights

B

the nurse is assessing the motor function of an unconscious client. the nurse should plan to use which techniques to test the clients peripheral response to pain? A. sternal rub B. nail bed pressure C. pressure on the orbital rim D. squeezing of the sternocleidomastoid muscle

B

the nurse is caring for a client who begins to experience seizure activity while in bed. which action by the nurse is contraindicated? A. loosening restrictive clothing B. restraining the clients limbs C. removing the pillow and raising padded side rails D. positioning the client to the side, if possible, with the head flexed forward

B

the nurse is caring for the client with increased intracranial pressure. the nurse would note which trend in vital sings if the intracranial pressure is rising? A. increasing temperature, increasing pulse, increasing respirations, decreasing blood pressure B. increasing temperature, decreasing pulse, decreasing respirations. increasing blood pressure C. decreasing temperature, decreasing pulse, increasing respirations, decreasing blood pressure D. decreasing temperature, increasing pulse, decreasing respirations, increasing blood pressure

B

the nurse is evaluating the pin sites of a client in skeletal traction. the nurse would be least concerned with which finding? A. inflammation B. serous drainage C. pain at pin site D. purulent drainage

B

When preparing a male client, age 51, for surgery to treat appendicitis, the nurse formulates a nursing diagnosis of Risk for infection related to inflammation, perforation, and surgery. What is the rationale for choosing this nursing diagnosis? a. Obstruction of the appendix may increase venous drainage and cause the appendix to rupture. b. Obstruction of the appendix reduces arterial flow, leading to ischemia, inflammation, and rupture of the appendix. c. The appendix may develop gangrene and rupture, especially in a middle-aged client. d. Infection of the appendix diminishes necrotic arterial blood flow and increases venous drainage.

B A client with appendicitis is at risk for infection related to inflammation, perforation, and surgery because obstruction of the appendix causes mucus fluid to build up, increasing pressure in the appendix and compressing venous outflow drainage. The pressure continues to rise with venous obstruction; arterial blood flow then decreases, leading to ischemia from lack of perfusion. Inflammation and bacterial growth follow, and swelling continues to raise pressure within the appendix, resulting in gangrene and rupture. Geriatric, not middle-aged, clients are especially susceptible to appendix rupture.

A 61-year-old patient with suspected bowel obstruction had a nasogastric tube inserted at 4:00 AM. The nurse shares in the morning report that the day shift staff should check the tube for patency at what times? A) 7:00 AM, 10:00 AM, and 1:00 PM B) 8:00 AM, 12:00 PM, and 4:00 PM C) 9:00 AM and 3:00 PM D) 9:00 AM, 12:00 PM, and 3:00 PM

B A nasogastric tube should be checked for patency routinely at 4-hour intervals. Thus if the tube were inserted at 4:00 AM, it would be due to be checked at 8:00 AM, 12:00 PM, and 4:00 PM.

What is the primary way that a nurse will evaluate the patency of an AVF? a. Palpate for pulses distal to the graft site. b. Auscultate for the presence of a bruit at the site. c. Evaluate the color and temperature of the extremity. d. Assess for the presence of numbness and tingling distal to the site.

B A patent arteriovenous fistula (AVF) creates turbulent blood flow that can be assessed by listening for a bruit or palpated for a thrill as the blood passes through the graft. Assessment of neurovascular status in the extremity distal to the graft site is important to determine that the graft does not impair circulation to the extremity but the neurovascular status does not indicate whether the graft is open.

A patient has a chest tube for treatment of a pneumothorax in the left lung. Which finding during your assessment requires immediate nursing intervention? A. The water seal chamber has intermittent bubbling. B. The patient has slight tracheal deviation to the right side. C. The water seal chamber fluctuates while the patient inhales and exhales. D. The patient complains of tenderness at the chest tube insertion site.

B A patient with a chest tube is at risk for a tension pneumothorax due to the risk of pressure building up in the intrapleural space. Therefore, the nurse would want to monitor the patient for this and if tracheal deviation is present this is a major sign a tension pneumothorax. All the other options are normal findings. The water seal chamber will have intermittent (not excessive) bubbling because of the air that will be leaving the intrapleural space. The water seal chamber will flucutate up and down when the patient breathes in and out, and it is normal for the patient to have tenderness at the insertion site of the chest tube.

When planning care for a patient with chronic lung disease who is receiving oxygen through a nasal cannula, what does the nurse expect? a) The oxygen must be humidified. b) The rate will be no more than 2 to 3 L/min or less. c) Arterial blood gases will be drawn every 4 hours to assess flow rate. d) The rate will be 6 L/min or more.

B A rate higher than 3 L/min may destroy the hypoxic drive that stimulates respirations in the medulla in a patient with chronic lung disease. Oxygen delivered at low rates does not necessarily have to be humidified, and arterial blood gases are not required at regular intervals to determine the flow rate.

The nurse conducts a complete physical assessment on a patient admitted with infective endocarditis. Which finding is significant? a. Respiratory rate of 18 and heart rate of 90 b. Regurgitant murmur at the mitral valve area c. Heart rate of 94 and capillary refill time of 2 seconds d. Point of maximal impulse palpable in fourth intercostal space

B A regurgitant murmur of the aortic or mitral valves would indicate valvular disease, which is a complication of endocarditis. All the other findings are within normal limits.

A patient is scheduled for a cardiac catheterization with coronary angiography. Before the test, the nurse informs the patient that a.it will be important to lie completely still during the procedure. b.a flushed feeling may be noted when the contrast dye is injected. c.monitored anesthesia care will be provided during the procedure. d.arterial pressure monitoring will be required for 24 hours after the test.

B A sensation of warmth or flushing is common when the contrast material is injected, which can be anxiety-producing unless it has been discussed with the patient. The patient may receive a sedative drug before the procedure, but monitored anesthesia care is not used. Arterial pressure monitoring is not routinely used after the procedure to monitor blood pressure. The patient is not immobile during cardiac catheterization and may be asked to cough or take deep breaths.

The client is taking fludrocortisone (Florinef) for adrenal hypofunction. The nurse instructs the client to report which symptom while taking this drug? a) Anxiety b) Headache c) Nausea d) Weight loss

B A side effect of fludrocortisone is hypertension. New onset of headache should be reported, and the client's blood pressure should be monitored. Anxiety is not a side effect of fludrocortisone and is not associated with adrenal hypofunction. Nausea is associated with adrenal hypofunction; it is not a side effect of fludrocortisone. Sodium-related fluid retention and weight gain, not loss, are possible with fludrocortisone therapy.

In a patient with AKI, which laboratory urinalysis result indicates tubular damage? a. Hematuria b. Specific gravity fixed at 1.010 c. Urine sodium of 12 mEq/L (12 mmol/L) d. Osmolality of 1000 mOsm/kg (1000 mmol/kg)

B A urine specific gravity that is consistently 1.010 and a urine osmolality of about 300 mOsm/kg is the same specific gravity and osmolality as plasma. This indicates that tubules are damaged and unable to concentrate urine. Hematuria is more common with postrenal damage. Tubular damage is associated with a high sodium concentration (greater than 40 mEq/L).

The nurse is caring for a 55-year-old man patient with acute pancreatitis resulting from gallstones. Which clinical manifestation would the nurse expect the patient to exhibit? A. Hematochezia B. Left upper abdominal pain C. Ascites and peripheral edema D. Temperature over 102o F (38.9o C)

B Abdominal pain (usually in the left upper quadrant) is the predominant manifestation of acute pancreatitis. Other manifestations of acute pancreatitis include nausea and vomiting, low-grade fever, leukocytosis, hypotension, tachycardia, and jaundice. Abdominal tenderness with muscle guarding is common. Bowel sounds may be decreased or absent. Ileus may occur and causes marked abdominal distention. Areas of cyanosis or greenish to yellow-brown discoloration of the abdominal wall may occur. Other areas of ecchymoses are the flanks (Grey Turner's spots or sign, a bluish flank discoloration) and the periumbilical area (Cullen's sign, a bluish periumbilical discoloration).

A patient who has been on continuous ambulatory peritoneal dialysis (CAPD) is hospitalized and is receiving CAPD with four exchanges a day. During the dialysate inflow, the patient complains of having abdominal pain and pain in the right shoulder. The nurse should a. massage the patient's abdomen and back. b. decrease the rate of dialysate infusion. c. stop the infusion and notify the health care provider. d. administer the PRN acetaminophen (Tylenol).

B Abdominal pain and referred shoulder pain can be caused by a rapid infusion of dialysate; the nurse should slow the rate of the infusion. Massage and administration of acetaminophen (Tylenol) would not address the reason for the pain. There is no need to notify the health care provider.

What is the first intervention for a client experiencing MI? A) Administer morphine B) Administer oxygen C) Administer sublingual nitroglycerin D) Obtain an ECG

B Administering supplemental oxygen to the client is the first priority of care. The myocardium is deprived of oxygen during an infarction, so additional oxygen is administered to assist in oxygenation and prevent further damage. Morphine and nitro are also used to treat MI, but they're more commonly administered after the oxygen. An ECG is the most common diagnostic tool used to evaluate MI

A hospital nurse has experienced percutaneous exposure to an HIV-positive patient's blood as a result of a needlestick injury. The nurse has informed the supervisor and identified the patient. What action should the nurse take next? A) Flush the wound site with chlorhexidine. B) Report to the emergency department or employee health department. C) Apply a hydrocolloid dressing to the wound site. D) Follow up with the nurse's primary care provider.

B After initiating the emergency reporting system, the nurse should report as quickly as possible to the employee health services, the emergency department, or other designated treatment facility. Flushing is recommended, but chlorhexidine is not used for this purpose. Applying a dressing is not recommended. Following up with the nurse's own primary care provider would require an unacceptable delay

After teaching a client who is recovering from an endoscopic trans-nasal hypophysectomy, the nurse assesses the client's understanding. Which statement made by the client indicates a correct understanding of the teaching? a. I will wear dark glasses to prevent sun exposure. b. I'll keep food on upper shelves so I do not have to bend over. c. I must wash the incision with peroxide and redress it daily. d. I shall cough and deep breathe every 2 hours while I am awake.

B After this surgery, the client must take care to avoid activities that can increase intracranial pressure. The client should avoid bending from the waist and should not bear down, cough, or lie flat. With this approach, there is no incision to clean and dress. Protection from sun exposure is not necessary after this procedure.

A malnourished patient is receiving a parenteral nutrition (PN) infusion containing amino acids and dextrose from a bag that was hung 24 hours ago. The nurse observes that about 50 mL remain in the PN container. Which action is best for the nurse to take? a. Ask the health care provider to clarify the written PN order. b. Add a new container of PN using the current tubing and filter. c. Hang a new container of PN and change the IV tubing and filter. d. Infuse the remaining 50 mL and then hang a new container of PN.

B All PN solutions are changed at 24 hours. PN solutions containing dextrose and amino acids require a change in tubing and filter every 72 hours rather than daily. Infusion of the additional 50 mL will increase patient risk for infection. Changing the IV tubing and filter more frequently than required will unnecessarily increase costs. The nurse (not the health care provider) is responsible for knowing the indicated times for tubing and filter changes.

The nurse is caring for a client who is receiving rituximab (Rituxan) for treatment of lymphoma. It is essential for the nurse to observe for which side effect? A. Alopecia B. Allergy C. Fever D. Chills

B Allergy is the most common side effect. Although fever & chills are side effect of monoclonal antibody therapy, they would not take priority over an allergic response that could potentially involve the airway.

A nurse is assessing clients on a medical-surgical unit. Which client should the nurse identify as being at greatest risk for atrial fibrillation? a. A 45-year-old who takes an aspirin daily b. A 50-year-old who is post coronary artery bypass graft surgery c. A 78-year-old who had a carotid endarterectomy d. An 80-year-old with chronic obstructive pulmonary disease

B Atrial fibrillation occurs commonly in clients with cardiac disease and is a common occurrence after coronary artery bypass graft surgery. The other conditions do not place these clients at higher risk for atrial fibrillation.

A nurse evaluates prescriptions for a client with chronic atrial fibrillation. Which medication should the nurse expect to find on this client's medication administration record to prevent a common complication of this condition? a. Sotalol (Betapace) b. Warfarin (Coumadin) c. Atropine (Sal-Tropine) d. Lidocaine (Xylocaine)

B Atrial fibrillation puts clients at risk for developing emboli. Clients at risk for emboli are treated with anticoagulants, such as heparin, enoxaparin, or warfarin. Sotalol, atropine, and lidocaine are not appropriate for this complication.

Which precaution is most important for the nurse to teach a client with leukemia to prevent an infection by autocontamination? A. Take antibiotics exactly as prescribed. B. Perform mouth care three times daily. C. Avoid the use of pepper and raw foods. D. Report any burning on urination immediately.

B Autocontamination is the overgrowth of the client's own normal flora or the translocation of his or her normal flora from its normal location to a different one. Performing frequent mouth care can reduce the number of normal flora organisms in the mouth and decrease the risk for developing an infection from autocontamination. Taking antibiotics does not prevent autocontamination, nor does reporting symptoms of an infection. Avoiding exposure to environmental organisms does not prevent autocontamination.

In which type of dialysis does the patient dialyze during sleep and leave the fluid in the abdomen during the day? a. Long nocturnal hemodialysis b. Automated peritoneal dialysis (APD) c. Continuous venovenous hemofiltration (CVVH) d. Continuous ambulatory peritoneal dialysis (CAPD)

B Automated peritoneal dialysis (APD) is the type of dialysis in which the patient dialyzes during sleep and leaves the fluid in the abdomen during the day. Long nocturnal hemodialysis occurs while the patient is sleeping and is done up to six times per week. Continuous venovenous hemofiltration (CVVH) is a type of continuous renal replacement therapy used to treat AKI. Continuous ambulatory peritoneal dialysis (CAPD) is dialysis that is done with exchanges of 1.5 to 3 L of dialysate at least four times daily.

A new graduate RN discovers that her client, who had a tracheostomy placed the previous day, has completely dislodged both the obturator and the tracheostomy tube. Which action should the nurse take first? A. Auscultate the client's breath sounds while applying a nasal cannula B. Direct someone to call the Rapid Response Team while using a resuscitation bag and facemask C. Apply a 100% non-rebreather mask while administering high-flow oxygen D. Replace the obturator while inserting the tracheostomy tube

B Because a fresh tracheostomy stoma will collapse, the client will lose his airway patency, which will require the nurse to ventilate the client through the mouth and nose while waiting for assistance to re-cannulate the client. Directing someone else to call the Rapid Response Team allows the nurse to provide immediate care required by the client.

The patient is receiving an IV vesicant chemotherapy drug. The nurse notices swelling and redness at the site. What should the nurse do first? A. Ask the patient if the site hurts. B. Turn off the chemotherapy infusion. C. Call the ordering health care provider. D. Administer sterile saline to the reddened area.

B Because extravasation of vesicants may cause severe local tissue breakdown and necrosis, with any sign of extravasation the infusion should first be stopped. Then the protocol for the drug-specific extravasation procedures should be followed to minimize further tissue damage. The site of extravasation usually hurts, but it may not. It is more important to stop the infusion immediately. The health care provider may be notified by another nurse while the patient's nurse starts the drug-specific extravasation procedures, which may or may not include sterile saline.

A 49-year-old man has been admitted with hypotension and dehydration after 3 days of nausea and vomiting. Which order from the health care provider will the nurse implement first? a. Insert a nasogastric (NG) tube. b. Infuse normal saline at 250 mL/hr. c. Administer IV ondansetron (Zofran). d. Provide oral care with moistened swabs.

B Because the patient has severe dehydration, rehydration with IV fluids is the priority. The other orders should be accomplished as quickly as possible after the IV fluids are initiated.

For a female client newly diagnosed with radiation-induced thrombocytopenia, the nurse should include which intervention in the plan of care? a. Administering aspirin if the temperature exceeds 102° F (38.8° C) b. Inspecting the skin for petechiae once every shift c. Providing for frequent rest periods d. Placing the client in strict isolation

B Because thrombocytopenia impairs blood clotting, the nurse should inspect the client regularly for signs of bleeding, such as petechiae, purpura, epistaxis, and bleeding gums. The nurse should avoid administering aspirin because it may increase the risk of bleeding. Frequent rest periods are indicated for clients with anemia, not thrombocytopenia. Strict isolation is indicated only for clients who have highly contagious or virulent infections that are spread by air or physical contact.

A client with a burn injury due to a house fire is admitted to the burn unit. The client's family asks the nurse why the client received a tetanus toxoid injection on admission. What is the nurse's best response to the client's family member? A. "The last tetanus injection was less than 5 years ago." B. "Burn wound conditions promote the growth of Clostridium tetani." C. "The wood in the fire had many nails, which penetrated the skin." D. "The injection was prescribed to prevent infection from Pseudomonas."

B Burn wound conditions promote the growth of Clostridium tetani, and all burn clients are at risk for this dangerous infection. Tetanus toxoid enhances acquired immunity to C. tetani, so this agent is routinely given when the client is admitted to the hospital. Regardless of when the last tetanus injection is given, it is still given on admission to prevent C. tetani. The fact that there were many nails in the wood in the fire is irrelevant. Tetanus toxoid injection does not prevent Pseudomonas infection.

The nurse recognizes that calcium channel blockers prescribed for treatment of angina exert their effect by: A) Increasing preload. B) Decreasing afterload. C) Positive chronotropic effect. D) Positive inotropic effect.

B Calcium channel blockers cause arteriolar smooth muscle relaxation, leading to lowered peripheral resistance and decreased blood pressure (decreased afterload). This decreases myocardial oxygen demand, and reduces frequency of anginal pain.

The nurse is teaching a wellness class to a group of women at their workplace. The nurse knows that which woman is at highest risk for developing cancer? A. A woman who obtains regular cancer screenings and consumes a high-fiber diet B. A woman who has a body mass index of 35 kg/m2 and smoked cigarettes for 20 years C. A woman who exercises five times every week and does not consume alcoholic beverages D. A woman who limits fat consumption and has regular mammography and Pap screenings

B Cancer prevention and early detection are associated with the following behaviors: limited alcohol use; regular physical activity; maintaining a normal body weight; obtaining regular cancer screenings; avoiding cigarette smoking and other tobacco use; using sunscreen with SPF 15 or higher; and practicing healthy dietary habits (e.g., reduced fat and increased fruits and vegetables).

A client who is receiving continuous oxygen therapy by nasal cannula for an acute respiratory problem is becoming increasingly confused. What does the nurse do first? a. Notify the health care provider. b. Assess the client's pulse oximetry. c. Document the observation. d. Raise the head of the bed.

B Cerebral hypoxia is a cause of confusion and is a sensitive indicator that the client needs more oxygen. Although you would want to notify the provider of the change in the client's condition, the best action is first to assess pulse oximetry and then to increase the oxygen. You would not just document the assessment finding without intervening. Raising the head of the bed would not help the client oxygenate better.

Which wound assessment characteristics suggest a superficial partial-thickness burn injury? A. Black-brown coloration B. Painful C. Moderate to severe edema D. Absence of blisters

B Characteristics of a superficial partial-thickness burn injury include pink to red coloration, mild to moderate edema, pain, and blisters. A black-brown coloration is more suggestive of full-thickness burn injury. Moderate to severe edema and absence of blisters may be present with deep partial-thickness to full-thickness burn injuries.

Which item should the nurse offer to the patient who is to restart oral intake after being NPO due to nausea and vomiting? a. Glass of orange juice b. Dish of lemon gelatin c. Cup of coffee with cream d. Bowl of hot chicken broth

B Clear cool liquids are usually the first foods started after a patient has been nauseated. Acidic foods such as orange juice, very hot foods, and coffee are poorly tolerated when patients have been nauseated.

The nurse is assessing a patient who is receiving peritoneal dialysis with 2-L inflows. Which information should be reported immediately to the health care provider? a. The patient complains of feeling bloated after the inflow. b. The patient's peritoneal effluent appears cloudy. c. The patient has abdominal pain during the inflow phase. d. The patient has an outflow volume of 1600 ml.

B Cloudy-appearing peritoneal effluent is a sign of peritonitis and should be reported immediately so that treatment with antibiotics can be started. The other problems can be addressed through nursing interventions such as slowing the inflow and repositioning the patient.

The nurse determines that teaching regarding cobalamin injections has been effective when the patient with chronic atrophic gastritis states which of the following? a. "The cobalamin injections will prevent gastric inflammation." b. "The cobalamin injections will prevent me from becoming anemic." c. "These injections will increase the hydrochloric acid in my stomach." d. "These injections will decrease my risk for developing stomach cancer."

B Cobalamin supplementation prevents the development of pernicious anemia. Chronic gastritis may cause achlorhydria, but cobalamin does not correct this. The loss of intrinsic factor secretion with chronic gastritis is permanent, and the patient will need lifelong supplementation with cobalamin. The incidence of stomach cancer is higher in patients with chronic gastritis, but cobalamin does not reduce the risk for stomach cancer.

An older adult client is being discharged home with a tracheostomy. Which nursing action is an acceptable assignment for an experienced LPN/LVN? A. Complete the referral form for a home health agency B. Suction the tracheostomy using sterile technique C. Teach the client and spouse about tracheostomy D. Consult with the physician about using a fenestrated tube

B Complex sterile procedures are within the education, scope, and practice of the experienced LPN/LVN.

The nurse is assessing a 31-year-old female patient with abdominal pain. Th nurse,who notes that there is ecchymosis around the area of umbilicus, will document this finding as a. Cullen sign. b. Rovsing sign. c. McBurney sign. d. Grey-Turner's sign.

B Cullen sign is ecchymosis around the umbilicus. Rovsing sign occurs when palpation of the left lower quadrant causes pain in the right lower quadrant. Deep tenderness at McBurney's point (halfway between the umbilicus and the right iliac crest), known as McBurney's sign, is a sign of acute appendicitis.

A patient has arrived for a scheduled hemodialysis session. Which nursing action is most appropriate for the registered nurse (RN) to delegate to a dialysis technician? a. Teach the patient about fluid restrictions. b. Check blood pressure before starting dialysis. c. Assess for causes of an increase in predialysis weight. d. Determine the ultrafiltration rate for the hemodialysis.

B Dialysis technicians are educated in monitoring for blood pressure. Assessment, adjustment of the appropriate ultrafiltration rate, and patient teaching require the education and scope of practice of an RN.

A nurse is performing an admission assessment on a patient with stage 3 HIV. After assessing the patient's gastrointestinal system and analyzing the data, what is most likely to be the priority nursing diagnosis? A) Acute Abdominal Pain B) Diarrhea C) Bowel Incontinence D) Constipation

B Diarrhea is a problem in 50% to 60% of all AIDS patients. As such, this nursing diagnosis is more likely than abdominal pain, incontinence, or constipation, though none of these diagnoses is guaranteed not to apply.

A patient with AKI has a serum potassium level of 6.7 mEq/L (6.7 mmol/L) and the following arterial blood gasresults: pH 7.28, PaCO230 mm Hg, PaO286 mm Hg, HCO3−18 mEq/L (18 mmol/L). The nurse recognizes thattreatment of the acid-base problem with sodium bicarbonate would cause a decrease in which value? a. pH b. Potassium level c. Bicarbonate level d. Carbon dioxide level

B During acidosis, potassium moves out of the cell in exchange for H+ions, increasing the serum potassium level.Correction of the acidosis with sodium bicarbonate will help to shift the potassium back into the cells. A decrease in pH and the bicarbonate and PaCO2levels would indicate worsening acidosis

A client is in the resuscitation phase of burn injury. Which route does the nurse use to administer pain medication to the client? A. Intramuscular B. Intravenous C. Sublingual D. Topical

B During the resuscitation phase, the IV route is used for giving opioid drugs because of problems with absorption from the muscle and stomach. When these agents are given by the intramuscular or subcutaneous route, they remain in the tissue spaces and do not relieve pain. In addition, when edema is present, all doses are rapidly absorbed at once when the fluid shift is resolving. This delayed absorption can result in lethal blood levels of analgesics. The sublingual route may not be effective, and because the skin is too damaged, the topical route is not indicated for administering drugs to the client in the resuscitation phase of burn injury.

After receiving a permanent pacemaker, the client asks the nurse if there are any activities to avoid during a vacation scheduled for 4 months after discharge. Which of the following is the most appropriate response by the nurse? a. "Swimming in the ocean should be avoided." b. "You should avoid working over a running engine." c. "There are no restrictions on your activity." d. "Avoid standing in front of microwave ovens."

B Early microwave ovens required avoidance by persons with pacemakers, but not current models. Swimming would be contraindicated for the first few weeks due to abduction of the arm while the leads were still adhering to the muscle of the heart. However, 4 months postoperatively, the client should be able to abduct arms for swimming. Working over a running engine, as well as being near high frequency power waves, is contraindicated for anyone with a permanent pacemaker.

Which manifestation of an oncologic emergency requires the nurse to contact the health care provider immediately? A. New onset of fatigue B. Edema of arms and hands C. Dry cough D. Weight gain

B Edema of the arms and hands indicates worsening compression of the superior vena cava consistent with superior vena cava syndrome. The compression must be relieved immediately, often with radiation therapy, because death can result without timely intervention.

After a total proctocolectomy and permanent ileostomy, the patient tells the nurse, "I cannot manage all these changes. I don't want to look at the stoma." What is the best action by the nurse? a. Reassure the patient that ileostomy care will become easier. b. Ask the patient about the concerns with stoma management. c. Develop a detailed written list of ostomy care tasks for the patient. d. Postpone any teaching until the patient adjusts to the ileostomy.

B Encouraging the patient to share concerns assists in helping the patient adjust to the body changes. Acknowledgment of the patient's feelings and concerns is important rather than offering false reassurance. Because the patient indicates that the feelings about the ostomy are the reason for the difficulty with the many changes, development of a detailed ostomy care plan will not improve the patient's ability to manage the ostomy. Although detailed ostomy teaching may be postponed, the nurse should offer teaching about some aspects of living with an ostomy.

Which of the following actions is the first priority of care for a client exhibiting signs and symptoms of coronary artery disease? A) Decrease anxiety B) Enhance myocardial oxygenation C) Administer sublingual nitroglycerin D) Educate the client about his symptoms

B Enhancing myocardial oxygenation is always the first priority when a client exhibits signs or symptoms of cardiac compromise. Without adequate oxygenation, the myocardium suffers damage. Sublingual nitroglycerin is administered to treat acute angina, but administration isn't the first priority. Although educating the client and decreasing anxiety are important in care delivery, neither are priorities when a client is compromised.

In a client with mitral regurgitation the nurse would expect to see which of the following signs and symptoms? a. Crushing chest pain b. Exertional dyspnea c. Elevated white blood cell count d. Low red blood cell count

B Fluid retention and diminished heart function cause exterional dyspnea in clients with mitral regurgitation as heart failure worsens. This is due to a rise in left atrial pressure and subsequent pulmonary and venous congestion.

The nurse is titrating the IV fluid infusion rate immediately after a patient has had kidney transplantation. Which parameter will be most important for the nurse to consider? a. Heart rate b. Urine output c. Creatinine clearance d. Blood urea nitrogen (BUN) level

B Fluid volume is replaced based on urine output after transplant because the urine output can be as high as a liter an hour. The other data will be monitored but are not the most important determinants of fluid infusion rate

A nurse assists with the cardioversion of a client experiencing acute atrial fibrillation. Which action should the nurse take prior to the initiation of cardioversion? a. Administer intravenous adenosine. b. Turn off oxygen therapy. c. Ensure a tongue blade is available. d. Position the client on the left side.

B For safety during cardioversion, the nurse should turn off any oxygen therapy to prevent fire. The other interventions are not appropriate for a cardioversion. The client should be placed in a supine position.

A client undergoing hemodialysis has an arteriovenous (AV) fistula in the left arm. A related nursing diagnosis for the client is risk for infection. The nurse should formulate which of the following outcome goals as most appropriate for this nursing diagnosis? a. The client's temperature remains less than 101F b. The client's WBC count remains within normal limits. c. The client washes hands at least once per day. d. The client states to avoid blood pressure measurement in the left arm.

B General indicators that the client is not experiencing infection include a temperature and WBC count within normal limits. The client also should use proper hand-washing technique as a general preventive measure. Hand washing once per day is insufficient. It is true that the client should avoid BP measurement in the affected arm; however, this would relate more closely to the nursing diagnosis risk for injury.

A patient with acute respiratory distress syndrome (ARDS) and acute renal failure has the following medications prescribed. Which medication should the nurse discuss with the health care provider before administration? a. ranitidine (Zantac) 50 mg IV b. gentamicin (Garamycin) 60 mg IV c. sucralfate (Carafate) 1 g per nasogastric tubed. d. methylprednisolone (Solu-Medrol) 40 mg IV

B Gentamicin, which is one of the aminoglycoside antibiotics, is potentially nephrotoxic, and the nurse should clarify the drug and dosage with the health care provider before administration. The other medications are appropriate for the patient with ARDS.

A nurse assesses clients for potential endocrine dysfunction. Which client is at greatest risk for a deficiency of gonadotropin and growth hormone? a. A 36-year-old female who has used oral contraceptives for 5 years b. A 42-year-old male who experienced head trauma 3 years ago c. A 55-year-old female with a severe allergy to shellfish and iodine d. A 64-year-old male with adult-onset diabetes mellitus

B Gonadotropin and growth hormone are anterior pituitary hormones. Head trauma is a common cause of anterior pituitary hypofunction. The other factors do not increase the risk of this condition.

A pregnant woman develops chest pain and is found to be in atrial fibrillation. Which medication would be appropriate to prescribe for this client? A. Warfarin B. Heparin C. Aspirin D. Atenolol

B Heparin is the only one of the medications that would be used for this problem and does not cross the placental barrier. Warfarin, atenolol and aspirin cross the placenta and are distributed in breast milk, leading to potential fetal death, neonatal hemorrhage, or intrauterine death.

Beep Beep Beep. The high pressure alarm is sounding in the patient's room. Which of the following is the most likely cause. A) The ventilator tubing has become disconnected B) The patient is trying to talk to his friend C) There is a leak in the cuff D) The patient is c/o pain 9/10

B High pressure alarms sound when anything is blocking the air from going down the tube. Some possible causes include biting the tube, excess secretions, kinking, condensation in tubing, the patient gagging, coughing, or talking, or a more serious complication like pneumothorax or bronchospasm. Disconnected tubing would most likely set off a low pressure alarm. A leak in the cough would prevent all the air to go into the lungs efficiently. Pain itself would not affect the pressure.

A patient with adrenocortical insufficiency is prescribed hydrocortisone. Which drugs should be avoided in the patient's prescription? a. Oral contraceptives, antiepileptics, and nonsteroidal antiinflammatory drugs (NSAIDs) b. Oral hypoglycemics, anticoagulants, and nonsteroidal antiinflammatory drugs (NSAIDs) c. Antihypertensives, oral hypoglycemics, and nonsteroidal antiinflammatory drugs (NSAIDs) d. Antiepileptics, antihypertensives, and oral hypoglycemics

B Hydrocortisone is a corticosteroid. Oral hypoglycemics, anticoagulants, and nonsteroidal antiinflammatory drugs (NSAIDs) have potential interactions with corticosteroids and should be avoided by a patient taking hydrocortisone. Note that antiepileptics and antihypertensives may or may not interact with corticosteroids.

What distinguishes hypercapnic respiratory failure from hypoxemic respiratory failure? A. Low oxygen saturation despite administration of supplemental oxygen B. Acidemia for which the body cannot compensate C. Respiration rate greater than 30 breaths/minute D. Heart rate increases above 100 beats/minute

B Hypercapnic respiratory failure is PaCO2 greater than 48 mm Hg in combination with acidemia. The body cannot compensate for the acidemia. Hypoxemic respiratory failure is a PaO2 less than 60 mm Hg despite receiving an inspired oxygen concentration greater than or equal to 60%. The respiratory rate and heart rate are not part of the definitions of these two conditions.

A nurse cares for a client with adrenal hyperfunction. The client screams at her husband, bursts into tears, and throws her water pitcher against the wall. She then tells the nurse, "I feel like I am going crazy." How should the nurse respond? a. I will ask your doctor to order a psychiatric consult for you. b. You feel this way because of your hormone levels. c. Can I bring you information about support groups? d. I will close the door to your room and restrict visitors.

B Hypercortisolism can cause the client to show neurotic or psychotic behavior. The client needs to know that these behavior changes do not reflect a true psychiatric disorder and will resolve when therapy results in lower and steadier blood cortisol levels. The client needs to understand this effect and does not need a psychiatrist, support groups, or restricted visitors at this time.

Which finding is consistent with a diagnosis of hyperaldosteronism?Multiple choice question a. Edema b. Hypernatremia c. Low blood pressure d. Potassium retention

B In hyperaldosteronism, elevated levels of aldosterone are associated with sodium retention, which leads to hypernatremia. Edema and low blood pressure are not caused by an increase in sodium excretion. Elevated levels of aldosterone lead to potassium excretion.

A nurse is assessing a 28-year-old man with HIV who has been admitted with pneumonia. In assessing the patient, which of the following observations takes immediate priority? A) Oral temperature of 100°F B) Tachypnea and restlessness C) Frequent loose stools D) Weight loss of 1 pound since yesterday

B In prioritizing care, the pneumonia would be assessed first by the nurse. Tachypnea and restlessness are symptoms of altered respiratory status and need immediate priority. Weight loss of 1 pound is probably fluid related; frequent loose stools would not take short-term precedence over a temperature or tachypnea and restlessness. An oral temperature of 100°F is not considered a fever and would not be the first issue addressed.

A nurse cares for a client with an intravenous temporary pacemaker for bradycardia. The nurse observes the presence of a pacing spike but no QRS complex on the client's electrocardiogram. Which action should the nurse take next? a. Administer intravenous diltiazem (Cardizem). b. Assess vital signs and level of consciousness. c. Administer sublingual nitroglycerin. d. Assess capillary refill and temperature.

B In temporary pacing, the wires are threaded onto the epicardial surface of the heart and exit through the chest wall. The pacemaker spike should be followed immediately by a QRS complex. Pacing spikes seen without subsequent QRS complexes imply loss of capture. If there is no capture, then there is no ventricular depolarization and contraction. The nurse should assess for cardiac output via vital signs and level of consciousness. The other interventions would not determine if the client is tolerating the loss of capture.

A patient with HIV will be receiving care in the home setting. What aspect of self-care should the nurse emphasize during discharge education? A) Appropriate use of prophylactic antibiotics B) Importance of personal hygiene C) Signs and symptoms of wasting syndrome D) Strategies for adjusting antiretroviral dosages

B Infection control is of high importance in patients living with HIV, thus personal hygiene is paramount. This is a more important topic than signs and symptoms of one specific complication (wasting syndrome). Drug dosages should never be independently adjusted. Prophylactic antibiotics are not normally prescribed unless the patient's CD4 count is below 50.

The patient with sudden pain in the left upper quadrant radiating to the back and vomiting was diagnosed with acute pancreatitis. What intervention(s) should the nurse expect to include in the patient's plan of care? A. Immediately start enteral feeding to prevent malnutrition. B. Insert an NG and maintain NPO status to allow pancreas to rest. C. Initiate early prophylactic antibiotic therapy to prevent infection. D. Administer acetaminophen (Tylenol) every 4 hours for pain relief.

B Initial treatment with acute pancreatitis will include an NG tube if there is vomiting and being NPO to decrease pancreatic enzyme stimulation and allow the pancreas to rest and heal. Fluid will be administered to treat or prevent shock. The pain will be treated with IV morphine because of the NPO status. Enteral feedings will only be used for the patient with severe acute pancreatitis in whom oral intake is not resumed. Antibiotic therapy is only needed with acute necrotizing pancreatitis and signs of infection.

A client is hospitalized for pituitary function testing. Which nursing action included in the client's plan of care will be most appropriate for the RN to delegate to the LPN/LVN? A. Assess the client for clinical manifestations of hypopituitarism. B. Inject regular insulin for the growth hormone stimulation test. C. Palpate the thyroid gland for size and firmness. D. Teach the client about the adrenocorticotropic hormone stimulation test.

B Injection of insulin is within the LPN/LVN scope of practice. Client assessment for clinical manifestations of hypopituitarism, palpating the thyroid gland, and client education are complex skills requiring training and expertise, and are best performed by an RN.

A dehydrated patient is in the Injury stage of the RIFLE staging of AKI. What would the nursefirst anticipate in the treatment of this patient? a. Assess daily weight b. IV administration of fluid and furosemide (Lasix) c. IV administration of insulin and sodium bicarbonate d. Urinalysis to check for sediment, osmolality, sodium, and specific gravity

B Injury is the stage of RIFLE classification when urine output is less than 0.5 mL/kg/hr for 12 hours, the serum creatinine is increased times two or the glomerular filtration rate (GFR) is decreased by 50%. This stage maybe reversible by treating the cause or, in this patient, the dehydration by administering IV fluid and a low dose of a loop diuretic, furosemide (Lasix). Assessing the daily weight will be done to monitor fluid changes but it is not the first treatment the nurse should anticipate. IV administration of insulin and sodium bicarbonate would be used for hyperkalemia. Checking the urinalysis will help to determine if the AKI has a prerenal, intrarenal, orpostrenal cause by what is seen in the urine but with this patient's dehydration, it is thought to be prerenal to begin treatment.

Which finding in a patient taking levothyroxine (Synthroid) and warfarin (Coumadin) would require follow-up by a nurse? A) Cardiac dysrhythmias B) Excessive bruising C) Weight loss of 5 kg D) Shortness of breath

B Levothyroxine intensifies the effect of warfarin, an anticoagulant that increases the patient's risk for bleeding. The warfarin dose may need to be reduced. Bruising, weight loss, and shortness of breath are not effects associated with interactions of levothyroxine and warfarin.

Which client is most at risk for the development of gallstones? A. 22-year-old woman who is 1 month postpartum B. 65-year-old woman after a liquid protein diet C. 70-year-old man with peptic ulcer disease D. 33-year-old man with type 2 diabetes

B Liquid protein diets increase susceptibility to gallstones by releasing cholesterol from tissues, which is then excreted as crystals in the bile.

Which client problem relating to altered nutrition is a consequence of AIDS? A. Increased appetite B. Decreased protein absorption C. Increased secretions of digestive juices D. Decreased gastrointestinal absorption

B Often the complications of the acquired immunodeficiency syndrome (AIDS) have a negative impact on nutritional status. Weight loss and protein depletion are commonly seen among the AIDS population.

A nurse is caring for a client who was prescribed high-dose corticosteroid therapy for 1 month to treat a severe inflammatory condition. The client's symptoms have now resolved and the client asks, "When can I stop taking these medications?" How should the nurse respond? a. It is possible for the inflammation to recur if you stop the medication. b. Once you start corticosteroids, you have to be weaned off them. c. You must decrease the dose slowly so your hormones will work again. d. The drug suppresses your immune system, which must be built back up.

B One of the most common causes of adrenal insufficiency, a life-threatening problem, is the sudden cessation of long-term, high-dose corticosteroid therapy. This therapy suppresses the hypothalamic-pituitary-adrenal axis and must be withdrawn gradually to allow for pituitary production of adrenocorticotropic hormone and adrenal production of cortisol. Decreasing hormone therapy slowly ensures self-production of hormone, not hormone effectiveness. Building the client's immune system and rebound inflammation are not concerns related to stopping high-dose corticosteroids.

A nurse is caring for a client who was prescribed high-dose corticosteroid therapy for 1 month to treat a severe inflammatory condition. The clients symptoms have now resolved and the client asks, When can I stop taking these medications? How should the nurse respond? a. It is possible for the inflammation to recur if you stop the medication. b. Once you start corticosteroids, you have to be weaned off them. c. You must decrease the dose slowly so your hormones will work again. d. The drug suppresses your immune system, which must be built back up.

B One of the most common causes of adrenal insufficiency, a life-threatening problem, is the sudden cessation of long-term, high-dose corticosteroid therapy. This therapy suppresses the hypothalamic-pituitary-adrenal axis and must be withdrawn gradually to allow for pituitary production of adrenocorticotropic hormone and adrenal production of cortisol. Decreasing hormone therapy slowly ensures self-production of hormone, not hormone effectiveness. Building the clients immune system and rebound inflammation are not concerns related to stopping high-dose corticosteroids.

What structural changes to the respiratory system should a nurse observe when caring for older adults? a) increased use of accessory muscles for breathing b) respiratory muscles become weaker c) increased mouth breathing and snoring d) diminished coughing and gag reflexes

B One of the structural changes affecting the respiratory system that a nurse should observe in an older adult is respiratory muscles becoming weaker. The nurse should also observe other structural changes: the chest wall becomes stiffer as a result of calcification of the intercostals cartilage, kyphoscoliosis, and arthritic changes to costovertebral joints; the ribs and vertebrae lose calcium; the lungs become smaller and less elastic; alveoli enlarge; and alveolar walls become thinner. Diminished coughing and gag reflexes, increased use of accessory muscles for breathing, and increased mouth breathing and snoring are functional changes to the respiratory system in older adults.

A patient receiving peritoneal dialysis using 2 L of dialysate per exchange has an outflow of 1200 ml. Which action should the nurse take first? a. Infuse 1200 ml of dialysate during the inflow. b. Assist the patient in changing position. c. Administer a laxative to the patient. d. Notify the health care provider about the outflow problem.

B Outflow problems may occur because the peritoneal catheter is collapsed by a portion of the intestine, and repositioning the patient will move the catheter and allow outflow to occur. If less than the ordered 2 L of dialysate is infused, the dialysis will be less effective. Administration of a laxative may also help if the patient's colon is full, but this should be tried after repositioning the patient. If the problem with outflow persists after the patient is repositioned, the health care provider should be notified.

A client has been brought in by the rescue squad to the emergency department. The client is having an acute exacerbation of chronic obstructive pulmonary disease (COPD) and is severely short of breath. On arrival, the client is on 15 L/min of oxygen via rebreather mask. Which action by the nurse takes priority? a. Immediately reduce the oxygen flow to 2 to 4 L/min via nasal cannula. b. Perform a thorough respiratory assessment and attach pulse oximetry. c. Call the laboratory to obtain arterial blood gases as soon as possible. d. Obtain a stat chest x-ray, then slowly wean the client's oxygen down.

B Oxygen-induced hypoventilation can occur in clients with chronically elevated PCO2 levels, such as those seen in COPD. Giving oxygen can eliminate their hypoxic drive to breathe and can cause respiratory arrest. However, hypoxemia is a greater threat to an acutely ill client than is the potential for oxygen-induced hypoventilation, and clients should be given the amount of oxygen they require. The nurse should perform a thorough respiratory assessment and should monitor the client for signs of this problem, rather than automatically reducing oxygen delivery. Blood gases and a chest x-ray will also be obtained, but they do not take priority over assessing and monitoring the client.

A patient who uses injectable illegal drugs asks the nurse about preventing acquired immunodeficiency syndrome (AIDS). Which response by the nurse is best? a. "Avoid sexual intercourse when using injectable drugs." b. "It is important to participate in a needle-exchange program." c. "You should ask those who share equipment to be tested for HIV." d. "I recommend cleaning drug injection equipment before each use."

B Participation in needle-exchange programs has been shown to decrease and control the rate of HIV infection. Cleaning drug equipment before use also reduces risk, but it might not be consistently practiced. HIV antibodies do not appear for several weeks to months after exposure, so testing drug users would not be very effective in reducing risk for HIV exposure. It is difficult to make appropriate decisions about sexual activity when under the influence of drugs

The patient with right upper quadrant abdominal pain has an abdominal ultrasound that reveals cholelithiasis. What should the nurse expect to do for this patient? A. Prevent all oral intake. B. Control abdominal pain. C. Provide enteral feedings. D. Avoid dietary cholesterol.

B Patients with cholelithiasis can have severe pain, so controlling pain is important until the problem can be treated. NPO status may be needed if the patient will have surgery but will not be used for all patients with cholelithiasis. Enteral feedings should not be needed, and avoiding dietary cholesterol is not used to treat cholelithiasis

When a patient with acute kidney injury (AKI) has an arterial blood pH of 7.30, the nurse will expect an assessment finding of a. persistent skin tenting b. rapid, deep respirations. c. bounding peripheral pulses. d. hot, flushed face and neck.

B Patients with metabolic acidosis caused by AKI may have Kussmaul respirations as the lungs try to regulate carbon dioxide. Bounding pulses and vasodilation are not associated with metabolic acidosis. Because the patient is likely to have fluid retention, poor skin turgor would not be a finding in AKI

To prevent the most common serious complication of PD, what is important for the nurse to do? a. Infuse the dialysate slowly. b. Use strict aseptic technique in the dialysis procedures. c. Have the patient empty the bowel before the inflow phase. d. Reposition the patient frequently and promote deep breathing.

B Peritonitis is a common complication of peritoneal dialysis (PD) and may require catheter removal and termination of dialysis. Infection occurs from contamination of the dialysate or tubing or from progression of exit-site or tunnel infections and strict sterile technique must be used by health professionals as well as the patient to prevent contamination. Too-rapid infusion may cause shoulder pain and pain may be caused if the catheter tip touches the bowel. Difficulty breathing, atelectasis, and pneumonia may occur from pressure of the fluid on the diaphragm, which may be prevented by elevating the head of the bed and promoting repositioning and deep breathing.

The nurse was accidently stuck with a needle used on an HIV-positive patient. After reporting this, what care should this nurse first receive? A. Personal protective equipment B. Combination antiretroviral therapy C. Counseling to report blood exposures D. A negative evaluation by the manage

B Postexposure prophylaxis with combination antiretroviral therapy can significantly decrease the risk of infection. Personal protective equipment should be available although it may not have stopped this needle stick. The needle stick has been reported. The negative evaluation may or may not be needed but would not occur first.

A client presents to the emergency department with acute adrenal insufficiency and the following vital signs: P 118 beats/min, R 18 breaths/min, BP 84/44 mm Hg, pulse oximetry 98%, and T 98.8° F oral. Which nursing intervention is the highest priority for this client? a) Administering furosemide (Lasix) b) Providing isotonic fluids c) Replacing potassium losses d) Restricting sodium

B Providing isotonic fluid is the priority intervention because this client's vital signs indicate volume loss that may be caused by nausea and vomiting and may accompany acute adrenal insufficiency. Isotonic fluids will be needed to administer IV medications such as hydrocortisone. Furosemide is a loop diuretic, which this client does not need. Potassium is normally increased in acute adrenal insufficiency, but potassium may have been lost if the client has had diarrhea; laboratory work will have to be obtained. Any restrictions, including sodium, should not be started without obtaining laboratory values to establish the client's baseline.

The nurse knows that which of the following conditions would most likely contribute to the development of ARDS? A) Simple Pneumothorax B) Right Lobular Pulmonary Contusion C) Cardiac Tamponade D) Subcutaneous Emphysema

B Pulmonary contusion causes fluid build-up to occur in the lungs which can in-turn impair gas exchange and and prevent oxygen and CO2 exchange. This fluid build-up can contribute to the development of ARDS (Acute Respiratory Distress Syndrome). This is the MOST likely to contribute this disorder.

The client is admitted to the hospital for COPD, and the physician requests a nasal cannula at 2 L/min. Within 30 minutes, the client's color improves. What does the nurse continue to monitor that may require immediate attention? A. Increasing carbon dioxide levels B. Decreasing respiratory rate C. Increasing adventitious breath sounds D. Increased coughing

B Respiratory rate and depth should be monitored closely while the client receives oxygen, because hypoventilation is seen during the first 30 minutes of oxygen therapy in clients with hypoxic drive for respiration. The client's color will improve (from ashen or gray to pink) because of an increase in PaO2 level before apnea or respiratory arrest occurs from loss of the hypoxic drive.

Nurse Vince sustained a dirty needle stick injury. Which diagnostic test would be ordered on a client? A. Enzyme-linked immunosorbent assay (ELISA) B. SUDS screening test C. Antibody titers D. Skin biopsy for Kaposi's sarcoma

B SUDS screening test results are available in 30 to 60 minutes. The test is performed on a client to determine if the health care worker with a dirty needle stick injury should begin antiretroviral treatment. ELISA test results indicate exposure to or infection with human immunodeficiency virus (HIV), but the test does not diagnose acquired immunodeficiency syndrome (AIDS). Antibody titers would not be appropriate to determine whether the health care worker has been exposed to HIV or hepatitis. Kaposi's sarcoma is usually associated with AIDS but not immediately after a needle stick.

The nurse on a burn unit has just received change-of-shift report about these clients. Which client does the nurse assess first? A. Adult client admitted a week ago with deep partial-thickness burns over 35% of the body who is reporting pain B. Firefighter with smoke inhalation and facial burns who has just arrived on the unit and whispers, "I can't catch my breath!" C. An electrician who suffered external burn injuries a month ago and is asking the nurse to contact the health care provider immediately about discharge plans D. Older adult client admitted yesterday with partial- and full-thickness burns over 40% of the body who is receiving IV fluids at 250 mL/hr

B Smoke inhalation and facial burns are associated with airway inflammation and obstruction; the client with difficulty breathing needs immediate assessment and intervention. Although the client admitted a week ago with deep partial-thickness burns is reporting pain, this client does not require immediate assessment. The electrician who suffered burn injuries a month ago is stable and has been in the burn unit for a month, so the client's condition does not warrant that the nurse should assess this client first. The older adult client admitted yesterday with burns over 40% of the body is stable; he is receiving IV fluids and does not need to be assessed first.

In replying to a patient's questions about the seriousness of her chronic kidney disease (CKD), the nurse knows that the stage of CKD is based on what? a. Total daily urine output b. Glomerular filtration rate c. Degree of altered mental status d. Serum creatinine and urea levels

B Stages of chronic kidney disease are based on theGFR. No specific markers of urinary output, mental status, or azotemia classify the degree of chronic kidney disease (CKD).

A nurse teaches a client with a cortisol deficiency who is prescribed prednisone (Deltasone). Which statement should the nurse include in this client's instructions? a. You will need to learn how to rotate the injection sites. b. If you work outside in the heat, you may need another drug. c. You need to follow a diet with strict sodium restrictions. d. Take one tablet in the morning and two tablets at night.

B Steroid dosage adjustment may be needed if the client works outdoors and might be difficult, especially in hot weather, when the client is sweating a great deal more than normal. Clients take prednisone orally, have no need for a salt restriction, and usually start the regimen with two tablets in the morning and one at night

A client has undergone a transsphenoidal hypophysectomy. Which intervention does the nurse implement to avoid increasing intracranial pressure (ICP) in the client? a) Encourages the client to cough and deep-breathe b) Instructs the client not to strain during a bowel movement c) Instructs the client to blow the nose for postnasal drip d) Places the client in the Trendelenburg position

B Straining during a bowel movement increases ICP and must be avoided. Laxatives may be given and fluid intake encouraged to help with this. Although deep breathing is encouraged, the client must avoid coughing early after surgery because this increases pressure in the incision area and may lead to a cerebrospinal fluid (CSF) leak. If the client has postnasal drip, he or she must inform the nurse and not blow the nose; postnasal drip may indicate leakage of CSF. The head of the bed must be elevated after surgery.

The nurse reviews the chart of the client admitted with a diagnosis of glioblastoma with a T1NXM0 classification. Which explanation does the nurse offer when the client asks what the terminology means? A. "Two lymph nodes are involved in this tumor of the glial cells, and another tumor is present." B. "The brain tumor measures about 1 to 2 cm and shows no regional lymph nodes and no distant metastasis." C. "This type of tumor in the brain is small with some lymph node involvement; another tumor is present somewhere else in your body." D. "Glioma means this tumor is benign, so I will have to ask your health care provider the reason for the chemotherapy and radiation."

B T1 means that the tumor is increasing in size to about 2 cm, and that no regional lymph nodes are present in the brain. M0 means that no distant metastasis has occurred.

Which assessment information will be most important for the nurse to report to the health care provider about a patient with acute cholecystitis? a. The patient's urine is bright yellow. b. The patient's stools are tan colored. c. The patient has increased pain after eating. d. The patient complains of chronic heartburn.

B Tan or grey stools indicate biliary obstruction, which requires rapid intervention to resolve. The other data are not unusual for a patient with this diagnosis, although the nurse would also report the other assessment information to the health care provider.

A nurse cares for a male client with hypopituitarism who is prescribed testosterone hormone replacement therapy. The client asks, "How long will I need to take this medication?" How should the nurse respond? a. When your blood levels of testosterone are normal, the therapy is no longer needed. b. When your beard thickens and your voice deepens, the dose is decreased, but treatment will continue forever. c. When your sperm count is high enough to demonstrate fertility, you will no longer need this therapy. d. With age, testosterone levels naturally decrease, so the medication can be stopped when you are 50 years old.

B Testosterone therapy is initiated with high-dose testosterone derivatives and is continued until virilization is achieved. The dose is then decreased, but therapy continues throughout life. Therapy will continue throughout life; therefore, it will not be discontinued when blood levels are normal, at the age of 50 years, or when sperm counts are high.

A nurse cares for a male client with hypopituitarism who is prescribed testosterone hormone replacement therapy. The client asks, How long will I need to take this medication? How should the nurse respond? a. When your blood levels of testosterone are normal, the therapy is no longer needed. b. When your beard thickens and your voice deepens, the dose is decreased, but treatment will continue forever. c. When your sperm count is high enough to demonstrate fertility, you will no longer need this therapy. d. With age, testosterone levels naturally decrease, so the medication can be stopped when you are 50 years old.

B Testosterone therapy is initiated with high-dose testosterone derivatives and is continued until virilization is achieved. The dose is then decreased, but therapy continues throughout life. Therapy will continue throughout life; therefore, it will not be discontinued when blood levels are normal, at the age of 50 years, or when sperm counts are high.

A nurse cares for a client who has an 80% blockage of the right coronary artery (RCA) and is scheduled for bypass surgery. Which intervention should the nurse be prepared to implement while this client waits for surgery? a. Administration of IV furosemide (Lasix) b. Initiation of an external pacemaker c. Assistance with endotracheal intubation d. Placement of central venous access

B The RCA supplies the right atrium, the right ventricle, the inferior portion of the left ventricle, and the atrioventricular (AV) node. It also supplies the sinoatrial node in 50% of people. If the client totally occludes the RCA, the AV node would not function and the client would go into heart block, so emergency pacing should be available for the client. Furosemide, intubation, and central venous access will not address the primary complication of RCA occlusion, which is AV node malfunction.

A nurse is caring for a patient with decreased triiodothyronine (T3) and thyroxine (T4) and elevated thyroid-stimulating hormone (TSH) levels. The nurse knows the patient is likely suffering from what? A) Thyrotoxicosis B) Hypothyroidism C) Hyperthyroidism D) Graves' disease

B The anterior pituitary increases production of TSH when thyroid hormone levels of T3 and T4, are reduced, reflecting primary hypothyroidism. Patients may experience fatigue caused by a lowered basal metabolic rate. Thyrotoxicosis, hyperthyroidism, and Graves' disease are medical conditions indicative of excessive thyroid activity.

A family member has been taught to provide oral care to a client with a tracheostomy. Which statement by the family member indicates an accurate understanding of the correct way to provide mouth care? a. "I can use glycerin swabs." b. "I'll use water and a toothette." c. "I can use hydrogen peroxide." d. "It is okay to use mouthwash."

B The best choice for mouth care is water and a toothette because these are the least irritating. Glycerin swabs, hydrogen peroxide, and mouthwash all are too irritating to the mucous membranes of the mouth.

A nurse is preparing to change the parenteral nutrition (PN) solution bag and tubing. The client's central venous line is located in the right subclavian vein. The nurse ask the client to take which essential action during the tube change? A. Turn the head to the right. B. Inhale deeply, hold it, and bear down. C. Breathe normally. D. Exhale slowly and evenly.

B The client should be asked to perform the Valsalva maneuver during tubing changes. This helps avoid air embolism during tube changes. The nurse asks the client to take a deep breath, hold it, and bear down. Option A is incorrect because if the intravenous line is on the right, the client turns his or head to the left. This position increases intrathoracic pressure. Options C and D can cause the potential for an air embolism during the tube change.

Several clients have been brought to the emergency department after an office building fire. Which client is at greatest risk for inhalation injury? A. Middle-aged adult who is frantically explaining to the nurse what happened B. Young adult who suffered burn injuries in a closed space C. Adult with burns to the extremities D. Older adult with thick, tan-colored sputum

B The client who suffered burn injuries in a closed space is at greatest risk for inhalation injury because the client breathed a greater concentration of confined smoke. Clients who experienced a fire typically have some type of respiratory distress. However, the client talking without difficulty demonstrates minimal respiratory distress. Extensive burns to the hands and face, not the extremities, would be a greater risk. Sputum would be carbonaceous, not tan, if the client had suffered inhalation injury.

The nurse is caring for a client with emphysema. A review of the client's chart reveals pH 7.36, paO2 73 mm Hg, PaCO2 64 mm Hg, and HCO3 35 mEq/L. The nurse would question which prescription, if prescribed by the health care practitioner? a) Pulse oximetry b) 4 L/minute O2 nasal cannula c) High-Fowler's position d) Increase fluid intake to 3 L/day

B The client with chronic lung disease, such as emphysema, becomes insensitive to carbon dioxide and responds to hypoxia to stimulate breathing. If given excessive oxygen (4 L/minute), the stimulus to breathe is removed. Clients with emphysema are most comfortable in high-Fowler's position because it aids in the use of the accessory muscles to promote respirations. Increasing fluid intake helps keep the client's secretions thin. Pulse oximetry monitors the client's arterial oxyhemoglobin saturation while receiving oxygen therapy.

Which of the following should the nurse include in the plan of care for a client following a coronary angiogram? a. Immediate cardiac stress test b. Encourage fluids c. Activity restriction for 4 to 6 weeks d. Vigorous leg exercises

B The client's leg on the side where the cardiologist entered the femoral artery needs to remain still for a period of time after the procedure (usually 2 to 4 hours) in order to allow the arterial site to seal. A cardiac stress test would not be indicated because the angiogram provides a more definitive diagnostic work-up. A 4- to 6-week activity restriction may be indicated after a large myocardial infarction, but not for a simple coronary angiogram, after which the client can begin walking hours later. The client should be encouraged to drink fluids to protect the kidneys from the contrast dye.

A 48-year-old patient with stage 2 chronic kidney disease (CKD) is scheduled for an intravenous pyelogram (IVP). Which order for the patient will the nurse question? a. NPO for 6 hours before procedure b. Ibuprofen (Advil) 400 mg PO PRN for pain c. Dulcolax suppository 4 hours before procedure d. Normal saline 500 mL IV infused before procedure

B The contrast dye used in IVPs is potentially nephrotoxic, and concurrent use of other nephrotoxic medications such as the nonsteroidal anti-inflammatory drugs (NSAIDs) should be avoided. The suppository and NPO status are necessary to ensure adequate visualization during the IVP. IV fluids are used to ensure adequate hydration, which helps reduce the risk for contrast-induced renal failure

The nurse is caring for a client with acute coronary syndrome (ACS) and atrial fibrillation who has a new prescription for metoprolol (Lopressor). Which monitoring is essential when administering the medication? a. ST segment b. Heart rate c. Troponin d. Myoglobin

B The effects of metoprolol are to decrease heart rate, blood pressure, and myocardial oxygen demand. ST segment elevation is consistent with MI; it does not address monitoring of metoprolol. Elevation in troponin is consistent with a diagnosis of MI but does not address needed monitoring for metoprolol. Elevation in myoglobin is consistent with myocardial injury in ACS but does not address needed monitoring related to metoprolol.

Following a laryngectomy a patient coughs violently during suctioning and dislodges the tracheostomy tube. Which action should the nurse take first? a. Cover stoma with sterile gauze and ventilate through stoma. b. Attempt to reinsert the tracheostomy tube with the obturator in place. c. Assess the patient's oxygen saturation and notify the health care provider. d. Ventilate the patient with a manual bag and face mask until the health care provider arrives.

B The first action should be to attempt to reinsert the tracheostomy tube to maintain the patient's airway. Assessing the patient's oxygenation is an important action, but it is not the most appropriate first action in this situation. Covering the stoma with a dressing and manually ventilating the patient may be an appropriate action if the nurse is unable to reinsert the tracheostomy tube. Ventilating with a facemask is not appropriate for a patient with a total laryngectomy because there is a complete separation between the upper airway and the trachea.

A 20-year-old man with extensive facial injuries from a motor vehicle crash is receiving tube feedings through a percutaneous endoscopic gastrostomy (PEG). Which action will the nurse include in the plan of care? a. Keep the patient positioned on the left side. b. Check the gastric residual volume every 4 to 6 hours. c. Avoid giving bolus tube feedings through the PEG tube. d. Obtain a daily abdominal x-ray to verify tube placement.

B The gastric residual volume is assessed every 4 to 6 hours to decrease the risk for aspiration. The patient does not need to be positioned on the left side. Bolus feedings can be administered through a PEG tube. An x-ray is obtained immediately after placement of the PEG tube to check position, but daily x-rays are not needed.

Which patient at the cardiovascular clinic requires the most immediate action by the nurse? a.Patient with type 2 diabetes whose current blood glucose level is 145 mg/dL b.Patient with stable angina whose chest pain has recently increased in frequency c.Patient with familial hypercholesterolemia and a total cholesterol of 465 mg/dL d.Patient with chronic hypertension whose blood pressure today is 172/98 mm Hg

B The history of more frequent chest pain suggests that the patient may have unstable angina, which is part of the acute coronary syndrome spectrum. This will require rapid implementation of actions such as cardiac catheterization and possible percutaneous coronary intervention. The data about the other patients suggest that their conditions are stable.

The nurse is preparing to receive a postoperative client who just had a tracheostomy. Which action by the nurse takes priority? a. Obtain report from the postanesthesia care unit. b. Place a second tracheostomy tube and obturator at the bedside. c. Review orders for postoperative pain medications. d. Order supplies for tracheostomy care for 24 hours.

B The nurse must ensure that a second tracheostomy tube with obturator is available at the bedside in case of accidental decannulation, because tube dislodgment in the first 72 hours is an emergency. Obtaining report and understanding pain medication orders are important for any postoperative client, but for the tracheostomy client, having the extra material on hand is critical. Obtaining supplies for tracheostomy care is not as high a priority as the other three.

The nurse is caring for a client with unstable angina whose cardiac monitor shows ventricular tachycardia. Which action is appropriate to implement first? a. Defibrillate the client at 200 J. b. Check the client for a pulse. c. Cardiovert the client at 50 J. d. Give the client IV lidocaine.

B The nurse needs to assess the client to determine stability before proceeding with further interventions. If the client has a pulse and is relatively stable, elective cardioversion or antidysrhythmic medications may be prescribed.

A severely malnourished patient reports that he is Jewish. The nurse's initial action to meet his nutritional needs will be to a. have family members bring in food. b. ask the patient about food preferences. c. teach the patient about nutritious Kosher foods. d. order nutrition supplements that are manufactured Kosher.

B The nurse's first action should be further assessment whether or not the patient follows any specific religious guidelines that impact nutrition. The other actions may also be appropriate, based on the information obtained during the assessment.

Which statement is CORRECT about a tension pneumothorax? A. This condition happens when an opening to the intrapleural space creates a two-way valve which causes pressure to build up in the space leading to shifting of the mediastinum. B. A tension pneumothorax is a medical emergency and is treated with needle decompression. C. Tracheal deviation is an early sign of a tension pneumothorax D. An open pneumothorax is the only cause of a tension pneumothorax.

B The only correct statement about a tension pneumothorax is option B. Option A is wrong because this condition happens when an opening to the intrapleural space creates a ONE-way (not two-way) valve which causes pressure to build up in the space leading to shifting of the mediastinum. Option C is wrong because tracheal deviation is a LATE (not early) sign of a tension pneumothorax. Option D is wrong because not only can an open pneumothorax cause a tension pneumothorax but a closed or spontaneous pneumothroax can as well.

After change-of-shift report, which patient will the nurse assess first? a. A 40-year-old woman whose parenteral nutrition infusion bag has 30 minutes of solution left b. A 40-year-old man with continuous enteral feedings who has developed pulmonary crackles c. A 30-year-old man with 4+ generalized pitting edema and severe protein-calorie malnutrition d. A 30-year-old woman whose gastrostomy tube is plugged after crushed medications were administered.

B The patient data suggest aspiration has occurred and rapid assessment and intervention are needed. The other patients should also be assessed as quickly as possible, but the data about them do not suggest any immediately life-threatening complications

When the nurse is caring for an obese patient with left lower lobe pneumonia, gas exchange will be best when the patient is positioned a. on the left side. b. on the right side. c. in the tripod position. d. in the high-Fowler's position.

B The patient should be positioned with the "good" lung in the dependent position to improve the match between ventilation and perfusion. The obese patient's abdomen will limit respiratory excursion when sitting in the high-Fowler's or tripod positions.

A transesophageal echocardiogram (TEE) is ordered for a patient with possible endocarditis. Which action included in the standard TEE orders will the nurse need to accomplish first? a.Start an IV line. b.Place the patient on NPO status. c.Administer O2 per nasal cannula. d.Give lorazepam (Ativan) 1 mg IV.

B The patient will need to be NPO for 6 hours preceding the TEE, so the nurse should place the patient on NPO status as soon as the order is received. The other actions also will need to be accomplished but not until just before or during the procedure.

The patient had a history of rheumatic fever and has been diagnosed with mitral valve stenosis. The patient is planning to have a biologic valve replacement. What protective mechanisms should the nurse teach the patient about using after the valve replacement? a. Long-term anticoagulation therapy b. Antibiotic prophylaxis for dental care c. Exercise plan to increase cardiac tolerance d. Take β-adrenergic blockers to control palpitations.

B The patient will need to use antibiotic prophylaxis for dental care to prevent endocarditis. Long-term anticoagulation therapy is not used with biologic valve replacement unless the patient has atrial fibrillation. An exercise plan to increase cardiac tolerance is needed for a patient with heart failure. Taking β-adrenergic blockers to control palpitations is prescribed for mitral valve prolapse, not valve replacement.

A patient whose heart monitor shows sinus tachycardia, rate 132, is apneic and has no palpable pulses. What is the first action that the nurse should take? a.Perform synchronized cardioversion. b.Start cardiopulmonary resuscitation (CPR). c.Administer atropine per agency dysrhythmia protocol. d.Provide supplemental oxygen via non-rebreather mask.

B The patient's clinical manifestations indicate pulseless electrical activity and the nurse should immediately start CPR. The other actions would not be of benefit to this patient.

A patient with respiratory failure has a respiratory rate of 8 and an SpO2 of 89%. The patient is increasingly lethargic. The nurse will anticipate assisting with a. administration of 100% oxygen by non-rebreather mask. b. endotracheal intubation and positive pressure ventilation. c. insertion of a mini-tracheostomy with frequent suctioning. d. initiation of bilevel positive pressure ventilation (BiPAP).

B The patient's lethargy, low respiratory rate, and SpO2 indicate the need for mechanical ventilation with ventilator-controlled respiratory rate. Administration of high flow oxygen will not be helpful because the patient's respiratory rate is so low. Insertion of a mini-tracheostomy will facilitate removal of secretions, but it will not improve the patient's respiratory rate or oxygenation. BiPAP requires that the patient initiate an adequate respiratory rate to allow adequate gas exchange.

After receiving change-of-shift report on a heart failure unit, which patient should the nurse assess first? a.Patient who is taking carvedilol (Coreg) and has a heart rate of 58 b.Patient who is taking digoxin and has a potassium level of 3.1 mEq/L c.Patient who is taking isosorbide dinitrate/hydralazine (BiDil) and has a headache d.Patient who is taking captopril (Capoten) and has a frequent nonproductive cough

B The patient's low potassium level increases the risk for digoxin toxicity and potentially fatal dysrhythmias. The nurse should assess the patient for other signs of digoxin toxicity and then notify the health care provider about the potassium level. The other patients also have side effects of their medications, but their symptoms do not indicate potentially life-threatening complications.

The nurse is administering IV fluid boluses and nasogastric irrigation to a patient with acute gastrointestinal (GI) bleeding. Which assessment finding is most important for the nurse to communicate to the health care provider? a. The bowel sounds are hyperactive in all four quadrants. b. The patient's lungs have crackles audible to the midchest. c. The nasogastric (NG) suction is returning coffee-ground material. d. The patient's blood pressure (BP) has increased to 142/84 mm Hg.

B The patient's lung sounds indicate that pulmonary edema may be developing as a result of the rapid infusion of IV fluid and that the fluid infusion rate should be slowed. The return of coffee-ground material in an NG tube is expected for a patient with upper GI bleeding. The BP is slightly elevated but would not be an indication to contact the health care provider immediately. Hyperactive bowel sounds are common when a patient has GI bleeding.

A 44-year-old man admitted with a peptic ulcer has a nasogastric (NG) tube in place. When the patient develops sudden, severe upper abdominal pain, diaphoresis, and a firm abdomen, which action should the nurse take? a. Irrigate the NG tube. b. Check the vital signs. c. Give the ordered antacid. d. Elevate the foot of the bed.

B The patient's symptoms suggest acute perforation, and the nurse should assess for signs of hypovolemic shock. Irrigation of the NG tube, administration of antacids, or both would be contraindicated because any material in the stomach will increase the spillage into the peritoneal cavity. Elevating the foot of the bed may increase abdominal pressure and discomfort, as well as making it more difficult for the patient to breathe.

The nurse is planning care for a patient with severe heart failure who has developed elevated blood urea nitrogen (BUN) and creatinine levels. The primary collaborative treatment goal in the plan will be a. augmenting fluid volume. b. maintaining cardiac output. c. diluting nephrotoxic substances. d. preventing systemic hypertension.

B The primary goal of treatment for acute kidney injury (AKI) is to eliminate the cause and provide supportive care while the kidneys recover. Because this patient's heart failure is causing AKI, the care will be directed toward treatment of the heart failure. For renal failure caused by hypertension, hypovolemia, or nephrotoxins, the other responses would be correct

A 68-year-old patient with a bleeding duodenal ulcer has a nasogastric (NG) tube in place, and the health care provider orders 30 mL of aluminum hydroxide/magnesium hydroxide (Maalox) to be instilled through the tube every hour. To evaluate the effectiveness of this treatment, the nurse a. monitors arterial blood gas values daily. b. periodically aspirates and tests gastric pH. c. checks each stool for the presence of occult blood. d. measures the volume of residual stomach contents.

B The purpose for antacids is to increase gastric pH. Checking gastric pH is the most direct way of evaluating the effectiveness of the medication. Arterial blood gases may change slightly, but this does not directly reflect the effect of antacids on gastric pH. Because the patient has upper gastrointestinal (GI) bleeding, occult blood in the stools will appear even after the acute bleeding has stopped. The amount of residual stomach contents is not a reflection of resolution of bleeding or of gastric pH.

When caring for a 63-year-old woman with a soft, silicone nasogastric tube in place for enteral feedings, the nurse will a. avoid giving medications through the feeding tube. b. flush the tubing after checking for residual volumes. c. administer continuous feedings using an infusion pump. d. replace the tube every 3 days to avoid mucosal damage.

B The soft silicone feeding tubes are small in diameter and can easily become clogged unless they are flushed after the nurse checks the residual volume. Either intermittent or continuous feedings can be given. The tubes are less likely to cause mucosal damage than the stiffer polyvinyl chloride tubes used for nasogastric suction and do not need to be replaced at certain intervals. Medications can be given through these tubes, but flushing after medication administration is important to avoid clogging.

The nurse is caring for a client with burns to the face. Which statement by the client requires further evaluation by the nurse? A. "I am getting used to looking at myself." B. "I don't know what I will do when people stare at me." C. "I know that I will never look the way I used to, even after the scars heal." D. "My spouse does not stare at the scars as much now as in the beginning."

B The statement about not knowing what to do when people stare indicates that the client is not coping effectively; the nurse should assist the client in exploring coping techniques. Visits from friends and short public appearances before discharge may help the client begin adjusting to this problem. The statement that the client is getting used to looking at himself or herself, the realization that he or she will always look different than before, and stating that the client's spouse doesn't stare at the scars as much all indicate that the client is coping effectively. Community reintegration programs can assist the psychosocial and physical recovery of the client with serious burns.

A client is being weaned from a tracheostomy tube and has tolerated capping of the tube for 24 hours. Which action by the nurse is most appropriate? a. Collect all materials needed for suturing the stoma shut. b. Place a dry dressing over the stoma and tape it securely. c. Assess the client for air leaking around the tube. d. Select a smaller tracheostomy tube to be inserted.

B The tube will be able to be removed after the client has tolerated capping of it for 24 hours. Therefore, a dry dressing will be able to be placed over the stoma. The stoma will not be sutured. It will heal on its own with a small scar. Airflow should be adequate around the capped tube. The physician will not likely insert the next smallest size tube but instead will remove the existing tube.

A client with chronic renal failure is about to begin hemodialysis therapy. The client asks the nurse about the frequency and scheduling of hemodialysis treatments. The nurse's response is based on an understanding that the typical schedule is: a. 5 hours of treatment 2 days per week. b. 3 to 4 hours of treatment 3 days per week c. 2 to 3 hours of treatment 5 days per week d. 2 hours of treatment 6 days per week

B The typical schedule for hemodialysis is 3 to 4 hours of treatment three days per week. Individual adjustments may be made according to variables such as the size of the client, type of dialyzer, the rate of blood flow, personal client preferences, and others.

A client is being discharged home with a tracheostomy. Which action does the nurse teach the client to decrease the risk for aspiration while eating? a. Swallow quickly. b. Thicken all liquids. c. Rinse all food with water. d. Chew food completely.

B Thickening liquids may assist the client in swallowing and may help prevent aspiration. Swallowing quickly will not decrease the risk of aspiration and may actually put the client at greater risk. It is not recommended that the client drink water to wash down food. Chewing food completely will help prevent choking but will not decrease aspiration risk.

When preparing the newly diagnosed client with HIV and significant other for discharge, which explanation by the nurse accurately describes proper condom use? A) ''Condoms should be used when lesions on the penis are present.'' B) ''Always position the condom with a space at the tip of an erect penis.'' C) ''Make sure it fits loosely to allow for penile erection.'' D) ''Use adequate lubrication such as petroleum jelly.''

B This allows for the collection of semen at the tip of the condom.

A patient is admitted with a chest wound and experiencing extreme dyspnea, tachycardia, and hypoxia. The chest wound is located on the left mid-axillary area of the chest. On assessment, you note there is unequal rise and fall of the chest with absent breath sounds on the left side. You also note a "sucking" sound when the patient inhales and exhales. The patient's chest x-ray shows a pneumothorax. What type of pneumothorax is this known as? A. Closed pneumothorax B. Open pneumothorax C. Tension pneumothorax D. Spontaneous pneumothorax

B This description is of an open pneumothorax. An open pneumothorax happens when there is an opening in the chest wall ( from a gun shot, stabbing etc.) that creates a passage between the outside air and intrapleural space. This allow air to pass back and forth during inspiration and expiration. The body will shunt air through the opening in the chest well instead of the trachea (if the opening on the chest is large enough) which will create a "sucking" sound.

A patient with chronic kidney disease (CKD) brings all home medications to the clinic to be reviewed by the nurse. Which medication being used by the patient indicates that patient teaching is required? a. Multivitamin with iron b. Milk of magnesia 30 mL c. Calcium phosphate (PhosLo) d. Acetaminophen (Tylenol) 650 mg

B This increases the magnesium level in the patient whom already has problems with hypermagnesemia

Spironolactone (Aldactone) is prescribed for a client with chronic cirrhosis and ascites. The nurse should monitor the client for which of the following medication-related side effects? A. Jaundice B. Hyperkalemia C. Tachycardia D. Constipation

B This is a potassium-sparing diuretic so clients should be monitored closely for hyperkalemia. Diarrhea, dizziness, and headaches are other more common side effects. Tachycardia, jaundice, and constipation are not expected side effects of spironolactone (Aldactone).

After insertion of a chest tube, fluctuations in the water-seal chamber that correspond with inspiration and expiration are an expected and normal finding. a) False b) True

B This much negative pressure is excessive and may cause excessive trauma, hypoxemia, and atelectasis.

The nursing student is performing tracheostomy care on a client. Which action by the student leads the supervising nurse to intervene? a. Using folded gauze dressings on both sides of the stoma b. Cutting a slit in a gauze 4 4 pad to fit around the stoma c. Applying new tracheostomy ties before removing old ones d. Tying the twill tape in a square knot on the side of the neck

B Tracheostomy dressings should be made from gauze pads with a manufactured slit in them that fits around the tube. If none are available, use two gauze pads folded in half placed on either side of the tube. Cutting a piece of gauze could result in entry of tiny shreds of the gauze the tracheostomy. The other interventions are appropriate.

A client is 24 hours postoperative after a tracheostomy has been performed. The nurse finds the client cyanotic, with the tracheostomy tube lying on his chest. Which action by the nurse takes priority? a. Auscultate breath sounds bilaterally. b. Ventilate with a resuscitation bag and mask. c. Call a code or the Rapid Response Team. d. Insert a new obturator into the neck.

B Tube dislodgment in the first 72 hours after surgery is an emergency because the tracheostomy tract has not matured and replacement is difficult. First, ventilate the client using a manual resuscitation bag and facemask while another nurse calls for help. Although auscultation of breath sounds is important, the client's airway must be opened and ventilation started. Ventilation should begin while another nurse calls the code. Reinsertion of a fresh tracheostomy tube will require the physician's intervention.

The nurse and unlicensed assistive personnel (UAP) on the telemetry unit are caring for four patients. Which nursing action can be delegated to the UAP? a.Teaching a patient scheduled for exercise electrocardiography about the procedure b.Placing electrodes in the correct position for a patient who is to receive ECG monitoring c.Checking the catheter insertion site for a patient who is recovering from a coronary angiogram d.Monitoring a patient who has just returned to the unit after a transesophageal echocardiogram

B UAP can be educated in standardized lead placement for ECG monitoring. Assessment of patients who have had procedures where airway maintenance (transesophageal echocardiography) or bleeding (coronary angiogram) is a concern must be done by the registered nurse (RN). Patient teaching requires RN level education and scope of practice.

Which information about an 80-year-old man at the senior center is of most concern to the nurse? a. Decreased appetite b. Unintended weight loss c. Difficulty chewing food d. Complaints of indigestion

B Unintentional weight loss is not a normal finding and may indicate a problem such as cancer or depression. Poor appetite, difficulty in chewing, and complaints of indigestion are common in older patients. These will need to be addressed but are not of as much concern as the weight loss.

A public health nurse is preparing an educational campaign to address a recent local increase in the incidence of HIV infection. The nurse should prioritize which of the following interventions? A) Lifestyle actions that improve immune function B) Educational programs that focus on control and prevention C) Appropriate use of standard precautions D) Screening programs for youth and young adults

B Until an effective vaccine is developed, preventing HIV by eliminating and reducing risk behaviors is essential. Educational interventions are the primary means by which behaviors can be influenced. Screening is appropriate, but education is paramount. Enhancing immune function does not prevent HIV infection. Ineffective use of standard precautions apply to very few cases of HIV infection.

Which action is most important for the nurse to implement to prevent nausea and vomiting in a client who is prescribed to receive the first round of IV chemotherapy? A. Keep the client NPO during the time chemotherapy is infusing. B. Administer antiemetic drugs before administering chemotherapy. C. Ensure that the chemotherapy is infused over a 4- to 6-hour period. D. Assess the client for manifestations of dehydration hourly during the infusion period.

B When emetogenic chemotherapy drugs are prescribed, the client should receive antiemetic drugs before the chemotherapy drugs are administered. This allows time for prevention of chemotherapy-associated nausea and vomiting; however, the antiemetic therapy cannot stop until all risks for nausea and vomiting have passed. Clients become nauseated and vomit even if they are NPO.

When evaluating the effectiveness of preoperative teaching with a patient scheduled for coronary artery bypass graft (CABG) surgery using the internal mammary artery, the nurse determines that additional teaching is needed when the patient says which of the following? a."They will circulate my blood with a machine during the surgery." b."I will have small incisions in my leg where they will remove the vein." c."They will use an artery near my heart to go around the area that is blocked." d."I will need to take an aspirin every day after the surgery to keep the graft open."

B When the internal mammary artery is used there is no need to have a saphenous vein removed from the leg. The other statements by the patient are accurate and indicate that the teaching has been effective.

A nurse cares for a client who possibly has syndrome of inappropriate antidiuretic hormone (SIADH). The client's serum sodium level is 114 mEq/L. Which action should the nurse take first? a. Consult with the dietitian about increased dietary sodium. b. Restrict the client's fluid intake to 600 mL/day. c. Handle the client gently by using turn sheets for re-positioning. d. Instruct unlicensed assistive personnel to measure intake and output.

B With SIADH, clients often have dilutional hyponatremia. The client needs a fluid restriction, sometimes to as little as 500 to 600 mL/24 hr. Adding sodium to the client's diet will not help if she is retaining fluid and diluting the sodium. The client is not at increased risk for fracture, so gentle handling is not an issue. The client should be on intake and output; however, this will monitor only the client's intake, so it is not the best answer. Reducing intake will help increase the client's sodium.

A nurse cares for a client who possibly has syndrome of inappropriate antidiuretic hormone (SIADH). The clients serum sodium level is 114 mEq/L. Which action should the nurse take first? a. Consult with the dietitian about increased dietary sodium. b. Restrict the clients fluid intake to 600 mL/day. c. Handle the client gently by using turn sheets for re-positioning. d. Instruct unlicensed assistive personnel to measure intake and output.

B With SIADH, clients often have dilutional hyponatremia. The client needs a fluid restriction, sometimes to as little as 500 to 600 mL/24 hr. Adding sodium to the clients diet will not help if he or she is retaining fluid and diluting the sodium. The client is not at increased risk for fracture, so gentle handling is not an issue. The client should be on intake and output; however, this will monitor only the clients intake, so it is not the best answer. Reducing intake will help increase the clients sodium.

A nurse is providing discharge teaching to a client who is postoperative following open cholecystectomywith T-tube placement. Which of the following instructions should the nurse include in the teaching?(Select all that apply. A. Take baths rather than showers. B. Clamp T-tube for 1 to 2 hr before and after meals. C. Keep the drainage system above the level of the abdomen. D. Expect to have the T-tube removed 3 days postoperatively.

BC

A patient in the emergency room has signs and symptoms associated with cholecystitis. What testing do you anticipate the physician will order to help diagnose cholecystitis? Select all that apply: A. Lower GI series B. Abdominal ultrasound C. HIDA Scan (Hepatobiliary Iminodiacetic AciD scan) D. Colonoscopy

BC

The nurse is planning care for a client with acute respiratory distress syndrome​ (ARDS). Which independent nursing intervention should the nurse include in the care of this​ client?(Select all that​ apply.) A. Prescribe surfactant therapy. B. Suction the airway as needed. C. Obtain a sputum culture. D. Maintain the head of the bed at less than 30 degrees. E. Position the client in a prone position for 60 minutes five times a day.

BC

A client with end-stage renal failure has an internal arteriovenous fistula in the left arm for vascular access during hemodialysis. What should the nurse instruct the client do to? Select all that apply a. remind the HCPs to draw blood from veins on the left side b. avoid sleeping on the left arm c. wear wristwatch on the right arm d. assess fingers on the left arm for warmth e. obtain BP from the left arm

BCD

What is the advantage of using Volume Cycled Ventilation? (SATA) A. the ventilator pushes air into lungs until a preset airway pressure is reached B. the ventilator pushes air into the lungs until a preset volume is reached C. a constant tidal volume is delivered regardless of pressure needed to deliver the tidal volume D. limits excessive pressure being exerted on the lungs E. allows the health care team to determine if intubation is really neccessary

BCD

Which statements below are CORRECT regarding the role of bile? Select all that apply: A. Bile is created and stored in the gallbladder. B. Bile aids in digestion of fat soluble vitamins, such as A, D, E, and K. C. Bile is released from the gallbladder into the duodenum. D. Bile contains bilirubin.

BCD

a nurse is planning care for a client who has bacterial meningitis. which of the following actions should the nurse include in the pan of care? (select all that apply) A. monitor for bradycardia B. provide an emesis basin at the bedside C. administer antipyretic medication D. perform a skin assessment E. keep the head of the bed flat

BCD

The nurse administers amiodarone (Cordarone) to a client with ventricular tachycardia. Which monitoring by the nurse is necessary with this drug? Select all that apply. a. Respiratory rate b. QT interval c. Heart rate and rhythm d. Magnesium level e. Urine output

BCD Amiodarone causes prolongation of the QT interval, which can precipitate dysrhythmia. Antidysrhythmic medications cause changes in cardiac rhythm and rate; therefore monitoring of heart rate and rhythm is needed.Electrolyte depletion, specifically potassium and magnesium, may predispose to further dysrhythmia. Although it is always important to monitor vital signs and urine output, these assessments are not specific to amiodarone.

Your patient is diagnosed with acute cholecystitis. The patient is extremely nauseous. A nasogastric tube is inserted with GI decompression. The patient reports a pain rating of 9 on 1-10 scale and states the pain radiates to the shoulder blade. Select all the appropriate nursing interventions for the patient: A. Encourage the patient to consume clear liquids. B. Administered IV fluids per MD order. C. Provide mouth care routinely. D. Keep the patient NPO. E. Administer analgesic as ordered. F. Maintain low intermittent suction to NG tube.

BCDEF

A patient with an oral ET tube has a nursing diagnosis of risk for aspiration related to presence of artificial airway. What are appropriate nursing interventions for this patient? Select all that apply a. assess gag reflex b. suction the patient's mouth frequently c. ensure that the cuff is properly inflated d. keep the ventilator tubing cleared of condensed water e. raise the head of bed 30-45 degrees unless the patient is unstable

BCE

At a visit to the pediatric clinic, a mother is concerned by her 4 year old's symptoms over the last few weeks. Which of the following symptoms described by the mother would lead the nurse to be concerned about an oncologic disorder? Select all that apply a. bruising in various stages, mainly on the legs b. frequent complaints of respiratory infections, while siblings remain healthy c. enlarged, firm lymph nodes d. asthma symptoms with increase in wheezing e. fever for more than 1 week

BCE

The nurse is administering aminoglutethimide to a patient and will monitor for which adverse effects? (Select all that apply) A. Constipation B. Dizziness C. Anorexia D. Hypotension E. Lethargy

BCE

Which conditions is aminoglutethimide (Cytadren) used to treat? (Select all that apply.) A. Thyroid cancer B. Adrenal cancer C. Testicular cancer D. Cushing's syndrome E. Metastatic breast cancer

BCE

A patient is diagnosed with adrenocortical insufficiency. Which laboratory findings would be consistent with this diagnosis? Select all that apply. (SATA) a. Serum sodium: 140 mEq/L b. Serum potassium: 6.5 mEq/L c. Blood glucose levels: 80 mg/dL d. Blood urea nitrogen (BUN): 30 mg/dL e. Electrocardiogram (ECG): Peaked T waves

BCE Adrenocortical insufficiency leads to hyperkalemia, hypoglycemia, peaked T waves in ECG, hyponatremia, and increased blood urea nitrogen levels. Normal serum electrolyte ranges include sodium from 135 to 145 mEq/L, potassium from 3.5 to 5 mEq/L, glucose from 70 to 99 mg/dL, and blood urea nitrogen from 6 to 20 mg/dL. A serum potassium level of 6.5 mEq/L shows increased serum potassium levels (hyperkalemia). A blood urea nitrogen level of 30 mg/dL shows increased levels. Peaked T waves are observed in electrocardiogram due to hyperkalemia.

A nurse cares for a client who is prescribed vasopressin (DDAVP) for diabetes insipidus. Which assessment findings indicate a therapeutic response to this therapy? (SATA) a. Urine output is increased. b. Urine output is decreased. c. Specific gravity is increased. d. Specific gravity is decreased. e. Urine osmolality is increased. f. Urine osmolality is decreased.

BCE Diabetes insipidus causes urine output to be greatly increased, with a low urine osmolality, as evidenced by a low specific gravity. Effective treatment results in decreased urine output that is more concentrated, as evidenced by an increased specific gravity.

the nurse is caring for a pt who had an open cholecystectomy 24 hrs ago. which of the follwing actions should the nurse take to assist the pt to maintain an effective breathing pattern? select all that apply a. place in a supine position b. provide analgesics for pain c. encourage coughing and deep breathing d. monitor bowel sounds e. assist with splinting during coughing f. maintain bedrest for 48 hrs after

BCE pain relief allows comfortable breathing, preventing shallow respiration's and guarding, splinting makes coughing more comfortable, encouraging coughing and deep breathing to keep lung clear

Select-all-that-apply: Which of the following are NOT typical signs and symptoms of pericarditis? A. Fever B. Increased pain when leaning forward C. ST segment depression D. Pericardial friction rub E. Radiating substernal pain felt in the left shoulder F. Breathing in relieves the pain

BCF

The diet order for a client receiving hemodialysis is written as 80-3-3. When the nurse explains the diet to the client, which of the following will be included in the teaching? Select all that apply. a. 80 grams of fat are allowed per day b. 80 grams of protein are allowed per day c. potassium is restricted to 3 grams a day d. phosphorus is restricted to 3 grams a day e. potassium is restricted to 80 mg per day f. sodium is restricted to 3 grams per day

BCF

A client receiving phenytoin (Dilantin) has been experiencing fluctuating serum blood levels of the medication. Development of which symptoms in the client should prompt the nurse to notify the primary health care provider immediately? (Select all that apply.) A. GI cramping and diarrhea B. Migraine headaches and nausea C. Dry skin and constipation D. Double vision and lethargy

BD

The nurse is conducting a community health education class on diet and cancer risk reduction. What should be included in the discussion? Select all the apply. A. Limit sodium intake. B. Avoid beef and processed meats. C. Increase consumption of whole grains. D. Eat "colorful fruits and vegetables," including greens. E. Avoid gas-producing vegetables such as cabbage.

BD Eating cruciferous vegetables such as broccoli, cauliflower, brussels sprouts, and cabbage may reduce cancer risk.

A nurse is caring for a patient with ARDS. Which of the following clinical indicators would signify that this client is in respiratory failure? Select all that apply. a. Pulse oximetry of 94% on room air b. A PaO2 level below 60 mmHg c. An ABG pH level of 7.35 d. A pCO2 level over 50 mmHg e. A respiratory rate of over 16/minute

BD Respiratory diseases can cause such compromise that the patient will suffer symptoms; however, there are certain clinical indicators that can clarify whether the patient is actually in respiratory failure. Clinical indicators of respiratory failure include pulse oximetry of less than 91% on room air, PaO2 level less than 60 mmHg, and a pCO2 level of over 50 mmHg.

The nurse is instructing a client on how to perform breast self-examination (BSE). Which techniques will the nurse include in teaching the client about BSE? Select all that apply. A. Instruct the client to keep her arm by her side while performing the examination. B. Ensure that the setting in which BSE is demonstrated is private and comfortable. C. Ask the client to remove her shirt. The bra may be left in place. D. Ask the client to demonstrate her own method of BSE. E. Use the fingertips, which are more sensitive than the finger pads, to palpate the breasts.

BD The setting should be private and comfortable to promote an environment conducive to learning and to prevent potential client embarrassment. Before teaching breast palpation, ask the client to demonstrate her own method, so that the nurse can assess the client's understanding of BSE. For better visualization, the arm should be placed over the head.The client should undress from the waist up. The finger pads, which are more sensitive than the fingertips, are used when palpating the breasts.

The nurse is caring for the patient with unstable angina. She is administering Cardizem as ordered to this client. The nurse knows that this medication is mainly used for the following reason. Select all that Apply: A) This medication will help increase myocardial contraction B) This medication will relax the heart and decrease the strength of the heart contractions C) This medication will increase the heart rate and therefore increase perfusion to the heart D) This medication will vasodilate blood vessels reducing preload and opening up coronary arteries E) This medication will protects the heart cells by preventing remodeling

BD This medication will "soften" the heart and the blood vessels. It will decrease strength of contractions, which will decrease the workload of the heart. This will also decrease the oxygen needs of the heart. It also has a vasodilating property which will allow more blood to flow through the coronary arteries and decrease blood pressure.

The patient is admitted to the ED with fever, swollen lymph glands, sore throat, headache, malaise, joint pain, and diarrhea. What nursing measures will help identify the need for further assessment of the cause of this patient's manifestations (select all that apply)? A. Assessment of lung sounds B. Assessment of sexual behavior C. Assessment of living conditions D. Assessment of drug and syringe use E. Assessment of exposure to an ill person

BD With these symptoms, assessing this patient's sexual behavior and possible exposure to shared drug equipment will identify if further assessment for the HIV virus should be made or the manifestations are from some other illness (e.g., lung sounds and living conditions may indicate further testing for TB).

The nurse taking care of a 5 year old cancer patient with ulcerative stomatitis is getting ready to perform mouth care. Which of the following principles should be followed? Select all that apply a. due to pain, viscous lidocaine should be used to swish the mouth 3 times per day b. a soft, bland diet, although not the favorite of the child, will help with the pain c. lemon glycerine swabs are helpful because they remind children of lemon drops d. using a soft sponge-type toothbrush will decrease the tendency for gums to bleed e. a solution of 1 tsp of baking soda and 1/2 tsp of table salt in 1 quart of water is helpful for mouth rinse

BDE

Which of the following skills can be delegated to the LPN? (Select all that apply.) A. Nasotracheal suctioning B. Oropharyngeal suctioning of a stable patient C. Suctioning a new artificial airway D. Permanent tracheostomy tube suctioning E. Care of an endotracheal tube (ETT)

BDE

A nurse prepares a client for a pharmacologic stress echocardiogram. Which actions should the nurse take when preparing this client for the procedure? (Select all that apply.) a. Assist the provider to place a central venous access device. b. Prepare for continuous blood pressure and pulse monitoring. c. Administer the client's prescribed beta blocker. d. Give the client nothing by mouth 3 to 6 hours before the procedure. e. Explain to the client that dobutamine will simulate exercise for this examination.

BDE Clients receiving a pharmacologic stress echocardiogram will need peripheral venous access and continuous blood pressure and pulse monitoring. The client must be NPO 3 to 6 hours prior to the procedure. Education about dobutamine, which will be administered during the procedure, should be performed. Beta blockers are often held prior to the procedure.

A nursing student is caring for a client with open-wound burns. Which nursing interventions does the nursing student provide for this client? (Select all that apply.) A. Provides cushions and rugs for comfort B. Performs frequent handwashing C. Places plants in the client's room D. Performs gloved dressing changes E. Uses disposable dishes

BDE Handwashing is the most effective technique for preventing infection. Gloves should be worn when changing dressings to reduce the risk for infection. Equipment is not shared with other clients to prevent the risk for infection. Disposable items (e.g., pillows, dishes) are used as much as possible. Cushions and rugs are difficult to clean and may harbor organisms, and so are not provided. To avoid exposure to Pseudomonas, having plants or flowers in the room is prohibited.

A nurse assesses a client who is recovering after a coronary catheterization. Which assessment findings in the first few hours after the procedure require immediate action by the nurse? (Select all that apply.) a. Blood pressure of 140/88 mm Hg b. Serum potassium of 2.9 mEq/L c. Warmth and redness at the site d. Expanding groin hematoma e. Rhythm changes on the cardiac monitor

BDE In the first few hours postprocedure, the nurse monitors for complications such as bleeding from the insertion site, hypotension, acute closure of the vessel, dye reaction, hypokalemia, and dysrhythmias. The client's blood pressure is slightly elevated but does not need immediate action. Warmth and redness at the site would indicate an infection, but this would not be present in the first few hours.

A nurse teaches a client with Cushing's disease. Which dietary requirements should the nurse include in this client's teaching? (SATA) a. Low calcium b. Low carbohydrate c. Low protein d. Low calories e. Low sodium

BDE The client with Cushing's disease has weight gain, muscle loss, hyperglycemia, and sodium retention. Dietary modifications need to include reduction of carbohydrates and total calories to prevent or reduce the degree of hyperglycemia. Sodium retention causes water retention and hypertension. Clients are encouraged to restrict their sodium intake moderately. Clients often have bone density loss and need more calcium. Increased protein intake will help decrease muscle loss.

A nurse teaches a client with Cushings disease. Which dietary requirements should the nurse include in this clients teaching? (Select all that apply.) a. Low calcium b. Low carbohydrate c. Low protein d. Low calories e. Low sodium

BDE The client with Cushings disease has weight gain, muscle loss, hyperglycemia, and sodium retention. Dietary modifications need to include reduction of carbohydrates and total calories to prevent or reduce the degree of hyperglycemia. Sodium retention causes water retention and hypertension. Clients are encouraged to restrict their sodium intake moderately. Clients often have bone density loss and need more calcium. Increased protein intake will help decrease muscle loss.

A nurse working in a long-term care facility is providing teaching to patients with altered oxygenation due to conditions such as asthma and COPD. Which measures would the nurse recommend? Select all that apply. a) Refrain from exercise. b) Reduce anxiety. c) Eat meals 1 to 2 hours prior to breathing treatments. d) Eat a high-protein/high-calorie diet. e) Maintain a high-Fowler's position when possible. f) Drink 2 to 3 pints of clear fluids daily.

BDE When caring for patients with COPD, it is important to create an environment that is likely to reduce anxiety and ensure that they eat a high-protein/high-calorie diet. People with dyspnea and orthopnea are most comfortable in a high Fowler's position because accessory muscles can easily be used to promote respiration. Patients with COPD should pace physical activities and schedule frequent rest periods to conserve energy. Meals should be eaten 1 to 2 hours after breathing treatments and exercises, and drinking 2 to 3 quarts (1.9-2.9 L) of clear fluids daily is recommended.

A nurse is caring for a client who is experiencing respiratory distress. Which of the following early manifestations of hypoxemia should the nurse recognize? Select all that apply a. confusion b. pale skin c. bradycardia d. hypotension e. elevated blood pressure

BE

Which patient's medical diagnoses should the nurse know are most likely to need mechanical ventilation? Select all that apply a. sleep apnea b. cystic fibrosis c. acute kidney injury d. type 2 DM e. acute respiratory distress syndrome (ARDS)

BE

A 55-year-old patient who is diagnosed with an evolving myocardial infarction (MI) insists on going home. The cardiac/vascular nurse encourages the patient to be admitted, because the greatest risk within the first 24 hours of sustaining an MI is: a.) Heart failure. b.) Pulmonary embolism. c.) Sudden death. d.) Ventricular aneurysm.

C

A client has a primary problem of inadequate nutrition caused by the effects of chemotherapy. The client is receiving continuous enteral feedings through a nasogastric tube (NG) tube. What does the RN ask the LPN/LVN to do for this client? A. Assess nutritional parameters on the client every 3 days. B. Check the residual volume of the NG tube every 4 hours. C. Monitor the client for signs and symptoms of pneumonia. D. Teach the client about the purpose of enteral feedings.

C

A client has been diagnosed to have chronic renal failure. Sodium polysterene sulfonate (exchange resin kayexalate) is prescribed. The action of the medication is that it releases a) bicarbonate in exchange for primarily sodium ions b) sodium ions in exchange for primarily bicarbonate ions c) sodium ions in exchange for primarily potassium ions d) potassium ions in exchange for primarily sodium ions

C

A client receiving total parenteral nutrition (TPN) exhibits symptoms of congestive heart failure (CHF) and pulmonary edema. Which complication of TPN is the client most likely experiencing? A. Calcium imbalance B. Fluid volume deficit C. Fluid volume overload D. Potassium imbalance

C

A client with a high genetic risk for breast cancer asks the nurse about options for prevention and early detection. Which option for prevention and early detection is the option of choice? A. Breast self-examination (BSE) beginning at 20 years of age B. Hormone replacement therapy combining estrogen and progesterone C. Magnetic resonance imaging (MRI) and mammography every year beginning at age 40 D. Prophylactic mastectomy

C

A comatose patient with a possible cervical spine injury is intubated with a nasal ET tube. The nurse recognizes that what is a disadvantage of a nasal ET tube in comparison with an oral ET tube? a. requires the placement of a bite block b. is more likely to cause laryngeal edema c. requires greater respiratory effort in breathing d. requires the placement of an additional airway to keep the trachea open

C

A male client with a cerebellar brain tumor is admitted to an acute care facility. The nurse formulates a nursing diagnosis of Risk for injury. Which "related-to" phrase should the nurse add to complete the nursing diagnosis statement? a. Related to visual field deficits b. Related to difficulty swallowing c. Related to impaired balance d. Related to psychomotor seizures

C

A male client with a nagging cough makes an appointment to see the physician after reading that this symptom is one of the seven warning signs of cancer. What is another warning sign of cancer? a. Persistent nausea b. Rash c. Indigestion d. Chronic ache or pain

C

A nurse is caring for a client who develops disequilibrium syndrome after receiving hemodialysis. Which of the following actions should the nurse take? a. administer an opioid medication b. monitor for hypertension c. assess level of consiousness d. increase the dialysis exchange rate

C

A nurse is caring for a group of clients on a medical-surgical nursing unit. The nurse recognizes that which of the following clients would be the least likely candidate for parenteral nutrition? A. A 55-year-old with persistent nausea and vomiting from chemotherapy. B. A 44-year old client with ulcerative colitis. C. A 59-year old client who had an appendectomy. D. A 25-year old client with a Hirschprung's Disease.

C

A nurse is changing the central line dressing of a client receiving parenteral nutrition (PN) and notes that the catheter insertion site appears reddened. The nurse next assesses which of the following items? A. Time of last dressing change. B. Tightness of the tuning connections. C. Client's temperature. D. Expiration date on the bag.

C

A nurse is providing discharge instructions to a female client who has a prescription for phenytoin. which of the following information should the nurse include? A.consider taking oral contraceptives when on this medication B.watch for receding gums when taking this medication C.take the medication at the same time everyday D.provide a urine sample to determine therapeutic levels f the medication

C

A nurse manager in a long-term care facility plans nutritional assessments of all residents. Which nutritional assessment activity does the nurse delegate to unlicensed assistive personnel (UAP) at the facility? A. Assessing residents' abilities to swallow B. Determining residents' functional status C. Measuring the daily food and fluid intake of residents D. Screening a portion of the residents with the Mini Nutritional Assessme

C

A particular area of concern for the adolescent receiving radiotherapy is a. frequent vomiting b. altered sexual function c. high risk for sterility d. precocious puberty

C

A patient admitted with a head injury has admission vital signs of temperature 98.6° F (37° C), blood pressure 128/68, pulse 110, and respirations 26. Which of these vital signs, if taken 1 hour after admission, will be of most concern to the nurse? a. Blood pressure 130/72, pulse 90, respirations 32 b. Blood pressure 148/78, pulse 112, respirations 28 c. Blood pressure 156/60, pulse 60, respirations 14 d. Blood pressure 110/70, pulse 120, respirations 30

C

A patient has a systemic blood pressure (BP) of 120/60 mm Hg and an intracranial pressure of 24 mm Hg. The nurse determines that the cerebral perfusion pressure (CPP) of this patient indicates a. high blood flow to the brain. b. normal intracranial pressure (ICP). c. impaired brain blood flow. d. adequate cerebral perfusion.

C

A patient has been hospitalized in the ICU for a near drowning event. The patient's respiratory function has been deteriorating over the last 24 hours. The physician suspects acute respiratory distress syndrome. A STAT chest x-ray is ordered. What finding on the chest x-ray is indicative of ARDS? A. infiltrates only on the upper lobes B. enlargement of the heart with bilateral lower lobe infiltrates C. white-out infiltrates bilaterally D. normal chest x-ray

C

A patient in the ICU has had an endotracheal tube in place for 3 weeks. The physician has ordered that a tracheostomy tube be placed. The patients family wants to know why the endotracheal tube cannot be left in place. What would be the nurses best response? A) The physician may feel that mechanical ventilation will have to be used long-term. B) Long-term use of an endotracheal tube diminishes the normal breathing reflex. C) When an endotracheal tube is left in too long it can damage the lining of the windpipe. D) It is much harder to breathe through an endotracheal tube than a tracheostomy.

C

A patient is admitted with sepsis. The patient has a temperature of 104.2 'F and is experiencing chills. On assessment, you note a mitral murmur which the patient states they've never had before, and dark, small lines on the patient's fingernails. The patient has a history of IV drug use in the past. However, the patient states they are no longer using drugs. The physician suspects possible infective endocarditis. What diagnostic test do you expect the physician to order in order to confirm the presence of infective endocarditis? A. Abdominal ultrasound B. Heart catheterization C. Transesophageal echocardiogram D. White blood cell count

C

A patient is admitted with severe lobar pneumonia. Which of the following assessment findings would indicate that the patient needs airway suctioning? A: Coughing up thick sputum only occasionally B: Coughing up thin, watery sputum easily after nebulization C: Decreased independent ability to cough D: Lung sounds clear only after coughing

C

A patient is hospitalized with chronic pericarditis. On assessment, you note the patient has pitting edema in lower extremities, crackles in lungs, and dyspnea on excretion. The patient's echocardiogram shows thickening of the pericardium. This is known as what type of pericarditis? A. Pericardial effusion B. Acute pericarditis C. Constrictive pericarditis D. Effusion-Constrictive pericarditis

C

A patient is receiving treatment for infective endocarditis. The patient has a history of intravenous drug use and underwent mitral valve replacement a year ago. The patient is scheduled for a transesophageal echocardiogram tomorrow. On assessment, you find tender, red lesions on the patient's hands and feet. You know that this is a common finding in patients with infective endocarditis and is known as? A. Janeway Lesions B. Roth Spots C. Osler's Nodes D. Trousseau's Sign

C

A patient is recovering from a myocardial infarction (MI) and develops chest pain on day 3 that increases when taking a deep breath and is relieved by leaning forward. Which action should the nurse take next? a. Assess the feet for pedal edema. b. Palpate the radial pulses bilaterally. c. Auscultate for a pericardial friction rub. d. Check the heart monitor for dysrhythmias.

C

A patient is recovering in the intensive care unit (ICU) 24 hours after receiving a kidney transplant. What is an expected assessment finding during the earliest stage of recovery? a. Hypokalemia b. Hyponatremia c. Large urine output d. Leukocytosis with cloudy urine output

C

A patient receiving mechanical ventilation is very anxious and agitated, and neuromuscular blocking agents are used to promote vasodilation. What should the nurse recognize about the care of this patient? a. the patient will be too sedated to be aware of the details of care b. caregivers should be encouraged to provide stimulation and diversion c. the patient should always be addressed and explanations of care given d. communication will not be possible with the use of neuromuscular blocking agents

C

A patient recently had a cardiac catheterization via right-radial approach. The patient has a compression device in place. The patient complains of numbness and pain in the right hand. The cardiac-vascular nurse notes a diminished pulse, with a cool and cyanotic hand. The nurse: a. calls the physician. b. performs an Allen's test. c. reduces the pressure on the puncture site. d. uses the Doppler to assess for pulse signals.

C

A patient returns from the OR and the nurses assesses for subcutaneous emphysema, which is a potential complication associated with tracheostomy. How does the nurse assess for this complication? A. checking the volume of the pilot balloon B. listening for airflow through the tube C. inspecting and palpating for air under the skin D. assessing the tube for patency

C

A patient taking phenytoin (Dilantin) and isoniazid reports feeling lethargic. Nystagmus is noted on physical examination. What problem does this information suggest to the nurse? A. Tubercular reactivation B. Abrupt discontinuation of isoniazid C. Phenytoin toxicity D. Liver damage

C

A patient who is breathing on his own has a fenestrated trach tube with a cuff. Which precaution must the nurse instruct the student about when caring for this patient? A. always keep the cuff inflated to prevent secretions from entering the lungs B. suction the patient every 30 to 60 minutes C. always deflate the cuff before capping the tube with the decannulation cuff D. To reduce the risk for tracheal damage, keep the cuff pressure between 22 and 30 mm Hg

C

A patient with ST-segment elevation in three contiguous electrocardiographic (ECG) leads is admitted to the emergency department (ED) and diagnosed as having an ST-segment-elevation myocardial infarction (STEMI). Which question should the nurse ask to determine whether the patient is a candidate for thrombolytic therapy? a. "Do you have any allergies?" b. "Do you take aspirin on a daily basis?" c. "What time did your chest pain begin?" d. "Can you rate your chest pain using a 0 to 10 scale?"

C

A patient with diabetes is started on phenytoin (Dilantin) for partial seizures. What does the nurse closely monitor in this individual? A. Blood pressure B. Hypoglycemia C. Hyperglycemia D. Weight loss

C

A patient with increased ICP has mannitol (Osmitrol) prescribed. Which option is the best indication that the drug is achieving the desired therapeutic effects? A. Urine output increases from 30 mL to 50 mL/hour. B. Blood pressure remains less than 150/90 mm Hg. C. The LOC improves. D. No crackles are auscultated in the lung fields.

C

A patient with increased ICP is being monitored in the intensive care unit (ICU) with a fiberoptic catheter. Which order is a priority for you? A. Perform hourly neurologic checks. B. Take a complete set of vital signs. C. Administer the prescribed mannitol (Osmitrol). D. Give an H2-receptor blocker.

C

A patient's sudden onset of hemiplegia has necessitated a computed tomography (CT) of her head. Which assessment should you complete before this diagnostic study? A. Assess the patient's immunization history. B. Screen the patient for any metal parts or a pacemaker. C. Assess the patient for allergies to shellfish, iodine, or dyes. D. Assess the patient's need for tranquilizers or antiseizure medications.

C

A pregnant woman has been diagnosed with acute respiratory distress syndrome​ (ARDS) and prescribed corticosteroids. The woman asks the nurse if it is safe to take this medication. Which is the best reply by the​ nurse? A. "That is an​ error; you​ shouldn't take corticosteroids during​ pregnancy." B. "Let's call the doctor and pharmacist to double check the safety profile of these​ drugs." C. "Corticosteroids are safe to take during pregnancy in some​ cases, and you need the​ medicine." D. ​"If the doctor prescribed the​ medication, I'm sure that​ it's fine to take​ it."

C

A pregnant woman who was tested and diagnosed with HIV infection is very upset. What should the nurse teach this patient about her baby's risk of being born with HIV infection? A. "The baby will probably be infected with HIV." B. "Only an abortion will keep your baby from having HIV." C. "Treatment with antiretroviral therapy will decrease the baby's chance of HIV infection." D. "The duration and frequency of contact with the organism will determine if the baby gets HIV infection."

C

After the nurse has finished teaching a patient about the use of sublingual nitroglycerin (Nitrostat), which patient statement indicates that the teaching has been effective? a. "I can expect some nausea as a side effect of nitroglycerin." b. "I should only take the nitroglycerin if I start to have chest pain." c. "I will call an ambulance if I still have pain after taking 3 nitroglycerin 5 minutes apart." d. "Nitroglycerin helps prevent a clot from forming and blocking blood flow to my heart."

C

An 8 year old is to receive a chemotherapeutic drug in the outpatient clinic; his mother comments that he is not eating well because he gets so nauseous with the chemo. One way to minimize these effects is to a. administer an antiemetic when the child complains of nausea b. avoid foods in the room that make him nauseous c. administer an antiemetic 30 minutes to an hour before the chemo infusion d. omit the chemo therapeutic drug from his regimen

C

A​ client's husband​ asks, "What should I do if my wife has a seizure to keep her​ safe?" Which response by the nurse is​ correct? A.​"Monitor your​ wife's blood​ pressure." B.​"Restrain your​ wife." C.​"Place your wife in the​ side-lying position." D.​"Insert a padded tongue blade in your​ wife's mouth."

C

Because corticosteroids may cause sodium retention, the nurse will closely monitor patients with which condition when administering corticosteroids? A. Diabetes mellitus B. Seizure disorders C. Heart failure D. Hyperthyroidism

C

Before administering sodium polystyrene sulfonate (Kayexalate) to a patient with hyperkalemia, the nurse should assess the a. blood urea nitrogen (BUN) and creatinine. b. blood glucose level. c. patients bowel sounds. d. level of consciousness (LOC).

C

For a male client with chronic obstructive pulmonary disease, which nursing intervention would help maintain a patent airway? A. Restricting fluid intake to 1,000 ml/day B. Enforcing absolute bed rest C. Teaching the client how to perform controlled coughing D. Administering prescribed sedatives regularly and in large amounts

C

For a male client with suspected increased intracranial pressure (ICP), a most appropriate respiratory goal is to: a. prevent respiratory alkalosis. b. lower arterial pH. c. promote carbon dioxide elimination. d. maintain partial pressure of arterial oxygen (PaO2) above 80 mm Hg

C

Heparin is ordered for a patient with a non-ST-segment-elevation myocardial infarction (NSTEMI). What is the purpose of the heparin? a. Heparin enhances platelet aggregation. b. Heparin decreases coronary artery plaque size. c. Heparin prevents the development of new clots in the coronary arteries. d. Heparin dissolves clots that are blocking blood flow in the coronary arteries.

C

Human Papilloma Virus in AIDS patients is manifested as: A. Cough, evening fever, night sweats, weight loss and anemia B. Persistent fever, tachypnoea, hypoxia, cyanosis and tachycardia. C. Genital warts, flat warts, skin warts, neoplasm of cervix, vagina and penis D. Watery diarrhea, abdominal pain, nausea and vomiting

C

In preparation for discharge, the nurse teaches a patient with chronic stable angina how to use the prescribed short-acting and long-acting nitrates. Which patient statement indicates that the teaching has been effective? a. "I will check my pulse rate before I take any nitroglycerin tablets." b. "I will put the nitroglycerin patch on as soon as I get any chest pain." c. "I will stop what I am doing and sit down before I put the nitroglycerin under my tongue." d. "I will be sure to remove the nitroglycerin patch before taking any sublingual nitroglycerin."

C

Mechanical ventilation with a rate and volume to maintain a mild hyperventilation is used for a patient with a head injury. To evaluate the effectiveness of the therapy, the nurse should a. monitor oxygen saturation. b. check arterial blood gases (ABGs). c. monitor intracranial pressure (ICP). d. assess patient breath sounds

C

Mr. D, a 28-year-old man, has been admitted to the intensive care unit for monitoring after a motor vehicle accident (MVA). Your physical assessment reveals multiple abrasions and bruising across the chest but an otherwise healthy young man. Suddenly, Mr. D complains of difficulty breathing. You quickly perform an assessment of his respiratory status and observe that his O2 saturation has dropped dramatically, there are decreased breath sounds on the left, and it appears that there is some tracheal deviation. What would be your next logical action? A) Notify Mr. D's physician and prepare for a stat V/Q scan. B) Start Mr. D on O2 at 4 L/min nasal cannula and prepare an aminophylline drip. C) Call the rapid response team and prepare for emergency insertion of a chest tube. D) Notify Mr. D's physician of these changes.

C

Nurse admitting post-craniotomy client from PACU. Client's incision is supratentorial, nurse will assist client into which position? A.Head of bed flat B.Supine C.Elevate HOB to 30 degrees. D.Lying on operative side

C

The nurse completes a history and physical on a client admitted with exacerbation of a seizure disorder. What datum collected by the nurse requires intervention? A.History of asthma B. History of diabetes mellitus C. Use of herb Ginkgo biloba D. Use of aspirin daily

C

The nurse counsels the family of a 4 year old child that in the period immediately following remission he must a. consume a large amount of food items containing potassium to maintain adequate potassium levels b. consume a large amount of foods containing iron to replenish deleted iron stores c. wear a surgical mask and practice meticulous hand washing when going out of his house d. all of the above are important in this time period

C

The nurse evaluates the effectiveness of a paracentesis in a patient who has ascites. Which measurement is most important for the nurse to note? a. Cardiac output b. Blood pressure c. Abdominal girth d. Intake and output

C

The nurse is caring for a patient with chronic kidney disease after hemodialysis. Which patient care action should the nurse delegate to the experienced unlicensed assistive personnel (UAP)? a. Assess the patient's access site for a thrill and bruit. b. Monitor for signs and symptoms of postdialysis bleeding. c. Check the patient's postdialysis blood pressure and weight. d. Instruct the patient to report signs of dialysis disequilibrium syndrome immediately.

C

The nurse is caring for a patient with emphysema and respiratory failure who is receiving mechanical ventilation through an endotracheal tube. To prevent ventilator-associated pneumonia (VAP), which action is most important to include in the plan of care? A.) Administer ordered antibiotics as scheduled. B.) Hyperoxygenate the patient before suctioning. C.) Maintain the head of bed at a 30- to 45-degree angle. D.) Suction the airway when coarse crackles are audible.

C

The nurse is caring for a patient with emphysema and respiratory failure who is receiving mechanical ventilation through an endotracheal tube. To prevent ventilator-associated pneumonia (VAP), which action is most important to include in the plan of care? a. Administer ordered antibiotics as scheduled. b. Hyperoxygenate the patient before suctioning. c. Maintain the head of bed at a 30- to 45-degree angle. d. Suction the airway when coarse crackles are audible

C

The nurse is caring for an older adult client who may need to be intubated. The family asks about the potential risks. Which is the best reply by the​ nurse? A. "There are no risks to​ intubation." B. "The doctor feels that intubation is the best option for your family​ member." C. "Older adults may be more likely to experience injury to the airway during​ intubation." D. "Intubation and mechanical ventilation are not​ long-term solutions. They just allow the lungs to​ rest."

C

The nurse is conducting a home visit for a​ 6-year-old client who has myoclonic and absence seizures. The parents are following a ketogenic diet for the child. Which observation requires​ follow-up by the​ nurse? A.Parents administer​ medium-chain-triglyceride (MCT) oil as needed. B.Parents include low carbohydrate foods. C.Parents include​ low-fat foods for each meal. D.Parents monitor urine ketone levels regularly.

C

The nurse is monitoring the effectiveness of antiretroviral therapy (ART) for a 56-year-old man with acquired immunodeficiency syndrome (AIDS). What laboratory study result indicates the medications have been effective? a. Increased viral load b. Decreased neutrophil count c. Increased CD4+ T cell count d. Decreased white blood cell count

C

The nurse is performing nasotracheal suctioning on a medical patient and obtains copious amounts of secretions from the patients airway, even after inserting and withdrawing the catheter several times. How should the nurse proceed? A) Continue suctioning the patient until no more secretions are obtained. B) Perform chest physiotherapy rather than nasotracheal suctioning. C) Wait several minutes and then repeat suctioning. D) Perform postural drainage and then repeat suctioning.

C

The nurse is preparing to suction a patient with an endotracheal tube. What should be the nurses first step in the suctioning process? A) Explain the suctioning procedure to the patient and reposition the patient. B) Turn on suction source at a pressure not exceeding 120 mm Hg. C) Assess the patients lung sounds and SAO2 via pulse oximeter. D) Perform hand hygiene and don nonsterile gloves, goggles, gown, and mask.

C

The nurse is suctioning a client via an endotracheal tube. During the suctioning procedure., the nurse notes on the monitor that the heart rate is decreasing. Which nursing intervention is appropriate? a. Continue to suction b. Notify the health care provider immediately c. Stop the procedure and re-oxygenate the client d. Ensure that the suction is limited to 15 seconds

C

The nurse is suctioning the secretions from a patient's endotracheal tube. The patient demonstrates a vagal response by a drop in heart rate to 54/min and a drop in BP to 90/55 mm Hg. After stopping suctioning, what is the nurses priority action? A. allow the patient to rest for at least 10 mins B. monitor the patient and call the Rapid Response team C. oxygenate with 100% oxygen and monitor the patient D. administer atropine according to standing orders

C

The nurse notes that a patient with a head injury has a clear nasal drainage. The most appropriate nursing action for this finding is to a. obtain a specimen of the fluid and send for culture and sensitivity. b. take the patient's temperature to determine whether a fever is present. c. check the nasal drainage for glucose with a Dextrostik or Testape. d. have the patient to blow the nose and then check the nares for redness.

C

The nursing instructor is teaching her students about the differences between ARF (Acute Respiratory Failure) and ARDS (Acute Respiratory Distress Syndrome). Which statement best describes this difference if made by the student? A) "ARF occurs in patients with chronic conditions while ARDS occurs in patients with trauma injuries" B) "They are almost the same thing except that ARDS is worse than ARF" C) "ARDS tends to occur up to a day or two after the initiating event, and unlike ARF requires mechanical ventilation to maintain oxygen status" D) "ARF is a disorder that mostly affects the breathing pattern while ARDS mostly affects the gas exchange by blocking the alveoli with fluid"

C

The physician orders a patient's T-Tube to be clamped 1 hour before and 1 hour after meals. You clamp the T-Tube as prescribed. While the tube is clamped which finding requires you to notify the physician? A. The T-Tube is not draining. B. The T-Tube tubing is below the patient's waist. C. The patient reports nausea and abdominal pain. D. The patient's stool is brown and formed.

C

To decrease the risk for ventilator-associated pneumonia, which action will the nurse include in the plan of care for a patient who requires intubation and mechanical ventilation? a. Avoid use of positive end-expiratory pressure (PEEP). b. Suction every 2 hours. c. Elevate head of bed to 30 to 45 degrees. d. Give enteral feedings at no more than 10 mL/hr.

C

To evaluate the effectiveness of prescribed therapies for a patient with ventilatory failure, which diagnostic test will be most useful to the nurse? a. Chest x-rays b. Pulse oximetry c. Arterial blood gas (ABG) analysis d. Pulmonary artery pressure monitoring

C

Two hours after a kidney transplant, the nurse obtains all of the following data when assessing the patient. Which information is most important to communicate to the health care provider? a. The urine output is 900 to 1100 mL/hr. b. The blood urea nitrogen (BUN) and creatinine levels are elevated. c. The patients central venous pressure (CVP) is decreased. d. The patient has level 8 (on a 10-point scale) incisional pain.

C

What causes the GI manifestation of stomatitis in the patient with CKD? a. high serum sodium levels b. irritation of the GI tract from creatinine c. increased ammonia from bacterial breakdown of urea d. iron salts, calcium-containing phosphate binders, and limited fluid intake

C

What is most important finding for you to act on for a patient who had a craniotomy? A. Sodium: 134 mEq/L B. While blood cell (WBC) count: 11,000/μL C. Urine specific gravity: 1.001 D. Blood urea nitrogen (BUN): 25 mg/dL

C

What is the correct sequence for suctioning a patient? 1. Open kit and basin. 2. Apply gloves. 3. Lubricate catheter. 4. Verify functioning of suction device and pressure. 5. Connect suctioning tubing to suction catheter. 6. Increase supplemental oxygen. 7. Reapply oxygen. 8. Suction airway. A. 6, 4, 3, 1, 2, 5, 8, 7 B. 4, 6, 1, 2, 3, 8, 5, 7 C. 4, 6, 1, 3, 2, 5, 8, 7 D. 6, 4, 1, 3, 2, 5, 7, 8

C

What nursing care is included for the patient with an ET tube? a. check the cuff pressure every hour b. keep a tracheosotmy tray at the bedside c. hyperoxygenate before and after suctioning d. reuse the suction catheter at the bedside for 24 hours

C

What pediatric oncologic emergencies are caused by the rapid release of intracellular metabolites during the initial treatment of some cancers? a. Hyperleukocytosis b. Overwhelming infection c. Acute tumor lysis syndrome d. Superior vena cava syndrome

C

What sign would make you suspect the cause of increased ICP involves the hypothalamus? A. Contralateral hemiparesis B. Ipsilateral pupil dilation C. Rise in temperature D. Decreased urine output

C

What teaching does the nurse include for a client with atrial fibrillation who has a new prescription for warfarin? a. It is important to consume a diet high in green leafy vegetables. b. You should take aspirin or ibuprofen for headache. c. Report nosebleeds to your provider immediately. d. Avoid caffeinated beverages.

C

When assessing a patient with a head injury, the nurse recognizes that the earliest indication of increased intracranial pressure (ICP) is a. vomiting. b. headache. c. change in level of consciousness (LOC). d. sluggish pupil response to light.

C

When assessing for potential serious adverse effects to propylthiouracil (PTU), the nurse will monitor which laboratory test? A. Kidney function B. Serum electrolytes C. Complete blood count (CBC) D. Brain natriuretic peptide

C

When caring for a patient who developed acute respiratory distress syndrome (ARDS) as a result of a urinary tract infection (UTI), the nurse is asked by the patient's family how a urinary tract infection could cause lung damage. Which response by the nurse is appropriate? a. "The infection spread through the circulation from the urinary tract to the lungs." b. "The urinary tract infection produced toxins that damaged the lungs." c. "The infection caused generalized inflammation that damaged the lungs." d. "The fever associated with the infection led to scar tissue formation in the lungs."

C

When should the nurse suction a patient's ET tube? a. when the patient has peripheral wheezes in all lobes b. when the patient has not been suctioned for the past 2 hours c. when the nurse auscultates adventitious sounds over the central airways d. when the nurse assesses a need to stimulate the patient to cough and deep breathe

C

When the nurse is weaning a patient who has chronic obstructive pulmonary disease (COPD) from mechanical ventilation, which patient assessment indicates that the weaning protocol should be discontinued? a. The patient heart rate is 98 beats/min. b. The patients oxygen saturation is 93%. c. The patient respiratory rate is 32 breaths/min. d. The patients spontaneous tidal volume is 500 mL.

C

Which clinical finding in a patient with a recent tracheostomy is the most serious and requires immediate intervention? A. increased cough and difficulty expectorating secretions B. food particles in the tracheal secretions C. pulsating tracheostomy tube in synchrony with the heartbeat D. set tidal volume on the ventilator not being received by the patient

C

Which factor indicates that tracheostomy placement would be preferable to endotracheal intubation? a. The patient is unable to clear secretions. b. The patient is at high risk for aspiration. c. A long-term airway is probably necessary. d. An upper airway obstruction is impairing the patient's ventilation.

C

Which finding by the nurse during abdominal auscultation indicates a need for a focused abdominal assessment? a. Loud gurgles b. High-pitched gurgles c. Absent bowel sounds d. Frequent clicking sounds

C

Which intervention is key to preventing ventilator-associated pneumonia as a complication in a patient with acute respiratory distress syndrome (ARDS)? A. Scheduled prophylactic nasopharyngeal suctioning B. Instilling normal saline down the endotracheal tube to loosen secretions C. Providing frequent mouth care and oral hygiene D. Using high tidal volumes on the ventilator

C

Which is part of the nursing management for ARDS? A. Aggressive use of intravenous (IV) fluids B. Administration of a β-blocker C. Use of positive end-expiratory pressure (PEEP) D. Use of the lateral recumbent position

C

Which of the following arteries primarily feeds the anterior wall of the heart? a.) Circumflex artery b.) Internal mammary artery c.) Left anterior descending artery d.) Right coronary artery

C

Which patient below is at MOST risk for developing ARDS and has the worst prognosis? A. A 52-year-old male patient with a pneumothorax. B. A 48-year-old male being treated for diabetic ketoacidosis. C. A 69-year-old female with sepsis caused by a gram-negative bacterial infection. D. A 30-year-old female with cystic fibrosis.

C

Which patient statement demonstrates understanding of radioactive iodine (I-131) therapy? A. "I will need to take this drug on a daily basis for at least 1 year." B. "This drug will help decrease my cold intolerance and weight gain." C. "This drug will be taken up by the thyroid gland and destroy thyroid tissue." D. "I will isolate myself from my family for 1 week so there is no risk of radiation exposure."

C

Which statement by a patient with stage 5 chronic kidney disease (CKD) indicates that the nurses teaching about management of CKD has been effective? a. I need to try to get more protein from dairy products. b. I will try to increase my intake of fruits and vegetables. c. I will measure my urinary output each day to help calculate the amount I can drink. d. I need to take the erythropoietin to boost my immune system and help prevent infection.

C

While caring for a patient with a suspected pneumothorax, you note there are several areas on the patient's skin that appear to be "bulging" out. These "bulging" areas are located on the patient's neck, face, and abdomen. On palpation on these areas, you note they feel "crunchy". When charting your findings you would refer to this finding as? A. Subcutaneous paresthesia B. Pigment molle C. Subcutaneous emphysema D. Veisalgia

C

While the nurse performs ROM on an unconscious patient with increased ICP, the patient experiences severe decerebrate posturing reflexes. The nurse should a. use restraints to protect the patient from injury b. administer CNS depressants to lightly sedate the patient c. perform the exercises less frequently because posturing can increase ICP d. continue the exercises because they are necessary to maintain musculoskeletal function

C

With which male client will the nurse conduct prostate screening and education? A. Young adult with a history of urinary tract infections. B. Client who has sustained an injury to the external genitalia. C. Adult who is older than 50 years. D. Sexually active client.

C

You are alerted to a possible acute subdural hematoma in the patient who A. has a linear skull fracture crossing a major artery. B. has focal symptoms of brain damage with no recollection of a head injury. C. develops decreasing LOC and a headache within 48 hours of a head injury. D. has an immediate loss of consciousness with a brief lucid interval followed by decreasing LOC.

C

You are answering questions at a class on brain tumors for nursing students. Which information related to brain tumors should you include in this class? A. Brain tumors are not removed unless than they cause headaches or seizures. B. Seizures are an uncommon symptom unless there is metastasis. C. The most common type of brain tumor is from metastasis of cancer outside of the brain. D. Brain tumors commonly metastasize to the lungs because of high vascularity.

C

You are caring for a patient who is admitted with a barbiturate overdose. The patient is unresponsive, with a blood pressure of 90/60 mm Hg, apical pulse of 110 beats/minute, and respiratory rate of 8 breaths/minute. Based on the initial assessment findings, you recognize that the patient is at risk for which type of respiratory failure? A. Hypoxemic respiratory failure related to shunting of blood B. Hypoxemic respiratory failure related to diffusion limitation C. Hypercapnic respiratory failure related to alveolar hypoventilation D. Hypercapnic respiratory failure related to increased airway resistance

C

You are providing care to a patient with pericarditis. Which of the following is NOT a proper nursing intervention for this patient? A. Monitor the patient for complications of cardiac tamponade. B. Administer Ibuprofen as scheduled. C. Place the patient in supine position to relieve pain. D. Monitor the patient for pulsus paradoxus and muffled heart sounds.

C

You're providing care to a patient who is being treated for aspiration pneumonia. The patient is on a 100% non-rebreather mask. Which finding below is a HALLMARK sign and symptom that the patient is developing acute respiratory distress syndrome (ARDS)? A. The patient is experiencing bradypnea. B. The patient is tired and confused. C. The patient's PaO2 remains at 45 mmHg. D. The patient's blood pressure is 180/96.

C

You're providing discharge teaching to a patient being treated for endocarditis. Which statement by the patient demonstrated they understood your teaching about this condition? A. "I will stop taking the antibiotics once my fever is gone in order to prevent antibiotic resistance." B. "I will only wash my hands with soap and water." C. "I will inform my dentist about my history of endocarditis prior to any invasive procedures." D. "I will avoid eating fish and organ meats."

C

Your patient is unable to have a cholecystectomy for the treatment of cholecystitis. Therefore, a cholecystostomy tube is placed to help treat the condition. Which statement about a cholecystostomy (C-Tube) is TRUE? A. The C-Tube is placed in the cystic duct of the gallbladder and helps drain infected bile from the gallbladder. B. Gallstones regularly drain out of the C-Tube, therefore, the nurse should flush the tube regularly to ensure patency. C. The C-Tube is placed through the abdominal wall and directly into the gallbladder where it will drain infected bile from the gallbladder. D. The tubing and drainage bag of the C-Tube should always be level with the insertion site to ensure the tube is draining properly.

C

a nurse is completing discharge teaching to a client who has seizures and received a vagal nerves stimulator to decrease seizure activity. which of the following statements should the nurse include in the teaching? A. it is safe to use microwave that are 1200 watts or less B. you should avoid the use of CT scans with contrast C. you should place a magnet over the implantable device when you feel an aura occurring D. it is recommended that you use ultrasound diathermy for pain management

C

a nurse is reviewing the use of the meningococcal vaccine for the prevention of meningitis with a newly licensed nurse. which of the following information should the nurse include? A. the vaccine is indicated to reduce the risk of respiratory infection B. the vaccine is administered in a series of four doses C. the vaccine is recommended for adolescents before starting college D. the vaccine is initially given at 2 months of agents

C

an 18 yr old is admitted with an acute onset of right lower quadrant pain. Appendicitis is suspected. For which clinical indicator should the nurse assess the client to determine if the pain is secondary to appendicitis A) urinary retention B) gastric hyperacidity C) rebound tenderness D) increased lower bowel motility

C

in which of the following positions should the nurse place a client following a craniotomy for evacuation of a subdural hematoma of the frontal lobe? A. supine B. Prone C. semi-fowlers D. sims

C

the nurse is collecting data for a pt who develops jaundice and dark, amber colored urine. the nurse recognizes that which of the following is most likely the cause? a. encephalopathy b. pancreatitis c. bile duct obstruction d. cholecystitis

C

the nurse is evaluating the status of a client who had a craniotomy 3 days ago. which assessment finding would indicate that the client is developing meningitis as a complication of surgery? A. negative kerning sign B. absence of nuchal rigidity C. a positive brudzinski sign D. a glasgow coma scale score of 15

C

The nurse is determining whether the client's rhythm strip demonstrates proper firing of the sinoatrial (SA) node. Which waveform indicates proper function of the SA node? a. The QRS complex is present. b. The PR interval is 0.24 second. c. A P wave precedes every QRS complex. d. The ST segment is elevated.

C A P wave is generated by the SA node and represents atrial depolarization.

Which of the following findings would alarm the nurse when caring for a client receiving chemotherapy who has a platelet count of 17,000/mm3? A. Increasing shortness of breath B. Diminished bilateral breath sounds C. Change in mental status D. Weight gain of 4 pounds in 1 day

C A change in mental status could result from spontaneous bleeding; in this case, a cerebral hemorrhage may have developed. Increasing shortness of breath is typically related to anemia, not to thrombocytopenia

A patient treated for human immunodeficiency virus (HIV) infection for 6 years has developed fat redistribution to the trunk, with wasting of the arms, legs, and face. What instructions will the nurse give to the patient? a. Review foods that are higher in protein. b. Teach about the benefits of daily exercise. c. Discuss a change in antiretroviral therapy. d. Talk about treatment with antifungal agents.

C A frequent first intervention for metabolic disorders is a change in antiretroviral therapy (ART). Treatment with antifungal agents would not be appropriate because there is no indication of fungal infection. Changes in diet or exercise have not proven helpful for this problem.

Which teaching is most appropriate for a client with chemotherapy-induced neuropathy? A. Bathe in cold water. B. Wear cotton gloves when cooking. C. Consume a diet high in fiber. D. Make sure shoes are snug.

C A high-fiber diet will assist with constipation due to neuropathy. Cotton gloves may prevent harm from scratching; protective gloves should be worn for washing dishes and gardening. Wearing cotton gloves while cooking can increase the risk for burns

A nurse assesses a client who is recovering after a left-sided cardiac catheterization. Which assessment finding requires immediate intervention? a. Urinary output less than intake b. Bruising at the insertion site c. Slurred speech and confusion d. Discomfort in the left leg

C A left-sided cardiac catheterization specifically increases the risk for a cerebral vascular accident. A change in neurologic status needs to be acted on immediately. Discomfort and bruising are expected at the site. If intake decreases, a client can become dehydrated because of dye excretion. The second intervention would be to increase the client's fluid status. Neurologic changes would take priority

A 58-year-old patient has just been admitted to the emergency department with nausea and vomiting. Which information requires the most rapid intervention by the nurse? a. The patient has been vomiting for 4 days. b. The patient takes antacids 8 to 10 times a day. c. The patient is lethargic and difficult to arouse. d. The patient has undergone a small intestinal resection.

C A lethargic patient is at risk for aspiration, and the nurse will need to position the patient to decrease aspiration risk. The other information is also important to collect, but it does not require as quick action as the risk for aspiration.

A nurse cares for a client who is recovering from a hypophysectomy. Which action should the nurse take first? a. Keep the head of the bed flat and the client supine. b. Instruct the client to cough, turn, and deep breathe. c. Report clear or light yellow drainage from the nose. d. Apply petroleum jelly to lips to avoid dryness.

C A light yellow drainage or a halo effect on the dressing is indicative of a cerebrospinal fluid leak. The client should have the head of the bed elevated after surgery. Although deep breathing is important postoperatively, coughing should be avoided to prevent cerebrospinal fluid leakage. Although application of petroleum jelly to the lips will help with dryness, this instruction is not as important as reporting the yellowish drainage.

The nurse provides discharge instructions for a 64-year-old woman with ascites and peripheral edema related to cirrhosis. Which statement, if made by the patient, indicates teaching was effective? A. "It is safe to take acetaminophen up to four times a day for pain." B. "Lactulose (Cephulac) should be taken every day to prevent constipation." C. "Herbs and other spices should be used to season my foods instead of salt." D. "I will eat foods high in potassium while taking spironolactone (Aldactone)."

C A low-sodium diet is indicated for the patient with ascites and edema related to cirrhosis. Table salt is a well-known source of sodium and should be avoided. Alternatives to salt to season foods include the use of seasonings such as garlic, parsley, onion, lemon juice, and spices. Pain medications such as acetaminophen, aspirin, and ibuprofen should be avoided as these medications may be toxic to the liver. The patient should avoid potentially hepatotoxic over-the-counter drugs (e.g., acetaminophen) because the diseased liver is unable to metabolize these drugs. Spironolactone is a potassium-sparing diuretic. Lactulose results in the acidification of feces in bowel and trapping of ammonia, causing its elimination in feces.

A patient on hemodialysis develops a thrombus of a subcutaneous arteriovenous (AV) graft, requiring its removal.While waiting for a replacement graft or fistula, the patient is most likely to have what done for treatment? a. Peritoneal dialysis b. Peripheral vascular access using radial artery c. Silastic catheter tunneled subcutaneously to the jugular vein d. Peripherally inserted central catheter (PICC) line inserted into subclavian vein

C A more permanent, soft, flexible Silastic double-lumen catheter is used for long-term access when other forms of vascular access have failed. These catheters are tunneled subcutaneously and have Dacron cuffs that prevent infection from tracking along the catheter.

A client has suspected alterations in antidiuretic hormone (ADH) function. Which diagnostic test does the nurse anticipate will be requested for this client? A. Adrenocorticotropic hormone (ACTH) suppression test B. Chest x-ray C. Cranial computed tomography (CT) D. Renal sonography

C ADH is a hormone of the posterior pituitary. Brain abscess, tumor, or subarachnoid hemorrhage could cause alterations in ADH levels. These can be seen on a CT scan of the brain. ACTH triggers the release of cortisol from the adrenal cortex and is not related to ADH. A chest x-ray would not show a pituitary tumor or brain abscess. Even though ADH acts on distal convoluted tubules in the kidneys, a renal sonogram would diagnose the cause of syndrome of inappropriate antidiuretic hormone.

A patient needing vascular access for hemodialysis asks the nurse what the differences are between an arteriovenous (AV) fistula and a graft. The nurse explains that one advantage of the fistula is that it a. can accommodate larger needles. b. increases patient mobility. c. is much less likely to clot. d. can be used sooner after surgery.

C AV fistulas are much less likely to clot than grafts, although it takes longer for them to mature to the point where they can be used for dialysis. The choice of an AV fistula or a graft does not impact on needle size or patient mobility

A client with a coronary occlusion is experiencing chest pain and distress. What is the primary reason that the nurse should administer oxygen to this client? A. Prevent dyspnea B. Prevent cyanosis C. Increase oxygen concentration to heart cells D. Increase oxygen tension in the circulating blood

C Administration of oxygen increases the transalveolar oxygenngradient, which improves the efficiency of the cardiopulmonary system; this increases the oxygen supply to the heart. Increased oxygen to the heart cells will improve cardiac output, which may or may not prevent dyspnea. Pallor, not cyanosis, is usually associated with MI. Although administrating oxygen will increase oxygen tension in the circulating blood, it is not specific to heart cells, which are hypoxic when there is a MI.

A patient with a positive rapid antibody test result for human immunodeficiency virus (HIV) is anxious and does not appear to hear what the nurse is saying. What action by the nurse is most important at this time? a. Teach the patient about the medications available for treatment. b. Inform the patient how to protect sexual and needle-sharing partners. c. Remind the patient about the need to return for retesting to verify the results. d. Ask the patient to notify individuals who have had risky contact with the patient.

C After an initial positive antibody test, the next step is retesting to confirm the results. A patient who is anxious is not likely to be able to take in new information or be willing to disclose information about HIV status of other individuals.

The nurse is caring for a patient who is 24 hours postpacemaker insertion. Which of the following nursing interventions is most appropriate at this time? a. Reinforcing the pressure dressing as needed b. Encouraging range-of-motion exercises of the involved arm c. Assessing the incision for any redness, swelling, or discharge d. Applying wet-to-dry dressings every 4 hours to the insertion site

C After pacemaker insertion, it is important for the patient to limit activity of the involved arm to minimize pacemaker lead displacement. The nonpressure dressing is kept dry until removed, usually 24 hours postoperative. It is important for the nurse to observe signs of infection by assessing for any redness, swelling, or discharge from the incision site.

A patient with cholelithiasis needs to have the gallbladder removed. Which patient assessment is a contraindication for a cholecystectomy? A. Low-grade fever of 100° F and dehydration B. Abscess in the right upper quadrant of the abdomen C. Activated partial thromboplastin time (aPTT) of 54 seconds D. Multiple obstructions in the cystic and common bile duct

C An aPTT of 54 seconds is above normal and indicates insufficient clotting ability. If the patient had surgery, significant bleeding complications postoperatively are very likely. Fluids can be given to eliminate the dehydration; the abscess can be assessed, and the obstructions in the cystic and common bile duct would be relieved with the cholecystectomy.

Which assessment information about a 60-kg client admitted 12 hours ago with a full-thickness burn over 30% of the total body surface area is of greatest concern to the nurse? A. Bowel sounds are absent. B. The pulse oximetry level is 91%. C. The serum potassium level is 6.1 mEq/L. D. Urine output since admission is 370 mL.

C An elevated serum potassium level can cause cardiac dysrhythmias and arrest, and so is of the most concern. Absence of bowel sounds, a pulse oximetry level of 91%, and urine output of 370 mL since admission are normal findings during the resuscitation phase of burn injury.

The nurse is assessing a patient who had a total gastrectomy 8 hours ago. What information is most important to report to the health care provider? a. Absent bowel sounds b. Complaints of incisional pain c. Temperature 102.1° F (38.9° C) d. Scant nasogastric (NG) tube drainage

C An elevation in temperature may indicate leakage at the anastomosis, which may require return to surgery or keeping the patient NPO. The other findings are expected in the immediate postoperative period for patients who have this surgery.

The patient has been diagnosed with having an open pneumothorax r/t penetrating injury. Which of the following symptoms would the nurse most expect to see in this patient? A) Chest pain and tracheal shifting B) Hyperresanance and hyperexpansion of the affected side C) High pitched respiratory sounds and SpO2 89% D) Muffled heart sounds and bradycardia

C An open pneumothorax is often also known as a sucking chest wound, producing a high pitched sucking sound coming from the wound. Diminished oxygen level, hyperresanance, and chest pain would also be expected in this patient. Tracheal shifting and hyperexpansion of the lung may be seen in a tension pneumothorax (an unlikely development of an open pneumothorax). Tachycardia, rather than bradycardia would be a common symptom in pneumothorax.

What is the most common complication of an MI? A) Cardiogenic shock B) Heart failure C) arrhythmias D) Pericarditis

C Arrhythmias, caused by oxygen deprivation to the myocardium, are the most common complication of an MI. Cardiogenic shock, another complication of an MI, is defined as the end stage of left ventricular dysfunction. This condition occurs in approximately 15% of clients with MI. Because the pumping function of the heart is compromised by an MI, heart failure is the second most common complication. Pericarditis most commonly results from a bacterial or viral infection but may occur after the MI.

A client who was the sole survivor of a house fire says, "I feel so guilty. Why did I survive?" What is the best response by the nurse? A. "Do you want to pray about it?" B. "I know, and you will have to learn to adapt to a new body image." C. "Tell me more." D. "There must be a reason."

C Asking the client to tell the nurse more encourages therapeutic grieving. Offering to pray with the client assumes that prayer is important to the client and does not allow for grieving; the nurse should never assume that the client is religious. The response, "I know, and you will have to learn to adapt to a new body image" only serves to add stress to the client's situation. The response, "There must be a reason" minimizes the grieving process by not allowing the client to express his or her concerns.

A client has just arrived in the PACU following a successful tracheostomy procedure. Which nursing action must be taken first? A. Suction as needed B. Clean the tracheostomy inner cannula and stoma C. Listen to lung sounds D. Change the tracheostomy dressing as needed

C Assessment is the first phase of the nursing process. All other actions and procedures are driven by assessment findings. The first nursing action for a client following an airway procedure is to assess the client's respiratory status; this requires auscultation of the lungs.

After abdominal surgery, a patient with protein calorie malnutrition is receiving parenteral nutrition (PN). Which is the best indicator that the patient is receiving adequate nutrition? a. Serum albumin level is 3.5 mg/dL. b. Fluid intake and output are balanced. c. Surgical incision is healing normally. d. Blood glucose is less than 110 mg/dL.

C Because poor wound healing is a possible complication of malnutrition for this patient, normal healing of the incision is an indicator of the effectiveness of the PN in providing adequate nutrition. Blood glucose is monitored to prevent the complications of hyperglycemia and hypoglycemia, but it does not indicate that the patient's nutrition is adequate. The intake and output will be monitored, but do not indicate that the PN is effective. The albumin level is in the low-normal range but does not reflect adequate caloric intake, which is also important for the patient.

A patient with a tracheostomy has a new order for a fenestrated tracheostomy tube. Which action should the nurse include in the plan of care in collaboration with the speech therapist? a. Leave the tracheostomy inner cannula inserted at all times. b. Place the decannulation cap in the tube before cuff deflation. c. Assess the ability to swallow before using the fenestrated tube. d. Inflate the tracheostomy cuff during use of the fenestrated tube.

C Because the cuff is deflated when using a fenestrated tube, the patient's risk for aspiration should be assessed before changing to a fenestrated tracheostomy tube. The decannulation cap is never inserted before cuff deflation because to do so would obstruct the patient's airway. The cuff is deflated and the inner cannula removed to allow air to flow across the patient's vocal cords when using a fenestrated tube.

A patient who is on the progressive care unit develops atrial flutter, rate 150, with associated dyspnea and chest pain. Which action that is included in the hospital dysrhythmia protocol should the nurse do first? a.Obtain a 12-lead electrocardiogram (ECG). b.Notify the health care provider of the change in rhythm. c.Give supplemental O2 at 2 to 3 L/min via nasal cannula. d.Assess the patient's vital signs including oxygen saturation.

C Because this patient has dyspnea and chest pain in association with the new rhythm, the nurse's initial actions should be to address the patient's airway, breathing, and circulation (ABC) by starting with oxygen administration. The other actions also are important and should be implemented rapidly.

The nurse caring for a client admitted for chest pain and a MI is preparing to apply nitroglycerin ointment. Before applying, the nurse should: A. Assess the client's pulse rate B. Prepare the site with an alcohol swab C. Remove ointment previously applied D. Expect the client to be relieved of pain within 20 minutes

C Before applying the nitro ointment, the nurse should remove the previous ointment. If the previously applied ointment is not removed, the client could receive too much medication. The nurse should assess blood pressure reading, not pulse rate. There is no need to clean the site with alcohol before administration. nitro ointment is not used to treat acute pain.

The nurse determines that treatment of a client with a beta-adrenergic blocker for myocardial infarction has been effective when: A) Tachycardia occurs. B) Blood pressure is 90/50. C) Decreased dysrhythmias occur. D) Decreased urinary output occurs.

C Beta blockers have the ability to decrease heart rate, decrease contractility, and decrease blood pressure, leading to decreased oxygen demand. They also slow conduction, which suppresses dysrhythmias. Tachycardia would not be desired with an MI. A low BP alone would not indicate effective treatment of the MI.

A client diagnosed with hyperpituitarism resulting from a prolactin-secreting tumor has been prescribed bromocriptine mesylate (Parlodel). As a dopamine agonist, what effect does this drug have by stimulating dopamine receptors in the brain? a) Decreases the risk for cerebrovascular disease b) Increases the risk for depression c) Inhibits the release of some pituitary hormones d) Stimulates the release of some pituitary hormones

C Bromocriptine mesylate inhibits the release of both prolactin and growth hormone. It does not decrease the risk for cerebrovascular disease leading to stroke. Increased risk for depression is not associated with the use of bromocriptine mesylate; however, hallucinations have been reported as a side effect. Bromocriptine mesylate does not stimulate the release of any hormones.

A client presenting to the ER with chest pain and dizziness was found to be having a MI and subsequently suffered cardiac arrest. The ER health care team was able to successfully resuscitate the client. Lab work shows that the client is now acidotic. The nurse understands that the acidic serum pH most likely is caused from: A. fat forming ketoacids that are broken down. B. The client receiving too much sodium bicarbonate during resuscitation efforts. C. The decreased tissue perfusion that subsequently caused lactic acid production. D. An irregular heartbeat the client experienced during cardiac arrest.

C Cardiac arrest causes decreased tissue perfusion, which results in ischemic and cardiac insufficiency. Cardiac insufficiency causes anaerobic metabolism, which leads to lactic acid production. Fat-forming ketoacids occur in diabetes. Too much sodium bicarbonate causes alkalosis, not acidosis. An irregular heartbeat does not cause acidosis necessarily.

The client who has undergone breast surgery is struggling with issues concerning her sexuality. What is the best way for the nurse to address the client's concerns? A. Allow the client to bring up the topic first. B. Remind the client to avoid sexual intercourse for 2 months after the surgery. C. Suggest that the client wear a bra during intercourse. D. Teach the client that birth control is a priority.

C Clients may prefer to lay a pillow over the surgical site or wear a bra or camisole to prevent contact with the surgical site during intercourse.

The nurse asks a client who has experienced ventricular dysrhythmias about substance abuse. The client asks, "Why do you want to know if I use cocaine?" How should the nurse respond? a. "Substance abuse puts clients at risk for many health issues." b. "The hospital requires that I ask you about cocaine use." c. "Clients who use cocaine are at risk for fatal dysrhythmias." d. "We can provide services for cessation of substance abuse."

C Clients who use cocaine or illicit inhalants are particularly at risk for potentially fatal dysrhythmias. The other responses do not adequately address the client's question.

A patient who has a right-sided chest tube following a thoracotomy has continuous bubbling in the suction-control chamber of the collection device. Which action by the nurse is most appropriate? a. Document the presence of a large air leak. b. Notify the surgeon of a possible pneumothorax. c. Take no further action with the collection device. d. Adjust the dial on the wall regulator to decrease suction.

C Continuous bubbling is expected in the suction-control chamber and indicates that the suction-control chamber is connected to suction. An air leak would be detected in the water-seal chamber. There is no evidence of pneumothorax. Increasing or decreasing the vacuum source will not adjust the suction pressure. The amount of suction applied is regulated by the amount of water in this chamber and not by the amount of suction applied to the system

A patient with AKI is a candidate for continuous renal replacement therapy (CRRT). What is the most commonindication for use of CRRT? a. Azotemia b. Pericarditis c. Fluid overload d. Hyperkalemia

C Continuous renal replacement therapy (CRRT) is indicated for the patient with AKI as an alternative or adjunct to hemodialysis to slowly remove solutes and fluid in the hemodynamically unstable patient. It is especially useful for treatment of fluid overload, but hemodialysis is indicated for treatment of hyperkalemia, pericarditis, or other serious effects of uremia.

Which client does the nurse identify as being at highest risk for acute adrenal insufficiency resulting from corticosteroid use? a) Client with hematemesis, upper epigastric pain for the past 3 days not relieved with food, and melena b) Client with right upper quadrant pain unrelieved for the past 2 days, dark-brown urine, and clay-colored stools c) Client with shortness of breath and chest tightness, nasal flaring, audible wheezing, and oxygen saturation of 85% for the second time this week d) Client with three emergency department visits in the past month for edema, shortness of breath, weight gain, and jugular venous distention

C Corticosteroids may be used to treat signs and symptoms of asthma, such as shortness of breath and chest tightness, nasal flaring, audible wheezing, and oxygen saturation of 85%. This places the client at risk for adrenal insufficiency. Corticosteroids are not used to treat signs and symptoms of GI bleeding or peptic ulcer disease (hematemesis, upper epigastric pain for the past 3 days not relieved with food, and melena), gallbladder disease (right upper quadrant pain unrelieved for the past 2 days, dark brown urine, and clay-colored stools), or congestive heart failure (edema, shortness of breath, weight gain, and jugular venous distention).

A nurse is reviewing the health record of a client who has pancreatitis. The physical exam report by the provider indicates the presence of Cullen's sign. Which of the following is an appropriate action by the nurse to identify this finding? A. Tap lightly at the costovertebral margin on the client's back. B. Palpate the client's right lower quadrant. C. Inspect the skin around the umbilicus. D. Auscultate the area below the client's scapula.

C Cullen's sign is indicated by a bluish-grey discoloration in the periumbilical area.

Which is a priority nursing diagnosis for a patient receiving desmopressin (DDAVP)? A. Risk for injury B. Acute pain C. Excess fluid volume D. Deficient knowledge regarding medication

C Desmopressin is a form of antidiuretic hormone, which increases sodium and water retention, leading to an alteration in fluid volume. Although the other nursing diagnoses may be appropriate, they are not a priority using Maslow's hierarchy of needs.

A patient who has a positive test for human immunodeficiency virus (HIV) antibodies is admitted to the hospital with Pneumocystis jiroveci pneumonia (PCP) and a CD4+ T-cell count of less than 200 cells/L. Based on diagnostic criteria established by the Centers for Disease Control and Prevention (CDC), which statement by the nurse is correct? a. "The patient meets the criteria for a diagnosis of an acute HIV infection." b. "The patient will be diagnosed with asymptomatic chronic HIV infection." c. "The patient has developed acquired immunodeficiency syndrome (AIDS)." d. "The patient will develop symptomatic chronic HIV infection in less than a year."

C Development of PCP meets the diagnostic criterion for AIDS. The other responses indicate earlier stages of HIV infection than is indicated by the PCP infection.

When assessing a patient's abdomen, what would be most appropriate for the nurse to do? A) Palpate the abdomen before auscultation. B) Percuss the abdomen before auscultation. C) Auscultate the abdomen before palpation. D) Perform deep palpation before light palpation.

C During examination of the abdomen, auscultation is done before percussion and palpation because these latter procedures may alter the bowel sounds.

An ECG is prescribed for a client who reports chest pain. What early finding does the nurse expect on the lead over the infarcted area? A. Flattened T waves B. Absence of P waves C. Elevated ST segments D. Disappearance of Q waves

C Elevated ST segments are an early typical finding after a MI, because of altered contractility of the heart. Flattened or depressed T waves indicate hypokalemia. Absence of P waves occurs in atrial and ventricular fibrillation Q waves may become distorted with conduction or rhythm problems but they do not disappear unless cardiac standstill occurs.

Medical treatment of coronary artery disease includes which of the following procedures? A) Cardiac catherization B) Coronary artery bypass surgery C) Oral medication therapy D) Percutaneous transluminal coronary angioplasty

C Enhancing myocardial oxygenation is always the first priority when a client exhibits signs or symptoms of cardiac compromise. Without adequate oxygenation, the myocardium suffers damage. Sublingual nitroglycerin is administered to treat acute angina, but administration isn't the first priority. Although educating the client and decreasing anxiety are important in care delivery, neither are priorities when a client is compromised.3. Oral medication administration is a noninvasive, medical treatment for coronary artery disease. Cardiac catherization isn't a treatment, but a diagnostic tool. Coronary artery bypass surgery and percutaneous transluminal coronary angioplasty are invasive, surgical treatments.

A client receiving high-dose chemotherapy who has bone marrow suppression has been receiving daily injections of epoetin alfa (Procrit). Which assessment finding indicates to the nurse that today's dose should be held and the health care provider notified? A. Hematocrit of 28% B. Total white blood cell count of 6200 cells/mm3 C. Blood pressure change from 130/90 mm Hg to 148/98 mm Hg D. Temperature change from 99° F (37.2 C) to 100 F (37.8 C)

C Epoetin alfa and other erythropoiesis-stimulating agents (ESAs) such as darbepoetin alfa (Aranesp) and epoetin alfa (Epogen, Procrit) increase the production of many blood cell types, not just erythrocytes, which increases the client's risk for hypertension, blood clots, strokes, and heart attacks, especially among older adults. Dosing is based on individual client hemoglobin and hematocrit levels to ensure that just enough red blood cells are produced to avoid the need for transfusion but not to bring hemoglobin or hematocrit levels up to normal. The increased blood pressure is an indication to stop this therapy immediately.

The nurse is planning care for a patient who is chronically malnourished. Which action is appropriate for the nurse to delegate to unlicensed assistive personnel (UAP)? a. Assist the patient to choose high-nutrition items from the menu. b. Monitor the patient for skin breakdown over the bony prominences. c. Offer the patient the prescribed nutritional supplement between meals. d. Assess the patient's strength while ambulating the patient in the room.

C Feeding the patient and assisting with oral intake are included in UAP education and scope of practice. Assessing the patient and assisting the patient in choosing high-nutrition foods require licensed practical/vocational nurse (LPN/LVN)-or registered nurse (RN)-level education and scope of practice.

A nurse is providing education to a patient with chronic kidney disease who has a newly placed peritoneal dialysis catheter. The patient will be managing their dialysis at home. Which statement, if made by the patient, indicates that they understand the teaching? A) I will eat a low fiber, high protein diet. B) The fluid in the drainage bag should be clear and colorless, and equal to the amount of dialysis solution infused. C) I can slow the rate of the dialysis solution during infusion if I have pain or cramping. D) I should inspect the catheter site every few days for signs of infection, and I must wear a dressing over my catheter site at all times.

C Flow rate may be adjusted if cramping or pain is experienced during the inflow stage in order to decrease discomfort.

The nurse assesses a client who is receiving oxygen via a partial rebreather mask. Which assessment finding does the nurse intervene to correct? a. The bag is two thirds inflated during inhalation. b. The client's pulse oximetry reading is 93%. c. The oxygen flow rate is 2 L/min. d. The arterial oxygen level is 90%.

C Flow rate should be 6 to 11 L/min. A flow rate of 2 L/min will not adequately inflate the bag. A bag that is two thirds inflated is desired. A pulse oximetry reading of 93% and higher is adequate, as is an arterial oxygenation of 90%.

The nurse is evaluating the effectiveness of fluid resuscitation for a client in the resuscitation phase of burn injury. Which finding does the nurse correlate with clinical improvement? A. Blood urea nitrogen (BUN), 36 mg/dL B. Creatinine, 2.8 mg/dL C. Urine output, 40 mL/hr D. Urine specific gravity, 1.042

C Fluid resuscitation is provided at the rate needed to maintain urine output at 30 to 50 mL/hr or 0.5 mL/kg/hr. A BUN of 36 mg/dL is above normal, a creatinine of 2.8 mg/dL is above normal, and a urine specific gravity of 1.042 is above normal.

Marie, a 51-year-old woman, is diagnosed with cholecystitis. Which diet, when selected by the client, indicates that the nurse's teaching has been successful? A. 4-6 small meals of low-carbohydrate foods daily B. High-fat, high-carbohydrate meals C. Low-fat, high-carbohydrate meals D. High-fat, low protein meals

C For the client with cholecystitis, fat intake should be reduced. The calories from fat should be substituted with carbohydrates. Reducing carbohydrate intake would be contraindicated. Any diet high in fat may lead to another attack of cholecystitis.

The nurse instructs a 50-year-old woman about cholestyramine to reduce pruritis caused by gallbladder disease. Which statement by the patient to the nurse indicates she understands the instructions? A. "This medication will help me digest fats and fat-soluble vitamins." B. "I will apply the medicated lotion sparingly to the areas where I itch." C. "The medication is a powder and needs to be mixed with milk or juice." D. "I should take this medication on an empty stomach at the same time each day."

C For treatment of pruritus, cholestyramine may provide relief. This is a resin that binds bile salts in the intestine, increasing their excretion in the feces. Cholestyramine is in powder form and should be mixed with milk or juice before oral administration.

These data are obtained by the RN who is assessing a client who had a transsphenoidal hypophysectomy yesterday. What information has the most immediate implications for the client's care? a) Dry lips and oral mucosa on examination b) Nasal drainage that tests negative for glucose c) Client report of a headache and stiff neck d) Urine specific gravity of 1.016

C Headache and stiff neck (nuchal rigidity) are symptoms of meningitis that have immediate implications for the client's care. Dry lips and mouth are not unusual after surgery. Frequent oral rinses and the use of dental floss should be encouraged because the client cannot brush the teeth. Any nasal drainage should test negative for glucose; nasal drainage that tests positive for glucose indicates the presence of a cerebrospinal fluid leak. A urine specific gravity of 1.016 is within normal limits.

A 55-year-old patient with end-stage kidney disease (ESKD) is scheduled to receive a prescribed dose of epoetin alfa (Procrit). Which information should the nurse report to the health care provider before giving the medication? a. Creatinine 1.6 mg/dL b. Oxygen saturation 89% c. Hemoglobin level 13 g/dL d. Blood pressure 98/56 mm Hg

C High hemoglobin levels are associated with a higher rate of thromboembolic events and increased risk of death from serious cardiovascular events (heart attack, heart failure, stroke) when erythropoietin (EPO) is administered to a target hemoglobin of >12 g/dL. Hemoglobin levels higher than 12 g/dL indicate a need for a decrease in epoetin alfa dose. The other information also will be reported to the health care provider but will not affect whether the medication is administered

The client with respiratory failure has been intubated and placed on a ventilator and is requiring 100% oxygen delivery to maintain adequate oxygenation. Twenty-four hours later, the nurse notes new-onset crackles and decreased breath sounds, and the most recent ABGs show a PaO2 level of 95 mm Hg. The ventilator is not set to provide positive end-expiratory pressure (PEEP). Why is the nurse concerned? A. The low PaO2 level may result in oxygen toxicity B. The 100% oxygen delivery requirement indicates immediate extubation C. Lung sounds may indicate absorption atelectasis D. The level of oxygen delivery may indicate absorption atelectasis

C High levels of oxygen delivery can result in collapsed alveoli and absorption atelectasis. PEEP can help alveoli remain properly inflated.

A nurse plans care for a client with a growth hormone deficiency. Which action should the nurse include in this client's plan of care? a. Avoid intramuscular medications. b. Place the client in protective isolation. c. Use a lift sheet to re-position the client. d. Assist the client to dangle before rising.

C In adults, growth hormone is necessary to maintain bone density and strength. Adults with growth hormone deficiency have thin, fragile bones. Avoiding IM medications, using protective isolation & assisting the client as she moves from sitting to standing will not serve as safety measures when the client is deficient in growth hormone.

In assessing a client in the rehabilitative phase of burn therapy, which priority problem does the nurse anticipate? A. Intense pain B. Potential for inadequate oxygenation C. Reduced self-image D. Potential for infection

C In the rehabilitative phase of burn therapy, the client is discharged and his or her life is not the same. A priority problem of reduced self-image is expected. Intense pain and potential for inadequate oxygenation are relevant in the resuscitation phase of burn injury. Potential for infection is relevant in the acute phase of burn injury.

Which laboratory result indicates that fluid restrictions have been effective in treating syndrome of inappropriate antidiuretic hormone (SIADH)? a) Decreased hematocrit b) Decreased serum osmolality c) Increased serum sodium d) Increased urine specific gravity

C Increased serum sodium due to fluid restriction indicates effective therapy. Hemoconcentration is a result of hypovolemic hyponatremia caused by SIADH and diabetes insipidus. Plasma osmolality is decreased as a result of SIADH. Urine specific gravity is decreased with diabetes insipidus and is increased with SIADH.

Which clinical manifestation is indicative of wound healing for a client in the acute phase of burn injury? A. Pale, boggy, dry, or crusted granulation tissue B. Increasing wound drainage C. Scar tissue formation D. Sloughing of grafts

C Indicators of wound healing include the presence of granulation, re-epithelialization, and scar tissue formation. Pale, boggy, dry, or crusted granulation tissue is indicative of infection, as are increasing wound drainage and sloughing of grafts.

A patient with Graves' disease is treated with iodine-131 (Iodope) therapy. Which statement by the patient would indicate understanding of the treatment's effects? A) "I'll have to isolate myself from my family so I don't expose them to radiation." B) "I'm looking forward to feeling better immediately after this treatment." C) "I'll tell my doctor if I have fatigue, hair loss, or cold intolerance." D) "I'll need to take this drug on a daily basis for at least 1 year."

C Iodine-131 usually is given as a single treatment to produce remission of Graves' disease. Fatigue, hair loss, and cold intolerance are signs of hypothyroidism, which is a complication of the treatment. Iodine-131 has a quick radioactive decay and half-life; therefore, isolation is not needed, but it can take up to 2 months for the desired response to develop.

You're providing care to a patient with a pneumothorax who has a chest tube. On assessment of the chest tube system, you note there is no fluctuation of water in the water seal chamber as the patient inhales and exhales. You check the system for kinks and find none. What is your next nursing action? A. Keep monitoring the patient because this is a normal finding. B. Increase wall suction to the system until the water fluctuates in the water seal chamber. C. Assess patient's lung sounds to assess if the affected lung has re-expanded. D. Notify the physician.

C It is normal for the water seal chamber to tidal up and down as the patient breathes in and out. If there is no fluctuation of water in the water seal chamber there may be a kink in the tubing or the lung has re-expanded. Therefore, it is important to check the system for kinks and if there are none then check the patient's lung sounds to see if lung sounds are present on the affected side (hence the lung has re-expanded).

The charge nurse is making assignments for the next shift. Which patient should be assigned to the fairly new nurse (6 months of experience) floated from the surgical unit to the medical unit? A.) A 58-year-old patient on airborne precautions for tuberculosis (TB) B.) A 65-year-old patient who just returned from bronchoscopy and biopsy C.) A 72-year-old patient who needs teaching about the use of incentive spirometry D.)A 69-year-old patient with chronic obstructive pulmonary disease (COPD) who is ventilator dependent

C Many surgical patients are taught about coughing, deep breathing, and the use of incentive spirometry preoperatively. Also, a fairly new nurse should be assigned more stable and less complicated patients. To care for the patient with TB in isolation, the nurse must be fitted for a high-efficiency particulate air (HEPA) respirator mask. The bronchoscopy patient needs specialized and careful assessment and monitoring after the procedure, and the ventilator-dependent patient needs a nurse who is familiar with ventilator care. Both of these patients need experienced nurses.

A patient with AIDS is admitted to the hospital with AIDS-related wasting syndrome and AIDS-related anorexia. What drug has been found to promote significant weight gain in AIDS patients by increasing body fat stores? A) Advera B) Momordicacharantia C) Megestrol D) Ranitidine

C Megestrol acetate (Megace), a synthetic oral progesterone preparation, promotes significant weight gain. In patients with HIV infection, it increases body weight primarily by increasing body fat stores. Advera is a nutritional supplement that has been developed specifically for people with HIV infection and AIDS. Momordicacharantia (bitter melon) is given as an enema and is part of alternative treatment for HIV/AIDS. Ranitidine prevents ulcers.

A patient's capillary blood glucose level is 120 mg/dL 6 hours after the nurse initiated a parenteral nutrition (PN) infusion. The most appropriate action by the nurse is to a. obtain a venous blood glucose specimen. b. slow the infusion rate of the PN infusion. c. recheck the capillary blood glucose in 4 to 6 hours. d. notify the health care provider of the glucose level.

C Mild hyperglycemia is expected during the first few days after PN is started and requires ongoing monitoring. Because the glucose elevation is small and expected, notification of the health care provider is not necessary. There is no need to obtain a venous specimen for comparison. Slowing the rate of the infusion is beyond the nurse's scope of practice and will decrease the patient's nutritional intake.

A 48-year-old male with chronic kidney disease is starting peritoneal dialysis today with the help and supervision of a nurse with the plan being that he eventually will be able to do this in the comfort of his own home. The nurse knows the patient needs more education in regards to peritoneal dialysis with which statement? a. "If I notice cloudy drainage I should contact my provider as soon as possible." b. "When I am filling up my abdomen if I feel pain it's okay to slow down the infusion." c. "My dietary intake will need to change, I will need to be sure to increase my sodium intake while eating less protein since I am losing all my salt." d. "When I am draining the fluid out I want more to come out than what I put in."

C More education needs to be given to the patient because he should be increasing his protein intake. Peritoneal dialysis leads to protein loss due to the permeability of the membrane in the peritoneum so his diet should reflect that.

A nurse assesses a client who is recovering from a transsphenoidal hypophysectomy. The nurse notes nuchal rigidity. Which action should the nurse take first? a. Encourage range-of-motion exercises. b. Document the finding and monitor the client. c. Take vital signs, including temperature. d. Assess pain and administer pain medication.

C Nuchal rigidity is a major manifestation of meningitis, a potential postoperative complication associated with this surgery. Meningitis is an infection; usually the client will also have a fever and tachycardia. Range-of-motion exercises are inappropriate because meningitis is a possibility. Documentation should be done after all assessments are completed and should not be the only action. Although pain medication may be a palliative measure, it is not the most appropriate initial action.

A client who is scheduled to undergo radiation for prostate cancer is admitted to the hospital by the registered nurse. Which statement by the client is most important to communicate to the physician? A. "I am allergic to iodine." B. "My urinary stream is very weak." C. "My legs are numb and weak." D. "I am incontinent when I cough."

C Numbness and weakness should be reported to the physician because paralysis caused by spinal cord compression can occur.

The nurse admitting a patient with acromegaly anticipates administering which medication? A. Corticotropin (Acthar) B. Desmopressin (DDAVP) C. Octreotide (Sandostatin) D. Somatropin (Genotropin)

C Octreotide suppresses growth hormone, the culprit of acromegaly.

A patient begins hemodialysis after having had conservative management of chronic kidney disease. The nurse explains that one dietary regulation that will be changed when hemodialysis is started is that a. unlimited fluids are allowed since retained fluid is removed during dialysis. b. increased calories are needed because glucose is lost during hemodialysis. c. more protein will be allowed because of the removal of urea and creatinine by dialysis.d. dietary sodium and potassium are unrestricted because these levels are normalized by dialysis.

C Once the patient is started on dialysis and nitrogenous wastes are removed, more protein in the diet is allowed. Fluids are still restricted to avoid excessive weight gain and complications such as shortness of breath. Glucose is not lost during hemodialysis. Sodium and potassium intake continues to be restricted to avoid the complications associated with high levels of these electrolytes

Your patient with chronic obstructive pulmonary disease suddenly complains of sharp pain that began with a coughing fit. You know the doctor will require the following: a) Surgical consent for lobectomy b) Nothing- this is normal c) Chest tube set-up d) Ventilator set-up

C Patients with COPD and certain other chronic lung conditions are at a high risk for spontaneous pneumothorax. A chest tube would be needed to treat this condition.

What nursing action should the nurse prioritize during the care of a patient who has recently recovered from rheumatic fever? a. Teach the patient how to manage his or her physical activity. b. Teach the patient about the need for ongoing anticoagulation. c. Teach the patient about the need for continuous antibiotic prophylaxis. d. Teach the patient about the need to maintain standard infection control procedures.

C Patients with a history of rheumatic fever frequently require ongoing antibiotic prophylaxis, an intervention that necessitates education. This consideration is more important than activity management in preventing recurrence. Anticoagulation is not indicated in this patient population. Standard precautions are indicated for all patients.

The nurse palpates enlarged cervical lymph nodes on a patient diagnosed with acute human immunodeficiency virus (HIV) infection. Which action would be most appropriate for the nurse to take? a. Instruct the patient to apply ice to the neck. b. Advise the patient that this is probably the flu. c. Explain to the patient that this is an expected finding. d. Request that an antibiotic be prescribed for the patient.

C Persistent generalized lymphadenopathy is common in the early stages of HIV infection. No antibiotic is needed because the enlarged nodes are probably not caused by bacteria. Applying ice to the neck may provide comfort, but the initial action is to reassure the patient this is an expected finding. Lymphadenopathy is common with acute HIV infection and is therefore not likely the flu.

Which menu choice by the patient who is receiving hemodialysis indicates that the nurse's teaching has been successful? a. Split-pea soup, English muffin, and nonfat milk b. Oatmeal with cream, half a banana, and herbal tea c. Poached eggs, whole-wheat toast, and apple juice d. Cheese sandwich, tomato soup, and cranberry juice

C Poached eggs would provide high-quality protein, and apple juice is low in potassium. Cheese is high in salt and phosphate, and tomato soup would be high in potassium. Split-pea soup is high in potassium, and dairy products are high in phosphate. Bananas are high in potassium, and the cream would be high in phosphate

The nurse at the dialysis clinic notes when she reviews a client's labs that the labs indicated hyperkalemia. She makes a note to make sure the client is adhering to all dietary restrictions. Of the following possibilities, which might the nurse ask about? a. fiber supplements b. intake of whole grains c. salt substitutes d. sugar substitutes

C Potassium may be restricted in some clients because hyperkalemia tends to occur in end-stage renal disease. Excess potassium can cause cardiac arrest. Because of this danger, renal clients should not use salt substitutes or low-sodium milk because the sodium in these products is replaced with potassium.

A patient with Graves disease exhibits tachycardia, heat intolerance, and exophthalmos. Prior to surgery, which drug is used to alter thyroid hormone levels? a. Liotrix (Thyrolar) b. Propranolol (Inderal) c. Propylthiouracil (PTU) d. Thyroid (Thyrotab)

C Propylthiouracil is a potent antithyroid drug used in preparation for a subtotal thyroidectomy. Liotrix and thyroid are used as thyroid replacement. Propranolol is used to treat hypertension associated with hyperthyroidism

A 52-year-old client relates to the nurse that she has never had a mammogram because she is terrified that she will have cancer. Which response by the nurse is therapeutic? A. "Don't worry, most lumps are discovered by women during breast self-examination." B. "Does anyone in your family have breast cancer?" C. "Finding a cancer in the early stages increases the chance for cure." D. "Have you noticed a lump or thickening in your breast?"

C Providing truthful information addresses the client's concern.

A female client with cancer is scheduled for radiation therapy. The nurse knows that radiation at any treatment site may cause a certain adverse effect. Therefore, the nurse should prepare the client to expect: a. hair loss. b. stomatitis. c. fatigue. d. vomiting.

C Radiation therapy may cause fatigue, skin toxicities, and anorexia regardless of the treatment site. Hair loss, stomatitis, and vomiting are site-specific, not generalized, adverse effects of radiation therapy.

Which precaution is most important for the nurse to teach a client receiving radiation therapy for head and neck cancer? A. Avoid eating red meat during treatment. B. Pace your leisure activities to prevent fatigue. C. See your dentist twice yearly for the rest of your life. D. Avoid using headphones or headsets until your hair grows back.

C Radiation therapy that is directed in or around the oral cavity has a variety of actions that increase the risk for dental caries (cavities) and tooth decay. The salivary glands are affected, which changes the composition of the person's saliva and often causes "dry mouth." This result allows rapid bacterial overgrowth, which leads to cavity formation. In addition, the radiation damages the integrity of the enamel and also damages some of the living cells in the tooth. All contribute to an increased risk for dental infections and cavities.

The nurse is caring for a group of patients. Which patient is at highest risk for pancreatic cancer? A. A 38-year-old Hispanic female who is obese and has hyperinsulinemia B. A 23-year-old who has cystic fibrosis-related pancreatic enzyme insufficiency C. A 72-year-old African American male who has smoked cigarettes for 50 years D. A 19-year-old who has a 5-year history of uncontrolled type 1 diabetes mellitus

C Risk factors for pancreatic cancer include chronic pancreatitis, diabetes mellitus, age, cigarette smoking, family history of pancreatic cancer, high-fat diet, and exposure to chemicals such as benzidine. African Americans have a higher incidence of pancreatic cancer than whites. The most firmly established environmental risk factor is cigarette smoking. Smokers are two or three times more likely to develop pancreatic cancer as compared with nonsmokers. The risk is related to duration and number of cigarettes smoked.

In the patient using Controlled Mandatory Ventilation, what drug are they on to suppress respiratory effort? A. Morphine, Fentanyl, or Hydromorphone B. Propofol C. Rocuronium, Pancuronium, or Nimbex D. Formoterol, Salmeterol, or Arformoterol

C Rocuronium, Pancuronium, & Nimbex are all non-Depolarizing Neuromuscular Blocking Agents. But, be considerate that the patient recieving this type of ventilation will also require sedation and opioids.

A client has surgery for a perforated appendix with localized peritonis. In which position should the nurse place the client? A) Sims position B) trendelenburg C) semi-fowlers D)dorsal recumbant

C Semi-fowlers aids in drainage and prevents spread of infection throughout the abodominal cavity.

Which intervention will be most helpful in preventing disseminated intravascular coagulation (DIC)? A. Monitoring platelets B. Administering packed red blood cells C. Using strict aseptic technique to prevent infection D. Administering low-dose heparin therapy for clients on bedrest

C Sepsis is a major cause of DIC, especially in the oncology client. Heparin may be administered to clients with DIC who have developed clotting, but this has not been proven to prevent the disorder.

The nurse care plan for a patient with AIDS includes the diagnosis of Risk for Impaired Skin Integrity. What nursing intervention should be included in the plan of care? A) Maximize the patient's fluid intake. B) Provide total parenteral nutrition (TPN). C) Keep the patient's bed linens free of wrinkles. D) Provide the patient with snug clothing at all times.

C Skin surfaces are protected from friction and rubbing by keeping bed linens free of wrinkles and avoiding tight or restrictive clothing. Fluid intake should be adequate, and must be monitored, but maximizing fluid intake is not a goal. TPN is a nutritional intervention of last resort.

Which strategy does the nurse include when teaching a college student about fire prevention in the dormitory room? A. Use space heaters to reduce electrical costs. B. Check water temperature before bathing. C. Do not smoke in bed. D. Wear sunscreen.

C Smoking in bed increases the risk for fire because the person could fall asleep. Use of space heaters may increase the risk for fire, especially if they are knocked over and left unattended. Checking water temperature does not prevent fires, but it should be checked if the client has reduced sensation in the hands or feet. Sunscreen is advised to prevent sunburn.

IV sodium nitroprusside (Nipride) is ordered for a patient with acute pulmonary edema. During the first hours of administration, the nurse will need to titrate the nitroprusside rate if the patient develops a.ventricular ectopy. b.a dry, hacking cough. c.a systolic BP <90 mm Hg. d.a heart rate <50 beats/minute.

C Sodium nitroprusside is a potent vasodilator, and the major adverse effect is severe hypotension. Coughing and bradycardia are not adverse effects of this medication. Nitroprusside does not cause increased ventricular ectopy.

The nurse is providing discharge instructions to a client on spironolactone (Aldactone) therapy. Which comment by the client indicates a need for further teaching? a) "I must call the provider if I am more tired than usual." b) "I need to increase my salt intake." c) "I should eat a banana every day." d) "This drug will not control my heart rate."

C Spironolactone increases potassium levels, so potassium supplements and foods rich in potassium, such as bananas, should be avoided to prevent hyperkalemia. While taking spironolactone, symptoms of hyponatremia such as drowsiness and lethargy must be reported; the client may need increased dietary sodium. Spironolactone will not have an effect on the client's heart rate.

The nurse is instructing an unlicensed health care worker on the care of the client with HIV who also has active genital herpes. Which statement by the health care worker indicates effective teaching of standard precautions? A) ''I need to know my HIV status, so I must get tested before caring for any clients." B) ''Putting on a gown and gloves will cover up the itchy sores on my elbows.'' C) ''Washing my hands and putting on a gown and gloves is what I must do before starting care.'' D) ''I will wash my hands before going into the room, and then put on gown and gloves only for direct contact with the client's genitals."

C Standard precautions include whatever personal protective equipment (PPE) is necessary for the prevention of transmission of HIV and genital herpes.

The registered nurse would correct the nursing student when caring for a client with neutropenia secondary to chemotherapy in which circumstance? A. Student scrubs the hub of IV tubing before administering an antibiotic. B. Nurse overhears the student explaining to the client the importance of handwashing. C. Student teaches the client that symptoms of neutropenia include fatigue and weakness. D. The nurse observes the student providing oral hygiene and perineal care.

C Symptoms of neutropenia include low neutrophil count, fever, and signs and symptoms of infection; the student should be corrected.

Six weeks after hematopoietic stem cell transplantation for leukemia, the client's white blood cell (WBC) count is 8200/mm3. What is the nurse's best action in view of this laboratory result? A. Notify the health care provider immediately. B. Assess the client for other symptoms of infection. C. Document the laboratory report as the only action. D. Remind the client to avoid crowds and people who are ill.

C The WBC count is now within the normal range (5000 to 10,000/mm3) and is an indicator of successful engraftment. The client is not at any particular risk for infection at this time, nor is there reason to believe an infection is present. (At any post-transplantation check-up, the client is assessed for infection.)

A patient was tested for HIV using enzyme immunoassay (EIA) and results were positive. The nurse should expect the primary care provider to order what test to confirm the EIA test results? A) Another EIA test B) Viral load test C) Western blot test D) CD4/CD8 ratio

C The Western blot test detects antibodies to HIV and is used to confirm the EIA test results. The viral load test measures HIV RNA in the plasma and is not used to confirm EIA test results, but instead to track the progression of the disease process. The CD4/CD8 ratio test evaluates the ratio of CD4 and CD8 cells but is not used to confirm results of EIA testing.

The client with chronic renal failure returns to the nursing unit following a hemodialysis treatment. On assessment, the nurse notes that the client's temperature is 100.2F. Which of the following is the appropriate nursing action? a. Encourage fluids. b. Notify the physician. c. Continue to monitor vital signs. d. Monitor the site of the shunt for infection.

C The client may have an elevated temperature following dialysis because the dialysis machine warms the blood slightly. If the temperature is elevated excessively and remains elevated, sepsis would be suspected and a blood sample would be obtained as prescribed for culture and sensitivity determinations.

Metabolic and nutritional needs of the patient with increased ICP are best met with a. enteral feedings that are low in sodium b. the simple glucose available in D5W IV solutions c. a fluid restriction that promotes a moderate dehydration d. balanced, essential nutrition in a form that the patient can tolerate

D

New bubbling is observed in the water seal chamber after a patient with a pleural chest tube returns from a test. The nurse clamps the chest tube momentarily with a tubing clamp at the dressing site. When this is done, bubbling in the water seal stops. The next appropriate nursing action is to: A. Continue to monitor the water seal chamber for bubbling every hour for the next four hours B. Do nothing. This bubbling is normal in patients with pleural chest tubes C. Call the physician immediately and do not leave the patient's bedside because of the risk of respiratory failure D. Remove the chest tube dressing to see if one or more eyelets of the chest tube have been pulled out of the chest

D

A client with Cushing's disease begins to laugh loudly and inappropriately, causing the family in the room to be uncomfortable. What is the nurse's best response? a) "Don't mind this. The disease is causing this." b) "I need to check the client's cortisol level." c) "The disease can sometimes affect emotional responses." d) "Medication is available to help with this."

C The client may have neurotic or psychotic behavior as a result of high blood cortisol levels. Being honest with the family helps them to understand what is happening. Telling the family not to mind the laughter and that the disease is causing it is vague and minimizes the family's concern. This is the perfect opportunity for the nurse to educate the family about the disease. Cushing's disease is the hypersecretion of cortisol, which is abnormally elevated in this disease and, because the diagnosis has already been made, blood levels do not need to be redrawn. Telling the family that medication is available to help with inappropriate laughing does not assist them in understanding the cause of or the reason for the client's behavior.

Which statement by a client indicates an accurate understanding of home self-care of a tracheostomy? a. "The stoma should be left uncovered during the day to dry." b. "I need to put normal saline in my airway twice daily." c. "While showering, I need to keep water out of my airway." d. "I don't need to use tracheostomy ties on a daily basis."

C The client should put a shield over the tracheostomy to keep water from entering the airway. The airway should remain covered during the day with cotton or foam. Saline should be put in the airway 10 to 15 times daily. Tracheostomy ties should be used daily.

Twenty-four hours after endoscopic retrograde cholangiopancreatography (ERCP), a client develops left upper quadrant abdominal pain and has a temperature of 101° F (38.3° C). What is the nurse's best action? A. Administer acetaminophen for control of fever and pain. B. Document the finding, because it is a normal postprocedure event. C. Notify the health care provider. D. Increase the IV fluid rate.

C The client who has undergone an ERCP may develop complications such as perforation or sepsis manifested by fever and abdominal pain. The nurse should report these symptoms to the health care provider immediately.

A client with burn injuries is being admitted. Which priority does the nurse anticipate within the first 24 hours? A. Range-of-motion exercises B. Emotional support C. Fluid resuscitation D. Sterile dressing changes

C The client will require fluid resuscitation because fluid does not stay in the vessels after a burn injury. Range-of-motion exercise is not the priority for this client. Although emotional support and sterile dressing changes are important, they are not the priority during the resuscitation phase of burn injury.

A client diagnosed with chronic renal failure (CRF) is scheduled to begin hemodialysis. The nurse assesses that which of the following neurological and psychosocial manifestations if exhibited by this client would be unrelated to the CRF? a. Labile emotions. b. Withdrawal. c. Euphoria. d. Depression.

C The client with CRF often experiences a variety of psychosocial changes. These are related to uremia, as well as the stress associated with living with a chronic disease that is life-threatening. Clients with CRF may have labile emotions or personality changes and may exhibit withdrawal, depression, or agitation. Delusions and psychosis also can occur. Euphoria is not part of the clinical picture for the client in renal failure.

Which assessment is the nurse's highest priority in caring for a client in the acute phase of burn injury? A. Bowel sounds B. Muscle strength C. Signs of infection D. Urine output

C The client with burn injury is at risk for infection as a result of open wounds and reduced immune function. Burn wound sepsis is a serious complication of burn injury, and infection is the leading cause of death during the acute phase of recovery. Assessing bowel sounds, assessing muscle strength, and assessing urine output are important but not the priority during the acute phase of burn injury.

A patient with acute kidney injury (AKI) has longer QRS intervals on the electrocardiogram (ECG) than were noted on the previous shift. Which action should the nurse take first? a. Notify the patient's health care provider. b. Document the QRS interval measurement. c. Check the medical record for most recent potassium level. d. Check the chart for the patient's current creatinine level.

C The increasing QRS interval is suggestive of hyperkalemia, so the nurse should check the most recent potassium and then notify the patient's health care provider. The BUN and creatinine will be elevated in a patient with AKI, but they would not directly affect the electrocardiogram (ECG). Documentation of the QRS interval is also appropriate, but interventions to decrease the potassium level are needed to prevent life-threatening dysrhythmias

The student nurse diligently assesses her patient with a chest tube. She notices that the suction control chamber of the chest tube is not bubbling. What is the first thing this student should do? A) Document this normal finding B) Encourage the patient to cough and deep breathe C) Check the level of the suction on the wall D) Clamp the chest tube and call for help

C The level of suction is controlled by the amount of water in the suction control chamber. However, it would be prudent of the student nurse to check and see if the suction is even turned on. This portion of the chest tube should be gently bubbling, indicating the system is working. Coughing and deep breathing would not help turn the suction on. The student should never clamp the chest tube.

A nurse is taking pulmonary artery catheter measurements of a male client with acute respiratory distress syndrome. The pulmonary capillary wedge pressure reading is 12mm Hg. The nurse interprets that this readings is: A. High and expected B. Low and unexpected C. Normal and expected D. Uncertain and unexpected

C The normal pulmonary capillary wedge pressure (PCWP) is 8 to 13 mm Hg, and the client is considered to have high readings if they exceed 18 to 20 mm Hg. The client with acute respiratory distress syndrome has a normal PCWP, which is an expected finding because the edema is in the interstitium of the lung and is noncardiac.

Skull radiographs and a computed tomography (CT) scan provide evidence of a depressed parietal fracture with a subdural hematoma in a patient admitted to the emergency department following an automobile accident. In planning care for the patient, the nurse anticipates that a. the patient will receive life-support measures until the condition stabilizes b. immediate burr holes will be made to rapidly decompress the intracranial activity c. the patient will be treated conservatively with close monitoring for changes in neurologic condition d. the patient will be taken to surgery for a craniotomy for evacuation of blood and decompression of the cranium

D

The ED nurse has inspected, auscultated, and palpated the abdomen with no obvious abnormalities, except pain. When the nurse palpates the abdomen for rebound tenderness, there is severe pain. The nurse should know that this could indicate what problem? A) Hepatic cirrhosis B) Hypersplenomegaly C) Gall bladder distention D) Peritoneal inflammation

D

The client newly diagnosed with CKD recently has begun HD. Knowing that the client is at risk for disequilibrium syndrome, the nurse should assess the client during dialysis for which associated manifestations? a. hypertension, tachycardia, and fever b. hypotension, bradycardia, and hypothermia c. restlessness, irritability, and generalized weakness d. headache, deteriorating LOC, and twitching

D

A patient with a recent diagnosis of HIV infection expresses an interest in exploring alternative and complementary therapies. How should the nurse best respond? A) "Complementary therapies generally have not been approved, so patients are usually discouraged from using them." B) "Researchers have not looked at the benefits of alternative therapy for patients with HIV, so we suggest that you stay away from these therapies until there is solid research data available." C) "Many patients with HIV use some type of alternative therapy and, as with most health treatments, there are benefits and risks." D) "You'll need to meet with your doctor to choose between an alternative approach to treatment and a medical approach."

C The nurse should approach the topic of alternative or complementary therapies from an open-ended, supportive approach, emphasizing the need to communicate with care providers. Complementary therapies and medical treatment are not mutually exclusive, though some contraindications exist. Research supports the efficacy of some forms of complementary and alternative treatment

The client with a new tracheostomy has a soiled dressing. What is the best nursing intervention? A. Cut sterile 4 x 4 gauze to fit around the tracheostomy tube B. Reinforce the dressing with sterile 4 x 4 gauze C. Replace the dressing with clean, folded 4 x 4 gauze D. Replace the dressing with sterile, folded 4 x 4 gauze

D

The patient has an open pneumothorax. Which of the following options below is a nursing intervention you would provide to this patient? A. Place the patient in supine position B. Place a non-occlusive dressing over the chest wound C. Place a sterile occlusive dressing over the chest wound and tape it on three sides D. Prepare the patient for a thoracentesis

C The nursing intervention would be to place a sterile occlusive dressing over the wound and tape it on 3 sides (leaving one side NOT taped). This will allow exhaled air to leave the opening but seal over the opening when inhaling (hence not letting pressure build in the intrapleural space and prevent a tension pneumothorax).

To prepare a 56-year-old male patient with ascites for paracentesis, the nurse a. places the patient on NPO status. b. assists the patient to lie flat in bed. c. asks the patient to empty the bladder. d. positions the patient on the right side.

C The patient should empty the bladder to decrease the risk of bladder perforation during the procedure

A patient with acute renal failure (ARF) requires hemodialysis and temporary vascular access is obtained by placing a catheter in the left femoral vein. The nurse will plan to a. restrict the patient's oral protein intake. b. discontinue the retention catheter. c. place the patient on bed rest. d. start continuous pulse oximetry.

C The patient with a femoral vein catheter must be on bed rest to prevent trauma to the vein. Protein intake is likely to be increased when the patient is receiving dialysis. The retention catheter is likely to remain in place because accurate measurement of output will be needed. There is no indication that the patient needs continuous pulse oximetry.

Which statement by a 62-year-old patient with stage 5 chronic kidney disease (CKD) indicates that the nurse's teaching about management of CKD has been effective? a. "I need to get most of my protein from low-fat dairy products." b. "I will increase my intake of fruits and vegetables to 5 per day." c. "I will measure my urinary output each day to help calculate the amount I can drink." d. "I need to take erythropoietin to boost my immune system and help prevent infection."

C The patient with end-stage kidney disease is taught to measure urine output as a means of determining an appropriate oral fluid intake. Erythropoietin is given to increase the red blood cell count and will not offer any benefit for immune function. Dairy products are restricted because of the high phosphate level. Many fruits and vegetables are high in potassium and should be restricted in the patient with CKD.

After the insertion of an arteriovenous graft (AVG) in the right forearm, a 54-year-old patient complains of pain and coldness of the right fingers. Which action should the nurse take? a. Teach the patient about normal AVG function. b. Remind the patient to take a daily low-dose aspirin tablet. c. Report the patient's symptoms to the health care provider. d. Elevate the patient's arm on pillows to above the heart level.

C The patient's complaints suggest the development of distal ischemia (steal syndrome) and may require revision of the AVG. Elevation of the arm above the heart will further decrease perfusion. Pain and coolness are not normal after AVG insertion. Aspirin therapy is not used to maintain grafts

The nurse obtains the vital signs for a patient admitted 2 days ago with gram-negative sepsis: temperature 101.2° F, blood pressure 90/56 mm Hg, pulse 92, respirations 34. Which action should the nurse take next? a. Administer the scheduled IV antibiotic. b. Give the PRN acetaminophen (Tylenol) 650 mg. c. Obtain oxygen saturation using pulse oximetry. d. Notify the health care provider of the patient's vital signs.

C The patient's increased respiratory rate in combination with the admission diagnosis of gram-negative sepsis indicates that acute respiratory distress syndrome (ARDS) may be developing. The nurse should check for hypoxemia, a hallmark of ARDS. The health care provider should be notified after further assessment of the patient. Administration of the scheduled antibiotic and administration of Tylenol also will be done, but they are not the highest priority for a patient who may be developing ARDS.

A client with renal failure is undergoing continuous ambulatory peritoneal dialysis. Which nursing diagnosis is the most appropriate for this client? a) Impaired urinary elimination b) Toileting self-care deficit c) Risk for infection d) Activity intolerance

C The peritoneal dialysis catheter and regular exchanges of the dialysis bag provide a direct portal for bacteria to enter the body. If the client experiences repeated peritoneal infections, continuous ambulatory peritoneal dialysis may no longer be effective in clearing waste products. Impaired urinary elimination, Toileting self-care deficit, and Activity intolerance may be pertinent but are secondary to the risk of infection.

The client with chronic renal failure who is scheduled for hemodialysis this morning is due to receive a daily dose of enalapril (Vasotec). The nurse should plan to administer this medication: a. during dialysis b. just before dialysis c. the day after dialysis d. on return form dialysis

D

The decision has been made to discharge a ventilator-dependent patient home. The nurse is developing a teaching plan for this patient and his family. What would be most important to include in this teaching plan? A) Administration of inhaled corticosteroids B) Assessment of neurologic status C) Turning and coughing D) Signs of pulmonary infection

D

The home care nurse is visiting a patient newly discharged home after a lobectomy. What would be most important for the home care nurse to assess? A) Resumption of the patients ADLs B) The familys willingness to care for the patient C) Nutritional status and fluid balance D) Signs and symptoms of respiratory complications

D

The nurse is caring for a client with hypercortisolism. The nurse begins to feel the onset of a cold but still has 4 hours left in the shift. What does the nurse do? a) Asks another nurse to care for the client b) Monitors the client for cold-like symptoms c) Refuses to care for the client d) Wears a facemask when caring for the client

D A client with hypercortisolism will be immune-suppressed. Anyone with a suspected upper respiratory infection who must enter the client's room must wear a mask to prevent the spread of infection. Although asking another nurse to care for the client might be an option in some facilities, it is not generally realistic or practical. The nurse, not the client, feels the onset of the cold, so monitoring the client for cold-like symptoms is part of good client care for a client with hypercortisolism. Refusing to care for the client after starting care would be considered abandonment.

A client is becoming frustrated because of an inability to communicate with a tracheostomy. Which intervention by the nurse most effectively enhances communication? a. Explain to the client that speech will be clear and distinct with a fenestrated tube. b. Reassure the client that in time he or she will get used to the speech difficulties. c. Place a sign above the client's bed indicating that the client cannot speak. d. Provide the client with a communication board and call light within easy reach.

D A communication board and the call light will reassure the client that needs will be communicated and met. It is doubtful that the client with a tracheostomy will ever speak clearly and distinctly, no matter what type of tube he or she uses. Reassuring the client that he or she will get used to the speech difficulties does nothing to alleviate the discomfort and fear associated with impaired communication. Placing a sign above the client's bed indicating that he cannot speak will not enhance his ability to communicate, although it may help staff remember that the client has impaired communication.

The nurse assesses a client during suctioning. Which finding indicates that the procedure should be stopped? a. Heart rate increases from 86 to 102 beats/min. b. Respiratory rate increases from 16 to 20 breaths/min. c. Blood pressure increases from 110/70 to 120/80 mm Hg. d. Heart rate decreases from 78 to 40 beats/min.

D A decrease in heart rate indicates that the client is not tolerating the procedure, and the vasovagal reflex may be stimulated. An increase in heart rate may be stimulated by suctioning and is expected, as is a slight increase in blood pressure. A slight increase in respiratory rate after the procedure might be caused by the feeling of oxygen being suctioned from the client's airway, along with secretions.

Which client being cared for on the medical-surgical unit will be best to assign to a nurse who has floated from the intensive care unit (ICU)? A. Recent radical mastectomy client who requires chemotherapy administration B. Modified radical mastectomy client who needs discharge teaching C. Stage III breast cancer client who is requesting information about radiation and chemotherapy D. A client with a Jackson-Pratt drain in place who has just arrived from the postanesthesia care unit (PACU) after a quadrantectomy

D A nurse working in the ICU would be familiar with postoperative monitoring and care of clients with Jackson-Pratt drains.

The decision to begin antiretroviral therapy is based on: A. The CD4 cell count B. The plasma viral load C. The intensity of the patient's clinical symptoms D. All of the above

D A person's CD4 count is an important factor in the decision to start ART. A low or falling CD4 count indicates that HIV is advancing and damaging the immune system. A rapidly decreasing CD4 count increases the urgency to start ART. Regardless of CD4 count, there is greater urgency to start ART when a person has a high viral load or any of the following conditions: pregnancy, AIDS, and certain HIV-related illnesses and co infections.

Which client requires immediate nursing intervention? The client who: a) complains of epigastric pain after eating. b) complains of anorexia and periumbilical pain. c) presents with ribbonlike stools. d) presents with a rigid, boardlike abdomen.

D A rigid, boardlike abdomen is a sign of peritonitis, a possibly life-threatening condition. Epigastric pain occurring 90 minutes to 3 hours after eating indicates a duodenal ulcer. Anorexia and periumbilical pain are characteristic of appendicitis. Risk of rupture is minimal within the first 24 hours, but increases significantly after 48 hours. A client with a large-bowel obstruction may have ribbonlike stools.

A client who has experienced a panic attack is being transferred to the medical-surgical ward. The transfer nurse reports that the client is doing much better after receiving bronchodilators via nebulizer and a small dose of oral Valium 4 hours ago in the emergency department. Vital signs are stable with oxygen delivered at 4 L/min via simple facemask. Why is this client at high risk for subsequent respiratory distress? A. The client is not being treated for asthma B. The client has a mental disorder C. The client received a dose of Valium D. The client is receiving oxygen at 4 L/min

D A simple facemask must receive oxygen at a rate of at least 5 L/min to prevent inhalation of exhaled breath, which has low levels of oxygen and can eventually suffocate the client.

A patient has been diagnosed with AIDS complicated by chronic diarrhea. What nursing intervention would be appropriate for this patient? A) Position the patient in the high Fowler's position whenever possible. B) Temporarily eliminate animal protein from the patient's diet. C) Make sure the patient eats at least two servings of raw fruit each day. D) Obtain a stool culture to identify possible pathogens.

D A stool culture should be obtained to determine the possible presence of microorganisms that cause diarrhea. Patients should generally avoid raw fruit when having diarrhea. There is no need to avoid animal protein or increase the height of the patient's bed.

Which of the following EKG changes on the patient with chest pain indicates the highest level of damage? A) T wave inversion B) T wave enlargement C) ST segment elevation D) Q wave widening

D Abnormal Q waves represent myocardial necrosis and can develop with 1-3 days of an MI. T wave inversion is often the first symptom of ischemia of the heart and may be seen with angina. T wave enlargement and ST segment elevation are often seen with injury to the heart.

Which clinical manifestation would the nurse expect a client diagnosed with acute cholecystitis to exhibit? A. Jaundice, dark urine, and steatorrhea B. Acute right lower quadrant (RLQ) pain, diarrhea, and dehydration C. Ecchymosis petechiae, and coffee-ground emesis D. Nausea, vomiting, and anorexia

D Acute cholecystitis is an acute inflammation of the gallbladder commonly manifested by the following: anorexia, nausea, and vomiting; biliary colic; tenderness and rigidity the right upper quadrant (RUQ) elicited on palpation (e.g., Murphy's sign); fever; fat intolerance; and signs and symptoms of jaundice. Ecchymosis, petechiae, and coffee-ground emesis are clinical manifestations of esophageal bleeding. The coffee-ground appearance indicates old bleeding. Jaundice, dark urine, and steatorrhea are clinical manifestations of the icteric phase of hepatitis.

In preparing a client for a trans-esophageal echocardiogram (TEE), the nurse should include which of the following in the client education? a. "You will be able to eat only soft foods for the first day after the procedure." b. "The procedure involves a series of x-rays that may require you to come back." c. "The procedure involves a balloon that will press plaque against the blocked walls of your coronary artery." d. "You will need a designated driver to take you home."

D After a trans-esophageal echocardiogram (TEE), the client should be able to eat lukewarm food as soon as the gag reflex returns, usually just a few hours after receiving anesthetizing spray in the back of the throat. X-rays are not involved, nor is the client undergoing a balloon procedure such as the coronary angioplasty. Having a designated driver will be needed after a trans-esophageal echocardiogram because the client has received IV sedation.

A nurse is caring a client who disconnected the tubing of the parenteral nutrition from the central line catheter. A nurse suspects an occurrence of an air embolism. Which of the following is an appropriate position for the client in this kind of situation? A. On the right side, with head higher than the feet. B. On the right side, with head lower than the feet. C. On the left side, with the head higher than the feet. D. On the left side, with head lower than the feet.

D Air embolism happens when air enters the catheter system when the IV tubing disconnects. If it is suspected, the client should be placed in a left-side-lying position. The head should be lower than the feet. This position will lessen the effect of the air traveling as a bolus to the lungs by trapping it on the right side of the heart.

In the preoperative holding area, the client who is scheduled to have an adrenalectomy for hypercortisolism is prescribed to receive cortisol by intravenous infusion. What is the nurse's best action? a. Request a "time-out" to determine whether this is a valid prescription. b. Ask the client whether he or she usually takes prednisone. c. Hold the dose because the client has a high cortisol level. d. Administer the drug as prescribed.

D Although the client has hypercortisolism, removal of the adrenal gland will stop the secretion of this important hormone that is essential for life. Further, the stress of surgery also increases the client's need for this hormone. Supplying the hormone throughout surgery prevents the complication (or at least reduces the risk for) acute adrenal crisis.

When delegating care for clients on the burn unit, which client does the charge nurse assign to an RN who has floated to the burn unit from the intensive care unit? A. Burn unit client who is being discharged after 6 weeks and needs teaching about wound care B. Recently admitted client with a high-voltage electrical burn C. A client who has a 25% total body surface area (TBSA) burn injury, for whom daily wound débridement has been prescribed D. Client receiving IV lactated Ringer's solution at 150 mL/hr

D An RN float nurse will be familiar with administration of IV fluids and with signs of fluid overload, such as shortness of breath, and so could be assigned to the client receiving IV lactated Ringer's solution at 150 mL/hr. The client needing teaching about wound care, the client with a high-voltage electrical burn, and the client with a 25% TBSA burn injury all require specialized knowledge about burn injuries and should be assigned to RNs who have experience caring for clients with burn injuries.

The charge nurse is making client assignments for the medical-surgical unit. Which client will be best to assign to an RN who has floated from the pediatric unit? a) Client in Addisonian crisis who is receiving IV hydrocortisone b) Client admitted with syndrome of inappropriate antidiuretic hormone (SIADH) secondary to lung cancer c) Client being discharged after a unilateral adrenalectomy to remove an adrenal tumor d) Client with Cushing's syndrome who has elevated blood glucose and requires frequent administration of insulin

D An RN who works with pediatric clients would be familiar with glucose monitoring and insulin administration. A client in Addisonian crisis would best be monitored by an RN from the medical-surgical floor. Although the float RN could complete the admission history, the client with SIADH secondary to lung cancer might require teaching and orientation to the unit that a nurse more familiar with that area would be better able to provide. Discharge teaching specific to adrenalectomy should be provided by the RN who is regularly assigned to the medical-surgical floor and is more familiar with taking care of postoperative adult clients with endocrine disorders.

The client with an external arteriovenous shunt in place for hemodialysis is at risk for bleeding. The priority nurse action would be to: a. Check the shunt for the presence of bruit and thrill. b. Observe the site once as time permits during the shift. c. Check the results of the prothrombin times as they are determined. d. Ensure that small clamps are attached to the arteriovenous shunt dressing.

D An arteriovenous shunt is a less common form of access site but carries a risk for bleeding when it is used because two ends of an external cannula are tunneled subcutaneously into an artery and a vein, and the ends of the cannula are joined. If accidental disconnection occurs, the client could lose blood rapidly. For this reason, small clamps are attached to the dressing that covers the insertion site for use if needed. The shunt site also should be assessed at least every 4 hours

You are the nursing instructor and you are taking your students to a unit where chest tubes are often in use. Which statement, if made by your students, is correct? A) "If a clot has formed in the tubing, it can be gently milked by fully completely compressing the tubing and milking it into the drainage container" B) "I should loop my patients tubing in order to keep it off of the floor" C) "Because my patient has a tube draining air out of their pleural space, it will not be necessary to have them use their incentive spirometer" D) "There can be an occasional bubble form in the water seal chamber of the chest tube"

D An occasional bubble formed in the water seal chamber indicates that air is being released from the pleural spaces. Gentle milking of the tube may be permitted, but the tube should never be fully compressed to do it. Looping or kinking of the tube may cause a backward pressure that could impede drainage or force air back into the pleural spaces. Incentive spirometer use will help improve lung expansion.

The client with chronic renal failure is scheduled for hemodialysis this morning is due to receive a daily dose of enalapril (Vasotec). The nurse should plan to administer this medication: a. During dialysis. b. Just before dialysis. c. The day after dialysis. d. On return from dialysis.

D Antihypertensive medications such as enalapril are given to the client following hemodialysis. This prevents the client from becoming hypotensive during dialysis and also from having the medication removed from the bloodstream by dialysis. No rationale exists for waiting an entire day to resume the medication. This would lead to ineffective control of the blood pressure.

The nurse observes a nursing student suctioning a client. Which intervention by the student nurse requires the supervising nurse to intervene? a. Checking oxygen saturation post suctioning b. Hyperoxygenating the client after removal of the catheter c. Applying intermittent suction during catheter removal d. Applying suction when the catheter is inserted

D Applying suction as the catheter is introduced allows the tubing to adhere to the airway and destroys cells. The other options are appropriate actions on the part of a nurse or student who is suctioning a client.

The nurse is assigned to care for a client immediately after breast-conserving surgery for cancer. What is the priority for care of the client at this time? A. Teach the client to sleep in the prone position each night. B. Empty wound drains and record the output amount. C. Remind the client how to perform breast self-examination. D. Monitor the incision and flap for adequate tissue perfusion.

D Assess the incision and flap for duskiness and decreased capillary refill during dressing changes, which are signs of poor tissue perfusion. The client should avoid sleeping in the prone position. Emptying drains, documenting output, and encouraging BSE are important but are not the priority in the immediate postoperative phase.

When analyzing the rhythm of a patient's electrocardiogram (ECG), the nurse will need to investigate further upon finding a(n) a.isoelectric ST segment. b.P-R interval of 0.18 second. c.Q-T interval of 0.38 second. d.QRS interval of 0.14 second.

D Because the normal QRS interval is 0.04 to 0.10 seconds, the patient's QRS interval of 0.14 seconds indicates that the conduction through the ventricular conduction system is prolonged. The P-R interval and Q-T interval are within normal range, and ST segment should be isoelectric (flat).

While suctioning a client who had a tracheostomy placed 4 days ago, the nurse notes particles of food in the tracheal secretions. Which action by the nurse is most appropriate? a. Increase the inflation pressure in the tracheostomy cuff. b. Add blue dye to a beverage to assess for aspiration. c. Make the client NPO and notify the health care provider. d. Perform a more thorough assessment of the client.

D Before calling the provider, the nurse needs more data, such as lung sounds, presence of cough, pulse oximetry reading, and possibly mental status. The nurse could temporarily make the client NPO while conducting this assessment, but calling the provider must wait until he or she has more complete data. The nurse should not decide to increase the inflation pressure in the tracheostomy cuff on his or her own. Adding dye to food, drink, or tube feeding formulas was commonly done in the past but should be avoided because the dye is toxic to lung tissues if aspirated.

A client with an exacerbation of ulcerative colitis has been placed on total parenteral nutrition (TPN). The client asks why nutrition is being supplied in this manner and not by mouth. What is the nurse's best response? A. "TPN contains a high percentage of glucose that is more readily absorbed into the bloodstream than into the ulcerated colon." B. "TPN will be given in addition to your meals to help you gain any weight that you may have lost through diarrhea." C. "TPN is considered an elemental formula and, as such, is easier to digest." D. "TPN will be given during this period to allow your bowel to rest."

D Bowel rest during severe exacerbations of ulcerative colitis is part of the nonsurgical management of the disease.

Which of the following is an expected outcome for a client on the second day of hospitalization after an MI? A) Has severe chest pain B) Can identify risks factors for MI C) Agrees to participate in a cardiac rehabilitation walking program D) Can perform personal self-care activities without pain

D By day 2 of hospitalization after an MI, clients are expected to be able to perform personal care without chest pain. Day 2 hospitalization may be too soon for clients to be able to identify risk factors for MI or begin a walking program; however, the client may be sitting up in a chair as part of the cardiac rehabilitation program. Severe chest pain should not be present.

Which statement by the nurse when explaining the purpose of positive end-expiratory pressure (PEEP) to the family members of a patient with ARDS is correct? a. "PEEP will prevent fibrosis of the lung from occurring." b. "PEEP will push more air into the lungs during inhalation." c. "PEEP allows the ventilator to deliver 100% oxygen to the lungs." d. "PEEP prevents the lung air sacs from collapsing during exhalation."

D By preventing alveolar collapse during expiration, PEEP improves gas exchange and oxygenation. PEEP will not prevent the fibrotic changes that occur with ARDS, push more air into the lungs, or change the fraction of inspired oxygen (FIO2) delivered to the patient.

In teaching clients at risk for bradydysrhythmias, what information does the nurse include? a. "Avoid potassium-containing foods." b. "Stop smoking and avoid caffeine." c. "Take nitroglycerin for a slow heartbeat." d. "Use a stool softener."

D Clients at risk for bradydysrhythmias should avoid bearing down or straining during a bowel movement; the Valsalva maneuver can cause bradycardia. Taking a stool softener helps to prevent this.

The nurse is assessing a dialysis patient who is asking to receive continuous ambulatory peritoneal dialysis (CAPD) instead of hemodialysis. Which of the following complications of CAPD will the nurse review with the client? a. hypercalcemia b. hypertension c. hyponatremia d. hypotension

D Clients on CAPD have a more normal lifestyle than do clients on either hemodialysis or peritoneal dialysis. Complications associated with CAPD include peritonitis, hypotension, and weight gain.

A 68 year-old client is admitted for mitral valve replacement surgery. The client has a history of mitral valve regurgitation and mitral stenosis since her teenage years. During the admission assessment, the nurse should ask the client if as a child she had? a. Encephalitis b. Hay fever c. Measles d. Rheumatic fever

D Clients that present with mitral stenosis have a history of rheumatic fever or bacterial endocarditis.

A client has been placed on 6 L of humidified oxygen via nasal cannula. Which action by the nurse is most appropriate? a. Drain condensation back into the humidifier, maintaining a closed system. b. Keep the water sterile by draining it from the water trap back into the humidifier. c. Turn down the humidity when condensation begins to collect in the tubing. d. Remove condensation in the tubing by disconnecting and emptying it appropriately.

D Condensation often forms in the tubing when a client receives humidified high-flow oxygen. Remove this condensation as it collects by disconnecting the tubing and emptying the water. Some humidifiers and nebulizers have a water trap that hangs from the tubing so the condensation can be drained without disconnecting. To prevent bacterial contamination, never drain the fluid back into the humidifier or the nebulizer. Do not turn down the humidity because the physician has ordered it and the client needs it. Minimize how long the tubing is disconnected because the client does not receive oxygen during this period.

The nurse sees the level of water in the water seal chamber rising very high. The nurse correlates which patient behavior with this rise? A) The patient is eating his lunch B) The patient is resting on his side C) The patient is squeezing the tubing D) The patient is coughing viciously.

D Coughing, sneezing or other forces can cause an increase in negative pressure which will in turn cause an increase in the water in the water seal chamber. Eating or resting should not affect the negative pressure in the tube. Squeezing, kinking, or somehow cutting off the flow into the chest tube would increase positive pressure, not negative.

How does the drug desmopressin (DDAVP) decrease urine output in a client with diabetes insipidus (DI)? a) Blocks reabsorption of sodium b) Increases blood pressure c) Increases cardiac output d) Works as an antidiuretic hormone (ADH) in the kidneys

D Desmopressin is a synthetic form of ADH that binds to kidney receptors and enhances reabsorption of water, thus reducing urine output. Desmopressin does not have any effect on sodium reabsorption. It may cause a slight increase or a transient decrease in blood pressure, but this does not affect urine output. Desmopressin does not increase cardiac output.

Which of the following clients is at the highest risk for experiencing an MI? a) The 71 year old African American female who smoked 10 years ago b) The 57 year old Caucasian male whose HDL is 30 and triglyceride level is 160 c) The 60 year old Native American male who has had an MI in the past d) The 65 year old Hispanic male with type II diabetes and peripheral arterial disease

D Diabetes and PAD are considered CAD risk equivalents, meaning that they place the client at the same risk of experiencing a cardiac event as someone who has already had an MI. Considering this patients age, race, and two CAD risk equivalents, the nurse knows that this patient is at the highest risk for experiencing an MI.

A client with atrial fibrillation is receiving digoxin (Lanoxin). It is MOST important for the nurse to? a. Record an EKG strip after administration b. Monitor blood pressure every 4 hours c. Maintain accurate intake and output records d. Measure apical pulse prior to administration

D Digitoxin decreases conduction velocity through the AV node and prolongs the refractory period. If the apical heart rate is less than 60 beats/minute, withhold the drug. The apical pulse should be taken with a stethoscope so that there will be no mistake about what the heart rate actually is.

The client newly diagnosed with chronic renal failure recently has begun hemodialysis. Knowing that the client is at risk for disequilibrium syndrome, the nurse assesses the client during dialysis for: a. Hypertension, tachycardia, and fever. b. Hypotension, bradycardia, and hypothermia. c. Restlessness, irritability, and generalized weakness. d. Headache, deteriorating level of consciousness, and twitching.

D Disequilibrium syndrome is characterized by headache, mental confusion, decreasing level of consciousness, nausea, vomiting, twitching, and possible seizure activity. Disequilibrium syndrome is caused by rapid removal of solutes from the body during hemodialysis. At the same time, the blood-brain barrier interferes with the efficient removal of wastes from brain tissue. As a result, water goes into cerebral cells because of the osmotic gradient, causing brain swelling and onset of symptoms. The syndrome most often occurs in clients who are new to dialysis and is prevented by dialyzing for shorter times or at reduced blood flow rates.

A client with a possible adrenal gland tumor is admitted for testing and treatment. Which nursing action is most appropriate for the charge nurse to delegate to the nursing assistant? a) Assess skin turgor and mucous membranes for hydration status. b) Discuss the dietary restrictions needed for 24-hour urine testing. c) Plan ways to control the environment that will avoid stimulating the client. d) Remind the client to avoid drinking coffee and changing position suddenly.

D Drinking caffeinated beverages and changing position suddenly are not safe for a client with a potential adrenal gland tumor because of the effects of catecholamines. Reminding the client about previous instructions is an appropriate role for a nursing assistant who may observe the client doing potentially risky activities. Client assessment, client teaching, and environment planning are higher-level skills that require the experience and responsibility of the RN, and are not within the scope of practice of the nursing assistant.

A patient who is human immunodeficiency virus (HIV)-infected has a CD4+ cell count of 400/µL. Which factor is most important for the nurse to determine before the initiation of antiretroviral therapy (ART) for this patient? a. HIV genotype and phenotype b. Patient's social support system c. Potential medication side effects d. Patient's ability to comply with ART schedule

D Drug resistance develops quickly unless the patient takes ART medications on a strict, regular schedule. In addition, drug resistance endangers both the patient and the community. The other information is also important to consider, but patients who are unable to manage and follow a complex drug treatment regimen should not be considered for ART

When the nurse is monitoring a patient who is undergoing exercise (stress) testing on a treadmill, which assessment finding requires the most rapid action by the nurse? a.Patient complaint of feeling tired b.Pulse change from 87 to 101 beats/minute c.Blood pressure (BP) increase from 134/68 to 150/80 mm Hg d.Newly inverted T waves on the electrocardiogram

D ECG changes associated with coronary ischemia (such as T-wave inversions and ST segment depression) indicate that the myocardium is not getting adequate oxygen delivery and that the exercise test should be terminated immediately. Increases in BP and heart rate (HR) are normal responses to aerobic exercise. Feeling tired is also normal as the intensity of exercise increases during the stress testing.

A client is admitted to the ER with crushing chest pain. A diagnosis of acute coronary syndrome is suspected. The nurse expects that the client's initial treatment will include which medication? A. Gabapentin B. Midazolam C. Alprazolam D. Aspirin

D Early administration of aspirin in the setting of acute MI has been demonstrated to significantly reduce mortality. Aspirin inhibits the action of platelets, preventing their ability to clump together and form clots. The mechanism of acute coronary syndrome usually is ruptured plaque in one of the coronary arteries with clot formation obstructing blood flow. Prompt administration of an antiplatelet agent, such as aspirin, can be lifesaving.

A patient has a normal cardiac rhythm and a heart rate of 72 beats/minute. The nurse determines that the P-R interval is 0.24 seconds. The most appropriate intervention by the nurse would be to a.notify the health care provider immediately. b.give atropine per agency dysrhythmia protocol. c.prepare the patient for temporary pacemaker insertion. d.document the finding and continue to monitor the patient.

D First-degree atrioventricular (AV) block is asymptomatic and requires ongoing monitoring because it may progress to more serious forms of heart block. The rate is normal, so there is no indication that atropine is needed. Immediate notification of the health care provider about an asymptomatic rhythm is not necessary.

A client with Cushing's disease says that she has lost 1 pound. What does the nurse do next? a) Auscultates the lungs for crackles b) Checks urine for specific gravity c) Forces fluids d) Weighs the client

D Fluid retention with weight gain is more of a problem than weight loss in clients with Cushing's disease. Weighing the client with Cushing's disease is part of the nurse's assessment. Crackles in the lungs indicate possible fluid retention, which would cause weight gain, not weight loss. Urine specific gravity will help assess hydration status, but this would not be the next step in the client's assessment. Forcing fluids is not appropriate because usually excess water and sodium reabsorption cause fluid retention in the client with Cushing's disease.

A 42-year-old woman is admitted to the outpatient testing area for an ultrasound of the gallbladder. Which information obtained by the nurse indicates that the ultrasound may need to be rescheduled? a. The patient took a laxative the previous evening. b. The patient had a high-fat meal the previous evening. c. The patient has a permanent gastrostomy tube in place. d. The patient ate a low-fat bagel 4 hours ago for breakfast.

D Food intake can cause the gallbladder to contract and result in a suboptimal study. The patient should be NPO for 8 to 12 hours before the test. A high-fat meal the previous evening, laxative use, or a gastrostomy tube will not affect the results of the study.

A 52-year-old man was referred to the clinic due to increased abdominal girth. He is diagnosed with ascites by the presence of a fluid thrill and shifting dullness on percussion. After administering diuretic therapy, which nursing action would be most effective in ensuring safe care? A. Measuring serum potassium for hyperkalemia B. Assessing the client for hypervolemia C. Measuring the client's weight weekly D. Documenting precise intake and output

D For the client with ascites receiving diuretic therapy, careful intake and output measurement is essential for safe diuretic therapy. Diuretics lead to fluid losses, which if not monitored closely and documented, could place the client at risk for serious fluid and electrolyte imbalances. Hypokalemia, not hyperkalemia, commonly occurs with diuretic therapy. Because urine output increases, a client should be assessed for hypovolemia, not hypervolemia. Weights are also an accurate indicator of fluid balance. However, for this client, weights should be obtained daily, not weekly.

A nurse is analyzing the posthemodialysis lab test results for a client with chronic renal failure (CRF). The nurse interprets that the dialysis is having an expected but nontherapeutic effect if the results indicate a decreased: a. Phosphorus. b. Creatinine. c. Potassium. d. Red blood cell count

D Hemodialysis typically lowers the amounts of fluid, sodium, potassium, urea nitrogen, creatinine, uric acid, magnesium, and phosphate levels in the blood. Hemodialysis also worsens anemia, because RBCs are lost in dialysis from blood sampling and anticoagulation during the procedure, and from residual blood that is left in the dialyzer. Although all of these results are expected, only the lowered RBC count is nontherapeutic and worsens the anemia already caused by the disease process.

While caring for the patient in the oliguric phase of AKI, the nurse monitors the patient for associated collaborative problems. When should the nurse notify the health care provider? a. Urine output is 300 mL/day. b. Edema occurs in the feet, legs, and sacral area. c. Cardiac monitor reveals a depressed T wave and elevated ST segment. d. The patient experiences increasing muscle weakness and abdominal cramping.

D Hyperkalemia is a potentially life-threateningcomplication of AKI in the oliguric phase. Muscle weakness and abdominal cramping are signs of the neuromuscular impairment that occurs with hyperkalemia. In addition, hyperkalemia can cause the cardiac conduction abnormalities of peaked T wave, prolonged PR interval, prolonged QRS interval, and depressed ST segment. Urine output of 300 mL/day is expected during the oliguric phase, as is the development of peripheral edema.

The nurse receives the following information about a 51-year-old woman who is scheduled for a colonoscopy. Which information should be communicated to the health care provider before sending the patient for the procedure? a. The patient has a permanent pacemaker to prevent bradycardia. b. The patient is worried about discomfort during the examination. c. The patient has had an allergic reaction to shellfish and iodine in the past. d. The patient refused to drink the ordered polyethylene glycol (GoLYTELY).

D If the patient has had inadequate bowel preparation, the colon cannot be visualized and the procedure should be rescheduled. Because contrast solution is not used during colonoscopy, the iodine allergy is not pertinent. A pacemaker is a contraindication to magnetic resonance imaging (MRI), but not to colonoscopy. The nurse should instruct the patient about the sedation used during the examination to decrease the patient's anxiety about discomfort

A nurse assessing a patient's respiratory effort notes that the client's breaths are shallow and 8 per minute. Shortly after, the client's respirations cease. Which of the following should the nurse use for this patient? a) Oxygen tent b) Oxygen mask c) Nasal cannula d) Ambu bag

D If the patient is not breathing with an adequate rate and depth, or if the patient has lost the respiratory drive, a manual rescucitation bag (Ambu bag)may be used to deliver oxygen until the patient is resuscitated or can be intubated with an endotracheal tube.

What indicates to the nurse that a patient with oliguria has prerenal oliguria? a. Urine testing reveals a low specific gravity. b. Causative factor is malignant hypertension. c. Urine testing reveals a high sodium concentration. d. Reversal of oliguria occurs with fluid replacement.

D In prerenal oliguria, the oliguria is caused by a decreasein circulating blood volume and there is no damageyet to the renal tissue. It can be reversed by correcting the precipitating factor, such as fluid replacement for hypovolemia. Prerenal oliguria is characterized by urine with a high specific gravity and a low sodium concentration, whereas oliguria of intrarenal failure is characterized by urine with a low specific gravity and a high sodium concentration. Malignant hypertension causes damage to renal tissue and intrarenal oliguria.

What accurately describes the care of the patient with CKD? a. A nutrient that is commonly supplemented for the patient on dialysis because it is dialyzable is iron. b. The syndrome that includes all of the signs and symptoms seen in the various body systems in CKD is azotemia. c. The use of morphine is contraindicated in the patient with CKD because accumulation of its metabolites maycause seizures. d. The use of calcium-based phosphate binders in the patient with CKD is contraindicated when serum calcium levels are increased.

D In the patient with CKD, when serum calcium levels are increased, calcium-based phosphate binders are not used. The nutrient supplemented for patients on dialysis is folic acid. The various body system manifestations occur with uremia, which includes azotemia. Meperidine is contraindicated in patients with CKD related to possible seizures.

A 70-year-old man who has end-stage lung cancer is admitted to the hospital with confusion and oliguria for 2 days. Which finding would the nurse report immediately to the health care provider? A. Weight gain of 2 lb D. Urine specific gravity of 1.015 C. Blood urea nitrogen of 20 mg/dL D. Serum sodium level of 118 mEq/L

D Lung cancer cells are able to manufacture and release antidiuretic hormone (ADH) with resultant water retention and hyponatremia. Hyponatremia (serum sodium levels less than 135 mEq/L) may lead to central nervous system symptoms such as confusion, seizures, coma, and death. A weight gain may be due to fluid retention. The urine specific gravity and blood urea nitrogen are normal.

A patient's current antiretroviral regimen includes nucleoside reverse transcriptase inhibitors (NRTIs). What dietary counseling will the nurse provide based on the patient's medication regimen? A) Avoid high-fat meals while taking this medication. B) Limit fluid intake to 2 liters a day. C) Limit sodium intake to 2 grams per day. D) Take this medication without regard to meals.

D Many NRTIs exist, but all of them may be safely taken without regard to meals. Protein, fluid, and sodium restrictions play no role in relation to these drugs.

In caring for the patient with AKI, what should the nurse be aware of? a. The most common cause of death in AKI is irreversible metabolic acidosis. b. During the oliguric phase of AKI, daily fluid intake is limited to 1000 mL plus the prior day's measured fluid loss. c. Dietary sodium and potassium during the oliguric phase of AKI are managed according to the patient's urinaryoutput. d. One of the most important nursing measures in managing fluid balance in the patient with AKI is taking accuratedaily weights.

D Measuring daily weights with the same scale at the same time each day allows for the evaluation and detection of excessive body fluid gains or losses. Infection is the leading cause of death in AKI, so meticulous aseptic technique is critical. The fluid limitation in the oliguric phase is 600 mL plus the prior day's measured fluid loss. Dietary sodium and potassium intake are managed according to the plasma levels.

An 18-year-old pregnant female has tested positive for HIV and asks the nurse if her baby is going to be born with HIV. What is the nurse's best response? A) "There is no way to know that for certain, but we do know that your baby has a one in four chance of being born with HIV." B) "Your physician is likely the best one to ask that question." C) "If the baby is HIV positive there is nothing that can be done until it is born, so try your best not to worry about it now." D) "It's possible that your baby could contract HIV, either before, during, or after delivery."

D Mother-to-child transmission of HIV-1 is possible and may occur in utero, at the time of delivery, or through breast-feeding. There is no evidence that the infant's risk is 25%. Deferral to the physician is not a substitute for responding appropriately to the patient's concern. Downplaying the patient's concerns is inappropriate.

The patient with pericarditis is complaining of chest pain. After assessment, which intervention should the nurse expect to implement to provide pain relief? a. Corticosteroids b. Morphine sulfate c. Proton pump inhibitor d. Nonsteroidal antiinflammatory drugs

D Nonsteroidal antiinflammatory drugs (NSAIDs) will control pain and inflammation. Corticosteroids are reserved for patients already taking corticosteroids for autoimmune conditions or those who do not respond to NSAIDs. Morphine is not necessary. Proton pump inhibitors are used to decrease stomach acid to avoid the risk of GI bleeding from the NSAIDs.

A nurse assesses a client's electrocardiograph tracing and observes that not all QRS complexes are preceded by a P wave. How should the nurse interpret this observation? a. The client has hyperkalemia causing irregular QRS complexes. b. Ventricular tachycardia is overriding the normal atrial rhythm. c. The client's chest leads are not making sufficient contact with the skin. d. Ventricular and atrial depolarizations are initiated from different sites.

D Normal rhythm shows one P wave preceding each QRS complex, indicating that all depolarization is initiated at the sinoatrial node. QRS complexes without a P wave indicate a different source of initiation of depolarization. This finding on an electrocardiograph tracing is not an indication of hyperkalemia, ventricular tachycardia, or disconnection of leads.

A client is in the acute phase of burn injury. For which action does the nurse decide to coordinate with the registered dietitian? A. Discouraging having food brought in from the client's favorite restaurant B. Providing more palatable choices for the client C. Helping the client lose weight D. Planning additions to the standard nutritional pattern

D Nutritional requirements for the client with a large burn area can exceed 5000 kcal/day. In addition to a high calorie intake, the burn client requires a diet high in protein for wound healing. Consultation with the dietitian is required to help the client achieve the correct nutritional balance. It is fine for the client with a burn injury to have food brought in from the outside. The hospital kitchen can be consulted to see what other food options may be available to the client. It is not therapeutic for the client with burn injury to lose weight.

A 16-year-old has come to the clinic and asks to talk to a nurse. The nurse asks the teen what she needs and the teen responds that she has become sexually active and is concerned about getting HIV. The teen asks the nurse what she can do keep from getting HIV. What would be the nurse's best response? A) "There's no way to be sure you won't get HIV except to use condoms correctly." B) "Only the correct use of a female condom protects against the transmission of HIV." C) "There are new ways of protecting yourself from HIV that are being discovered every day." D) "Other than abstinence, only the consistent and correct use of condoms is effective in preventing HIV."

D Other than abstinence, consistent and correct use of condoms is the only effective method to decrease the risk of sexual transmission of HIV infection. Both female and male condoms confer significant protection. New prevention techniques are not commonly discovered, though advances in treatment are constant.

A client is scheduled for a Percutaneous Transluminal Coronary Angioplasty (PTCA). The nurse knows that a PTCA is the? a. Surgical repair of a diseased coronary artery b. Placement of an automatic internal cardiac defibrillator c. Non-invasive radiographic examination of the heart d. Procedure that compresses plaque against the wall of the diseased coronary artery to improve blood flow

D PTCA is performed to improve coronary artery blood flow in a diseased artery. It is performed during a cardiac catheterization. Aorta Coronary Bypass Graft is the surgical procedure to repair a diseased coronary artery.

Bobby, a 13 year old is being seen in the emergency room for possible appendicitis. An important nursing action to perform when preparing Bobby for an appendectomy is to:" a) administer saline enemas to cleanse the bowels b) apply heat to reduce pain c) measure abdominal girth d) continuously monitor pain

D Pain is closely monitored in appendicitis. In most cases, pain medication is not given until prior to surgery or until the diagnosis is confirmed to be able to closely monitor the progression of the disease. A sudden change in the character of pain may indicate rupture or bowel perforation. Administering an enema or applying heat may cause perforation and abdominal girth may not change with appendicitis.

A patient receiving parenteral nutrition is administered via the following routes except: A. Subclavian line. B. Central Venous Catheter. C. PICC (Peripherally inserted central catheter) line. D. PEG tube.

D Percutaneous endoscopic gastrostomy (PEG tube) is inserted into a person's stomach through the abdominal wall that is used to provide a means of feeding when oral intake is not adequate. While Parenteral nutrition bypasses the digestive system by the administration to the bloodstream

A patient has come into the free clinic asking to be tested for HIV infection. The patient asks the nurse how the test works. The nurse responds that if the testing shows that antibodies to the AIDS virus are present in the blood, this indicates what? A) The patient is immune to HIV. B) The patient's immune system is intact. C) The patient has AIDS-related complications. D) The patient has been infected with HIV.

D Positive test results indicate that antibodies to the AIDS virus are present in the blood. The presence of antibodies does not imply an intact immune system or specific immunity to HIV. This finding does not indicate the presence of AIDS-related complications.

A 51-year-old woman had an incisional cholecystectomy 6 hours ago. The nurse will place the highest priority on assisting the patient to a. choose low-fat foods from the menu. b. perform leg exercises hourly while awake. c. ambulate the evening of the operative day. d. turn, cough, and deep breathe every 2 hours.

D Postoperative nursing care after a cholecystectomy focuses on prevention of respiratory complications because the surgical incision is high in the abdomen and impairs coughing and deep breathing. The other nursing actions are also important to implement but are not as high a priority as ensuring adequate ventilation.

A patient has come into the ED with a hemothorax and has had a chest tube inserted 2 hours ago. Which of the following would be most concerning if observed by the nurse? A) Tidaling in the water seal of the chest tube with a popping sensation in the skin around the chest tube B) The patient is complaining of pain 8/10 and is taking shallow breaths with a RR of 27 C) There is intermittent bubbling in the water seal of the chest tube with 200 ml of bright red drainage D) The patient begins to pick at his IV lines and tries to get out of bed and is sweating profusely

D Restlessness and Diaphoresis symptoms of hypoxemia and possible development of ARF. This requires immediate intervention. Tidaling in the water seal portion of the chest tube and subcutaneous emphysema are normal/benign findings and should be documented. Severe pain and elevated RR would be expected in this patient, but should be monitored for worsening severity. While 200 ml of bright red drainage would be expected after immediate insertion of the chest tube, intermittent bubbling would NOT be expected in the case of hemothorax. This indicates and air leak and should be investigated, but is not the most concerning in this situation.

A nurse cares for a client after a pituitary gland stimulation test using insulin. The client's post-stimulation laboratory results indicate elevated levels of growth hormone (GH) and adrenocorticotropic hormone (ACTH). How should the nurse interpret these results? a. Pituitary hypofunction b. Pituitary hyperfunction c. Pituitary-induced diabetes mellitus d. Normal pituitary response to insulin

D Some tests for pituitary function involve administering agents that are known to stimulate the secretion of specific pituitary hormones and then measuring the response. Such tests are termed stimulation tests. The stimulation test for GH or ACTH assessment involves injecting the client with regular insulin (0.05 to 1 unit/kg of body weight) and checking circulating levels of GH and ACTH. The presence of insulin in clients with normal pituitary function causes increased release of GH and ACTH.

Respirations of the sedated client with a new tracheostomy have become noisy, and the ventilator alarms indicate high peak pressures. The ventilator tube is clear. What is the best immediate action by the nurse? A. Humidifying the oxygen source B. Increasing oxygenation C. Removing the inner cannula of the tracheostomy D. Suctioning the client

D Suctioning the client will likely result in clear lung sounds and lower peak pressure, and the appearance of the sputum will indicate whether bleeding is a concern.

A client who is receiving total enteral nutrition (TEN) exhibits acute confusion and shallow breathing and says, "I feel weak." As the client begins to have a generalized seizure, how does the nurse interpret this client's signs and symptoms? A. The enteral tube is misplaced or dislodged. B. Abdominal distention is present. C. A fluid and electrolyte imbalance is present. D. This is refeeding syndrome.

D Symptoms of refeeding syndrome include shallow respirations, weakness, acute confusion, seizures, and increased bleeding tendency.

Which of the following tests is used most often to diagnose angina? A) Chest x-ray B) Echocardiogram C) Cardiac catherization D) 12-lead electrocardiogram (ECG)

D The 12-lead ECG will indicate ischemia, showing T-wave inversion. In addition, with variant angina, the ECG shows ST-segment elevation. A chest x-ray will show heart enlargement or signs of heart failure, but isn't used to diagnose angina.

The nurse obtains a rhythm strip on a patient who has had a myocardial infarction and makes the following analysis: no visible P waves, P-R interval not measurable, ventricular rate 162, R-R interval regular, and QRS complex wide and distorted, QRS duration 0.18 second. The nurse interprets the patient's cardiac rhythm as a.atrial flutter. b.sinus tachycardia. c.ventricular fibrillation. d.ventricular tachycardia.

D The absence of P waves, wide QRS, rate >150 beats/minute, and the regularity of the rhythm indicate ventricular tachycardia. Atrial flutter is usually regular, has a narrow QRS configuration, and has flutter waves present representing atrial activity. Sinus tachycardia has P waves. Ventricular fibrillation is irregular and does not have a consistent QRS duration.

Which nursing action will be most useful in assisting a college student to adhere to a newly prescribed antiretroviral therapy (ART) regimen? a. Give the patient detailed information about possible medication side effects. b. Remind the patient of the importance of taking the medications as scheduled. c. Encourage the patient to join a support group for students who are HIV positive. d. Check the patient's class schedule to help decide when the drugs should be taken.

D The best approach to improve adherence is to learn about important activities in the patient's life and adjust the ART around those activities. The other actions also are useful, but they will not improve adherence as much as individualizing the ART to the patient's schedule.

A patient who was admitted with a myocardial infarction experiences a 45-second episode of ventricular tachycardia, then converts to sinus rhythm with a heart rate of 98 beats/minute. Which of the following actions should the nurse take next? a.Immediately notify the health care provider. b.Document the rhythm and continue to monitor the patient. c.Perform synchronized cardioversion per agency dysrhythmia protocol. d.Prepare to give IV amiodarone (Cordarone) per agency dysrhythmia protocol.

D The burst of sustained ventricular tachycardia indicates that the patient has significant ventricular irritability, and antidysrhythmic medication administration is needed to prevent further episodes. The nurse should notify the health care provider after the medication is started. Defibrillation is not indicated given that the patient is currently in a sinus rhythm. Documentation and continued monitoring are not adequate responses to this situation.

A client with acquired immunodeficiency syndrome (AIDS) has a nursing diagnosis of Imbalanced nutrition: less than body requirements. The nurse plans which of the following goals with this client? a) consume foods and beverages that are high in glucose b) plan large menus and cook meals in advance c) eat low-calorie snacks between meals d) eat small, frequent meals throughout the day

D The client should eat small, frequent meals throughout the day. The client also should take in nutrient-dense and high-calorie meals and snacks rather than those that are high in glucose only. The client is encouraged to eat favorite foods to keep intake up and plan meals that are easy to prepare. The client can also avoid taking fluids with meals to increase food intake before satiety sets in.

The nurse is addressing condom use in the context of a health promotion workshop. When discussing the correct use of condoms, what should the nurse tell the attendees? A) Attach the condom prior to erection. B) A condom may be reused with the same partner if ejaculation has not occurred. C) Use skin lotion as a lubricant if alternatives are unavailable. D) Hold the condom by the cuff upon withdrawal.

D The condom should be unrolled over the hard penis before any kind of sex. The condom should be held by the tip to squeeze out air. Skin lotions, baby oil, petroleum jelly, or cold cream should not be used with condoms because they cause latex deterioration/condom breakage. The condom should be held during withdrawal so it does not come off the penis. Condoms should never be reused.

A nurse is assessing a female client with multiple trauma who is at risk for developing acute respiratory distress syndrome. The nurse assesses for which earliest sign of acute respiratory distress syndrome? A. Bilateral wheezing B. Inspiratory Crackles C. Intercostal retractions D. Increased respiratory rate

D The earliest detectable sign of acute respiratory distress syndrome is an increased respiratory rate, which can begin from 1 to 96 hours after the initial insult to the body. This is followed by increasing dyspnea, air hunger, retraction of accessory muscles, and cyanosis. Breath sounds may be clear or consist of fine inspiratory crackles or diffuse coarse crackles.

The nurse is teaching a family member how to suction the client's tracheostomy at home. Which information does the nurse include in the teaching plan? a. Always suction using sterile technique. b. Suction the mouth first and then the airway. c. Be prepared to recannulate the tube frequently. d. Suctioning with clean technique is acceptable.

D The family member can suction using clean technique because fewer organisms are present in the home than in the hospital. Never suction the mouth first because airway pathogenic organisms could be introduced into the airway. The family member should not be required to recannulate the tube except in an emergency.

Which of the following should the nurse include in the preoperative teaching for a client scheduled for coronary artery bypass graft (CABG) surgery? a. Coughing is to be avoided in order to protect the sternal incision b. The hospital stay is generally about 10 days c. A liquid diet will be ordered for the first 4 to 5 days postoperatively d. High-calorie supplements are encouraged in the first few weeks postoperative

D The first meal after coronary artery bypass graft surgery may be clear liquids, but the client quickly progresses to a low-fat and low-salt diet as soon as it can be tolerated. Coughing is important to clear the airways, and is done by splinting the sternum with a pillow. Hospital stays are generally 4 to 5 days. A poor appetite may be present for the first few weeks and clients are encouraged to try high-calorie supplements.

A nurse is completing the admission assessment of a client who has acute pancreatitis. Which of the following findings is the priority to be reported to the provider? A. History of cholelithiasis B. Serum amylase levels three times greater than the expected value C. Client report of severe pain radiating to the back that is rated at an "8" D. Hand spasms present when blood pressure is checked

D The greatest risk to the client is hypocalcemia due to the risk of cardiac dysrhythmia. Hand spasms when taking a blood pressure is an indication of hypocalcemia and is the priority finding to report to the provider

After vertical banded gastroplasty, a 42-year-old male patient returns to the surgical nursing unit with a nasogastric tube to low, intermittent suction and a patient-controlled analgesia (PCA) machine for pain control. Which nursing action should be included in the postoperative plan of care? a. Offer sips of fruit juices at frequent intervals. b. Irrigate the nasogastric (NG) tube frequently. c. Remind the patient that PCA use may slow the return of bowel function. d. Support the surgical incision during patient coughing and turning in bed.

D The incision should be protected from strain to decrease the risk for wound dehiscence. The patient should be encouraged to use the PCA because pain control will improve the cough effort and patient mobility. NG irrigation may damage the suture line or overfill the stomach pouch. Sugar-free clear liquids are offered during the immediate postoperative time to decrease the risk for dumping syndrome.

The client is a burn victim who is noted to have increasing edema and decreased urine output as a result of the inflammatory compensation response. What does the nurse do first? A. Administer a diuretic. B. Provide a fluid bolus. C. Recalculate fluid replacement based on time of hospital arrival. D. Titrate fluid replacement.

D The intravenous fluid rate should be adjusted on the basis of urine output plus serum electrolyte values (titration of fluids). A common mistake in treatment is giving diuretics to increase urine output. Giving a diuretic will actually decrease circulating volume and cardiac output by pulling fluid from the circulating blood volume to enhance diuresis. Fluid boluses are avoided because they increase capillary pressure and worsen edema. Fluid replacement formulas are calculated from the time of injury, not from the time of arrival at the hospital.

The nurse is reviewing the laboratory test results for a client admitted with a possible pituitary disorder. Which information has the most immediate implication for the client's care? A. Blood glucose 125 mg/dL B. Blood urea nitrogen (BUN) 40 mg/dL C. Serum potassium 5.2 mEq/L D. Serum sodium 110 mEq/L

D The normal range for serum sodium is 135 to 145 mEq/L; a result of 110 mEq/L is considered hyponatremia and is extremely dangerous. The client is at risk for increased intracranial pressure, seizures, and death. The RN must act rapidly because this situation requires immediate intervention. The normal range for fasting blood glucose is 60 to 110 mg/dL; 125 mg/dL is high, but is not considered dangerous. The normal range for BUN is 7 to 20 mg/dL; 40 mg/dL is high. An elevated BUN can be an indication of kidney failure, dehydration, fever, increased protein intake, and shock, so the client should have a creatinine drawn for a more complete picture of kidney function. The normal range for serum potassium is 3.5 to 5.2 mEq/L; 5.2 mEq/L is high normal.

The nurse is caring for a patient suffering from anorexia secondary to chemotherapy. Which of the following strategies would be most appropriate for the nurse to use to increase the patient's nutritional intake? A. Increase intake of liquids at mealtime to stimulate the appetite. B. Serve three large meals per day plus snacks between each meal. C. Avoid the use of liquid protein supplements to encourage eating at mealtime. D. Add items such as skim milk powder, cheese, honey, or peanut butter to selected foods.

D The nurse can increase the nutritional density of foods by adding items high in protein and/or calories (such as peanut butter, skim milk powder, cheese, honey, or brown sugar) to foods the patient will eat.

Which assessment should the nurse perform first for a patient who just vomited bright red blood? a. Measuring the quantity of emesis b. Palpating the abdomen for distention c. Auscultating the chest for breath sounds d. Taking the blood pressure (BP) and pulse

D The nurse is concerned about blood loss and possible hypovolemic shock in a patient with acute gastrointestinal (GI) bleeding. BP and pulse are the best indicators of these complications. The other information is important to obtain, but BP and pulse rate are the best indicators for assessing intravascular volume

The nurse and the UAP are helping to take care of the patient who is on a mechanical ventilator. Which of the following, if done by the UAP, requires intervention by the nurse? A) Once a day the UAP moves the ETT tube from one side of the mouth to the other B) The UAP monitors for any alarms coming from the machine C) Performs ROM exercises with the client D) Asks the patient to rate his pain using his marker board

D The nurse is responsible for assessing pain on a patient, not the UAP. All other parts are fully within the UAP's scope of practice.

A male client has doubts about performing peritoneal dialysis at home. He informs the nurse about his existing upper respiratory infection. Which of the following suggestions can the nurse offer to the client while performing an at-home peritoneal dialysis? a) Perform deep-breathing exercises vigorously. b) Avoid carrying heavy items. c) Auscultate the lungs frequently. d) Wear a mask when performing exchanges.

D The nurse should advise the client to wear a mask while performing exchanges. This prevents contamination of the dialysis catheter and tubing, and is usually advised to clients with upper respiratory infection. Auscultation of the lungs will not prevent contamination of the catheter or tubing. The client may also be advised to perform deep-breathing exercises to promote optimal lung expansion, but this will not prevent contamination. Clients with a fistula or graft in the arm should be advised against carrying heavy items.

A client with complaints of right lower quadrant pain is admitted to the emergency department. Blood specimens are drawn and sent to the laboratory. Which laboratory finding should be reported to the physician immediately? a) Hematocrit 42% b) Serum potassium 4.2 mEq/L c) Serum sodium 135 mEq/L d) White blood cell (WBC) count 22.8/mm3.

D The nurse should report the elevated WBC count. This finding, which is a sign of infection, indicates that the client's appendix might have ruptured. Hematocrit of 42%, serum potassium of 4.2 mEq/L, and serum sodium of 135 mEq/L are within normal limits. Alterations in these levels don't indicate appendicitis.

A client is admitted with a diagnosis of acute appendicitis. When assessing the abdomen, the nurse would expect to find rebound tenderness at which location? a) Left lower quadrant b) Left upper quadrant c) Right upper quadrant d) Right lower quadrant

D The pain of acute appendicitis localizes in the right lower quadrant (RLQ) at McBurney's point, an area midway between the umbilicus and the right iliac crest. Often, the pain is worse when manual pressure near the region is suddenly released, a condition called rebound tenderness.

A patient with chronic kidney disease (CKD) is started on hemodialysis, and after the first treatment, the patient complains of nausea and a headache. The nurse notes mild jerking and twitching of the patient's extremities. The nurse will anticipate the need to a. increase the time for the next dialysis to remove wastes more completely. b. switch to continuous renal replacement therapy (CRRT) to improve dialysis efficiency. c. administer medications to control these symptoms before the next dialysis. d. slow the rate for the next dialysis to decrease the speed of solute removal.

D The patient has symptoms of disequilibrium syndrome, which can be prevented by slowing the rate of dialysis so that fewer solutes are removed during the dialysis. Increasing the time of the dialysis to remove wastes more completely will increase the risk for disequilibrium syndrome. CRRT is a less efficient means of removing wastes and, because it is continuous, would not be used for a patient with CKD. Administration of medications to control the symptoms is not an appropriate action; rather, the disequilibrium syndrome should be avoided.

After providing a patient with discharge instructions on the management of a new permanent pacemaker, the nurse knows that teaching has been effective when the patient states a."I will avoid cooking with a microwave oven or being near one in use." b."It will be 1 month before I can take a bath or return to my usual activities." c."I will notify the airlines when I make a reservation that I have a pacemaker." d."I won't lift the arm on the pacemaker side up very high until I see the doctor."

D The patient is instructed to avoid lifting the arm on the pacemaker side above the shoulder to avoid displacing the pacemaker leads. The patient should notify airport security about the presence of a pacemaker before going through the metal detector, but there is no need to notify the airlines when making a reservation. Microwave oven use does not affect the pacemaker. The insertion procedure involves minor surgery that will have a short recovery period.

The nurse is teaching a patient with Addison's disease about corticosteroid therapy. The nurse should prioritize which of these teaching points?Multiple choice question a. "Plan a high-carbohydrate diet." b. "Increase your daily intake of sodium." c. "Decrease your daily intake of calcium." d. "Do not stop taking the medication abruptly."

D The patient should be instructed to not stop the medication abruptly because this can cause adverse side effects. Patients taking corticosteroids should not consume a high-carbohydrate diet, because corticosteroids increase blood sugar. Patients should also increase their daily intake of calcium to prevent bone loss due to the side effects of corticosteroids. Patients should also decrease, not increase, their daily intake of sodium to avoid fluid retention.

After receiving change-of-shift report, which patient should the nurse assess first? a. Patient who is scheduled for the drain phase of a peritoneal dialysis exchange b. Patient with stage 4 chronic kidney disease who has an elevated phosphate level c. Patient with stage 5 chronic kidney disease who has a potassium level of 3.4 mEq/L d. Patient who has just returned from having hemodialysis and has a heart rate of 124/min

D The patient who is tachycardic after hemodialysis may be bleeding or excessively hypovolemic and should be assessed immediately for these complications. The other patients also need assessments or interventions but are not at risk for life-threatening complications

When caring for a patient with a history of a total gastrectomy, the nurse will monitor for a. constipation. b. dehydration. c. elevated total serum cholesterol. d. cobalamin (vitamin B12) deficiency.

D The patient with a total gastrectomy does not secrete intrinsic factor, which is needed for cobalamin (vitamin B12) absorption. Because the stomach absorbs only small amounts of water and nutrients, the patient is not at higher risk for dehydration, elevated cholesterol, or constipation.

When assessing a 22-year-old patient who required emergency surgery and multiple transfusions 3 days ago, the nurse finds that the patient looks anxious and has labored respirations at a rate of 38 breaths/min. The oxygen saturation is 90% with the oxygen delivery at 6 L/min via nasal cannula. Which action is most appropriate? A.) Increase the flow rate on the oxygen to 10 L/min and reassess the patient after about 10 minutes. B.) Assist the patient in using the incentive spirometer and splint his chest with a pillow while he coughs. C.) Administer the ordered morphine sulfate to the patient to decrease his anxiety and reduce the hyperventilation. D.)Switch the patient to a nonrebreather mask at 95% to 100% fraction of inspired oxygen (FIO2) and call the health care provider to discuss the patient's status.

D The patient's history and symptoms suggest the development of acute respiratory distress syndrome (ARDS), which will require intubation and mechanical ventilation to maintain oxygenation and gas exchange. The HCP must be notified so that appropriate interventions can be taken. Application of a nonrebreather mask can improve oxygenation up to 95 to 100%. The maximum oxygen delivery with a nasal cannula is an Fio2 of 44%. This is achieved with the oxygen flow at 6 L/min, so increasing the flow to 10 L/min will not be helpful. Helping the patient to cough and deep breathe will not improve the lung stiffness that is causing his respiratory distress. Morphine sulfate will only decrease the respiratory drive and further contribute to his hypoxemia.

An 80-year-old patient with uncontrolled type 1 diabetes mellitus is diagnosed with aortic stenosis. When conservative therapy is no longer effective, the nurse knows that the patient will need to do or have what done? a. Aortic valve replacement b. Take nitroglycerin for chest pain. c. Open commissurotomy (valvulotomy) procedure d. Percutaneous transluminal balloon valvuloplasty (PTBV) procedure

D The percutaneous transluminal balloon valvuloplasty (PTBV) procedure is best for this older adult patient who is a poor surgery candidate related to the uncontrolled type 1 diabetes mellitus. Aortic valve replacement would probably not be tolerated well by this patient, although it may be done if the PTBV fails and the diabetes is controlled in the future. Nitroglycerin is used cautiously for chest pain because it can reduce BP and worsen chest pain in patients with aortic stenosis. Open commissurotomy procedure is used for mitral stenosis.

The patient is being treated with brachytherapy for cervical cancer. What factors must the nurse be aware of to protect herself when caring for this patient? A. The medications the patient is taking B. The nutritional supplements that will help the patient C. How much time is needed to provide the patient's care D. The time the nurse spends at what distance from the patient

D The principles of ALARA (as low as reasonably achievable) and time, distance, and shielding are essential to maintain the nurse's safety when the patient is a source of internal radiation. The patient's medications, nutritional supplements, and time needed to complete care will not protect the nurse caring for a patient with brachytherapy for cervical cancer

A 45-year-old patient is admitted to the emergency department with severe abdominal pain and rebound tenderness. Vital signs include temperature 102° F (38.3° C), pulse 120, respirations 32, and blood pressure (BP) 82/54. Which prescribed intervention should the nurse implement first? a. Administer IV ketorolac (Toradol) 15 mg. b. Draw blood for a complete blood count (CBC). c. Obtain a computed tomography (CT) scan of the abdomen. d. Infuse 1 liter of lactated Ringer's solution over 30 minutes.

D The priority for this patient is to treat the patient's hypovolemic shock with fluid infusion. The other actions should be implemented after starting the fluid infusion.

A client with acquired immunodeficiency syndrome has a respiratory infection from Pneumocystis jiroveci and a nursing diagnosis of Impaired Gas Exchange written in the plan of care. Which of the following indicates that the expected outcome of care has not yet been achieved? A. Client has clear breath sounds B. Client now limits his fluid intake C. Client expectorates secretions easily D. Client is free of complaints of shortness of breath

D The status of the client with a diagnosis of Impaired gas exchange would be evaluated against the standard outcome criteria for this nursing diagnosis. These would include the client stating that breathing is easier and is coughing up secretions effectively, and has clear breath sounds. The client should not limit fluid intake because fluids are needed to decrease the viscosity of secretions for expectoration.

The nurse is instructing the patient with stable angina about the use of nitroglycerin. Which of the following statements, made by the patient, indicates the need for further teaching? A) "When the pain is really bad, I can crush the tablet with my teeth to make it work faster" B) "I will be sure to never remove the pills from the container they are in" C) "It's probably a good idea to take the medication before I have sex with my wife" D) "I should call my doctor if I experience flushing of the face or a HA"

D These are common side effects of this medication and there is not a need to call the physician for this issue. The patient can crush the tablet between his teeth, then place sublingually for hastening of effect. The nitroglycerin pills must be stored in a dry, cool place and should be kept in the dark bottles they come in. Nitroglycerin should be taken in advance of any activity that may produce pain, such as sexual activity.

A client who smokes is being discharged home on oxygen. The client states, "My lungs are already damaged, so I'm not going to quit smoking." What is the discharge nurse's best response? A. "You can quit when you are ready." B. "It's never too late to quit." C. "Just turn off your oxygen when you smoke." D. "You are right, the damage has been done. But let's talk about why smoking around oxygen is dangerous."

D This is a great opening for the nurse to educate the client about the dangers of smoking in the presence of oxygen, as well as the benefits of quitting.

After change-of-shift report, which patient should the nurse assess first? a. 42-year-old who has acute gastritis and ongoing epigastric pain b. 70-year-old with a hiatal hernia who experiences frequent heartburn c. 53-year-old who has dumping syndrome after a recent partial gastrectomy d. 60-year-old with nausea and vomiting who has dry oral mucosa and lethargy

D This older patient is at high risk for problems such as aspiration, dehydration, and fluid and electrolyte disturbances. The other patients will also need to be assessed, but the information about them indicates symptoms that are typical for their diagnoses and are not life threatening.

The nurse is caring for a patient who has hypothyroidism. To assist in differentiating between primary and secondary hypothyroidism, the nurse will expect the provider to order which drug? a. Liothyronine sodium (Cytomel) b. Liotrix (Thyrolar) c. Methimazole (Tapazole) d. Thyrotropin (Thytropar)

D Thyrotropin is a purified extract of thyroid-stimulating hormone and is used as a diagnostic agent todifferentiate between primary and secondary hypothyroidism. Liothyronine and liotrix are thyroid replacementdrugs. Methimazole is used to decrease thyroid hormone secretion

A nurse prepares to defibrillate a client who is in ventricular fibrillation. Which priority intervention should the nurse perform prior to defibrillating this client? a. Make sure the defibrillator is set to the synchronous mode. b. Administer 1 mg of intravenous epinephrine. c. Test the equipment by delivering a smaller shock at 100 joules. d. Ensure that everyone is clear of contact with the client and the bed.

D To avoid injury, the rescuer commands that all personnel clear contact with the client or the bed and ensures their compliance before delivery of the shock. A precordial thump can be delivered when no defibrillator is available. Defibrillation is done in asynchronous mode. Equipment should not be tested before a client is defibrillated because this is an emergency procedure; equipment should be checked on a routine basis. Epinephrine should be administered after defibrillation.

A client diagnosed with widespread lung cancer asks the nurse why he must be careful to avoid crowds and people who are ill. What is the nurse's best response? A. "With lung cancer, you are more likely to develop pneumonia and could pass this on to other people who are already ill." B. "When lung cancer is in the bones, it becomes a bone marrow malignancy, which stops producing immune system cells." C. "The large amount of mucus produced by the cancer cells is a good breeding ground for bacteria and other microorganisms." D. "When lung cancer is in the bones, it can prevent production of immune system cells, making you less resistant to infection."

D Tumor cells that enter the bone marrow reduce the production of healthy white blood cells (WBCs), which are needed for normal immune function. Therefore clients who have cancer, especially leukemia, are at an increased risk for infection. Other people are not at risk for becoming infected as a result of contact with a person who has lung cancer. Lung cancer that has spread to the bone is still lung cancer; it is not a bone marrow malignancy. It is true that the person with lung cancer may produce more mucus, which can harbor microorganisms, but this is not the main reason why the client should avoid crowds and people who are ill.

The nurse is discussing treatment options with the client newly diagnosed with breast cancer. Which statement by the client indicates a need for further teaching? A. "Hormonal therapy is only used to prevent the growth of cancer. It won't get rid of it." B. "I might have chemotherapy before surgery." C. "If I get radiation, I am not radioactive to others." D. "Radiation will remove the cancer, so I might not need surgery."

D Typically, radiation therapy follows surgery to kill residual tumor cells. Radiation therapy plays a critical role in the therapeutic regimen and is effective treatment for almost all sites where breast cancer can metastasize. The purpose of radiation therapy is to reduce the risk for local recurrence of breast cancer.

Which of the following types of angina is most closely related with an impending MI? A) Angina decubitus B) Chronic stable angina C) Noctural angina D) Unstable angina

D Unstable angina progressively increases in frequency, intensity, and duration and is related to an increased risk of MI within 3 to 18 months.

A client with chronic renal failure who is not receiving dialysis is suffering from uremia. What nutrient will the nurse tell this client to limit in an attempt to control the uremia? a. carbohydrate b. potassium c. magnesium d. protein

D Uremia is a condition in which protein wastes that should normally have been excreted are instead circulating in the blood. The diet may limit protein to as little as 40 grams a day for predialysis clients.

A patient has developed DI after a head injury. Which medication should the nurse anticipate to be prescribed for the management of DI? A. Corticotrophin (Acthar) B. Octreotide (Sandostatin) C. Somatropin (Genotropin) D. Desmopressin (DDAVP)

D Vasopressin (Pitressin) and desmopressin (DDAVP) are used to prevent or control polydipsia (excessive thirst), polyuria, and dehydration in patients with DI caused by a deficiency of endogenous antidiuretic hormone.

A client has frequent bursts of ventricular tachycardia on the cardiac monitor. A nurse is most concerned with this dysrhythmia because: A) It is uncomfortable for the client, giving a sense of impending doom. B) It produces a high cardiac output that quickly leads to cerebral and myocardial ischemia. C) It is almost impossible to convert to a normal sinus rhythm. D) It can develop into ventricular fibrillation at any time.

D Ventricular tachycardia is a life-threatening dysrhythmia that results from an irritable ectopic focus that takes over as the pacemaker for the heart. The low cardiac output that results can lead quickly to cerebral and myocardial ischemia. Client's frequently experience a feeling of impending death. Ventricular tachycardia is treated with antidysrhythmic medications or magnesium sulfate, cardioversion (client awake), or defibrillation (loss of consciousness), Ventricular tachycardia can deteriorate into ventricular defibrillation at any time.

A nurse providing care of a patient's chest drainage system observes that the chest tube has become separated from the drainage device. What would be the first action that should be taken by the nurse in this situation? a) Notify the physician. b) Apply an occlusive dressing on the site. c) Assess the patient for signs of respiratory distress. d) Put on gloves and insert the chest tube in a bottle of sterile saline.

D When a chest tube becomes separated from the drainage device, the nurse should first put on gloves, open a sterile bottle of normal saline or water, and insert the chest tube into the bottle without contaminating the chest tube. This creates a water seal until a new drainage unit can be attached. Then the nurse should assess vital signs and notify the physician.

Which of the following would indicate that Bobby's appendix has ruptured? a) diaphoresis b) anorexia c) pain at Mc Burney's point d) relief from pain

D all are normal signs of having appendicits and once you have relief from pain means you could have a rupture

During the assessment of a patient with acute abdominal pain, the nurse should: a. perform deep palpation before auscultation b. obtain blood pressure and pulse rate to determine hypervolemic changes c. auscultate bowel sounds because hyperactive bowel sounds suggest paralytic ileus d. measure body temperature because an elevated temperature may indicate an inflammatory or infectious process.

D for the patient complaining of acute abdominal pain, nurse should take vital signs immediately. Increased pulse and decreasing blood pressure are indicative of hypovolemia. An elevated temperature suggests an inflammatory infectious process. Intake and output measurements provide essential information about the adequate of vascular volume. Inspect abdomen first and then auscultate bowel sounds. Palpation is performed next and should be gentle.

The nurse is caring for a 47-year-old female patient who is comatose and is receiving continuous enteral nutrition through a soft nasogastric tube. The nurse notes the presence of new crackles in the patient's lungs. In which order will the nurse take action? a. Check the patient's oxygen saturation. b. Notify the patient's health care provider. c. Measure the tube feeding residual volume. d. Stop administering the continuous feeding.

DACB The assessment data indicate that aspiration may have occurred, and the nurse's first action should be to turn off the tube feeding to avoid further aspiration. The next action should be to check the oxygen saturation because this may indicate the need for immediate respiratory suctioning or oxygen administration. The residual volume should be obtained because it provides data about possible causes of aspiration. Finally, the health care provider should be notified and informed of all the assessment data the nurse has just obtained.

A client with a tracheostomy is at increased risk for aspiration. Which nursing intervention(s) will reduce this risk? SELECT ALL THAT APPLY. A. Encourage frequent sipping from a cup B. Encourage water with meals C. Inflate the tracheostomy cuff during meals D. Maintain the client upright for 30 minutes after eating E. Provide small, frequent meals F. Teach the client to "tuck" the chin down in the forward position to swallow

DEF At least 30 minutes is required for thinner liquids in the stomach to be thickened in combination with stomach contents and/or removed from the stomach; this reduces the chance of aspiration. Eating requires significant time and energy. When the client becomes tired, he is more likely to aspirate. Shorter and more frequent intervals of eating tire the client less and reduce the chance of aspiration. Tucking the chin downward helps to open the upper esophageal sphincter.

The nurse is teaching a client who is receiving an anti-estrogen drug about the side effects she may encounter. Which of these should the nurse include in the discussion? Select all that apply. A. Heavy menses B. Smooth facial skin C. Hyperkalemia D. Breast tenderness E. Weight loss F. Deep vein thrombosis (DVT)

DF Breast tenderness and shrinking breast tissue, Venous thromboembolism, Irregular menses or no menstrual period, Acne may develop, Hypercalcemia, not hyperkalemia, is typical and Fluid retention with weight gain may occur.

A

Diabetes insipidus puts you at risk for?

A 6-year-old child is seen in the urgent care unit for a history of seizures at home. He begins to have seizures in the urgent care unit that last more than 5 minutes. IV access has not been successful. The nurse caring for this child is knowledgeable that either of these medications may be given to stop the child's seizures: a. IM phenytoin b. Rectal diazepam c. Buccal midazolam d. a and c e. b and c

E

C

Early stage of severe burns puts you at risk for? A. Hypokalemia B. Hyponatremia C. Hyperkalemia D. Hypernatremia

True or False: Endocarditis only affects the atrioventricular and semi-lunar valves in the heart.

False

A

Metabolic alkalosis puts you at risk for? A. Hypokalemia B. Hyponatremia C. Hyperkalemia D. Hypernatremia

A

Severe malnutrition in an older adult man can make him at risk for? A. Hypokalemia B. Hyponatremia C. Hyperkalemia D. Hypernatremia

A

Short bowel syndrome with use of total parentaeral nutrition can put you at risk for? A. Hypokalemia B. Hyponatremia C. Hyperkalemia D. Hypernatremia

C

The advanced practice nurse is assessing the patient with a risk for hypocalcemia. What is the correct technique to test for Chvostek's sign? a. Patient flexes arms against the chest and examiner attempts to pull the arms away from the chest. b. Place a blood pressure cuff around the upper arm and inflate the cuff to greater than the patient's systolic pressure c. Tap the face just below and in front of the ear to trigger facial twitching of one side of the mouth, nose, and cheek d. Lightly tap the patellar and Achilles tendons with a reflex hammer and measure the movement.

D

The client admitted with a serum sodium level of 110 mEq/L. Which nursing intervention should be implemented? a. Encourage fluids orally b. Administer 10% saline solution IVPB c. Administer antidiuretic hormone intranasally d. Place on seizure precautions

C

The client has been vomiting and has had numerous episodes of diarrhea. Which laboratory test should the nurse monitor? a. Serum calcium b. Serum phosphorus c. Serum potassium d. Serum sodium

B

The client has received IV solutions for three days through a 20-gauge IV catheter placed in the left cephalic vein. On morning rounds, the nurse notes the IV site is tender to palpation and a red streak has formed. Which intervention should the nurse implement first? a. Start a new IV in the right hand b. Discontinue the intravenous line c. Complete an incident record d. Place a warm washrag over the site

ABE

The client is NPO and is receiving total parenteral nutrition (TPN) via a subclavian line. Which precautions should the nurse implement? (Select all that apply) a. Place the solution on an IV pump at the prescribed rate b. Monitor blood glucose every six hours c. Weigh the client weekly, first thing in the morning d. Change the IV tubing every three days e. Monitor intake and output every shift

B

The nurse administering potassium to the patient carefully monitors the infusion because of the risk for which condition? a. Pulmonary edema b. Cardiac dysrhythmia c. Postural hypotension d. Renal failure

C

The nurse and an unlicensed assitive personnel (UAP) are caring for a group of clients. Which nursing intervention should the nurse perform? a. Measure the client's output from the indwelling catheter b. Record the client's intake and output on the I & O sheet c. Instruct the client on appropriate fluid restriction d. Provide water for a client diagnosed with diabetes insipidus

A

The nurse assessing the patient notes a bounding pulse quality, neck vein distention when supine, presence of crackles in the lungs, and increasing peripheral edema. What condition does the nurse suspect? a. Fluid excess b. Fluid deficit c. Electrolyte imbalance d. Serum protein increase

A

The nurse identifies the nursing diagnosis of Risk for Injury for the patient with hyponatremia. What is the etiology of this diagnosis? a. Altered mental capabilities b. Fragility of bones c. Immobility d. Altered senses

A

The nurse is assessing skin turgor in the 65-year-old patient. What is the correct technique to use with this patient? a. Pinch the skin over the sternum and observe for tenting and resumption of skin to its normal position after release. b. Observe the skin for a dry, scaly appearance and compare it to a previous assessment. c. Pinch the skin over the back of the hand and observe for tenting; count the number of seconds for the skin to recover position. d. Observe the mucous membranes and tongue for cracks, fissures, or a pasty coating.

A

The nurse is assessing the patient with severe hypermagnesemia. Which assessment findings are associated with this electrolyte imbalance? a. Bradycardia and hypotension b. Tachycardia and weak palpable pulse c. Hypertension and irritability d. Irregular pulse and deep respirations

A

The nurse is assessing the patient's neuromuscular status to obtain a baseline because the patient is at risk for electrolyte imbalances. What technique does the nurse use to assess muscle strength in the legs? a. Ask the patient to push the feet against a flat surface and apply resistance to the opposite side of the flat surface. b. Ask the patient to walk around the room and observe for stride, gait, balance, and endurance. c. Instruct the patient to stand at the side of the bed and abduct each leg as high as possible. d. Support the patient's lower leg with the palm and move the knee through flexion and extension.

C

The nurse is assessing the weight of the patient with chronic renal failure. The patient shows a 2 kg weight gain since the last clinic appointment. This is equivalent to how many liters of fluid? a. 0.5 b. 1 c. 2 d. 3

C

The nurse is caring for a client diagnosed with diabetic ketoacidosis (DKA). Which statement best explains the scientific rationale for the client's Kussmaul's respirations? a. The kidneys produce excess urine and the lungs try to compensate b. The respirations increase the amount of carbon dioxide in the bloodstream c. The lungs speed up to release carbon dioxide and increase the pH d. The shallow and slow respirations will increase the HCO3 in the serum.

C

The nurse is caring for a client with congestive heart failure. On assessment, the nurse notes that the client is dyspneic and that crackles are audible on auscultation. The nurse suspects excess fluid volume. What addition signs would the nurse expect to note in this client if excess fluid volume is present? a. Weight loss b. Flat neck and hand veins c. An increase in blood pressure d. A decreased central venous pressure (CVP)

B

The nurse is caring for a patient who takes potassium and digoxin. For what reason does the nurse monitor both laboratory results? a. Digoxin increases potassium loss through the kidneys. b. Digoxin toxicity can result if hypokalemia is present. c. Digoxin may cause potassium levels to rise to toxic levels d. Hypokalemia causes the cardiac muscle to be less sensitive to digoxin

C

The nurse is caring for several patients with electrolyte imbalances. Which condition may require the patient to be put on seizure precautions? a. Hypercalcemia b. Hyperphosphatemia c. Hypocalcemia d. Hypokalemia

A

The nurse is giving discharge instructions to the patient with advanced congestive heart failure who is at continued risk for fluid volume excess. For which physical change does the nurse instruct the patient to call the health care provider? a. Greater than 3 lbs gained in a week or greater than 1 to 2 lbs gained in a 24-hour period. b. Greater than 5 lbs gained in a week or greater than 1 to 2 lbs gained in a 24-hour period. c. Greater than 15 lbs gained in a month or greater than 5 lbs gained in a week. d. Greater than 20 lbs gained in a month or greater than 5 lbs gained in a week.

B

The nurse is reviewing the laboratory calcium level results for the patient. Which value indicates mild hypocalcemia? a. 5.0 mg/dL b. 8.0 mg/dL c. 10.0 mg/dL d. 12.0 mg/dL

ADEG

The nurse is teaching the patient to recognize food sthat are high in sodium. Which food items does the nurse use as examples? (Select all that apply) a. Egg roll with soy sauce b. Whate rice c. Salad with oil and vinegar dressing d. Bacon and eggs e. Cottage cheese and tomato f. Steak g. Soup with saltine crackers h. Steamed vegetable

A

The nurse is working in a long-term care facility where there are numerous patients who are immobile and at risk for dehydration. Which task is best to delegate to the unlicensed assistive personnel (UAP)? a. Offer patients a choice of fluids every 1 to 2 hours. b. Check patients at the beginning of the shift to see who is thirsty. c. Give patients extra fluids around medication times. d. Evaluate oral intake and urinary output.

A

The nurse monitors the effectiveness of magnesium sulfate by assessing which factor every hour? a. Deep tendon reflexes b. Vital signs c. Serum laboratory values d. Urine output

BCEF

The nurse provides instructions to a client with a low potassium level about the foods that are high in potassium and tells the client to consume which foods? (Select all that apply) a. Peas b. Raisins c. Potatoes d. Cauliflower e. Cantaloupe f. Strawberries

B

The older adult patient at risk for fluid and electrolyte problems is vigilantly monitored by the nurse for the first indication of a fluid balance problem. What is this indication? a. Fever b. Mental status change c. Poor skin turgor d. Dry mucous membranes

D

The patient complains of muscle cramping in the calves, paresthesia of the toes, and the sensation of the heart skipping a beat. These symptoms may indicate which one of the following imbalances? a. Hypernatremia b. Hypercalcemia c. Hypoglycemia d. Hyperkalemia

A

The patient has a low potassium level and the physician has ordered an IV infusion. Before starting an IV potassium infusion, what does the nurse assess? a. Adequate urine output b. Oxygen saturation level c. Baseline mental status d. Apical pulse

BCD

The patient has an elevated potassium level. Which assessment findings are associated with hyperkalemia? (Select all that apply) a. Wheezing on exhalation b. Numbness in hands, feet, and around the mouth c. Frequent, explosive diarrhea stools d. Irregular heart rate and hypotension e. Circumoral cyanosis

B

The patient has chronic renal failure (CRF). Which electrolyte imbalance often associated with hypocalcemia and CRF does the nurse monitor for? a. Hypophosphatemia b. Hyperphosphatemia c. Hyperkalemia d. Hyponatremia

A

The patient has hyperkalemia resulting from dehydration. Which additional laboratory findings does the nurse anticipate for this patient? a. Increased hematocrit and hemoglobin levels b. Decreased serum electrolyte levels c. Increased urine potassium levels d. Decreased serum creatinine

B

The patient has hyperphosphatemia. Which accompanying and potentially life-threatening electrolyte imbalance does the nurse monitor for? a. Hypercalcemia b. Hypocalcemia c. Hyponatremia d. Hyperkalemia

D

The patient is at risk for fluid volume excess. For self-management at home, what does the nurse teach the patient to do? a. Increase diuretic dose if swelling occurs. b. Limit the amount of free water in relation to sodium intake. c. Monitor his or her skin turgor. d. Weigh self each day on the same scale.

C

The patient is talking to the nurse about sodium intake. Which statement by the patient indicates an understanding of high sodium food sources? a. "I have bacon and eggs every morning for breakfast." b. "We never eat seafood because of the salt water." c. "I love Chinese food, but I gave it up because of the soy sauce." d. "Pickled herring is a fish and my doctor told me to eat alot of fish.

A

The patient's laboratory results show a decrease in serum phosphorus level. The nurse expects to see a reciprocal increased change in which serum level? a. Calcium b. Potassium c. Sodium d. Magnesium

C

The patient's potassium level is 2.5 mEq/L. Which clinical findings does the nurse expect to see when assessing this patient? a. Hypertension, bounding pulses, and bradycardia b. Moist crackles, tachypnea, and diminished breath sounds c. General skeletal muscle weakness, lethargy, and weak hand grasps d. Increased specific gravity and decreased urine output

B

The patient's serum potassium value is below 2.8 mEq/L. The patient is also on digoxin. The nurse quickly assesses the patient for which cardiac problem before notifying the physician? a. Cardiac murmur b. Cardiac dysrhythmia c. Congestive heart failure d. Cardiac tamponade

A

The physician has ordered therapy for the patient with low sodium and signs of fluid volume excess. Which diuretic is best for this patient? a. Conivaptan (Vaprisol) b. Furosemide (Lasix) c. Hydrochlorothiazide (HydroDiuril) d. Bumetanide (Bumex)

D

The physician orders magnesium sulfate (MgSO4) for the patient with severe hypomagnesemia. What is the preferred route of administration for this drug? a. Oral b. Subcutaneous c. Intramuscular d. Intravenous

C

What is the consequence and clinical manifestation for the patient who does not meet the obligatory urine output? a. Increased salivation and alkalosis b. Increased thirst with dry mucous membranes c. Lethal electrolyte imbalance and acidosis d. Bradycardia and decreased nitrogen level.

A

Which analogy best approximates the principles of diffusion and concentration gradient? a. Game with four players on one side and eight on the other; two move over to create six per side b. Community fun run where 2000 participants move across the line in a mass start. c. Basketball game of five players per side; all players move across the court. d. Concert where 1000 people are trying to enter through a single gate.

AEFGH

Which assessment findings are related to hypercalcemia? (Select all that apply) a. Bradycardia b. Paresthesia c. Leg cramping d. Hyperactive bowel sounds e. Ineffective respiratory movements f. Shortened QT interval g. Impaired blood flow h. Profound muscle weakness

B

Which intake-output record represents the norm for the average adult? a. 500 mL of fluid per day, ingesting an additional 200 mL of fluid from food. b. 1500 mL of fluid per day, ingesting an additional 800 mL of fluid from food. c. 3000 mL of fluid per day, ingesting an additional 500 mL of fluid from food. d. 5000 mL of fluid per day, ingesting an additional 100 mL of fluid from food.

D

Which intervention does the nurse implement for the patient with hypocalcemia? a. Encourage activitiy by thepatient as tolerated, including weight-lifting b. Encourage socialization and active participation in stimulating activities c. Include a tracheostomy tray at the bedside for emergency use d. Provide adequate intake of vitamin D and calcium rich foods

A

Which is a preventive nursing intervention for patients at risk for developing hypercalcemia? a. Ensure adequate hydration b. Discourage weight-bearing activity such as walking c. Monitor the patient for fluid volume excess d. Administer multivitamin tablets twice per day

B

Which nursing interventions does the nurse include for the patient with hypophosphatemia? a. Aggressive treatment with parenteral p;hosphorous b. Administration of oral vitamin D and phosphorus supplements c. Concurrent administration of calcium supplements d. Elimination of beef, pork, and legumes from the diet

B

Which of the following nursing interventions is most important when caring for a patient receiving a plasma volume expander? a. Assess the patient for deep vein thrombosis b. Observe for signs of fluid overload c. Encourage fluid intake d. Monitor arterial blood gases

C

Which patient condition places the patient at risk for hypocalcemia, hyperkalemia, and hypernatremia? a. Hypothyroidism b. Diabetes mellitus c. Chronic renal failure d. Adrenal insufficiency

ACDF

Which patients are at risk for developing hyponatremia? (Select all that apply) a. Postoperative patient who has been NPO for 24 hours. b. Patient with decreased fluid intake for several days c. Patient with excessive intake of 5% dextrose solution d. Diabetic patient with blood glucose of 250 mg/dL e. Patient with overactive adrenal glands f. Tennis player in 100 degree F weather

D

Which person is most likely to have symptoms related to poor lymph circulation? a. Person with carpal tunnel syndrome b. Marathon runner c. Person with a history of myocardial infarction d. Frequent overseas flyer

D

Which serum laboratory value does the nurse expect to see in the patient with hyperkalemia? a. Calcium greater than 8.0 mg/dL b. Potassium greater than 3.5 mEq/L c. Calcium greater than 11.0 mg/dL d. Potassium greater than 5.0 mEq/L

D

Which serum laboratory value does the nurse expect to see in the patient with hypokalemia? a. Calcium less than 8.0 mg/dL b. Potassium less than 5.0 mEq/L c. Calcium less than 11.0 mg/dL d. Potassium less than 3.5 mEq/L

A

Which serum level of phosphorus represents hypophosphatemia? a. 2.5 mg/dL b. 3.5 mg/dL c. 4.5 mg/dL d. 5.5 mg/dL

A

the patient shows a positive Trousseau's or Chvostek's sign. The nurse prepares to give the patient which urgent treatment? a. IV calcium b. Calcitonin (Calcimar) c. IV potassium chloride d. Large doses of oral calcium

B

the patient's potassium level is low. What change in the cardiovascular system does the nurse expect to see related to hypokalemia? a. Tall, peaked T waves b. Weak, thready pulse c. Malignant hypertension d. Distended neck veins

Under secretion of the adrenocortical hormones leads to: A. Cushing's syndrome B. Addison's disease

B

The public health nurse is giving a lecture on potential outbreaks of infectious meningitis. Which population is most at risk for an outbreak? 1. Clients recently discharged from the hospital. 2. Residents of a college dormitory. 3. Individuals who visit a third world country. 4. Employees in a high-rise office building.

2

The nurse is caring for a child who had a seizure 15 minutes after sustaining a head injury. After assuring a patent airway, which of the following is the priority intervention? 1 Assess fluid and electrolyte status 2 Administer prescribed benzodiazepine 3 Monitor for postconcussive syndrome 4 Observe for signs of increased intracranial pressure

2

A client is admitted to an emergency department, and a diagnosis of myxedema coma is made. Which action would the nurse prepare to carry out initially? 1. Warm the client. 2.Maintain a patent airway. 3.Administer thyroid hormone. 4.Administer fluid replacement.

2

The nurse is assessing a client with hyperthyroidism. What findings should the nurse expect? 1. Weight gain, constipation, and lethargy 2. Weight loss, nervousness, and tachycardia 3. Exophthalmos, diarrhea, and cold intolerance 4. Diaphoresis, fever, and decreased sweating

2 Weight loss, nervousness, and tachycardia are signs of hyperthyroidism. Other signs of hyperthyroidism include exophthalmos, diaphoresis, fever, and diarrhea. Weight gain, constipation, lethargy, decreased sweating, and cold intolerance are signs of hypothyroidism.

A nurse is planning care for a child with acute bacterial meningitis. Based on the mode of transmission of this infection, which of the following would be included in the plan of care? 1. No precautions are required as long as antibiotics have been started 2. Maintain enteric precautions 3. Maintain respiratory isolation precautions for at least 24 hours after the initiation of antibiotics 4. Maintain neutropenic precautions

3

The client is scheduled for a bilateral adrenalectomy for Cushing's disease. Which information regarding the prescribed prednisone should the nurse teach? Select all that apply. 1. When discontinuing this medication, it must be tapered. 2. Take the medication regularly; do not skip doses. 3. Stop taking the medication if you develop a round face. 4. Notify the HCP if you start feeling thirsty all the time. 5. Wear a MedicAlert bracelet in case of an emergency.

245

the client is admitted to the medical dept. with a dx of r/o acute pancreatitis. which laboratory values should the nurse monitor to confirm this dx? 1. creatinine and BUN 2. troponin and CK-MB 3. serum amylase and lipase 4. serum bilirubin and calcium

3 serum amylase increases within 2 to 12 hrs of the onset of acute pancreatitis to 2 to 3 times normal and returns to normal in 3 to 4 days; lipase elevates and remains elevated for 7 to 14 days

A client who was tested for human immunodeficiency virus (HIV) after a recent exposure had a negative result. During the post-test counseling session, the nurse tells the client which of the following? a) the test should be repeated in 6 months b) this ensures that the client is not infected with the HIV virus c) the client no longer needs to protect himself from sexual partners d) the client probably has immunity to the acquired immunodeficiency virus

A

B

A nurse caring for a group of clients reviews the electrolyte laboratory results and notes a sodium level of 130 mEq/L on one client's laboratory report. The nurse understands that which client is at highest risk for the development of a sodium value at this level? a. The client with Cushing's syndrome b. The client who is taking diuretics c. The client with hyperaldosteronism d. The client who is taking corticosteroids

C

A nursing student needs to administer potassium chloride intravenously as prescribed to a client with hypokalemia. The nursing instructor determines that the student is unprepared for this procedure if the student states that which of the following is part of the plan for preparation and administration of the potassium? a. Obtaining a controlled intravenous (IV) infusion pump b. Monitoring urine output during administration c. Preparing the medication for bolus administration d. Diluting the medication in appropriate amount of normal saline

C

A patient will be sent home on diuretic therapy and will need to increase the amount of potassium in the diet. What food choices would the nurse suggest be added? a. Liver, red meats, lettuce b. Apples, pears, celery, onions c. Bananas, tomatoes, beans, fresh meats d. Potato chips, licorice, rice, corn

The nurse should include all of the following in the plan of care for the client with a chest tube r/t hemothorax? SATA: A) Report drainage of 100 ml/hr B) Teach the patient to cough and deep breath frequently C) Report intermittent bubbling in the water seal chamber D) Keep the patient on bedrest with bedside commode E) Loop tubing to keep it off of the floor

ABC

A client with CKD returns to the nursing unit following a HD treatment. On assessment, the nurse notes that the client's temperature is 101.2. Which nursing action is most appropriate? a. encourage fluid intake b. notify the HCP c. continue to monitor vital signs d. monitor the site of the shunt for infection

B

A patient who is receiving mechanical ventilation is anxious and is fighting the ventilator. Which action should the nurse take first? a. Ventilate the patient with a manual resuscitation bag. b. Verbally coach the patient to breathe with the ventilator. c. Sedate the patient with the ordered PRN lorazepam (Ativan). d. Increase the rate for the ordered propofol (Diprivan) infusion.

B

Premature ventricular contractions (PVCs) occur while the nurse is suctioning a patients endotracheal tube. Which action by the nurse is best? a. Decrease the suction pressure to 80 mm Hg. b. Stop and ventilate the patient with 100% oxygen. c. Document the dysrhythmia in the patients chart. d. Give prescribed PRN antidysrhythmic medications.

B

During the PD, the nurse observes that the solution draining from the client's abdomen is consistently blood tinged. The client has a permanent peritoneal catheter in place. The nurse should recognize that the bleeding a. is expected with a permanent peritoneal catheter b. indicates abdominal blood vessel damage c. can indicate kidney damage d. is caused by too-rapid infusion of the dialysate

B

During the exudative phase of acute respiratory distress syndrome (ARDS), the patient's lung cells that produce surfactant have become damaged. As the nurse you know this will lead to? A. bronchoconstriction B. atelectasis C. upper airway blockage D. pulmonary edema

B

During the postictal period of a seizure, you would expect the patient to A. demonstrate minor jerking and eye fluttering. B. sleep for several hours. C. be incontinent of urine and feces. D. require ventilator assistance.

B

Synchronized Intermittent Mechanical Ventilation is best for which patient? A. The patient with sleep apnea B. The patient trying to wean from mechanical ventilation C. The patient who is receiving neuromuscular blocking agents D. The patient who has respiratory drive but cannot sustain normal tidal volume

B

How do you assess the accessory nerve? A. Assess the gag reflex by stroking the posterior pharynx. B. Ask the patient to shrug the shoulders against resistance. C. Ask the patient to push the tongue to either side against resistance. D. Have the patient say "ah" while visualizing elevation of the soft palate

B

Is responsible for maintaining adequate levels of calcium in the extracellular fluid A. Thyroid Gland B. Parathyroid gland

B

The acute medical nurse is preparing to wean a patient from the ventilator. Which assessment parameter is most important for the nurse to assess? A) Fluid intake for the last 24 hours B) Baseline arterial blood gas (ABG) levels C) Prior outcomes of weaning D) Electrocardiogram (ECG) results

B

The nurse is caring for a patient who has decreased mobility. Which intervention is a simple and cost-effective method for reducing the risks of stasis of pulmonary secretions and decreased chest wall expansion? A: Antibiotics B: Frequent change of position C: Oxygen humidification D: Chest physiotherapy

B

The nurse is caring for a patient who is scheduled to have a thoracotomy. When planning preoperative teaching, what information should the nurse communicate to the patient? A) How to milk the chest tubing B) How to splint the incision when coughing C) How to take prophylactic antibiotics correctly D) How to manage the need for fluid restriction

B

A nurse is completing teaching to a client who has a new prescription for famotidine (Pepcid). Which ofthe following statements by the client indicates understanding of the teaching? A. "This medicine coats the lining of my stomach." B. "This should stop the pain right away." C. "I will take my pill at meal time." D. "I will monitor for bleeding from my nose."

C

When caring for a client with a tracheostomy, the nurse would perform which recommended action? a) Clean the wound around the tube and inner cannula at least every 24 hours. b) Assess a newly inserted tracheostomy every 3 to 4 hours. c) Suction the tracheostomy tube using sterile technique. d) Use gauze dressings over the tracheostomy that are filled with cotton.

C

The nurse is caring for a patient with chronic obstructive pulmonary disease (COPD). The patient has been receiving high-flow oxygen therapy for an extended time. What symptoms should the nurse anticipate if the patient were experiencing oxygen toxicity? A) Bradycardia and frontal headache B) Dyspnea and substernal pain C) Peripheral cyanosis and restlessness D) Hypotension and tachycardia

B

Nurse is assessing neuro status of client who had craniotomy 3 days ago. Nurse should notify surgeon immediately if client exhibits which of the following? A.Pupils equal and reactive at 4 mm in size B.Pain with forward flexion of the neck onto the chest. C.Mild headache relieved by codeine sulfate D.Disorientation to date

B

Nurse is giving hydralazine hydrochloride (apresoline) to a patient with autonomic dysreflexia. which of the following findings indicates that the medication is effective? A. patient's muscle spasms subside B. patient's blood pressure declines C. intensity of patient's seizures decline D. patient states they feel better

B

Of what precautions should a client receiving radioactive iodine-131 be made aware? a.) Drink plenty of fluids, especially those high in calcium. b.) Avoid close contact with children or pregnant women for one week after administration of drug. c.) Be aware of the symptoms of tachycardia, increased metabolic rate, and anxiety. d.) Wear a mask if around children or pregnant women.

B

A patient who is recovering from an acute myocardial infarction (AMI) asks the nurse about when sexual intercourse can be resumed. Which response by the nurse is best? a. "Most patients are able to enjoy intercourse without any complications." b. "Sexual activity uses about as much energy as climbing two flights of stairs." c. "The doctor will provide sexual guidelines when your heart is strong enough." d. "Holding and cuddling are good ways to maintain intimacy after a heart attack."

B

A black male client with asthma seeks emergency care for acute respiratory distress. Because of this client's dark skin, the nurse should assess for cyanosis by inspecting the: a. lips. b. mucous membranes. c. nail beds. d. earlobes

B

Antiretroviral drugs are used to a. cure acute HIV infection. b. decrease viral RNA levels. c. treat opportunistic diseases. d. decrease pain and symptoms in terminal disease.

B

Aspirin is administered to the client experiencing an MI because of its: A) Antipyrectic action B) Antithrombotic action C) Antiplatelet action D) Analgesic action

B

Which statement regarding continuous ambulatory peritoneal dialysis (CAPD) would be most important when teaching a patient new to the treatment? a. "Maintain a daily written record of blood pressure and weight." b. "It is essential that you maintain aseptic technique to prevent peritonitis." c. "You will be allowed a more liberal protein diet once you complete CAPD." d. "Continue regular medical and nursing follow-up visits while performing CAPD."

B

Which teaching is essential for a client who has had a permanent pacemaker inserted? a. Avoid talking on a cell phone. b. Avoid contact sports and blows to the chest. c. Avoid sexual activity. d. Do not take tub baths.

B No pressure should be applied over the generator site.

The nurse will plan to monitor a patient with an obstructed common bile duct for a. melena. b. steatorrhea. c. decreased serum cholesterol levels. d. increased serum indirect bilirubin levels.

B A common bile duct obstruction will reduce the absorption of fat in the small intestine, leading to fatty stools. Gastrointestinal (GI) bleeding is not caused by common bile duct obstruction. Serum cholesterol levels are increased with biliary obstruction. Direct bilirubin level is increased with biliary obstruction.

Which action should the nurse perform when preparing a patient with supraventricular tachycardia for cardioversion who is alert and has a blood pressure of 110/66 mm Hg? a.Turn the synchronizer switch to the "off" position. b.Give a sedative before cardioversion is implemented. c.Set the defibrillator/cardioverter energy to 360 joules. d.Provide assisted ventilations with a bag-valve-mask device.

B When a patient has a nonemergency cardioversion, sedation is used just before the procedure. The synchronizer switch is turned "on" for cardioversion. The initial level of joules for cardioversion is low (e.g., 50). Assisted ventilations are not indicated for this patient.

a nurse is caring for a client who has just been admitted following surgical evacuation of a subdural hematoma. which of the following is the priority assessment? A. glasgow coma scale B. cranial nerve function C. oxygen saturation D. pupillary response

C

A 54-year-old man has just arrived in the recovery area after an upper endoscopy. Which information collected by the nurse is most important to communicate to the health care provider? a. The patient is very drowsy. b. The patient reports a sore throat. c. The oral temperature is 101.6° F. d. The apical pulse is 104 beats/minute.

C A temperature elevation may indicate that a perforation has occurred. The other assessment data are normal immediately after the procedure.

Which of the following ECG characteristics is consistent with a diagnosis of ventricular tachycardia (VT)? a. Unmeasurable rate and rhythm b. Rate 150 beats/min; inverted P wave c. Rate 200 beats/min; P wave not visible d. Rate 125 beats/min; normal QRS complex

C VT is associated with a rate of 150 to 250 beats/min; the P wave is not normally visible. P wave inversion and a normal QRS complex are not associated with VT. Rate and rhythm are not measurable in ventricular fibrillation.

You're providing a community in-service about gastrointestinal disorders. During your teaching about cholecystitis, you discuss how cholelithiasis can lead to this condition. What are the risk factors for cholelithiasis that you will include in your teaching to the participants? Select all that apply: A. Being male B. Underweight C. Being female D. Older age E. Native American F. Caucasian G. Pregnant H. Family History I. Obesity

CDEGHI

A female client with asthma is receiving a theophylline preparation to promote bronchodilation. Because of the risk of drug toxicity, the nurse must monitor the client's serum theophylline level closely. The nurse knows that the therapeutic theophylline concentration falls within which range? a. 1 to 2 mcg/ml b. 2 to 5 mcg/ml c. 5 to 10 mcg/ml d. 10 to 20 mcg/ml

D

A client is discharged home with an enteral feeding tube. What does the home health nurse do to determine the patency of the client's enteral tube? A. Arranges for the client to have an x-ray performed periodically B. Auscultates the client's abdomen for bowel sounds before each feeding C. Instills air into the tube to check for placement and patency before each feeding D. Tests aspirated tube contents for pH level before each feeding

D

A client is receiving nutrition via parenteral nutrition (PN). A nurse assess the client for complications of the therapy and assesses the client for which of the following signs of hyperglycemia? A. High-grade fever, chills, and decreased urination. B. Fatigue, increased sweating, and heat intolerance. C. Coarse dry hair, weakness, and fatigue. D. Thirst, blurred vision, and diuresis

D

A client admitted for uncontrolled hypertension and chest pain was prescribed a low sodium diet and started on furosemide (Lasix). The nurse should instruct the client to include which foods in the diet? A. cabbage B. Liver C. Apples D. Bananas

D

A client asks the nurse about early detection of breast masses. Which statement by the nurse about early detection of breast masses is correct? A. "A yearly breast examination by a health care provider can substitute for breast self-examination (BSE)." B. "Detection of breast cancer before axillary node invasion yields the same survival rate." C. "Mammography as a baseline screening is recommended by the American Cancer Society at 30 years of age." D. "The goal of screening for breast cancer is early detection because BSE does not prevent breast cancer."

D

A nurse is completing an admission assessment of a client who has pancreatitis. Which of the following is an expected finding? A. Pain in right upper quadrant radiating to right shoulder B. Report of pain being worse when sitting upright C. Pain relieved with defecation D. Epigastric pain radiating to left shoulder

D A client who has pancreatitis will report severe, boring epigastric pain that radiates to the back, left flank, or left shoulder.

The nurse's client asks, "How did I get rheumatic heart disease?" The most appropriate response by the nurse is that rheumatic heart disease is frequently a result of? a. Hypertension. b. Pregnancy. c. Genetic tendency. d. Streptococcal infection.

D Rheumatic heart disease commonly occurs in children after an infection of group a beta hemolytic streptococcal pharyngitis.

Which of the nursing interventions should be implemented to manage appendicitis? a. Assess pain b. encourage oral intake of clear fluids. c. provide discharge teaching D. assess for symptoms of peritonitis.

D monitor for peritonitis because if the appendix ruptures, bacteria can enter the peritoneum. Pain will be managed with analgesics, and pt should be NPO for surgery. Discharge is not done at this time

D

The nurse is assisting a community group to plan a family sports day. In order to prevent dehydration, what beverage does the nurse suggest be supplied? a. Iced tea b. Light beer c. Diet soda d. Bottled water

A

The nurse is assessing the patient with a mild increase in sodium level. What early manifestation does the nurse observe in this patient? a. Muscle twitching and irregular muscle contractions. b. Inability of muscles and nerves to respond to a stimulus c. Muscle weakness occurring bilaterally with no specific pattern. d. Reduced or absent deep tendon reflexes.

B

The nurse weighs the patient and finds that there has been a weight gain of 1.5 kg since the previous day. What would be the nurse's next highest priority? a. Check with the patient to see if there have been any dietary changes in the last few days b. Assess the patient for signs of edema and BP for possible hypertension c. Contact dietary to change the patient's diet to reduced sodium d. Request a diuretic from the patient's provider

B

The nurse writes the client problem of "fluid volume excess" (FVE). Which intervention should be included in the plan of care? a. Change the IV fluid from 0.9% NS to D5W b. Restrict the sodium in the client's diet c. Monitor blood glucose levels d. Prepare the client for hemodialysis

The client diagnosed with Addison's disease is being discharged. Which statement indicates the client needs more discharge teaching? 1. "I will be sure to keep my dose of steroid constant and not vary." 2. "I may have to take two forms of steroids to remain healthy." 3. "I will get weak and dizzy if I don't take my medication." 4. "I need to notify any new HCP of the medications I take."

1 The dose of corticosteroids may have to be increased during the stress of an infection or surgery. It is imperative that under these circumstances the client receives enough medication to replicate the body's own responses to stress.

The client is admitted to the intensive care department diagnosed with myxedemacoma. Which assessment data warrant immediate intervention by the nurse? 1. Serum blood glucose level of 74 mg/dL. 2. Pulse oximeter reading of 90%. 3. Telemetry reading showing sinus bradycardia. 4. The client is lethargic and sleeps all the time.

2 A pulse oximeter reading of less than 93% is significant. A 90% pulse oximeter reading indicates a PaO2 of approximately 60 on an arterial blood gas test; this is severe hypoxemia and requires immediate intervention.

The client is admitted to rule out Cushing's syndrome. Which laboratory tests should the nurse anticipate being ordered? 1. Plasma drug levels of quinidine, digoxin, and hydralazine. 2. Plasma levels of ACTH and cortisol. 3. A 24-hour urine for metanephrine and catecholamine. 4. Spot urine for creatinine and white blood cells (WBCs).

2 adrenal gland secretes cortisol & pituitary gland scrte adrenocorticotropic hormone (ACTH), used by the body to stimulate the production of cortisol.

The nurse enters the patient's room at the beginning of her shift. The patient is 3 days post-op right-sided pneumonectomy. Which of the following findings requires most immediate intervention by the nurse? 1) The patient is slowly sipping iced water. 2) The CNA reports that urinary output for the last 6 hours is 200 mL. 3) The patient is positioned on her left side with SCDs in place. 4) The patient reports pain at 9/10.

3 The post-op pneumonectomy patient should be positioned on the OPERATIVE (bad) side OR on the back. Sipping iced water in itself isn't harmful to this patient. Urinary output is sufficient. Pain is expected, although this would be the nurse's second concern.

A client is admitted for treatment of the syndrome of inappropriate antidiuretic hormone (SIADH). Which nursing intervention is appropriate? 1. Infusing I.V. fluids rapidly as ordered 2. Encouraging increased oral intake 3. Restricting fluids 4. Administering glucose-containing I.V. fluids as ordered

3 To reduce water retention in a client with the SIADH, the nurse should restrict fluids. Administering fluids by any route would further increase the client's already heightened fluid load.

the client is immediate postprocedure endoscopic retrograde cholangiopancreatogram (ERCP). which intervention should the nurse implement? 1. assess for rectal bleeding, 2, increase fluid intake 3. assess gag reflex 4. keep in supine position

3 the gag reflex will be suppressed as a result of the local anesthesia applied to the throat to insert the endoscope into the esophagus; therefore, the gag reflex must be assessed prior to allowing the client to resume eating or drinking

The nurse admitting a patient with acromegaly anticipates administering which medication? 1. desmopressin (DDAVP) 2. corticotropin (Acthar) 3. somatropin (Nutropin) 4. octreotide (Sandostatin)

4

The nurse should assess a client with hypothyroidism for: 1. corneal abrasion due to inability to close the eyelids. 2. weight loss due to hypermetabolism. 3. fluid loss due to diarrhea. 4. decreased activity due to fatigue.

4

The nurse would suspect excessive thyroid replacement in a patient taking levothyroxine (Synthroid) when the patient is exhibiting which adverse effect? 1. Depression 2. Intolerance to cold 3. Weight gain 4. Irritability

4

The nurse uses the minimal occluding volume technique to inflate the cuff on an ET tube to minimize the incidence of what? a. infection b. hypoexmia c. tracheal damage d. accidental extubation

C

The nurse will teach a patient with chronic pancreatitis to take the prescribed pancrelipase (Viokase) a. at bedtime. b. in the morning. c. with each meal. d. for abdominal pain.

C

A HD client with a left arm fistula is at risk for arterial steal syndrome. The nurse should assess for which manifestations of this complication? a. warmth, redness, and pain in the left hand b. ecchymosis and audible bruit over the fistula c. edema and reddish discoloration of the left arm d. pallor, diminished pulse, and pain in the left hand

D

What indicates to the nurse that a patient with AKI is in the recovery phase? a. A return to normal weight b. A urine output of 3700 mL/day c. Decreasing sodium and potassium levels d. Decreasing blood urea nitrogen (BUN) and creatinine levels

The blood urea nitrogen (BUN) and creatinine levels remain high during the oliguric and diuretic phases ofAKI. The recovery phase begins when the glomerular filtration returns to a rate at which BUN and creatinine stabilize and then decrease. Urinary output of 3 to 5 L/day, decreasing sodium and potassium levels, and fluid weight loss are characteristic of the diuretic phase of AKI.

C

The client is admitted to a nursing unit from a long-term care facility with a hematocrit of 56% and a serum sodium level of 152 mEq/L. Which condition is a cause for these findings? a. Overhydration b. Anemia c. Dehydration d. Renal failure


Ensembles d'études connexes

Psych test 3 (anxiety, personality disorders, & substance abuse)

View Set

Missouri Life and Health Insurance Final Exam

View Set

Chapter 4 Market Failures: Public Goods and externalities

View Set

BHAN Exam 2 - Chapter 11 & 11A: Maintaining a Healthy Weight & Body Image

View Set